Chuyên đề bồi dưỡng học sinh giỏi toán 9 và ôn thi vào chuyên phần số học

245 145 0
Chuyên đề bồi dưỡng học sinh giỏi toán 9 và ôn thi vào chuyên phần số học

Đang tải... (xem toàn văn)

Tài liệu hạn chế xem trước, để xem đầy đủ mời bạn chọn Tải xuống

Thông tin tài liệu

Hy vọng rằng qua bài viết này, các bạn đã nắm được một số vấn đề cơ bản liên quan đến định lý rồi từ đó, tìm hiểu, mở rộng thêm để định lý Thặng dư Trung Hoa trong tay chúng ta không chỉ[r]

(1)(2) TRẦN NAM DŨNG (chủ biên) VÕ QUỐC BÁ CẨN - LÊ PHÚC LỮ - PHẠM HY HIẾU TỪ NGUYỄN THÁI SƠN - LÊ VIỆT HẢI Chuyên đề TOÁN HỌC Số d TP HỒ CHÍ MINH, THÁNG 10 NĂM 2010 (3) d (4) LỜI NÓI ĐẦU Chuyên đề Toán học số trường Phổ thông Năng khiếu mà các bạn cầm trên tay là ấn phẩm có nhiều điều đặc biệt Thứ nhất, nó thai nghén khoảng thời gian dài kỷ lục: Ít là năm Thứ hai, nó đời cách chuyên đề trước đó năm Thứ ba, tham gia đóng góp cho Chuyên đề lần này không là các học sinh và thầy cô trường Phổ thông Năng khiếu mà còn nhiều bạn học sinh, sinh viên, các thầy cô các trường khác Internet đã tạo giới khác hẳn, và Toán học không nằm ngoài thay đổi đó Làm Chuyên đề Toán học số này, chúng tôi nhớ các hệ học sinh trường Phổ thông Năng khiếu, tác giả Chuyên đề Toán học số 1, 2, 3, 4, 5, 6, 7, Đó là Lê Long Triều, Trần Quang Ánh, Võ Tâm Vân, Lê Quang Nẫm, Lưu Minh Đức, Nguyễn Lê Lực, Trịnh Lê Tuấn, Lê Minh Tuấn, Phạm Quốc Việt, Hoàng Thanh Lâm, Trần Đình Nguyên, Nguyễn Cẩm Thạch, Phạm Tuấn Anh, Lương Thế Nhân, Trần Vĩnh Hưng, Nguyễn Tiến Khải, Trần Quang, Trần Anh Hoàng, Nguyễn Đăng Khoa, Nguyễn Anh Cường, Trần Chiêu Minh, Kha Tuấn Minh, Chúng tôi có mong muốn thực tuyển chọn các bài viết từ các Chuyên đề này Hy vọng Chuyên đề toán học số với nội dung khá phong phú và hình thức đẹp là món quà tặng ý nghĩa các bạn trẻ yêu Toán Trong biển mênh mông kiến thức, người thực chuyên đề mong ấn phẩm mình là giọt nước có ích Chuyên đề Toán học số hoàn thành với hỗ trợ Công ty cổ phần giáo dục Titan Ban biên tập xin chân thành cảm ơn hỗ trợ này BAN BIÊN TẬP (5) d Chuyên đề Toán học số (6) VỀ NHÀ TÀI TRỢ Công ty cổ phần giáo dục Ti tan (Titan Education) hoạt động lĩnh vực cung cấp dịch vụ giáo dục phổ thông, đó đặc biệt là các sản phẩm có nội dung Toán học Sản phẩm chính công ty là trang web đào tạo Toán học trực tuyến cho học sinh lớp – 12, kho liệu Toán học dành cho học sinh, sinh viên, giáo viên và các nhà nghiên cứu Toán học, các lớp học trực tiếp rèn luyện tư Toán học, bồi dưỡng và phát triển khiếu Toán học, các lớp học nghiệp vụ dành cho giáo viên Toán Ngoài ra, công ty còn tư vấn, cung cấp tài liệu, sách tham khảo Toán cho học sinh và giáo viên các cấp trình độ Titan Education mong muốn nhiệt huyết và lực mình góp phần nhỏ bé vào nghiệp giáo dục và đào tạo hệ trẻ cho đất nước Việt Nam Mọi chi tiết xin liên hệ: Công ty cổ phần giáo dục Ti tan Địa chỉ: 18A Nam Quốc Cang, P Phạm Ngũ Lão, Q 1, TP Hồ Chí Minh Điện thoại: 0854 542305 – 0839 260137 Fax: 0839 260140 – Hotline: 012345 39976 Website: www.titan.edu.vn Email: khotailieu@titan.edu.vn (7) d Chuyên đề Toán học số (8) MỤC LỤC Lời nói đầu Về nhà tài trợ Duyên số Toán học Việt Nam với giải Fields Ngô Việt Trung Thông tin Toán học 15 Khám phá số tính chất dãy số truy hồi tuyến tính cấp hai Đào Hoàng Nhã 17 Định lý thặng dư Trung Hoa Phạm Hy Hiếu 29 Rèn luyện kỹ giải các bài toán Hình học phẳng Lê Phúc Lữ 41 Nhìn Hình học mắt Đại số Từ Nguyễn Thái Sơn 69 Nguồn gốc bài toán Hình học số đề thi Việt Nam TST 2009 Lê Bá Khánh Trình 85 Nhỏ mà không nhỏ Võ Quốc Bá Cẩn 97 Bất đẳng thức Bernoulli Trương Tấn Sang 113 Enlightening Trigonometrical Substitutions Vardan Verdiyan, Daniel Campos Salas 141 Về bài toán Bất đẳng thức Nguyễn Văn Huyện 161 Tổ hợp và công thức Ck2 Đặng Hoàng Linh 167 Mở rộng từ bài toán Từ Nguyễn Thái Sơn 171 (9) Chuyên đề Toán học số Ứng dụng Toán học việc hiển thị hình ảnh máy vi tính Phạm Mộng Bảo 177 Cấp số cộng và Phương trình hàm trên N Nguyễn Trọng Tuấn 187 Sáng tạo Phương trình hàm từ các đẳng thức Lê Việt Hải, Đào Thái Hiệp 193 Lời giải đề thi vào lớp 10 chuyên Toán năm học 2010 - 2011 213 Đáp án đề thi chọn đội tuyển Toán năm học 2008 - 2009 219 Đáp án đề thi chọn đội tuyển Toán năm học 2009 - 2010 229 Một số đề thi Olympic Toán 239 (10) DUYÊN SỐ CỦA TOÁN HỌC VIỆT NAM VỚI GIẢI FIELDS Ngô Việt Trung Viện Toán học Việt Nam Nói là duyên là vì nhiều nước giàu hơn, có truyền thống Toán học Việt Nam không có giải Fields Nước Đức, cường quốc Toán học có giải Fields Ấn Độ và Trung Quốc là cái nôi Toán học lịch sử và là nước có nhiều nhà Toán học tiếng, chưa mon men đến giải Fields Cả châu Á có ba giải Fields, là Nhật Châu Mỹ Latin và châu Phi không có giải Fields Theo nghĩa nào đó, đoạt giải Fields còn khó giải Nobel vì giải Nobel trao hàng năm, lúc giải Fields trao năm lần Nói cách nôm na, người giải Fields phải có kết xuất sắc năm không phải năm giải Nobel Ngoài giải Fields trao cho người không quá 40 tuổi Andrew Wiles, người giải bài toán Fermat không giải Fields còn đủ tuổi vì lời giải ban đầu có lỗ hổng Đến khắc phục lỗ hổng lời giải thì ông đã quá 40 tuổi Nói để thấy giải Fields là duyên thực Trong lịch sử 70 năm giải Fields có 48 người giải Thế mà Việt Nam lại có mối liên hệ chặt chẽ với nhiều người số họ Chúng ta hãy bắt đầu câu chuyện với GS Lê Văn Thiêm Ông Thiêm là người Việt có học vị tiến sĩ Toán học đầu tiên và coi là cha đẻ Toán học Việt Nam Sau bảo vệ tiến sĩ ông Thiêm làm trợ lý cho GS Rolf Nevanlinna trường Đại học Zurich hai lần vào năm 1946 và 1948 Sinh thời ông Thiêm thường coi mình là học trò Nevanlinna, công trình tiếng ông Thiêm là bài toán ngược Nevanlinna Trong Toán học có tạp chí tên là Tạp chí trung tâm Toán học, chuyên đăng bài giới thiệu các công trình công bố Người giới thiệu bài báo ông Thiêm là ông Lars Ahlfors, hai nhà Toán học trao giải Fields lần đầu năm 1936 Ahlfors là học trò ông Nevanlinna (theo nghĩa bảo vệ luận án tiến sĩ) Do ông Nevanlinna không phải là giáo sư hướng dẫn luận án ông Thiêm nên ta có thể coi ông Thiêm là nuôi ông Nevanlinna mặt Toán học Vì ta có thể coi Ahlfors là anh ông Thiêm Giải Fields trao lần thứ hai năm 1950 Một hai người giải là nhà Toán học Pháp Laurent Schwartz Ông Schwartz là người sáng lập Ủy ban quốc gia vì Việt Nam Pháp năm 1966 và Tòa án quốc tế Russel xử tội diệt chủng Mỹ Việt Nam năm 1967 Ông Schwartz sang thăm Việt Nam nhiều lần, lần đầu tiên vào năm 1968 chiến tranh chống Mỹ Năm 1990 ông Bộ Đại học mời sang tham quan và đánh giá giáo dục Việt Nam Ông đã viết báo cáo 40 (11) 10 Chuyên đề Toán học số trang, đó ông đã đưa kết luận tiếng “Việt Nam đã thắng chiến tranh và thua hòa bình” Thắng là vì chiến tranh Việt Nam đã gửi học sinh tốt học nước ngoài Khi trở nước người này đã làm cho Việt Nam trở thành cường quốc khoa học vùng sau chiến tranh Nhưng hòa bình Việt Nam đã không coi trọng việc sử dụng và đãi ngộ đội ngũ khoa học với yếu điểm chính là chế độ lương bổng Điều này đã làm xói mòn khoa học Việt Nam lượng và chất, làm cho khoa học và giáo dục Việt Nam thua nước vùng Trong hồi ký mình (được dịch nhiều thứ tiếng) ông Schwartz dành chương dài viết Việt Nam và kết thúc chương này với câu “Việt Nam đã đánh dấu đời tôi” Có điều thú vị là ông Lê Văn Thiêm và ông Schwartz có chung thầy hướng dẫn luận án là GS Georg Valiron Ông Valiron còn có học trò Việt Nam là GS Phạm Tĩnh Quát Ta có thể coi ông Quát, ông Thiêm và ông Schwartz là anh em ruột mặt Toán học Giải Fields trao lần thứ ba vào năm 1954 Một hai người giải là nhà Toán học Nhật Kunihiko Kodaira Ông có người rể là GS Mutsuo Oka, là nhà Toán học Ông Oka là người bạn lớn Toán học Việt Nam Ông đã thu xếp cho nhiều nhà Toán học Việt Nam sang Nhật làm việc và tham gia quyên góp tiền cho việc xây dựng nhà khách Viện Toán học Khi ông Kodaira năm 1997, gia đình đã định tặng tủ sách chuyên môn ông Kodaira cho thư viện Viện Toán học Năm 1966 giải Fields trao lần đầu tiên cho nhà Toán học, đó có nhà Toán học Pháp Alexander Grothendieck và nhà Toán học Mỹ Steffen Smale Cả hai người tiếng hoạt động chống chiến tranh Mỹ Việt Nam Ông Grothendieck coi là nhà Toán học có ảnh hưởng nửa sau kỷ 20 và là học trò (bảo vệ luận án tiến sĩ) ông Schwartz Để tỏ thái độ chống chiến tranh ông Grothendieck sang thăm Việt Nam năm 1967 lúc Mỹ ném bom Hà Nội ác liệt Ông đã giảng loạt các bài giảng các hướng nghiên cứu Toán học đại, chủ yếu Đại số đồng điều Trong báo cáo chuyên Việt Nam ông viết “có Toán học Việt Nam thật đúng nghĩa nước Việt Nam Dân chủ Cộng hòa” Câu này ông gạch thêm bên để nhấn mạnh Sau ông viết là “tôi chứng minh “Định lý tồn tại” này và giới thiệu tương đối chi tiết Toán học Việt Nam thời giờ” Ông đặc biệt ấn tượng với khả các nhà Toán học trẻ Việt Nam và nêu tên đích danh ba người là Đoàn Quỳnh, Hoàng Xuân Sính và Trần Văn Hạo Ông có kế hoạch đưa người này sang đào tạo bên Pháp Sau này có Hoàng Xuân Sính sang Paris làm luận án tiến sĩ hướng dẫn ông Tham gia Hội đồng bảo vệ luận án có đến ba người giải Fields là Grothendiẹck, Schwartz và Pierre Deligne Có lẽ chưa có Hội đồng bảo vệ luận án tiếng Rất tiếc là GS Hoàng Xuân Sính không công bố các kết luận án Gần đây có bài báo tổng quan hướng nghiên cứu đó có nhắc đến các kết tiên phong GS Hoàng Xuân Sính Ông Grothendieck là thầy (hướng dẫn luận án tiến sĩ) ông Luc Illusie, ông này lại là thầy Gerard Laumon là thầy Ngô Bảo Châu Như Grothendieck là cụ Ngô Bảo Châu và Ngô Bảo Châu có họ hàng với GS Lê Văn Thiêm và GS Hoàng Xuân Sính mặt Toán học (12) Duyên số Toán học Việt Nam với giải Fields 11 Ông Smale coi là nhà bác học Toán học vì ông quan tâm ngiên cứu nhiều chuyên ngành Toán học khác và chuyên ngành nào ông đạt kết xuất sắc Những năm 60 ông là lãnh tụ phong trào trí thức chống chiến tranh Việt Nam Mỹ Năm 1965 ông tổ chức cho sinh viên bãi khóa Đại học California và chặn tàu chở lính Mỹ Berkeley Năm 1966 ông tổ chức họp báo chống chiến tranh Việt Nam bên thềm Đại hội Toán học giới nhận giải Fields Vì hoạt động chống chiến tranh mà ông bị Quỹ khoa học quốc gia Mỹ cắt tiền tài trợ nghiên cứu Năm 2004 Viện Toán học mời GS Smale sang Việt Nam giảng bài với tài trợ Quỹ giáo dục Việt Nam (VEF) Trong buổi nói chuyện với sinh viên Đại học bách khoa Hà Nội ông đã khóc và xin lỗi chiến tranh Việt Nam Ông Smale có học trò người Việt là Hà Quang Minh, làm việc Đại học Humboldt Berlin Đại hội Toán học giới năm 1970 có hai giải Fields liên quan đến Việt Nam Người thứ là nhà Toán học Nhật Heisuke Hironaka Năm 1968 ông Hironaka dạy Lý thuyết kỳ dị cho các nhà Toán học trẻ châu Âu Trong lớp học đó có sinh viên Việt Nam tên là Lê Dũng Tráng tuổi đôi mươi Sau này Lê Dũng Tráng trở thành chuyên gia hàng đầu giới Lý thuyết kỳ dị GS Lê Dũng Tráng là người đưa Hội Toán học Việt Nam gia nhập Liên đoàn Toán học giới là tổ chức xét và trao giải Fields GS Hironaka quan tâm đến việc giúp đỡ Toán học Việt Nam Ông là người đã vận động Hội Toán học Nhật thành lập Chương trình trao đổi Toán học Nhật và Việt Nam Ông đã sang thăm Việt Nam vài lần với tư cách cá nhân Năm 1977 ông công bố công trình Toán học tiếng mình Tạp chí Acta Mathematica Vietnamica Viện Toán, trích dẫn nhiều Người thứ hai là nhà Toán học Nga Sergey Novikov Ông Novikov là thầy Lê Tự Quốc Thắng, huy chương vàng Olympic Toán quốc tế năm 1982 Hiện Lê Tự Quốc Thắng là chuyên gia hàng đầu giới lĩnh vực Tô pô chiều thấp Còn hai giải Fields đã sang làm việc Việt Nam Người thứ là nhà Toán học Mỹ David Mumford giải Fields năm 1974 Ông này đã làm báo cáo mời tai Hội nghị Toán quốc tế Viện Toán phối hợp với Đai học Quy Nhơn tổ chức năm 2005 Người thứ hai là nhà Toán học New Zealand Vaughan Jones, người giải Fields năm 1990 Ông này đã làm báo cáo mời Hội nghị quốc tế Tôpô lượng tử Viện Toán tổ chức năm 2007 và công bố công trình mình tạp chí Acta Mathematica Vietnamica Viện Toán Năm 1978 có nhà Toán học Pháp Pierre Deligne giải Fields Ông Deligne là học trò ông Grothendieck và là thầy GS Lê Dũng Tráng (đồng hướng dẫn) Ông là thành viên Hội đồng bảo vệ GS Hoàng Xuân Sính Do GS Hoàng Xuân Sính là học trò ông Grothendieck nên có thể coi GS Hoàng Xuân Sính là em và Ngô Bảo Châu là cháu họ GS Deligne mặt Toán học Đặc biêt hơn, bạn cùng thầy Ngô Bảo Châu là Laurent Lafforgue giải Fields năm 2002 Học trò đầu tiên Lafforgue là Ngô Đắc Tuấn, người đã đoạt huy chương vàng hai lần thi Olympic Toán quốc tế năm 1995 và 1996 Hiện Ngô (13) 12 Chuyên đề Toán học số Đắc Tuấn làm việc Đại học Paris 13 Gần đây có Terence Tao là nhà Toán học Úc giải Fields năm 2006 có liên quan đến VN Tao có mối quan hệ cộng tác thân thiết với Vũ Hà Văn, là chuyên gia hàng đầu giới lĩnh vực Tổ hợp Họ đã viết chung 15 công trình và sách chuyên khảo Ngoài ra, Tao có cùng thầy với Dương Hồng Phong, là nhà Toán học VN hàng đầu Mỹ Hiện nay, Tao có nghiên cứu sinh người Việt là Lê Thái Hoàng, huy chương vàng Olympic Toán quốc tế năm 1999 Với người trẻ tuổi Ngô Đắc Tuấn và Lê Thái Hoàng theo đuổi nghiệp Toán, biết đâu VN lại có may giải Fields lần Grothendieck và GS Ngô Thúc Lanh (phía sau) và GS Hoàng Tụy (bên phải ảnh) nơi sơ tán Đại học Tổng hợp Đại Từ, Thái Nguyên Bản chụp trích đoạn báo cáo đánh máy Grothendieck: “Có Toán học thật đúng nghĩa nước Việt Nam Dân chủ Cộng hòa” (gạch dưới) (14) Duyên số Toán học Việt Nam với giải Fields Bản chụp đoạn Hồi ký Schwartz báo cáo giáo dục đại học, đó có câu “Việt Nam đã thắng chiến tranh và thua hòa bình” Vợ chồng ông Schwartz và GS Tạ Quang Bửu (bên trái ảnh) thăm Việt Bắc 13 (15) 14 d Chuyên đề Toán học số (16) THÔNG TIN TOÁN HỌC • Một mùa hè với nhiều hoạt động sôi dành cho giáo viên và học sinh chuyên Toán đã kết thúc Năm nay, lần đầu tiên Khóa bồi dưỡng chuyên môn nghiệp vụ hè dành cho giáo viên Toán (gọi tắt là Trường hè bồi dưỡng giáo viên Toán) đã tổ chức ĐHQG TP Hồ Chí Minh từ – 11/8 với tham gia trên 150 giáo viên đến từ các trường THPT chuyên trên toàn quốc GS TSKH Nguyễn Văn Mậu, thầy Nguyễn Khắc Minh, TS Nguyễn Chu Gia Vượng, TS Lê Bá Khánh Trình, TS Nguyễn Văn Minh Mẫn, TS Trần Nam Dũng, ThS Nguyễn Trọng Tuấn, thầy Nguyễn Đức Tấn đã đến giảng bài và chia sẻ nhiều kinh nghiệm quý báu cho đội ngũ các thầy cô giáo dạy Toán • Tiếp nối thành công Trường hè dành cho giáo viên, từ ngày 16 – 22/8, chương trình Gặp gỡ Toán học lần đã Đại học Quốc gia thành phố Hồ Chí Minh phối hợp với trường THCS – THPT Âu Lạc (quận Gò Vấp) tổ chức với tham gia 50 học sinh đến từ các địa phương: thành phố Hồ Chí Minh, Cần Thơ, Đồng Nai, Ninh Thuận, Quảng Nam và Đà Nẵng Bên cạnh bài giảng các thầy cô giáo có nhiều kinh nghiệm, các bạn học sinh còn hướng dẫn nhiệt tình các anh chị sinh viên trưởng thành từ phong trào chuyên Toán Chương trình hoạt động văn nghệ thể thao sôi với các buổi chiều luyện tập và thi đấu thể thao, ngày dã ngoại thác Giang Điền, đêm liên hoan văn nghệ hết mình Đặc biệt các học sinh tham gia chương trình đã GS Egorov và học trò ông là TS Nguyễn Thành Nam, tổng giám đốc Tập đoàn FPT ghé thăm và trao đổi thân mật Toán học • Ngày 19/9, khóa câu lạc Toán học dành cho các học sinh lớp 10 đã khai giảng trường THPT chuyên Lê Hồng Phong Các học sinh sinh hoạt vòng tháng với nhiều hình thức hoạt động phong phú: Nghe giảng chuyên đề, tham gia seminar giải toán, làm bài kiểm tra, tham gia các thi đồng đội, nghe nói chuyện Toán học và Khoa học, dã ngoại, Có trên 60 học sinh đến từ các trường THPT thành phố đã tham gia câu lạc bộ, đó đông là các trường Lê Hồng Phong, Phổ thông Năng khiếu, Trần Đại Nghĩa • Theo thông tin từ ban tổ chức thi Giải thưởng Toán học Shing Tung Yau dành cho học sinh phổ thông thì hai đề án (project) hai nhóm học sinh Phổ thông Năng khiếu (gồm bạn Phạm Hy Hiếu, Nguyễn Mạnh Tiến, Từ Nguyễn Thái Sơn, Phạm Anh Tuấn) đã lọt vào vòng bán kết thi và trình bày trực tiếp trước ban giám khảo Singapore vào chiều 16/10/2010 Được biết project lọt vào bán kết thưởng 1000 USD Các project lọt vào chung kết thưởng 2000 USD và các tác giả bảo vệ Mỹ Thông tin thi có thể xem chi tiết tại: http://www.yau-awards.org/overseas/index.html • Theo thông báo từ Hội Toán học Việt Nam, ngày 22/4/2011, GS Frank Morgan, chuyên gia hàng đầu giới nhân chuyến thăm và làm việc Việt Nam ghé 15 (17) 16 Chuyên đề Toán học số thành phố Hồ Chí Minh và đọc bài giảng public “Bong bóng xà phòng và Toán học” Được biết Frank Morgan không tiếng với công trình khoa học tầm cỡ mình mà có tiếng diễn giả xuất sắc, có thể làm cho học sinh phổ thông hiểu điều cao siêu Thông tin chi tiết Frank Morgan có thể đọc đây: http://math.williams.edu/morgan/ • Phong trào tổ chức seminar Toán học và câu lạc Toán học lan rộng Hiện ngoài seminar Giải tích và Toán sơ cấp tổ chức trường Đại học Khoa học Tự Nhiên – Đại học Quốc gia Hà Nội (do GS Nguyễn Văn Mậu chủ trì), seminar các phương pháp Toán sơ cấp trường Phổ thông Năng khiếu đã có thêm seminar toán sơ cấp Vĩnh Long, An Giang, Long An Đặc biệt, kể từ tháng 10/2010, Viện Toán học Việt Nam tổ chức câu lạc Toán học dành cho học sinh THPT Viện Toán (1 tháng buổi) Được biết bài giảng đầu tiên (vào các tháng 10, 11, 12/2010) GS Hà Huy Khoái, TS Phan Thị Hà Dương, TS Nguyễn Chu Gia Vượng đảm trách (18) KHÁM PHÁ MỘT SỐ TÍNH CHẤT CỦA DÃY SỐ TRUY HỒI TUYẾN TÍNH CẤP HAI Đào Hoàng Nhã HS chuyên Toán khóa 2009 - 2012 Trước hết, để bước vào chuyên đề này, chúng ta cần biết nào là dãy số truy hồi tuyến tính cấp hai Định nghĩa Dãy số truy hồi tuyến tính cấp hai là dãy số có dạng sau un+2 = aun+1 + bun , đó a, b là các số thực khác Dãy số này khá đơn giản đằng sau nó là bí ẩn thú vị chờ đợi chúng ta Nào, chúng ta hãy bắt đầu hành trình Mở đầu cho khám phá các bí ẩn, chúng ta hãy cùng tìm hiểu tính chất thú vị sau Tính chất Cho dãy số {un } thỏa mãn un+2 = aun+1 + bun với n ∈ N∗ Khi đó ta có đẳng thức sau un+2 un − u2n+1 = (−b)n−1 (u3 u1 − u22 ), ∀n ∈ N∗ (1) Chứng minh Trước hết, ta chứng minh un+2 un − u2n+1 = −b(un+1 un−1 − u2n ) (2) Thật vậy, ta có un+2 un − u2n+1 + b(un+1 un−1 − u2n ) = un (un+2 − bun ) − un+1 (un+1 − bun−1 ) = un · aun+1 − un+1 · aun = Vậy (2) chứng minh Từ đây, cách áp dụng liên tiếp (2), ta dễ dàng thu đẳng thức (1) trên Bây giờ, (1) cho b = −1, ta có un+2 un − u2n+1 = u3 u1 − u22 Giả sử un 6= với n ≥ Khi đó, đặt c = u3 u1 − u22 , từ (3) ta suy un+2 = u2n+1 + c un Từ đó ta rút tính chất sau 17 (3) (19) 18 Chuyên đề Toán học số Tính chất Cho dãy {un } xác định u1 = m, u2 = p, u3 = q (mpq 6= 0) và un+2 u2n+1 + c = , un ∀n ≥ 1, đó c = mq − p2 Khi đó, ta có {un } chính là dãy số truy hồi tuyến tính cấp dạng un+2 = aun+1 − un , với a = q+m p Chứng minh Từ giả thiết, ta suy c = un+2 un − u2n+1 (4) c = un+1 un−1 − u2n (5) Thay n n − 1, ta Từ (4) và (5), ta có un+2 un − u2n+1 = un+1 un−1 − u2n , hay un (un+2 + un ) = un+1 (un+1 + un−1 ), un+2 + un un+1 + un−1 = un+1 un Thay n n − 1, n − 2, , 2, ta un+2 + un un+1 + un−1 u3 + u1 q+m = a = = ··· = = un+1 un u2 p Từ đó suy un+2 = aun+1 − un Tính chất chứng minh Các tính chất 1, thường sử dụng cho các dạng toán mà dãy số xác định trên Sau đây là số ví dụ Ví dụ (VMO 1999) Cho dãy số {an } xác định a1 = 1, a2 = 2, an+2 = 3an+1 − an , ∀n ≥ Chứng minh an+2 + an ≥ + a2n+1 , an ∀n ≥ Lời giải Ta dễ dàng chứng minh dãy số trên là dãy tăng, suy an > với n Sử dụng tính chất 1, ta có a2 + an+2 = n+1 an Từ đây, sử dụng bất đẳng thức AM-GM cho hai số dương, ta   a2n+1 + a2n+1 a2 an+2 + an = + an = + + an ≥ n+1 + an an an an Bài toán chứng minh (20) Khám phá số tính chất dãy số truy hồi tuyến tính cấp hai 19 Ví dụ (Bulgaria 1978) Cho dãy số {un } thỏa mãn đồng thời các tính chất sau un 6= 0, ∀n = 1, 2, u1 , u2 ∈ Z u21 + u22 + a ∈ Z u1 u2 un+2 u2n+1 + a = , ∀n = 1, 2, un Chứng minh dãy {un } chứa toàn số nguyên Lời giải Sử dụng tính chất 2, ta có un+2 = kun+1 − un , với k = ∀n ≥ 1, (6) u3 + u1 u2 u22 + a u3 + u1 u2 + u22 + a nên ta có = ∈ Z, suy k ∈ Z Mà theo giả thiết u1 u2 u1 u2 thì u1 , u2 ∈ Z nên từ (6), ta dễ dàng suy un ∈ Z, ∀n ≥ Do u3 = Nhận xét Ví dụ trên cho ta tiêu chuẩn để đánh giá dãy số các số hạng dãy u2 + a {un } với un+2 = n+1 có là các số nguyên hay không un Bây giờ, chúng ta hãy tiếp tục xem xét dãy số {un } thỏa mãn u1 = a, u2 = b, un+2 = dun+1 − un Rõ ràng, a, b là các số nguyên và d là số nguyên thì hiển nhiên số hạng dãy {un } luôn luôn là số nguyên Vậy, d không là số nguyên thì d phải thỏa mãn điều kiện gì để số hạng dãy là các số nguyên? Chính câu hỏi này đã đưa ta đến với bài toán sau Bài toán Cho dãy số {un } thỏa mãn u1 = a, u2 = b, un+2 = dun+1 − un , ∀n ≥ 1, đó a, b là các số nguyên khác và d là số thực khác Tìm giá trị d để số hạng dãy là các số nguyên Bây chúng ta cùng tìm hiểu lời giải bài toán này Lời giải Nếu d là số vô tỉ thì rõ ràng u3 = bd − a là số vô tỉ vì a, b là các số nguyên Do đó ta cần xét d tập số hữu tỉ (21) 20 Chuyên đề Toán học số Giả sử d là số hữu tỉ thỏa mãn yêu cầu bài toán Vì d là số hữu tỉ nên ta có thể đặt p p d = với p, q là các số nguyên khác và (p, q) = Từ đó un+2 = un+1 − un , hay q q q(un+2 + un ) = pun+1 (7) Với n = thì (7) trở thành q(u3 + u1 ) = pu2 q, suy u2 q (do (p, q) = 1) Tương tự, cách cho n nhận giá trị 3, 4, ta có u4 q và u5 q Từ đó cho n = 2, ta suy pu3 = q(u2 + u4 ) q Mà (p, q) = nên (p, q ) = 1, đó từ trên suy u3 q Do pu4 = q(u3 + u5 ) q Vì (p, q ) = nên u4 q Cứ làm ta rút nhận xét là un q k−1 , ∀n ≥ k ≥ (8) Điều này ta có thể chứng minh quy nạp theo k Thật vậy, dễ thấy (8) đúng với k = Giả sử nó đúng tới k, tức là un q k−1 , ∀n ≥ k ≥ Khi đó từ (7) ta suy un+1 q k , tức là un q k , ∀n ≥ k + Nhận xét chứng minh Bây giờ, sử dụng tính chất 1, ta có un+2 un − u2n+1 = c, với c = u3 u1 − u22 là số Từ đây kết hợp với (8), ta suy c q 2k với k lớn tùy ý Do c là số nên c 6= thì q có thể (vì q khác 1, ta có q 2k > c k tăng đến giá trị nào đó), đó d là số nguyên (vì mẫu 1) Xét c = 0, đó ta có un+2 un = u2n+1 (9) Dễ thấy, vì a, b là các số nguyên khác cho nên ta luôn chứng minh un 6= với số nguyên dương n (có thể quy nạp từ (9)) Từ (9) suy un+1 un+2 = , un+1 un ∀n ≥ Suy un+1 un u2 b = = ··· = = = r, un un−1 u1 a ∀n ≥ (22) Khám phá số tính chất dãy số truy hồi tuyến tính cấp hai 21 Do đó un+1 = run , ∀n ≥ Rõ ràng, lúc này dãy số {un } là cấp số nhân, cho nên un+1 = rn a Đến đây, ta viết lại r dạng phân số tối giản r = un là số nguyên với n nên un+1 = (10) u với u, v ∈ Z và (u, v) = Do v un a ∈ Z Từ đó suy a v n (do (un , v n ) = 1) Mà a lại là số nên v có thể nhận giá trị là 1, đó r ∈ Z hay b a Ta lại có ar2 + a r2 + a2 + b u3 + u1 = = = d= u2 ar r ab Thử lại giá trị d, ta thấy số hạng dãy nguyên Từ đó ta rút kết luận • Nếu b không chia hết cho a thì d là số nguyên • Nếu b chia hết cho a thì d là số nguyên d = a2 + b ab Chính bài toán trên đã cho ta tính chất Tính chất Cho dãy số {un } thỏa mãn u1 = a, u2 = b, un+2 = dun+1 − un , ∀n ≥ 1, với a, b là các số nguyên và ab 6= Khi đó, số hạng dãy là các số nguyên và ta có các điều sau • d là số nguyên b không chia hết cho a; • d là số nguyên d = a2 + b b chia hết cho a ab Không dừng lại đây, chúng ta hãy tiếp tục cùng khám phá tiếp, còn nhiều bí ẩn đằng sau chờ đợi chúng ta Xét dãy số {un } thỏa mãn u1 = m, u2 = p và un+2 = aun+1 − un (11) Theo tính chất thì dãy số trên luôn có thể biến đổi dạng un+2 un − u2n+1 = u3 u1 − u22 = c (12) Nào, bây chúng ta hãy nhìn thật kĩ (11) và (12), chúng liệu có mối liên hệ nào không? Các bạn đã nhận chưa nào? Chúng ta cùng xem nhé! (23) 22 Chuyên đề Toán học số Từ (11) và (12), ta suy ( un+2 + un = aun+1 un+2 un = c + u2n+1 Do đó un+2 và un chính là nghiệm phương trình bậc hai X − aun+1 X + u2n+1 + c = 0, với X là ẩn số, un+1 chính là tham số Từ đó ta rút tính chất Tính chất Cho dãy số {un } thỏa mãn u1 = m, u2 = p, un+2 = aun+1 − un Khi đó, un+2 và un chính là nghiệm phương trình bậc hai X − aun+1 X + u2n+1 + c = 0, (13) với X là ẩn số, un+1 là tham số và c = u3 u1 − u22 Có lẽ, đến bây các bạn tự hỏi, tính chất có thể suy điều gì nữa? Bây giờ, ta hãy xem, số hạng dãy {un } là các số nguyên và giả sử a là số nguyên, thì rõ ràng phương trình (13) có hai nghiệm nguyên Mà điều kiện cần để phương trình bậc hai có các hệ số nguyên có nghiệm nguyên là biệt thức ∆ nó phải là số chính phương Mà ∆ = a2 u2n+1 − 4(u2n+1 + c) = (a2 − 4)u2n+1 − 4c Do đó ta có (a2 − 4)u2n+1 − 4c là số chính phương Ta rút tính chất sau Tính chất Cho dãy số {un } thỏa mãn un+2 = aun+1 − un , ∀n ≥ Giả sử số hạng dãy là số nguyên và a là số nguyên Khi đó ta có (a2 − 4)u2n+1 − 4c là số chính phương (c = u3 u1 − u22 ) Ví dụ Cho dãy số {un } xác định sau u1 = 5, u2 = 16, un+2 = 1994un+1 − un , ∀n ≥ Chứng minh 3976032u22508 − 159239 là số chính phương Lời giải Ta có 3976032 = 19942 − 4, u3 = 1994 · 16 − = 31899 và 159239 = 31899 · − 162 Do đó theo tính chất thì 3976032u22508 − 159239 là số chính phương (24) Khám phá số tính chất dãy số truy hồi tuyến tính cấp hai 23 Ví dụ (VMO 1997) Cho dãy số nguyên {an } xác định a0 = 1, a1 = 45 và an+2 = 45an+1 − 7an , với n = 0, 1, 2, (a) Tìm số ước dương a2n+1 − an an+2 theo n (b) Chứng minh với n thì 1997a2n + · 7n+1 là số chính phương Lời giải (a) Ta có a2 = 452 − · = 2018 Sử dụng tính chất 1, ta an+2 an − a2n+1 = 7n (a2 a0 − a21 ), suy a2n+1 − an+2 an = 7n+1 Do là số nguyên tố nên số các ước nguyên dương a2n+1 − an an+2 là n + số (b) Ta dùng tư tưởng các tính chất trên để giải câu hỏi này Từ giả thiết an+2 = 45an+1 − 7an và a2n+1 − an an+2 = 7n+1 (theo tính chất 1), ta suy ( an+2 + 7an = 45an+1 (7an )an+2 = 7a2n+1 − 7n+2 Do đó 7an và an+2 là các nghiệm phương trình bậc hai X − 45an+1 X + 7a2n+1 − 7n+2 = Vì 7an , an+2 là các số nguyên nên phương trình trên có hai nghiệm nguyên, suy biệt thức ∆ nó là số chính phương Mà ∆ = (45an+1 )2 − 4(7a2n+1 − 7n+2 ) = 1997a2n+1 + · 7n+2 , nên ta có 1997a2n+1 + · 7n+2 là số chính phương Hơn nữa, dễ thấy 1997a20 + · 71 = 452 Do ta có 1997a2n + · 7n+1 là số chính phương với số tự nhiên n Nhận xét Qua ví dụ trên, ta thấy tính chất và có thể tổng quát cho trường hợp un+2 = pun+1 + qun , với u1 , u2 là các số nguyên và p, q là các số nguyên Khi đó un+2 − qun = pun+1 và un+2 (−qun ) = −qu2n+1 + (−q)n c với c = u3 u1 − u22 Từ đó ta có un+2 và −qun là hai nghiệm phương trình bậc hai X − pun+1 X − qu2n+1 + (−q)n c = Và đó (p2 + 4q)u2n+1 − 4(−q)n c là số chính phương (25) 24 Chuyên đề Toán học số Bây giờ, chúng ta tiếp tục khám phá tiếp, liệu từ tính chất 4, ta có thể khai thác điều thú vị nào nữa? Các bạn hãy cùng tôi thử giải phương trình (13) Theo công thức nghiệm phương trình bậc hai, ta suy p aun+1 ± (a2 − 4)u2n+1 − 4c X1, = Giả sử un+2 > un , đó ta dãy số truy hồi có dạng q a un+2 = un+1 + (a2 − 4)u2n+1 − 4c 2 Như vậy, ta có thể rút tính chất sau Tính chất Mọi dãy số {un } thỏa mãn u1 = m, a un+2 = un+1 + 2 q (a2 − 4)u2n+1 + 4c, (ac 6= 0) luôn có thể đưa dãy số truy hồi tuyến tính cấp hai có dạng sau un+2 = aun+1 − un a un+1 sang vế trái và bình phương hai vế, ta  a2 1 u2n+2 − aun+2 un+1 + u2n+1 = (a − 4)u2n+1 + 4c , 4 Lời giải Chuyển u2n+2 − aun+2 un+1 + u2n+1 − c = Thay n n − 1, ta có u2n+1 − aun+1 un + u2n − c = Từ đây suy un+2 và un là các nghiệm phương trình bậc hai X − aun+1 X + u2n+1 − c = Áp dụng định lý Viette, ta có un+2 + un = aun+1 , hay un+2 = aun+1 − un Tính chất chứng minh Ví dụ Cho a, b, c là ba số nguyên thỏa mãn a2 = b + và dãy số {un } xác định sau p u0 = 0, un+1 = aun + bu2n + c2 , ∀n = 0, 1, 2, Chứng minh số hạng dãy trên là số nguyên Lời giải Ta có p p 2a 1p un+1 = aun + bun + c2 = aun + (a2 − 1)u2n + c2 = un + (4a − 4)u2n + 4c2 2 Theo tính chất 6, ta suy un+2 = 2aun+1 − un Vì u0 = 0, u1 = |c| ∈ Z nên từ trên ta có un là số nguyên với số tự nhiên n Từ tính chất và tính chất 6, ta có thể suy tính chất sau (26) Khám phá số tính chất dãy số truy hồi tuyến tính cấp hai 25 Tính chất Xét dãy số {un } thỏa mãn u1 , u2 ∈ Z và p un+1 = aun + bu2n + c d d2 − và b = với d là u2 + u22 số nguyên (nếu u2 chia hết không chia hết cho u1 ) d = (nếu u2 chia u1 u2 hết cho u1 ) thì số hạng dãy là nguyên Nếu c là số nguyên và tồn số d cho a = Đây có thể xem tiêu chuẩn để đánh giá dãy số dạng trên là dãy nguyên Trước kết thúc chuyên đề, chúng ta hãy cùng xem xét bài toán sau Bài toán (Bulgaria 1987) Xét dãy số {xn } xác định x1 = x2 = và xn+2 = 14xn+1 − xn − 4, ∀n ≥ Chứng minh với n ≥ 1, ta có xn là bình phương số nguyên Lời giải Để giải thành công bài toán này, trước hết chúng ta cần phải nghiên cứu các giá trị các số hạng đầu tiên dãy và sau đó rút nhận xét mối quan hệ chúng Thử vài giá trị đầu, ta có x1 = 12 , x2 = 12 , x3 = 32 , x4 = 112 , x5 = 412 , x6 = 1532 , Hãy quan sát và để ý, các số trên hoàn toàn có quy luật Ta nhận thấy = · − 1, 11 = · − 1, 41 = · 11 − 3, 153 = · 41 − 11, Từ các yếu tố trên, các số 1, 1, 3, 11, 41, 153, hình thành dựa vào dãy sau y1 = 1, y2 = 1, yn+2 = 4un+1 − yn , ∀n ≥ Bây giờ, việc chúng ta là chứng minh xn = yn2 (14) Cách chứng minh đơn giản chính là sử dụng quy nạp: Dễ thấy (14) đúng với n = 1, Giả sử nó đúng với n = 1, 2, , k, tức là xn = yn2 , ∀n = 1, 2, , k Ta cần chứng minh nó đúng với n = k + Ta có 2 yk+1 = (4yk − yk−1 )2 = 16yk2 − 8yk yk−1 + yk−1 + 4) = 14xk − xk−1 − + (2yk2 − 8yk yk−1 + 2yk−1 = xk+1 + 2yk − 2(4yk − yk−1 )yk−1 + = xk+1 + 2yk2 − 2yk+1 yk−1 + (27) 26 Chuyên đề Toán học số Sử dụng tính chất 1, ta có yk−1 yk+1 − yk2 = 2 = xk+1 Vậy ta có điều phải chứng minh Từ đó suy yk+1 Nhận xét Ta hoàn toàn có thể tổng quát bài toán trên Xét dãy số {un } thỏa mãn un+2 = aun+1 − un và c = u1 u3 − u22 , ta có un+2 un − u2n+1 = c Xét tiếp dãy số {vn } cho = u2n với n, ta có vn+2 = u2n+2 = a2 u2n+1 − 2aun+1 un + u2n = a2 u2n+1 − 2un (aun+1 − un ) − u2n = a2 u2n+1 − u2n − 2un un+2 = (a2 − 2)u2n+1 − 2(un un+2 − u2n+1 ) − u2n = (a2 − 2)vn+1 − − 2c Suy vn+2 = (a2 − 2)vn+1 − − 2c Mặt khác, cách sử dụng quy nạp, ta dễ dàng chứng minh điều ngược lại đúng, tức là: Nếu hai dãy {un }, {vn } thỏa mãn v1 = u21 , v2 = u22 , c = u1 u3 − u22 và un+2 = aun+1 − un , vn+2 = (a2 − 2)vn+1 − − 2c, thì ta có = u2n với n Và vậy, ta có bài toán tổng quát sau: Cho hai dãy số {un } và {vn } thỏa mãn v1 = u21 , v2 = u2 , c = u1 u3 − u22 và un+2 = aun+1 − un Khi đó vn+2 = (a2 − 2)vn+1 − − 2c và = u2n với số nguyên dương n Từ cách chứng minh trên, ta có thể rút phương pháp giải các bài toán dãy số có liên quan đến số chính phương Với tư tưởng trên, ta có thể giải các bài toán tương tự sau Bài toán Cho dãy số {yn } thỏa mãn y1 = y2 = và yn+2 = (4k − 5)yn+1 − yn + − 2k, ∀n ≥ Tìm k ∈ Z cho số hạng dãy là số chính phương (Ta có thể chứng minh dãy số thỏa mãn bài toán trên có dạng sau yn+2 = 7yn+1 − yn − Từ đó, áp dụng cách làm bài toán thì điều phải chứng minh.) (28) Khám phá số tính chất dãy số truy hồi tuyến tính cấp hai 27 Bài toán (Canada 1988) Cho hai dãy số {xn }, {yn } xác định x0 = 0, x1 = 1, xn+1 = 4xn − xn−1 và y0 = 1, y1 = 2, yn+1 = 4yn − yn−1 Chứng minh yn2 = 3x2n + 1, ∀n ∈ N Bài toán Cho dãy {un } thỏa mãn u1 = k, u2 = k + với k là số nguyên và un+1 = un (un − 1) + 2, ∀n = 2, 3, Chứng minh số A = (u21 + 1)(u22 + 1) · · · (u22005 + 1) − là số chính phương (Gợi ý, quy nạp ta chứng minh An = (un+1 − 1)2 với An = (u21 + 1)(u22 + 1) · · · (u2n + 1) − 1.) Sau đây là số bài tập dành cho bạn đọc Bài tập Cho dãy số {un } thỏa mãn u1 = 1, u2 = −1 và un = −un−1 − 2un−2 , ∀n = 3, 4, Xét dãy {vn } sau = 2n+1 − 7u2n−1 , ∀n = 2, 3, Chứng minh số hạng dãy {vn } là số chính phương Bài tập Cho dãy số {un } thỏa mãn u1 = 2, u2 = và un = nun−1 − (n − 2)un−2 − 2n + với n = 3, 4, (a) Tìm n để |un − 2007| có giá trị nhỏ (b) Tìm số dư chia u2007 cho 2006 Bài tập (VMO 1998) Cho dãy số {an } xác định a0 = 20, a1 = 100 và an+2 = 4an+1 + 5an + 20, ∀n ∈ N Tìm số nguyên dương h nhỏ thỏa mãn điều kiện an+h − an chia hết cho 1998 với số tự nhiên n Bài tập (TST Việt Nam 1993) Gọi ϕ(n) là phi hàm Euler Tìm tất các số nguyên dương k > thỏa mãn điều kiện: với a là số nguyên > bất kỳ, đặt x0 = a, xn+1 = kϕ(xn ) với n = 0, 1, 2, thì {xn } luôn bị chặn Bài tập (Ba Lan 2002) Cho k ∈ N∗ và dãy số {an } xác định a1 = k + 1, an+1 = a2n − kan + k, ∀n ∈ N∗ Chứng minh với m, n ∈ N∗ , m 6= n, ta có (an , am ) = (29) 28 Chuyên đề Toán học số Bài tập Cho số thực r và dãy số {xn } xác định x0 = 0, x1 = 1, xn+2 = rxn+1 − xn , ∀n ∈ N Chứng minh x1 + x3 + · · · + x2m−1 = x2m , ∀m ∈ N∗ Tài liệu tham khảo [1] Phan Huy Khải, Số học và dãy số, Nhà xuất Giáo Dục, 2009 [2] Nguyễn Văn Mậu (chủ biên), Chuyên đề chọn lọc dãy số và áp dụng, Nhà xuất Giáo Dục, 2008 [3] Các tài liệu sưu tầm từ internet (30) ĐỊNH LÝ THẶNG DƯ TRUNG HOA Phạm Hy Hiếu HS chuyên Toán khóa 2007 - 2010 Lược dẫn Xưa Trung Quốc, có vị tướng không có tài thao lược chiến trương mà còn có hiểu biết sâu sắc Toán học tên là Hàn Tín Tương truyền điểm quân, ông lệnh cho quân đội mình xếp thành hàng người, người 11 người, sau đó dựa vào quân số dư còn lại sau lần xếp hàng cùng với việc ước lượng quân số mình khoảng bao nhiêu, ông suy quân số chính xác mình Những gì Hàn Tín làm đã các nhà Toán học Trung Quốc và giới ghi nhận lại thành “định lý Thặng dư Trung Hoa” Đây thực là kết đẹp và có nhiều ứng dụng việc giải các bài toán Lý thuyết số và vấn đề cao cấp Toán học Bài viết này hy vọng trao đổi với các bạn định lý này Nhìn lại định lý Thặng dư Trung Hoa Định lý (Định lý Thặng dư Trung Hoa) Cho k là số nguyên dương, m1 , m2 , , mk là các số nguyên đôi nguyên tố cùng và a1 , a2 , , ak là các số nguyên Khi đó hệ phương trình đồng dư  x ≡ a1 (mod m1 )     x ≡ a (mod m ) 2     x ≡ ak (mod mk ) có nghiệm modulo m1 m2 · · · mk Chứng minh Từ phát biểu định lý, ta thấy việc chứng minh cần phải trải qua hai bước sau: (1) Chứng minh tồn nghiệm x modulo m1 m2 · · · mk (2) Chứng minh tính nghiệm theo modulo m1 m2 · · · mk (1) Để chứng minh tồn nghiệm hệ đồng dư, ta sử dụng nguyên lý quy nạp Giả sử kết luận định lý đã đúng với 1, 2, , k Xét k + số nguyên m1 , m2 , , mk+1 đôi nguyên tố cùng Đặt m0k = mk mk+1 Vì (mk , mk+1 ) = nên theo giả thiết quy nạp, hệ đồng dư ( x ≡ ak (mod mk ) x ≡ ak+1 (mod mk+1 ) 29 (31) 30 Chuyên đề Toán học số có nghiệm x ≡ a0k (mod m0k ) Lại theo giả thiết quy nạp thì vì m1 , m2 , , m0k là k số nguyên đôi nguyên tốt cùng nên hệ đồng dư  x ≡ a1 (mod m1 )     x ≡ a (mod m ) 2     x ≡ a0k (mod m0k ) có nghiệm theo modulo m1 m2 · · · m0k nên định lý đúng với k + Cuối cùng ta cần chứng minh định lý trường hợp k = Để làm điều đó, ta cần đến bổ đề sau Bổ đề (Bổ đề Bezóut) Nếu m, n là hai số nguyên thỏa mãn (m, n) = thì tồn các số nguyên u, v thỏa mãn mu + nv = Chứng minh Xét các số có dạng {1 − mu | u ∈ Z}, vì (m, n) = nên tồn u − mu cho n | − mu Khi đó ta cần chọn v = Bổ đề chứng minh  n Vào bài Theo bổ đề Bezóut suy tồn các số nguyên u, v cho m2 v − m1 u = Đặt r = (a1 − a2 )u, s = (a1 − a2 )v thì m2 s − m1 r = a1 − a2 ⇔ a1 + m1 r = a2 + m2 s Đặt x = a1 + m1 r = a2 + m2 s, ta có ( x ≡ a1 (mod m1 ) x ≡ a2 (mod m2 ) Vậy x là nghiệm hệ đồng dư với k = (2) Bây ta chứng minh tính nghiệm Thật giả sử tồn x0 cho    x ≡ x (mod m1 )   x ≡ x0 (mod m )     x ≡ x0 (mod mk ) Thế thì giả thiết m1 , m2 , , mk đôi nguyên tố cùng nên ta có x ≡ x0 (mod m1 m2 · · · mk ) Định lý chứng minh hoàn toàn Có nhiều cách chứng minh định lý Thặng dư Trung Hoa Sau đây ta đến với chứng minh khác cho phần tồn nghiệm (tức phần (1) trên) định lý này theo hướng xây dựng và không cần sử dụng nguyên lý quy nạp (32) Định lý Thặng dư Trung Hoa 31 Chứng  minh Do giả thiết m1 , m2 , , mk đôi nguyên tố cùng nên m gcd , mi = (m = m1 m2 · · · mk ) với i = 1, 2, , k Do vậy, tương tự với mi cách lý luận chứng minh bổ đề Bezóut, ta suy với i tồn bi cho m bi ≡ (mod mi ), ∀i = 1, 2, , k mi Từ đó m bi ≡ (mod mi ), ∀i = 1, 2, , k mi k X m bi với i = 1, 2, , k, ta có Xét số nguyên x0 = m i i=1 k X m bi (mod mi ) x0 = mi i=1 m ≡ bi (mod mi ) mi m bi ≡ (mod )) ≡ (mod mi ) (vì mi Vậy x0 là nghiệm hệ đồng dư bài Nghiên cứu hai chứng minh trên, ta nhận thấy chúng dựa trên tư tưởng bổ đề Bezóut Thực tế, bổ đề này có hệ quan trọng sau Hệ 1.1 Nếu m, n là các số nguyên và (m, n) = d thì với số nguyên h mà d | h thì tồn các số nguyên u, v thỏa mãn h = mu + nv Từ hệ này ta suy mở rộng sau đây định lý Thặng dư Trung Hoa Định lý (Mở rộng định lý Thặng dư Trung Hoa) Cho k là số nguyên dương, m1 , m2 , , mk là các số nguyên và a1 , a2 , , ak là các số nguyên Khi đó hệ phương trình đồng dư  x ≡ a1 (mod m1 )     x ≡ a (mod m ) 2     x ≡ ak (mod mk ) có nghiệm modulo lcm (m1 , m2 , , mk ) và ≡ aj (mod (mi , mj )), ∀1 ≤ i < j ≤ k Mở rộng này hoàn toàn có thể chứng minh phương pháp tương tự hóa từ hai cách chứng minh định lý lúc đầu, khác chỗ sử dụng hệ bổ đề Bezóut thay vì chính bổ đề này Không Lý thuyết số mà định lý Thặng dư Trung Hoa còn có các mở rộng tương tự Lý thuyết nhóm, Lý thuyết vành, trường, ideal, , điều đó vượt quá tầm quan tâm bài viết này (33) 32 Chuyên đề Toán học số Định lý Thặng dư Trung Hoa giải toán phổ thông Trong mục này ta tìm hiểu các bài toán ứng dụng định lý Thặng dư Trung Hoa Trước hết ta đến với ba bài toán đầu tiên mang tính kinh điển cho kỹ thuật sử dụng định lý thặng dư Trung Hoa Bài toán Chứng minh với số nguyên dương k lớn tùy ý tồn k số nguyên dương liên tiếp gồm toàn hợp số Lời giải Gọi p1 < p2 < · · · < pk là các số nguyên tố Theo định lý Thặng dư Trung Hoa thì tồn số nguyên dương n lớn tùy ý cho  n ≡ −1 (mod p1 )     n ≡ −2 (mod p )     n ≡ −k (mod pk ) Vậy với i = 1, 2, , k thì pi | n + i, có thể chọn n > pk , đó n + i là hợp số Bài toán (Problem E27, PEN, Hojoo Lee) Chứng minh với số nguyên dương k lớn tùy ý tồn k số nguyên dương liên tiếp không là lũy thừa số nguyên dương nào khác Lời giải Gọi p1 < p2 < · · · < pk là các số nguyên tố Theo định lý Thặng dư Trung Hoa thì tồn số nguyên dương n lớn tùy ý cho  n ≡ p1 − (mod p21 )     n ≡ p − (mod p2 ) 2     n ≡ pk − k (mod p2k ) Vậy với i = 1, 2, , k thì pi k n + i, ta có thể chọn n > pk , đó n + i là hợp số Lại lý luận trên, biểu diễn sở n + i có thừa số pi nên n + i không phải là lũy thừa số nguyên nào Bài toán (VMO 2008) Cho m = 20072008 Hỏi có tất bao nhiêu số tự nhiên n < m cho m | n(2n + 1)(5n + 2)? Lời giải Xét phân tích tiêu chuẩn m = 20072008 = 34016 · 2232008 Dễ dàng kiểm tra thấy gcd(i, j) < với i, j ∈ {n, 2n + 1, 5n + 2} nên ta có m | n(2n + 1)(5n + 2) và xảy các trường hợp sau: m | n (34) Định lý Thặng dư Trung Hoa 33 m | 2n + m | 5n + 34016 | n và 2232008 | 2n + 34016 | n và 2232008 | 5n + 34016 | 2n + và 2232008 | 5n + 34016 | 2n + và 2232008 | n 34016 | 5n + và 2232008 | n 34016 | 5n + và 2232008 | 2n + Trong trường hợp trên, theo định lý Thặng dư Trung Hoa, tồn giá trị n modulo m thỏa mãn trường hợp Do có tất số tự nhiên n thỏa mãn yêu cầu đề bài Thực chất bài toán VMO 2008 là trường hợp riêng bài toán tổng quát sau Bài toán Cho số nguyên dương n có phân tích tiêu chuẩn n = pa11 pa22 · · · pas s Xét đa thức P (x) có hệ số nguyên Nghiệm x0 phương trình đồng dư P (x) ≡ (mod n) (1) là lớp đồng dư x0 ∈ {0, 1, 2, , n − 1} thỏa mãn P (x0 ) ≡ (mod n) Khi đó, điều kiện cần và đủ để phương trình (1) có nghiệm là với i = 1, 2, , s, phương trình P (x) ≡ (mod pai i ) có nghiệm Hơn nữa, với i = 1, 2, , s, phương trình P (x) ≡ (mod pai i ) có ri nghiệm modulo pai i thì phương trình (1) có r = r1 r2 · · · rs nghiệm modulo n Lời giải Nếu các phương trình P (x) ≡ (mod pai i ) vô nghiệm thì hiển nhiên (1) vô nghiệm Ngược lại, giả sử với i = 1, 2, , s, phương trình P (x) ≡ (mod pai i ) có ri nghiệm là xi1 , xi2 , , xiri Theo định lý Thặng dư Trung Hoa, với cách chọn (x1j1 , x2j2 , , xsjs ), hệ đồng dư  a1   x ≡ x1j1 (mod p1 )   a2 x ≡ x 2j2 (mod p2 )     x ≡ x (mod pas ) sjs s có nghiệm x(1j1 , 2j2 , , sjs ) theo modulo n Rõ ràng nghiệm này là nghiệm phương trình (1) Vậy (1) có nghiệm Hơn số nghiệm (1) số cách chọn các (j1 , j2 , , js ) từ các tập có độ dài tương ứng là ri với i = 1, 2, , s Vậy nên r = r1 r2 · · · rs (35) 34 Chuyên đề Toán học số Bài toán trên không có ý nghĩa bài toán tổng quát Thực tế, nó cho phép ta giới hạn số lượng modulo cần khảo sát việc giải các phương trình dạng (1) Thật vậy, từ kết bài toán này, ta cần xét trường hợp modulo n phương trình đồng dư là lũy thừa số nguyên tố Kết hợp với bổ đề Hensel, ta còn có thể giới hạn bài toán trường hợp modulo n bài toán là số nguyên tố Lúc này, các định lý Lý thuyết đồng dư số nguyên tố định lý nhỏ Fermat, tồn nguyên thủy, có thể áp dụng Nói chung, bài toán trên có ý nghĩa lớn việc nghiên cứu các phương trình đồng dư dạng (1) Qua các bài toán trên, có lẽ các bạn đã nắm tư tưởng các bài toán cần đến định lý Thặng dư Trung Hoa Các bài toán trình bày đòi hỏi khéo léo việc sử dụng định lý Thặng dư Trung Hoa các kiến thức Số học khác Bài toán (Taiwan TST 2002) Trong lưới điểm nguyên mặt phẳng tọa độ Oxy, điểm A với tọa độ (x0 , y0 ) ∈ Z2 gọi là nhìn thấy từ O đoạn thẳng OA không chứa điểm nguyên nào khác ngoài O và A Chứng minh với n nguyên dương lớn tùy ý, tồn hình vuông n × n có các đỉnh nguyên, tất các điểm nguyên nằm bên và trên biên hình vuông không nhìn thấy từ O Lời giải Dễ thấy rằngđiều kiện cần và đủ để A(xA , yA ) nhìn thấy từ O là gcd(xA , yA ) = Để giải bài toán, ta xây dựng hình vuông n × n với n nguyên dương tùy ý cho với điểm nguyên (x, y) nằm trên biên hình vuông không thể nhìn thấy từ O Thật vậy, chọn pij là các số nguyên tố đôi khác với ≤ i, j ≤ n Xét hai hệ đồng dư     x ≡ (mod p p · · · p ) y≡0 (mod p10 p20 · · · pn0 ) 0 n            x + ≡ (mod p11 p12 · · · p1n )  y + ≡ (mod p11 p21 · · · pn1 ) x + ≡ (mod p21 p22 · · · p2n ) và y + ≡ (mod p12 p22 · · · pn2 )              x + n ≡ (mod p p · · · p )  y + n ≡ (mod p p · · · p ) n1 n2 nn nn 1n 2n Theo định lý Thặng dư Trung Hoa thì tồn (x0 , y0 ) thỏa mãn hai hệ đồng dư trên Khi đó, rõ ràng gcd(x0 + i, y0 + i) > với i, j = 0, 1, 2, , n Điều đó có nghĩa là điểm nằm trên biên hình vuông n × n xác định điểm phía bên trái là (x0 , y0 ) không thể nhìn thấy từ O Bài toán chứng minh Nhận xét Yêu cầu bài toán trên khiến ta phải lựa chọn các số dư và các modulo nguyên tố thích hợp trên diện rộng Điều này khiến cho việc áp dụng định lý Thặng dư Trung Hoa trở nên không đơn giản ba bài toán lúc đầu Bài toán (Problem E30, PEN, Hojoo Lee) Cho n là số nguyên dương lẻ và n > Gọi k, t là các số nguyên dương nhỏ cho kn + và tn là số chính phương n Chứng minh điều kiện cần và đủ để n là số nguyên tố là min{k, t} > (36) Định lý Thặng dư Trung Hoa 35 Lời giải Trước hết ta chứng minh điều kiện cần Giả sử n là số nguyên tố thì vì n tn là số chính phương và n | tn nên n2 | tn, suy n | t, từ đó t ≥ n > Hơn nữa, đặt a2 = kn + thì a2 ≡ (mod n), suy a ≡ ±1 (mod n) Nhưng vì a > nên a ≥ n − n n Từ đó kn + ≥ (n − 1)2 ⇒ k ≥ n − ⇒ k > (vì n > nên n − > ) Vậy điều 4 kiện cần chứng minh Ngược lại, giả sử min{k, t} > n Ta xét hai trường hợp • Trường hợp n có ước nguyên tố Do n lẻ nên n = pa với p ≥ n Nếu a chẵn, ta lấy t = < , rõ ràng tn = pa là số chính phương, mâu thuẫn 4a p Nếu a lẻ ≥ 3, ta lấy t = p < , thì tn là số chính phương và ta lại có mâu thuẫn Vậy a = 1, suy n = p là số nguyên tố • Trường hợp n có ít hai ước nguyên tố phân biệt Khi đó ta có thể viết n = pa m đó p là số nguyên tố, m là số nguyên dương lẻ và (m, p) = Theo định lý Thặng dư Trung Hoa, tồn số nguyên s cho ( s≡1 (mod pa ) s ≡ −1 (mod m) n Ta lại có s 6≡ (mod m) nên s 6= và s ≡ −1 (mod pa ) nên s = −1, tức là s2 6= Do s −1 ta đặt k = thì k là số nguyên dương, kn + = s2 là số chính n phương và n2 2 s −1 s n k= < ≤ = n n n n Mâu thuẫn với điều kịn min{k, t} > Vậy trường hợp này không thể xảy Tóm lại, n là số nguyên tố Từ đó n | s2 − Hơn ta có thể chọn s cho |s| ≤ Bài toán chứng minh hoàn toàn Bài toán (Saint Petersburg City Mathematical Olympiad) Cho S = {a1 , a2 , , an } ⊂ N∗ và P (x) ∈ Z[x] Biết với số nguyên dương k tồn số i ∈ {1, 2, , n} cho | P (k) Chứng minh tồn số i0 nào đó cho ai0 | P (k) với số nguyên dương k Lời giải Phản chứng Giả sử với i tồn số nguyên dương bi cho - P (bi ) Gọi P(S) = {p1 , p2 , , pk } là tập hợp tất các ước nguyên tố các phần tử S và mi là số nguyên dương nhỏ cho tồn số nguyên dương ki thỏa mãn (37) 36 Chuyên đề Toán học số i pm - P (ki ) Chú ý các số mi luôn tồn Theo định lý Thặng dư Trung i Hoa thì tồn số nguyên dương K cho  K ≡ k1 (mod pm  )    K ≡ k (mod pm2 ) 2     s K ≡ ks (mod pm s ) i Ta có pm i - P (K) với i = 1, 2, , s Hơn nữa, giả thiết phản chứng, với i tồn bi cho - P (bi ) nên phải có ước nguyên tố pij nào đó ri ri cho pij j k pij j - P (bi ) Do cách định nghĩa các mi ta trên, mij ≤ rij , mà mi ri pij j - P (K) nên pij j - P (K), dẫn đến - P (K) với i = 1, 2, , n Điều này mâu thuẫn với giả thiết Do đó điều giả sử là sai và bài toán chứng minh Nhận xét Nếu bài toán cho thêm giả thiết a1 , a2 , , an đôi nguyên tố cùng thì nó dễ nhiều vì ta có thể sử dụng trực tiếp định lý Thặng dư Trung Hoa Lời giải trên dựa trên ý tưởng sở là đặc biệt hóa bài toán trường hợp a1 , a2 , , an đôi nguyên tố cùng cách sử dụng phân tích tiêu chuẩn số tự nhiên, kết hợp với nguyên tắc cực hạn việc chọn các mi Bài toán còn có lời giải khác, không sử dụng đến định lý Thặng dư Trung Hoa mà vận dụng nhiều các tính chất Số học đa thức có hệ số nguyên Các bạn hãy thử tìm cách giải ấy, vì với cách giải này, có bài toán tổng quát khá thú vị Với định lý Fermat nhỏ và định lý Euler, từ Lý thuyết đồng dư các số nguyên, ta có thể giải các bài toán liên quan đến lũy thừa Tính chia hết các lũy thừa có ảnh hưởng đến việc số nguyên có là lũy thừa số nguyên nào khác hay không Hai bài toán sau đưa các ví dụ tình này cùng với việc giải chúng định lý Thặng dư Trung Hoa Bài toán (IMO Shortlist) Cho S = {a1 , a2 , , an } ⊂ Z Chứng minh tồn b ∈ Z cho tập bS = {ba1 , ba2 , , ban } chứa toàn lũy thừa bậc lớn số nguyên nào đó Lời giải Trong bài toán này, ta quy ước nói “m là lũy thừa số nguyên dương” thì lũy thừa hiểu là lũy thừa thực sự, tức là lũy thừa bậc lớn Gọi P(S) = {p1 , p2 , , pk } là tập hợp tất các ước nguyên tố các phần tử S Thế thì số ∈ S viết dạng r r ri = p1i1 p2i2 · · · pk k , đó các số rij có thể Ta chứng minh tồn số nguyên b có dạng b = ps11 ps22 · · · pskk cho bai là lũy thừa số nguyên Thật vậy, điều kiện cần và đủ để bai là lũy thừa số nguyên là tồn số nguyên tố qi cho qi | sj + rij với i = 1, 2, , k Chọn q1 , q2 , , qk là các số nguyên tố Theo (38) 37 Định lý Thặng dư Trung Hoa định lý Thặng dư Trung Hoa thì tồn s1 thỏa mãn hệ đồng dư  s1 + r11 ≡ (mod q1 )     s + r ≡ (mod q ) 21     s1 + rk1 ≡ (mod qk ) Khi đó, rõ ràng qi | s1 + ri1 với i = 1, 2, , k Tương tự, tồn s2 , s3 , , sk thỏa mãn các điều kiện giống Lúc này, số b = ps11 ps22 · · · pskk thỏa mãn yêu cầu đề bài Bài toán (Mathlinks) Cho m, n là hai số nguyên dương và n ≥ Chứng minh điều kiện cần và đủ để {1n , 2n , , mn } lập thành hệ thặng dư đầy đủ modulo m là m là số square-free đồng thời gcd (n, ϕ(m)) = Lời giải Điều kiện đủ là hiển nhiên Ta chứng minh điều kiện cần Trước hết ta chứng minh m phải là số square-free Thật vậy, giả sử tồn số nguyên tố p cho p2 | m, với n ≥ 2, ta có  n m ≡ mn ≡ (mod m), p nên {1n , 2n , , mn } không lập thành hệ thặng dư đầy đủ modulo m, mâu thuẫn Vậy m là số square-free Đặt m = p1 p2 · · · pk thì ϕ(m) = (p1 − 1)(p2 − 1) · · · (pk − 1) Theo định lý Thặng dư Trung Hoa, ta cần có 1n , 2n , , (pi − 1)n lập thành hệ thặng dư đầy đủ modulo pi với i = 1, 2, , k Gọi gi là nguyên thủy modulo pi thì n(pi −1) {1n , 2n , , (pi − 1)n } ≡ {gin , gi2n , , gi } (mod pi ) n(p −1) Nhưng {gin , gi2n , , gi i } lập thành hệ thặng dư đầy đủ modulo pi và (n, ϕ(pi )) = hay (n, pi − 1) = Do đó (n, ϕ(m)) = Bài toán chứng minh hoàn toàn Bài toán 10 (Câu chuyện số Carmichael) Ta đã biết định lý nhỏ Fermat: Nếu p là số nguyên tố và a là số nguyên dương cho (a, p) = thì ap−1 ≡ (mod p) Tuy nhiên định lý đảo định lý nhỏ Fermat này không đúng Ví dụ ta có thể kiểm tra với số nguyên dương a mà (a, 561) = thì a560 ≡ (mod 561) Nhưng 561 = · 11 · 17 không phải là số nguyên tố Những số có tính chất đặc biệt gọi là số giả nguyên tố với sở, số Carmichael Ta có định lý quan trọng số Carmichael sau: Nếu n là số giả nguyên tố với sở, tức là ∀a ∈ N∗ , (a, n) = ⇒ an−1 ≡ (mod n), thì n = p1 p2 · · · pk với pi là các số nguyên tố cho pi − | n với i (39) 38 Chuyên đề Toán học số Lời giải Xét phân tích tiêu chuẩn n = pa11 pa22 · · · pakk Gọi gi là nguyên thủy modulo pai i Theo định lý Thặng dư Trung Hoa thì với số i tồn số nguyên dương a cho   a ≡ gi (mod pa11 ) n  a ≡ (mod a1 ) p1 Khi đó dễ thấy (a, n) = Theo giả thiết trên thì an−1 ≡ (mod n), suy gin−1 ≡ an−1 ≡ (mod pai i ) Theo tính chất nguyên thủy thì ϕ(pai i ) = piai −1 (pi − 1) | n − Nhưng pai i | n nên = với i Vậy n = p1 p2 · · · pk và theo trên thì pi − | n − Bài toán chứng minh Nhận xét Bằng định lý Thặng dư Trung Hoa, ta đã xác định dạng phân tích sở các số Carmichael Tuy nhiên các số này Hai số Carmichael đầu tiên là 561 và 41041 Một số bài tập áp dụng Bài tập (Czech - Slovakia 1999) Chứng minh tồn dãy số nguyên dương ∞ {an }∞ n=0 thỏa mãn điều kiện: Với số nguyên dương k, {k + an }n=0 chứa hữu hạn các số nguyên tố Bài tập (France TST 2006) Cho a, b là các số nguyên dương thỏa mãn điều kiện bn + n | an + n với số nguyên dương n Chứng minh a = b Bài tập (MEMO 2009) Tìm tất các số nguyên dương k cho với m, n là các số nguyên dương phân biệt và không lớn k thì k - nn−1 − mm−1 Bài tập d (a) Chứng minh tồn cấp số cộng có độ dài hữu hạn lớn tùy ý cho số hạng cấp số cộng này là lũy thừa số nguyên nào đó (b) Chứng minh không tồn cấp số cộng vô hạn thỏa mãn điều kiện trên Bài tập (Mathlinks) Với số nguyên n, ký hiệu τ (n) là số các ước nguyên dương n Giả sử tồn hai đa thức f, g có hệ số nguyên thỏa mãn τ (f (n)) = τ (g(n)) với số nguyên n Chứng minh f (n) ≡ ±g(n) Bài tập Với p là số nguyên tố và n là số nguyên dương, ta ký hiệu fp (n) là lũy thừa p phân tích sở n! Tìm tất các số nguyên tố p thỏa mãn điều kiện: Với cặp số nguyên dương (m, a) tồn n cho fp (n) ≡ a (mod m) (40) Định lý Thặng dư Trung Hoa 39 Lời kết Định lý Thặng dư Trung Hoa là viên gạch quan trọng để xây nên tòa nhà Lý thuyết số và lý thuyết cao cấp Toán học Nó giúp ta giải nhiều bài toán khó, làm cho nhiều bài toán vốn khó trở nên đơn giản và đặc biệt, đôi cho lời giải khá bất ngờ Vượt ngoài biên giới Toán học, định lý Thặng dư Trung Hoa còn đóng vai trò quan trọng việc xây dựng Lý thuyết mật mã, đó tiêu biểu là Lý thuyết mật mã RSA Hy vọng qua bài viết này, các bạn đã nắm số vấn đề liên quan đến định lý từ đó, tìm hiểu, mở rộng thêm để định lý Thặng dư Trung Hoa tay chúng ta không đơn là công cụ giải toán mạnh mẽ mà còn là nét đẹp Toán học Tài liệu tham khảo [1] Hojoo Lee, Peter Vandendriessche, Problems in Elementary Number Theory, 2007 [LINK: http://www.problem-solving.be/pen/published/pen-20070711.pdf] [2] Titu Andreescu, Dorin Andrica, Number Theory - Structures, Examples and Problems, Birkhauser, 2009 [3] Nguyễn Thọ Tùng, Định lý Thặng dư Trung Hoa, 2009 [LINK: http://tungtho.wordpress.com/2009/07/06/8/] [4] Từ điển mã nguồn mở Wikipedia [LINK: http://en.wikipedia.org] [5] MathLinks Forum [LINK: http://www.mathlinks.ro] (41) 40 d Chuyên đề Toán học số (42) RÈN LUYỆN KỸ NĂNG GIẢI CÁC BÀI TOÁN HÌNH HỌC PHẲNG Lê Phúc Lữ SV Đại học FPT thành phố Hồ Chí Minh Suy nghĩ việc học Toán Hình học phẳng Có nào chúng ta tự hỏi làm nào để giải bài toán Hình học phẳng (HHP) chưa? Hay làm để có thể giỏi môn HHP, làm bạn nào đó có thể giải nhanh gọn và ấn tượng bài toán HHP, còn mình thì không? Đúng là vấn đề này thường đặt muốn trả lời cách thỏa đáng và đầy đủ thì là điều không đơn giản! Cũng giống các dạng toán khác, để giải bài toán HHP nào đó, chúng ta cần phải từ giả thiết, thông qua các suy luận để tìm đường đến kết luận yêu cầu nào đó đặt đề bài Nhưng đặc biệt hơn, môn HHP, ngoài tư logic thông thường, chúng ta còn cần phải có tư hình tượng, chúng ta cần phải tìm quan hệ các yếu tố hình học thông qua cái nhìn trực quan Với đặc trưng đó, mặt làm cho chúng ta có thể thấy vấn đề cần giải cách rõ ràng mặt khác đòi hòi chúng ta khả tưởng tượng phong phú và sâu sắc muốn học tốt dạng toán này Trên thực tế, học sinh giỏi Toán, không có nhiều người giỏi HHP; tham gia các kỳ thi HSG, họ sẵn sàng bỏ câu HHP nào đó để có thời gian dành cho bài toán khác Nhưng tất các kỳ thi, ta thấy góp mặt hai bài toán HHP với khoảng 15 − 25% số điểm đề và nó thực quan trọng! Có điều lạ là chúng ta học Hình học với thời gian dài dạng toán nào khác Ngay từ lớp chúng ta đã làm quen với các khái niệm điểm, đoạn thẳng, đường thẳng, góc, Đến lớp chúng ta đã biết định lý là gì và học cách chứng minh chúng: chứng minh hai góc đối đỉnh thì nhau, chứng minh tổng ba góc tam giác là 180◦ , Và chúng ta đã học và rèn luyện chúng suốt bây giờ, thời gian dài việc học bài toán sử dụng đạo hàm, bài giới hạn hay lượng giác nào đó Thế nhưng, dường Hình học luôn không là lựa chọn hàng đầu bắt đầu cho lời giải đề thi HSG Thậm chí nó còn là nỗi ám ảnh, lo sợ nhiều bạn HSG Toán Khi nhìn thấy bài hình nào đó, họ cố đưa Đại số càng nhanh càng tốt và sẵn sàng chấp nhận biến đổi, khai thác biểu thức cồng kềnh bài toán đó có thể giải cách nhẹ nhàng hình học túy Ta không phủ nhận học và giỏi HHP không phải là chuyện dễ, có cần khiếu và rèn luyện lâu dài, phải làm nhiều dạng bài tập để tích lũy cho mình 41 (43) 42 Chuyên đề Toán học số kinh nghiệm và nhạy bén cần thiết để đối mặt với bài HHP nào đó mà không bị ngỡ ngàng, lúng túng Chẳng hạn có nhiều học sinh THCS có thể giỏi HHP học sinh THPT là lí khiếu này Thế nhưng, chẳng may không có khiếu thì sao, lại bỏ cuộc? Tất nhiên là còn cách giải quyết, chúng ta hãy tham khảo số hướng giải và gợi ý rèn luyện sau đây để khắc phục và mong điều này có thể giúp các bạn rút cho thân ý tưởng nào đó cho việc học HHP thời gian tới Cũng phải nói thêm là đa số các bạn chưa giỏi HHP thường ghét phần này và tránh làm các bài toán hình học; đó, trước hết các bạn hãy làm quen và tiếp xúc nhiều với nó, và lâu dần các bạn có thể tìm thấy thú vị mà bài toán HHP đem lại tiến nào đó cho mình Chúng ta hãy suy nghĩ các vấn đề sau • Làm để rút ngắn đường từ giả thiết đến kết luận? • Làm để tận dụng hết giả thiết đề bài cho? • Làm đưa các kiến thức hình học sẵn có (như phương pháp định lý nào đó) cho việc giải bài toán HHP? • Làm cách nào để có thể kẻ đường phụ giải bài toán? • Làm để nâng cao trình độ giải toán HHP chúng ta đã có lực định? Trước phân tích kĩ các vấn đề trên, ta thử xem xét hai ví dụ nhỏ các vấn đề thường gặp phải giải toán HHP Ví dụ Cho tam giác ABC nội tiếp đường tròn (O) có phân giác góc A cắt (O) D Chứng minh 2AD > AB + AC A E O B C D (44) Rèn luyện kỹ giải các bài toán Hình học phẳng 43 Khi đứng trước bài toán này, ta thường nghĩ đến các phương pháp sau • Sử dụng phương pháp tọa độ: Dựng hệ trục tọa độ với B(−1, 0), C(1, 0) và A(a, b) Tính tọa độ điểm D dùng bất đẳng thức • Kẻ thêm các đường phụ: Dựng DE k AB (như hình vẽ trên) chứng minh AC = DE, BE = AD đưa AD + BE > AB + DE • Sử dụng định lý quen thuộc: Dùng định lý Ptolemy có AD · BC = AB · CD + AC · BD, mà BC < BD + CD = 2BD nên 2AD > AB + AC • Sử dụng phương pháp lượng giác: Tính toán trực tiếp ta có     A A B−C 2AD = 2R sin B + + 2R sin C + = 4R cos , 2 suy AB + AC = 2R(sin B + sin C) = 4R sin B−C B+C cos < 2AD 2 Như vậy: Với đề bài ban đầu, ta đã có nhiều ý tưởng riêng mà từ đó cho ta hướng giải khác nhau; hướng đó có điểm mạnh, điểm yếu nào đó chúng đưa ta đến điều phải chứng minh Ví dụ Cho tam giác ABC có phân giác AD Trên đoạn AD lấy hai điểm E, F cho ∠ABE = ∠DBF Chứng minh ∠ACE = ∠DCF A A ??? E P E F B N F D D C B C M Ta có thể giải bài toán trên sau: Gọi M là điểm đối xứng với F qua BC, N, P là điểm đối xứng với E qua AC, AB Chứng minh 4BEM = 4BP F (c.g.c), suy 4CEM = 4CN F (c.c.c) Từ đó dễ dàng suy ∠ACE = ∠DCF Cách giải là vậy, nhiên vấn đề đặt đây là: • Tại lại biết lấy các điểm đối xứng? (45) 44 Chuyên đề Toán học số • Tại lại biết xét các tam giác nhau? • Nếu không dựng đường phụ thì giải không? Trong phần sau chúng ta cùng tìm cách giải các vấn đề trên Lời giải các ví dụ trình bày chủ yếu là dựa trên hướng suy nghĩ chính, chú trọng phân tích các bước lập luận không sâu vào xét các trường hợp hình vẽ có thể xảy nhằm hạn chế phức tạp Dù trên thực tế, giải các bài toán HHP, chúng ta nên chú ý điều này, nên xét hết các trường hợp (vị trí các điểm, các tia; phân giác trong, ngoài; tam giác cân, không cân; đường tròn thực và suy biến, ) để đảm bảo lời giải đầy đủ và chính xác! Một số phương pháp rèn luyện tư hình học và nâng cao kỹ giải toán HHP 2.1 Lựa chọn công cụ thích hợp để giải bài toán HHP Chúng ta hãy thử ngẫm nghĩ lại, là học sinh THPT nay, chúng ta đã biết bao nhiêu phương pháp giải bài toán HHP Có thể chúng ta biết nhiều định lý, bổ đề đó chưa thể gọi là phương pháp theo nghĩa tổng quát Ở đây, ta nói đến phương pháp là định hướng, là tư tưởng chính lời giải; giải cách nào chưa sâu vào việc giải nào Xin nêu số phương pháp sau • Phương pháp hình học túy (quan hệ song song, vuông góc; tam giác đồng dạng, nhau; tính chất tam giác, đường tròn; các định lý hình học quen thuộc; các phép biến hình, ) • Phương pháp lượng giác (đưa yếu tố bài lượng giác các góc và biến đổi) • Phương pháp vector (dùng vector chứng minh tính chất hình học dựng hệ vector đơn vị để giải bài toán) • Phương pháp đại số (đưa các yếu tố bài độ dài cạnh và biến đổi) • Phương pháp tọa độ (đưa giả thiết đã cho vào hệ trục tọa độ và tìm tọa độ điểm, phương trình đường thẳng, đường tròn liên quan) Trong đó, dễ thấy mức độ tư hình học thể giảm dần qua thứ tự các phương pháp trên Nếu chúng ta là học sinh chưa giỏi HHP thì thường với các bài toán có giả thiết “thuận lợi” thì sử dụng tọa độ, điều đó tất nhiên có ích cho kỹ tính toán, biến đổi đại số chúng ta nói chung không có lợi cho việc rèn luyện tư hình học Và đa số các bài toán hình khó có thể sử dụng phương pháp này, cần đường tròn tâm đường tròn nội tiếp đã khiến cho việc dùng phương pháp tọa độ thật khó khăn Thế không phải nói mà ta lại quên phương pháp đó Có vài bạn đã khá nội dung này thì lại không thích sử dụng tọa độ và cố tìm cách giải túy cho nó Công việc này không phải lúc nào đúng, là các kỳ thi HSG có thời gian “gấp rút” và số lượng bài toán cần giải lại tương đối nhiều Chúng ta hãy thử nói bài toán đơn giản sau (46) Rèn luyện kỹ giải các bài toán Hình học phẳng 45 Ví dụ Cho đoạn thẳng AB cố định và đường thẳng d cố định song song với AB Điểm C di động trên d Tìm quỹ tích trực tâm tam giác ABC C H O A B Phân tích Một số bạn thấy bài toán này có giả thiết thật đơn giản, có đoạn thẳng cố định, điểm di động trên đường thẳng song song tìm trực tâm; thêm nữa, bài toán này có vẻ quen thuộc nên họ vẽ hình và cố gắng kẻ đường phụ để giải Thế nhưng, chắn các bạn khó mà tìm lời giải hình học túy cho bài toán này mà trên thực tế quỹ tích H là đường parabol! Nếu không cẩn thận vẽ hình trước nhiều lần để dự đoán quỹ tích để chắn đây không còn là quỹ tích đường thẳng, đường cong thông thường mà mò mẫn tìm không đúng cách không đến kết muốn có Bài toán này không khó không lựa chọn đúng công cụ thì không thể nhanh chóng thành công việc giải nó Lời giải Trong mặt phẳng tọa độ Oxy, xét A(−1, 0), B(1, 0) và đường thẳng d có phương trình y = a, a 6= 0, C di động trên đó nên có tọa độ là C(m, a), m ∈ R Ta tìm tọa độ trực tâm tam giác ABC Phương trình đường cao tam giác ứng với đỉnh C là x = m Phương trình đường cao ứng với đỉnh A là (m − 1)(x + 1) + ay = 0, hay (m − 1)x + ay + m − = Từ đây suy tọa độ trực tâm tam giác ABC là nghiệm hệ ( (m − 1)x + ay + m − = x=m Giải hệ này ta tìm xH = m và yH = − x2H − m2 Suy yH = a a Vậy quỹ tích H là parabol có phương trình y = − x2 a Ví dụ Cho tam giác ABC có cạnh BC cố định, A di động mặt phẳng Gọi G, H là trọng tâm, trực tâm tam giác Biết đoạn GH cắt BC trung điểm GH, tìm quỹ tích A (47) 46 Chuyên đề Toán học số A G B C H Phân tích Ta thấy giả thiết bài toán không phức tạp điều kiện GH cắt BC trung điểm GH thật khó vận dụng; ta có thể hiểu đơn giản là trung điểm GH thuộc BC thì không đem lại nhiều gợi ý cho lời giải bài toán Và đứng trước bài toán có giả thiết đơn giản khó vận dụng thì hãy thử nghĩ đến phương pháp tọa độ Khi đó, dù các tính chất hình học chưa thể đầy đủ các điều kiện hình học thì đảm bảo chặt chẽ Cũng tiến hành lựa chọn hệ trục tọa độ thích hợp tương tự trên tính tọa độ các điểm G, H và viết phương trình các đường thẳng cần thiết, đặt vào điều kiện bài toán, ta tìm quỹ tích điểm A chính là đường hypebol Các bạn thử giải lại bài toán này với việc giữ nguyên các giả thiết ban đầu, thay trực tâm H tâm đường tròn ngoại tiếp O, các công việc nói chung tiến hành tương tự dù ta có thêm khám phá Và được, hãy giải lại hai bài toán vừa phương pháp hình học túy dựa trên định nghĩa các đường conic, tìm tiêu điểm và đường chuẩn chúng! Đây là vấn đề khá thú vị khá khó! Ta hãy so sánh hai phương pháp giải bài toán sau để rút tầm quan trọng việc lựa chọn phương pháp phù hợp giải các bài toán HHP Ví dụ Cho tam giác ABC Về phía ngoài tam giác ABC dựng các điểm D, E, F cho các tam giác BCD, CAE, ABF là các tam giác Chứng minh hai tam giác ABC và DEF có cùng trọng tâm Lời giải Sử dụng phương pháp vector (khá nhẹ nhàng và không cần tốn nhiều thời gian để nghĩ cách giải này) Gọi M, N, P là trung điểm BC, CA, AB Ta có −−→ −−→ −−→ −−→ −−→ −−→ −−→ −−→ −−→ AD + BE + CF = AM + M D + BN + N E + CP + P F −−→ −−→ −−→ −−→ −−→ −−→ = AM + BN + CP + M D + N E + P F Dễ thấy −−→ −−→ −−→ −−→ −→ −−→ −−→ −→ −−→ → − AM + BN + CP = AB + AC + BA + BC + CA + CB = 2 −−→ −−→ −−→ → −−→ −−→ −−→ → − − và M D + N E + P F = theo định lý nhím nên AD + BE + CF = Vậy hai tam giác ABC và DEF có cùng trọng tâm Lời giải Sử dụng hình học phẳng túy (dựng nhiều đường phụ, hướng suy nghĩ thiếu tự nhiên và đòi hỏi có kinh nghiệm các bài toán có giả thiết tương tự này) (48) Rèn luyện kỹ giải các bài toán Hình học phẳng 47 E Q I A F N P G M B C D Gọi I là trung điểm EF và Q là điểm đối xứng với D qua BC, đó 4BCQ là tam giác Ta thấy phép quay tâm B góc quay 60◦ biến C thành Q, biến A thành F nên 4ABC = 4F BQ Tương tự 4ABC = 4EQC Từ đây ta có 4F BQ = 4EQC Suy F Q = AC = AE, QE = AB = AF và tứ giác AEQF là hình bình hành Do đó I chính là trung điểm AQ, mà M là trung điểm QD nên IM chính là đường trung bình tam giác QAD, suy IM = AD và IM k AD Gọi G là giao điểm AM và ID thì theo định lý Thalès GM GI IM = = = GA GD AD Hơn G cùng thuộc hai trung tuyến tam giác ABC và DEF nên nó chính là trọng tâm chung hai tam giác ABC và DEF Từ đây ta có điều phải chứng minh Trong việc giải các bài toán phương pháp tọa độ, ta cần chú ý đến việc chọn các hệ trục tọa độ hợp lý: tọa độ các điểm, phương trình đường thẳng cần viết đơn giản; có nhiều liên hệ với các điểm đã cho giả thiết, tận dụng các yếu tố đường song song, vuông góc, trung điểm hình cần dựng đơn giản, Chẳng hạn chúng ta có bài toán sau Ví dụ Cho tam giác ABC có D là trung điểm cạnh BC Gọi d là đường thẳng qua D và vuông góc với đường thẳng AD Trên đường thẳng d lấy điểm M Gọi E, F là trung điểm các đoạn thẳng M B, M C Đường thẳng qua E vuông góc với d cắt đường thẳng AB P, đường thẳng qua F vuông góc với d cắt đường thẳng AC Q Chứng minh đường thẳng qua M, vuông góc với đường thẳng P Q luôn qua điểm cố định M di động trên đường thẳng d (49) 48 Chuyên đề Toán học số Phân tích Ta thấy đề bài này các giả thiết đưa xoay quanh các yếu tố trung điểm, đường vuông góc, đoạn thẳng, vì có nhiều yếu tố nên việc liên kết chúng lại và đảm bảo sử dụng tất các giả thiết là điều không dễ dàng Chúng ta có lời giải cách sử dụng phương pháp hình học túy nhờ kiến thức trục đẳng phương sau nó phức tạp vì cần phải kẻ nhiều đường phụ Lời giải P A K M E F K0 B d D H0 H d0 C Q I Gọi H, K là hình chiếu B, C lên đường thẳng d Do D là trung điểm BC nên DH = DK, suy AD là trung trực HK, đó AH = AK Gọi (ω) là đường tròn tâm A qua H và K Gọi H , K là các điểm đối xứng với H, K qua các đường thẳng AB, AC Khi đó dễ thấy H , K thuộc (ω) Giả sử các đường thẳng HH , KK cắt I thì I là điểm cố định (∗) Ta có P E k BH (cùng vuông góc với d) mà P E qua trung điểm M B nên qua trung điểm M H, suy P E là trung trực M H và vì P H = P M Gọi (ω1 ) là đường tròn tâm P qua H và M, tính đối xứng nên H thuộc (ω1 ) Hoàn toàn tương tự, ta có QF là trung trực M K; gọi (ω2 ) là đường tròn tâm Q qua K và M thì K thuộc (ω2 ) Ta lại có • (ω), (ω1 ) cắt tai H, H nên HH là trục đẳng phương (ω), (ω1 ) • (ω), (ω2 ) cắt tai K, K nên KK là trục đẳng phương (ω), (ω2 ) Mặt khác, M cùng thuộc (ω1 ), (ω2 ) và P, Q là tâm (ω1 ), (ω2 ) nên đường thẳng d0 qua M, vuông góc với P Q chính là trục đẳng phương (ω1 ), (ω2 ) Từ đó suy HH , KK , d0 đồng quy tâm đẳng phương ba đường tròn (ω), (ω1 ), (ω2 ) (∗∗) Từ (∗) và (∗∗) suy d0 qua I là điểm cố định Vậy đường thẳng qua M, vuông góc với đường thẳng P Q luôn qua điểm cố định M di động trên đường thẳng d Ta có điều phải chứng minh (50) Rèn luyện kỹ giải các bài toán Hình học phẳng 49 Nhận xét Ta có thể sử dụng phương pháp tọa độ để giải nhẹ nhàng bài toán trên vì việc xác định tọa độ trung điểm và viết phương trình đường vuông góc cho các biểu thức đơn giản, đó chính là đáp án chính thức đề thi HSG quốc gia này Thế nhưng, không phải cách chọn trục tọa độ nào cho ta lời giải nhanh gọn Nếu chọn hệ trục tọa độ gốc D và trục hoành trùng với BC theo suy nghĩ thông thường thì lời giải dài và phức tạp so với chọn gốc tọa độ là D và trục hoành là đường thẳng d Các bạn hãy thử với cách này thấy khác biệt đó! Qua các ví dụ vừa nêu, ta thấy việc lựa chọn công cụ thích hợp để giải các bài toán hình học là yếu tố quan trọng để có thể đến kết cách đơn giản và ngắn gọn hơn, nhiều đó là cách có thể giải vấn đề 2.2 Về việc tận dụng giả thiết đề bài Trong bài toán thông thường, các giả thiết đưa ra, dù ít hay nhiều, dù gián tiếp hay trực tiếp, thì lời giải nào bài toán tận dụng Một bài toán càng có ít giả thiết thì nói chung việc sử dụng chúng càng đơn giản không phải dễ dàng gì cho việc đưa hàng loạt giả thiết, yếu tố, các quan hệ hình học vào lời giải mình Mỗi giả thiết đưa có mục đích và tầm quan trọng định; nhiệm vụ chúng ta là xác định xem cái nào là quan trọng và làm để tận dụng và liên kết tất vào lời giải bài toán mình! Trước hết, ta hãy đặt câu hỏi: “giả thiết đó nói lên điều gì?”, chẳng hạn cho giả thiết: tam giác ABC có M, N, P là trung điểm các cạnh, điều đó gợi cho ta suy nghĩ • Các cạnh 4M N P song song và nửa các cạnh 4ABC tương ứng; • Tam giác M N P đồng dạng với tam giác ABC với tỉ số đồng dạng là • Diện tích tam giác M N P ; diện tích tam giác ABC; • Phép vị tự tâm G – trọng tâm tam giác ABC với tỉ số − biến tam giác ABC đã cho thành tam giác M N P ; • Hai tam giác này có cùng trọng tâm; • Đường tròn ngoại tiếp tam giác M N P chính là đường tròn Euler nên nó qua chân các đường cao và trung điểm các đoạn nối trực tâm và đỉnh tam giác ABC; • Trực tâm tam giác M N P là tâm đường tròn ngoại tiếp tam giác ABC, Có thật nhiều suy nghĩ từ giả thiết và ta bỏ sót số chúng thì có thể không giải bài toán vì đó chính là chìa khóa vấn đề (tất nhiên không phải dùng hết các ý) Chúng ta càng có nhiều liên tưởng kiến thức hình học chúng ta càng nhiều và kinh nghiệm càng sâu sắc, điều đó đòi hỏi ta cần làm số lượng định các bài toán HHP Tiếp theo ta lại hỏi: “vậy chưa có nhiều kinh nghiệm thì sao?”, tất nhiên có cách nhỏ này giúp ta có thể thấy trực quan giả thiết đó Chúng ta hãy thử tìm cách dựng các “giả thiết” đó thước và compa, là với các giả thiết có phần phức tạp, điều này nhiều lúc có ích Chúng ta thử tìm hiểu rõ điều đó qua bài toán sau (51) 50 Chuyên đề Toán học số Ví dụ Cho tam giác ABC có K là điểm nằm tam giác và thỏa ∠KAB = ∠KBC = ∠KCA Gọi D, E, F là tâm đường tròn ngoại tiếp các tam giác KBC, KCA, KAB Gọi M, N, P là giao điểm BC, F D; CA, DE; AB, EF Chứng minh các tam giác ABC, DEF, M N P đồng dạng với Phân tích Ta thấy điểm K cho trên là giả thiết quen thuộc (điểm Brocard) nói chung các tính chất ta đã biết nó không phục vụ nhiều cho điều cần chứng minh đây Nếu ta vẽ hình đơn điệu bên thì việc giải và định hướng cho bài toán không đơn giản Ta thử dùng phép dựng hình xác định điểm K giả thiết thước và compa để xem thử nó có tính chất gì đặc biệt không Ta dễ dàng có phép dựng hình sau • Dựng trung trực đoạn AB và đường thẳng vuông góc với BC B, gọi F là giao điểm hai đường thẳng trên • Dựng đường tròn tâm F bán kính F A • Tương tự, dựng điểm E là giao điểm trung trực AC và đường thẳng vuông góc với AC A • Dựng đường tròn tâm E, bán kính EA • Giao điểm hai đường tròn trên chính là điểm K cần tìm A E P F N K B C M D Từ việc tìm cách dựng cho điểm K, ta đã có thêm trên hình số đường phụ cần thiết, bài toán đã rõ ràng nhiều Với gợi ý có từ hình vẽ ta vừa dựng, có thể giải bài toán này theo cách sau (52) Rèn luyện kỹ giải các bài toán Hình học phẳng 51 Hướng dẫn giải A E P F N K B C M D • Chứng minh AK ⊥ EF, BK ⊥ DE • Chứng minh ∠AKB + ∠DF E = 180◦ • Chứng minh ∠AKB + ∠ABC = 180◦ • Suy 4ABC ∼ 4EF D (g.g) • Suy tứ giác BM P F nội tiếp và M P ⊥ EF • Chứng minh ∠M P N = ∠F ED • Chứng minh 4M P N ∼ 4F ED (g.g) Từ đó suy điều phải chứng minh Còn các bài toán mà hình vẽ không thể dựng thước và compa thì sao, chẳng hạn định lý Morley: “Cho tam giác ABC Các đường chia ba các góc tam giác cắt các điểm M, N, P Khi đó ta có M N P là tam giác đều.” Ta biết việc chia ba góc không thể giải thước và compa nên cách tìm gợi ý từ việc dựng hình không thể thực Và có lẽ vì mà đến sau 50 năm xuất hiện, bài toán tiếng này có lời giải HHP túy đẹp và hoàn chỉnh Nhưng đó là câu chuyện bài toán tiếng giới; trên thực tế, cần thiết, chúng ta luôn có thể dùng cách dựng hình này cho việc tìm gợi ý cho bài toán và tận dụng giả thiết đề bài (53) 52 2.3 Chuyên đề Toán học số Về việc rút ngắn đường từ giả thiết đến kết luận Cũng tương tự phần trên, ta đặt các câu hỏi: “kết luận đó từ đâu mà ra?”, “những điều đó có liên hệ gì đến giả thiết chúng ta có?” Chúng ta tiến hành ngược lên từ điều cần chứng minh, tìm các điều cần phải có để có kết luận Ví dụ Cho tam giác ABC có đường cao BD và CE cắt H M là trung điểm BC, N là giao điểm DE và BC Chứng minh N H vuông góc với AM A D I E H K N C M F B Phân tích Từ giả thiết ta dễ dàng thấy tứ giác DEF M nội tiếp đường tròn Euler tam giác ABC nên NE · ND = NF · NM Mặt khác D, E nằm trên đường tròn đường kính AH; còn F, M nằm trên đường tròn đường kính HM nên N nằm trên trục đẳng phương đường tròn đường kính M H và đường tròn đường kính AH Đến đây ta chưa có kết N H ⊥ AM Ta thấy thiếu vài yếu tố hình, yếu tố nào đó cần có để kết nối các điều ta vừa phân tích từ giả thiết đến kết luận bài, yếu tố đó vừa phải đảm bảo có liên quan đến N H các phương tích trên, vừa đảm bảo có liên hệ đến đoạn AM Và việc chọn hai điểm phụ dựng thêm là trung điểm AH và HM (I là trung điểm AH, K là trung điểm HM ) là điều tự nhiên vì đó IK là đường trung bình tam giác HAM, I và K là tâm các đường tròn vừa nêu trên nên trục đẳng phương N H vuông góc với đường nối hai tâm đó Ví dụ Cho tam giác ABC có O nằm tam giác Các tia AO, BO, CO cắt các cạnh đối diện M, N, P Qua O kẻ đường thẳng song song với BC cắt M N, M P H, K Chứng minh OH = OK A N P O H K B M C (54) Rèn luyện kỹ giải các bài toán Hình học phẳng 53 Phân tích Ta thấy các giả thiết bài cho “thoáng” kết luận có thật thú vị Rõ ràng, O là điểm nằm tam giác thì không thể có tính chất nào đặc biệt có thể khai thác; đó, ta vào phân tích các tỉ số có từ đường thẳng song song đã kẻ Nếu chúng ta đã quen với các bài toán tỉ số này thì ta thấy có số công cụ hỗ trợ cho chúng ta tỉ số diện tích, tỉ số đồng dạng, định lý Thalès, định lý Menelaus, định lý Céva, Trước tiên, đường thẳng song song đề bài gợi ý cho ta sử dụng định lý Thales để đưa các đoạn thẳng OH, OK các đoạn thẳng “dễ giải quyết” Ta có OH ON ON • = , suy OH = · BM BM BN BN OK OP OP • = , suy OK = · CM CM CP CP ON OP Do đó, muốn có OH = OK thì · BM = · CM, hay BN CP ON CP CM · = BN OP BM Nếu biến đổi các tỉ số này tiếp tục thì dần dần, ta bị ngộ nhận với kết luận có sẵn; thay vào đó, ta đưa các tỉ số đoạn thẳng này tỉ số diện tích các tam giác • Hai tam giác có cùng cạnh đáy thì tỉ số diện tích tỉ số chiều cao • Hai tam giác có cùng chiều cao thì tỉ số diện tích tỉ số cạnh đáy Ta có thể dễ dàng thay các tỉ số trên có liên quan trên sau ON SAON SCON SAON + SCON SAOC = = = = , BN SABN SCBN SABN + SCBN SABC OP SAOB = CP SABC Do đó ON CP SAOC SABC SAOC CM · = · = = BN OP SABC SAOB SAOB BM Đến đây kết luận đã hoàn toàn rõ ràng Ví dụ 10 Cho tam giác ABC nhọn có đường cao AD thỏa AD = BC Gọi H là trực tâm tam giác, M và N là trung điểm BC và AD Chứng minh HN = HM A N E H B C D M (55) 54 Chuyên đề Toán học số Phân tích Ta thấy giả thiết bài toán có hai điều đáng chú ý là: đường cao AD tam giác ABC BC và M, N lại chính là trung điểm hai cạnh Ta có thể suy nghĩ rằng: BC • Diện tích tam giác ABC là SABC = AD · BC = 2 • Sử dụng lượng giác, ta có BC = BD +CD = AD cot B +AD cot C = AD(cot B +cot C), từ đó suy cot B + cot C = và cot A + cot B cot C = • Tứ giác AN M C có hai cạnh đối AN và CM và đường thẳng qua hai cạnh đó vuông góc với nên có thể có số tính chất đặc biệt • Do M, N là trung điểm BC, AD nên ta vẽ các đường tròn đường kính BC, AD thì M, N chính là tâm của các đường tròn này Hơn nữa, AD = BC nên hai đường tròn này và M, N đối xứng qua dây chung Và còn nhiều điều có thể suy luận từ giả thiết đó, mục đích ta là tìm cách giải hợp lý và đơn giản Ta thấy suy luận thứ trên có thể sử dụng có xuất đối xứng các yếu tố và giả thiết sử dụng cách triệt để Do đó, ta thử theo đường đó cách dựng thêm hai đường tròn Đến đây, có vẻ giả thiết trực tâm H chưa dùng đến chưa có đường rõ ràng cho ta cách sử dụng nó Ta hãy dừng việc phân tích giả thiết lại và xem đến kết luận: “chứng minh HM = HN ” Kết luận này có thể có từ nhiều hướng, chẳng hạn từ tỉ số các cạnh, từ hai tam giác nhau, từ hai hệ thức lượng giác nhau, hay từ phép biến hình nào đó Tất nhiên, với các đòi hỏi cần thiết để đến kết luận đó, ta có thể hình thành nhiều ý tưởng cho lời giải đã chọn cách dựng đường tròn nên ta thử bám theo tính đối xứng hai đường tròn Muốn có HM = HN thì H phải nằm trên trung trực M N, mà M, N đã đối xứng qua dây chung nên H phải nằm trên dây chung đó! Đến đây, ta thấy có thể đã gần liên kết các kiện Ta tiến thêm chút nữa! Như vậy, muốn có dây chung thì phải gọi tên hai giao điểm hai đường tròn, trên thực tế hai giao điểm đó nằm quá rời rạc, khó mà chứng minh chúng và H thẳng hàng Ta không chọn cách này Thử nhìn dây chung đó phương diện khác, không phải là điểm chung hai đường tròn túy mà là trục đẳng phương hai đường tròn, muốn H nằm trên đó thì H phải có cùng phương tích đến hai đường tròn Nhưng phương tích đó có dễ dàng tính không? Với đường tròn đường kính AD thì quá đơn giản, đó chính là HA · HD; còn với đường tròn đường kính BC thì chưa có, ta có thể vẽ qua H dây cung đường tròn này gắn với đầu mút là B C, ta thử vẽ dây BE và phương tích có là HB · HE, cần chứng minh HA · HD = HB · HE là xong! Hơn nữa, ta vẽ thì BE phải vuông góc với AC H là trực tâm tam giác; mà E thuộc đường tròn đường kính BC nên BE vuông góc với EC Do đó, hóa A, E, C thẳng hàng hay E chính là chân đường cao tam giác ABC, cộng với H là trực tâm thì đẳng thức cần có là HA · HD = HB · HE không có khó khăn gì Và các mắc xích trên đã nối liền, bài toán đã giải Việc phải làm còn lại là trình bày lời giải mà thôi Rõ ràng bài toán này không quá khó và còn nhiều cách giải khác cho nó mà chúng ta có thể thấy tọa độ là cách tốt Thế nhưng, có đủ thời gian, chúng ta hãy phân tích bài toán từ từ để tìm lời giải hình học túy thật đẹp trên! (56) Rèn luyện kỹ giải các bài toán Hình học phẳng 55 Có thể nói trước bài toán HHP khó, các công việc phân tích bài toán trên là cần thiết Đó là cách chúng ta mò mẫn, dò tìm cách giải bài toán, cách có đpcm từ yếu tố cho trước thông qua việc kết nối các “mắc xích” liên hệ chúng Ta hiểu “mắc xích” đây có thể là “một bước xuống dòng”, “một dấu suy (⇒), tương đương (⇔) ”, “một phép biến đổi”, nào đó; tất nhiên là không dễ dàng gì mà ta có chúng Chúng ta phân tích càng nhiều điều từ giả thiết và kết luận càng tốt, có thì việc dùng liên tưởng, phán đoán, kinh nghiệm cho việc viết tiếp “mắc xích” quan trọng vào bài nhằm hoàn chỉnh lời giải dễ dàng Đó chính là tầm quan trọng việc rút ngắn đường từ giả thiết đến kết luận 2.4 Dựng thêm yếu tố phụ các bài toán hình học Ta thấy đa số các ví dụ trên có đưa thêm các yếu tố phụ vào, đó có thể là giao điểm, trung điểm, chân đường vuông góc, đường thẳng song song hay chí là đường tròn Yếu tố phụ chính là cầu nối giả thiết và kết luận, nó liên kết các yếu tố rời rạc có sẵn lại và giúp tận dụng triệt để phát triển giả thiết đã cho thành nhiều kết quả, cuối cùng đến kết luận Nếu không có chúng, ta có thể giải bài toán khó khăn không thể giải Có thể nói học sinh đã biết cách kẻ đường phụ việc giải các bài HHP thì đó không thể nào là học sinh kém phần này Muốn kẻ đường phụ, đòi hỏi chúng ta phải có quan sát, đánh giá vấn đề tốt; có kinh nghiệm sâu sắc và khả phân tích, sáng tạo mức độ định Việc gọi tên cho điểm chưa có tên hình vẽ trên thực tế là chuyện không đơn giản dù điểm đó đã có sẵn nói chi đến việc dựng thêm nhiều yếu tố phụ, cái không có trước đó Điều này không khó hiểu vì làm các bài toán Đại số – Giải tích, chúng ta thường quen với các lập luận logic có sẵn, thứ xuất phải có lí rõ ràng Còn HHP thì không phải vậy, cứng nhắc cho đường phụ nào đó muốn kẻ cần phải có lập luận logic nào đó cho nó thì khó mà thực công việc này trên thực tế, nhiều ta kẻ đường phụ mà không có lí xác đáng! Do đó phần này ta suy nghĩ thêm việc kẻ đường phụ và vai trò quan trọng kinh nghiệm qua quá trình rèn luyện lâu dài để giải toán HHP cách kẻ thêm đường phụ Ta xét bài toán sau Ví dụ 11 Cho tứ giác ABCD nội tiếp (O, R) có M, N là giao điểm các cặp cạnh đối Chứng minh −−→ −−→ OM · ON = R2 Phân tích Khi giải bài toán này, chắn các bạn mò mẫn biến đổi tích vô hướng hai vector vế trái để đến kết cuối cùng bị ngộ nhận càng lúc càng phức tạp thêm Do đó, việc dựng thêm yếu tố phụ là điều tất yếu Chúng ta đừng lầm tưởng hình thức đơn giản bài toán này! Việc dựng đường phụ đây có thể là khó với số bạn chúng ta đã quen với bài toán sau thì chuyện trở nên đơn giản nhiều: “Cho tứ giác ABCD nội tiếp (O) có M, N là giao điểm AD, BC và AB, CD Chứng minh M A · M B + N A · N D = M N ” (57) 56 Chuyên đề Toán học số N A P B O D C M Ta giải bài toán này xem bổ đề và áp dụng nó vào bài toán đã cho: Gọi P là giao điểm đường tròn ngoại tiếp tam giác BCM với M N Ta thấy BP M C rõ ràng là tứ giác nội tiếp nên ∠BP M = ∠BCD = ∠N AB, suy tứ giác AN BP nội tiếp Theo tính chất phương tích, ta có M A · M B = M P · M N, N A · N D = N P · N M Từ đó suy M A · M B + N A · N D = M N (M P + N P ) = M N Ta quay trở lại bài toán đã cho, biến đổi biểu thức cần chứng minh chút cho vấn đề rõ ràng (giả sử M là giao điểm AB và CD, N là giao điểm AD và BC) −−→ −−→ OM · ON = R2 ⇔ OM + ON − M N = 2R2 Áp dụng bổ đề trên, thay M A · M B + N A · N D = M N vào biểu thức trên OM + ON − (M A · M B + N A · N D) = 2R2 Nhưng điều này là đúng theo tính chất phương tích M A · M B = OM − R2 , N A · N D = ON − R2 Từ đó, ta đã giải thành công bài toán Thử nghĩ không có hỗ trợ bổ đề trên thì việc kẻ đường tròn ngoại tiếp tam giác BCM chứng minh trên là chuyện không đơn giản Và phải công nhận kinh nghiệm giải toán HHP thể bài này không ít! Ta tiếp tục phân tích ví dụ khác Ví dụ 12 Trong mặt phẳng cho hai điểm A, B cố định (A khác B) Một điểm C di động mặt phẳng cho góc ∠ACB = α không đổi (0◦ < α < 180◦ ) Đường tròn tâm I nội tiếp tam giác ABC và tiếp xúc với các cạnh AB, BC, CA D, E, F Đường thẳng AI, BI cắt đường thẳng EF M, N Chứng minh (a) Đoạn M N có độ dài không đổi (b) Đường tròn ngoại tiếp tam giác DM N luôn qua điểm cố định (58) Rèn luyện kỹ giải các bài toán Hình học phẳng 57 C E M N F I A D P B Phân tích Nếu bài này ta đọc kĩ giả thiết thì thấy các điểm M và N xác định trên đã xuất nhiều bài toán quen thuộc trước đó mà yêu cầu đề dừng lại việc chứng minh các tam giác M BC, N BC vuông Nếu đã biết điều này, ta chứng minh lại kết đó và sử dụng vào việc giải bài toán đã cho bổ đề (trong bài này không xét 180◦ − ∠C các vị trí có thể có M, N ) Ta thấy ∠M EB = ∠CEF = và 180◦ − ∠C ∠ABC + ∠ACB = ∠M IB = ∠IAB + ∠IBA = 2 Do đó ∠M EB = ∠M IB Từ đây suy tứ giác EM BI nội tiếp và ∠IM B = ∠IEB = 90◦ , suy tam giác AM B vuông M Tương tự, ta có tam giác N AB vuông N Áp dụng điều này vào bài toán, ta tứ giác AN M B nội tiếp đường tròn đường kính AB AB IA Suy ∆AIB ∼ ∆N IM, từ đó ta = , suy MN NI   IN ∠CAB + ∠CBA ◦ M N = AB · = AB sin ∠N AI = AB sin 90 − IA ∠C α = AB sin = AB sin = const 2 Hơn nữa, ta thấy   ∠CAB + ∠CBA ∠M DN = ∠IDN + ∠IDM = 90◦ − = ∠C Gọi P là trung điểm AB thì P chính là tâm đường tròn ngoại tiếp tứ giác AN M B, suy ∠M P N = ∠M P A − ∠N P A = 2(∠M BA − ∠N BA) = 2(90◦ − ∠M AB − ∠N BA)   ∠CAB + ∠CBA = 90◦ − = ∠C Do đó ∠M P N = ∠M DN, suy tứ giác M N DP nội tiếp hay đường tròn ngoại tiếp tam giác DM N luôn qua P cố định Đây chính là điều phải chứng minh Bài toán này còn nhiều cách giải khác có lẽ cách này đơn giản, ngắn gọn Một số bài toán có thể giải nhiều cách dựng đường phụ và chúng ta càng có nhiều công cụ hỗ trợ bổ đề, định lý quen thuộc thì việc dựng hình đơn giản và lời giải nhẹ nhàng hơn, chúng ta hãy xét việc chứng minh định lý Pascal sau đây (59) 58 Chuyên đề Toán học số Ví dụ 13 Cho lục giác ABCDEF nội tiếp đường tròn (O) có M, N, P là giao điểm AB, DE; BC, EF ; CD, F A Chứng minh M, N, P thẳng hàng Phân tích Việc chứng minh định lý này đã quá quen thuộc cách sử dụng định lý Menelaus thuận và đảo cho các tam giác Cách đó tương đối ngắn gọn và không kẻ nhiều đường phụ Nhưng ta không biết trước định lý Menelaus và sử dụng cách chứng minh khác thì mời các bạn hãy theo dõi lời giải sau đây với việc kẻ thêm hai đường tròn phụ A O F B C E D M P N H Gọi I là giao điểm đường tròn ngoại tiếp tam giác BDM và F DQ Ta chứng minh bốn điểm M, N, P, I thẳng hàng cách chứng minh ba điểm thẳng hàng (Việc nghĩ hai đường tròn phụ này có thể xuất phát từ bài toán quen thuộc là: “Cho ba đường tròn (1), (2), (3) cùng qua D (1) cắt (2) A, (2) cắt (3) B, (3) cắt (1) C (A, B, C khác D) Với M nằm trên (1), gọi P, Q là giao điểm M A với (2), M C với (3) Chứng minh P Q qua B.”) Thật vậy, từ các tứ giác nội tiếp BDIM, F DIP, ta có ∠DIM + ∠DIP = ∠DBA + ∠DF A = 180◦ , suy M, I, P thẳng hàng Tiếp theo ta chứng minh M, N, I thẳng hàng Ta có tứ giác 1 BDIM nội tiếp nên ∠BIM = ∠BDM = 180◦ − ∠BDE = 180◦ − sđ(BAE) = sđ(BDE) 2 (60) Rèn luyện kỹ giải các bài toán Hình học phẳng 59 Mặt khác, lại có ∠BIF = ∠BID + ∠F ID = ∠BM D + ∠F P D = [(sđ(AF E) − sđ(BCD)) + (sđ(ABC) − sđ(DEF ))] = [(sđ(AF ) + sđ(AB)) − (sđ(DC) + sđ(DE))] = (sđ(BAF ) − sđ(CDE)) = ∠BN F, nên tứ giác BN IF nội tiếp Do đó ∠BIN = ∠BF N = sđ(BDE) = ∠BIM, suy M, N, I thẳng hàng Tương tự N, I, P thẳng hàng Vậy ta có M, N, P thẳng hàng (đpcm) Qua các ví dụ trên, ta thấy việc dựng đường phụ là công việc đòi hỏi phải có quá trình rèn luyện và tích lũy kinh nghiệm lâu dài Có thể nói chúng ta đã kẻ thành công đường phụ để giải bài toán nào đó chính là lúc chúng ta có bước tiến dài việc học tập HHP 2.5 Về việc học tập và rèn luyện HHP mức độ nâng cao Có nào chúng ta đặt câu hỏi: “Tại người ta lại có thể nghĩ bài toán hay nhỉ?” Thông thường, chúng ta giải bài toán với lời giải thật hay và đẹp gác nó lại mà không dành thời gian tìm hiểu thêm điều lý thú đằng sau nó hay chí là đưa bài toán từ bài toán cũ đó Việc tìm tòi giúp chúng ta chủ động các bài toán và phát triển kỹ HHP tốt Khi chúng ta tìm tòi sáng tác các bài toán chính là lúc chúng ta trên đường mà người đề đã và tìm hiểu xem họ đã làm nào để có bài toán Thông thường các bài toán HHP đặt dạng che giấu các vấn đề và công việc chúng ta là lần mò theo các giả thiết có sẵn để giải Việc che giấu càng hay mà số điểm và đường hình bị xóa mà yêu cầu bài toán lại không bị ảnh hưởng, người giải các bài phải khôi phục lại các điểm đó thông qua cách kẻ các yếu tố phụ; có thể là việc biến đổi các yếu tố bài, thêm các đường để che giấu chất vấn đề Và việc tự nghĩ các bài toán HHP phát triển từ bài toán cũ là việc làm có ích cho chúng ta mà ta trở thành thí sinh kỳ thi nào đó, đối mặt với bài toán HHP khó, không rơi vào hoàn cảnh bị động và lúng túng Ta thử xem các bài toán sau đây Ví dụ 14 Cho tam giác ABC nhọn có A là góc lớn nhất, nội tiếp đường tròn (O) và ngoại tiếp đường tròn (I), H là trực tâm Trung tuyến đỉnh I tam giác IOH cắt (I) P Gọi M, N là trung điểm AB, AC Chứng minh ∠BAC = ∠M P N (61) 60 Chuyên đề Toán học số A N M I O H E B C Phân tích Ta thấy giả thiết bài toán không quá phức tạp các yếu tố rời rạc hình việc dựng hình phức tạp có thể khiến ta khó tìm lời giải Thực ra, bài toán này phát triển từ định lý “đường tròn Euler tiếp xúc với đường tròn nội tiếp” Các vấn đề bị che lấp là • Trung điểm đoạn OH chính là tâm đường tròn Euler • Giao điểm P chính là tiếp điểm đường tròn Euler với đường tròn nội tiếp nên tất nhiên nó thuộc đường tròn Euler • Đường tròn Euler qua trung điểm các cạnh nên gọi Q là trung điểm BC thì M N P Q nội tiếp và ∠M P N = ∠M QN • Do M, N, Q là trung điểm các cạnh nên ∠BAC = ∠M QN Từ đó ta dễ dàng đến lời giải cho bài toán Nếu chúng ta chưa quen lần mò theo đường người cho đề để tìm lời giải thì bài toán trên thật không đơn giản chút nào, là chúng ta có khiếu HHP hay không Chẳng hạn bạn là người cho đề, bạn có sẵn bài toán chứng minh các điểm M, N, P nào đó cùng nằm trên đường thẳng d, bạn muốn bài toán này khó và bạn dễ dàng nghĩ A là điểm nào đó nằm ngoài đường thẳng d thì trực tâm ba tam giác AM N, AN P, AP M thẳng hàng (cùng nằm trên đường thẳng qua A vuông góc với d) và thế, bạn đã tích lũy thêm kinh nghiệm cho việc chứng minh ba điểm thẳng hàng Thử hỏi là người tìm lời giải bài toán thì việc nhìn cách chứng minh đó có dễ dàng không? Ví dụ 15 Cho tam giác ABC nội tiếp (O) có A là góc lớn Trung trực AB, AC cắt cạnh BC D, E Đường thẳng AD, AE cắt (O) M, N Gọi K là giao điểm BM và CN ; d là đường thẳng đối xứng với phân giác góc DAE qua phân giác góc BAC Đường thẳng OK cắt d I Chứng minh ∠BIC = ∠DOE (62) 61 Rèn luyện kỹ giải các bài toán Hình học phẳng A I B E D O K C M N Phân tích Chắc hẳn chúng ta đã nghe đến bài toán sau: “Cho tứ giác ABCD thỏa mãn ∠DAB = ∠ABC = ∠BCD Chứng minh đường thẳng Euler 4ABC qua D” Bài toán này là bài toán khó nó đã khá quen thuộc với nhiều cách giải Tưởng chừng bài toán này và ví dụ trên không có liên hệ gì thực ví dụ trên là phát triển bài toán vừa nêu với việc che lấp và bổ sung thêm hàng loạt vấn đề Nếu chưa biết đến nó thì ví dụ này là bài toán khó Chúng ta thử chứng minh xem tứ giác ABKC hình vẽ có tính chất ba góc không, rõ ràng điều đó là đúng Khi đó đưa kết trên vào thì đường thẳng Euler tam giác ABC qua K hay ngược lại OK qua trực tâm tam giác ABC Đường thẳng d đề bài thực chất là đường cao tam giác ABC và I chính là trực tâm Đây là các yếu tố đã bị che lấp đi, chúng ta không tiến hành bước để khai thác giả thiết thì khó có thể thấy điều này Đến đây thì rõ ràng BI k OE, CI k OD nên ∠BIC = ∠DOE là đúng Vấn đề đã giải quyết! Ta phân tích thêm ví dụ để thấy rõ vai trò kinh nghiệm tích lũy thân việc giải các bài toán HHP Ví dụ 16 Cho hai đường tròn (O), (O0 ) cắt A và B Gọi CC là tiếp tuyến chung (gần A hơn) hai đường tròn, C ∈ (O), C ∈ (O0 ) Gọi D, D0 là hình chiếu C, C trên đường thẳng OO0 Giả sử AD cắt (O) E, AD0 cắt (O0 ) E Chứng minh E, B, E thẳng hàng C K A C0 I O O0 D0 D E0 F B E F0 (63) 62 Chuyên đề Toán học số Phân tích Đây là bài toán hình học kỳ thi HSG quốc gia và các lời giải nó nói chung mang nhiều tính chất phép vị tự Đáp án chính thức là lời ngắn gọn và đẹp Dù vậy, các bạn đã nhiều lần giải các bài toán hai đường tròn cắt cùng với tiếp tuyến nó thì nhiều kinh nghiệm dạng này và có thể dùng kinh nghiệm đó bổ đề để giải bài toán này cách ấn tượng Hãy suy nghĩ cách giải sau Gọi F, F là giao điểm khác A các đường thẳng AO với (O), AO0 với (O0 ) Do AF, AF là đường kính các đường tròn (O), (O0 ) tương ứng nên ∠ABF = ∠ABF = 90◦ , suy F, B, F thẳng hàng Gọi R, R0 là bán kính hai đường tròn (O), (O0 ) Ta chứng minh ∠O0 AD0 = ∠OAD (∗) Thật vậy, gọi I là giao điểm CC với OO0 và K là giao điểm IA với (O) Dễ dàng thấy I chính là tâm vị tự hai đường tròn Do đó IO0 R0 O0 A IA IC ID0 = = = = = IO R OK IK IC ID Suy AD0 k KD hay ∠IAD0 = ∠IKD Mặt khác, AC k KC nên ∠IAC = ∠IKC = ∠ICA, IC IA = Từ đây ta có IA2 = IC · IC IC IA Do tứ giác CC D0 O nội tiếp nên IC · IC = ID0 · IO Kết hợp với trên ta có IA2 = ID0 · IO, IA IO hay = , từ đó suy 4IAD0 ∼ 4IOA Do đó ∠IAD0 = ∠IOA, mà ∠IAD0 = ∠IKD ID IA nên ∠IKD = ∠IOA, suy tứ giác ADOK nội tiếp Từ đây ta có suy 4IAC ∼ 4ICA và ∠OAD = ∠OKD Hơn OK k O0 A, DK k D0 A nên ∠O0 AD0 = ∠OKD Suy ∠O0 AD0 = ∠OAD, (∗) chứng minh Áp dụng vào bài toán, theo tính chất góc nội tiếp ta có ∠OAD = ∠F BE, ∠O0 AD0 = ∠F BE Do đó, kết hợp với (∗), ta ∠F BE = ∠F BE Mà F, B, F thẳng hàng nên theo tính chất góc đối đỉnh, ta có E, B, E thẳng hàng Ta có điều phải chứng minh Bên cạnh đó, ta cần phải nhắc đến số công cụ gọi là “cao cấp” để giải các bài toán HHP như: góc định hướng, độ dài đại số, tích có hướng và diện tích đại số, phương tích và trục đẳng phương, hàng điểm điều hòa, cực và đối cực, phép nghịch đảo và đồng dạng, định lý Carno, Michael, Cũng tương tự điều gọi là kinh nghiệm hay bổ đề trên, công cụ này có thể giúp ta giải nhanh gọn và dễ dàng nhiều bài toán khó mà sử dụng công cụ thông thường thì lời giải dài dòng và phức tạp; có nhiều ta không đủ (64) Rèn luyện kỹ giải các bài toán Hình học phẳng 63 khả nhìn lời giải kiểu Thế nhưng, muốn áp dụng định lý nào đó vào việc giải bài toán là điều không đơn giản mà số lượng định lý có sẵn tương đối lớn và càng khó khăn đặc trưng định lý đó chưa thể mặt nào bài toán, chúng ta phải lần mò theo giả thiết để đặt các yêu cầu cần có nhằm đến kết luận và có thể định lý nào đó xuất hỗ trợ cho ta Ví dụ 17 Cho tam giác ABC Dựng phía ngoài tam giác ABC các tam giác và gọi M, N, P là tâm chúng Chứng minh tam giác M N P Ta thấy đây là nội dung định lý Napoléon với cách chứng minh quen thuộc là dựng thêm các đường tròn ngoại tiếp các tam giác đã có gọi tên giao điểm chúng Sau đây, ta cùng xem hai cách chứng minh khác và đưa nhận xét so sánh Lời giải Sử dụng phương pháp thông thường Q A N P C B M Dựng điểm Q khác phía M so với N P cho ∠QP A = ∠M P B và P Q = M Q Ta có 4AP Q = 4BP M (c.g.c), suy ∠P AQ = ∠M BP = ∠ABC + 60◦ , AQ = BM = CM Do đó ∠N AQ = 360◦ − (∠P AQ + ∠P AN ) = 360◦ − (∠ABC + 60◦ + ∠CAB + 60◦ ) = ∠ACB + 60◦ = ∠M CN Suy 4AQN = 4CM N (c.g.c) và N Q = N M Mà P Q = P M 4AP Q = 4BP M nên P N là trung trực M Q, tức là M, Q đối xứng qua P N hay ∠QP N = ∠M P N, ∠QN P = ∠M N P (65) 64 Chuyên đề Toán học số Mặt khác, ta có ∠QP M = ∠AP M + ∠QP A = ∠AP M + ∠M P B = ∠AP B = 120◦ và tương 120◦ tự ∠QN M = 120◦ nên ∠M P N = ∠M N P = = 60◦ , suy tam giác M N P Lời giải Sử dụng phép quay vector B0 A N C0 P C B M A0 Gọi A0 , B , C là đỉnh các tam giác tương ứng dựng trên các đoạn CA, AB  Vì −−→BC,−− → − − → − − → 0 các điểm M, N là trọng tâm 4BA0 C, 4ACB nên M N = BA + A C + CB Tương tự, ta có −−→ −−→0 −−0→ −→ MP = BC + A B + CA π Xét phép quay vector góc quay là , ta có −−→ −−→ −−→ −−→ h −−→ −−→ −−→i Q π3 M N = Q π3 BA + A0 C + CB = Q π3 BA + Q π3 A0 C + Q π3 CB 3 −−→0 −−0→ −→ −−→ = BC + A B + CA = M P Suy tam giác M N P Ta thấy phép quay vector sử dụng bài toán đã giúp hạn chế nhiều lập luận hình học phức tạp và cho ta lời giải nhẹ nhàng, vấn đề là chúng ta phải biết vào các đặc trưng bài toán để vận dụng cho phù hợp và chính xác Nói chung ba công cụ (66) Rèn luyện kỹ giải các bài toán Hình học phẳng 65 sau: vector (tương ứng với đoạn thẳng), góc định hướng (tương ứng với góc), diện tích đại số và tích ngoài (tương ứng với diện tích) là ba công cụ mạnh, phát triển từ các yếu tố hình học Chúng ta nên tìm hiểu thêm chúng để có thêm trợ giúp tốt đứng trước bài toán HHP nào đó mà các phương pháp hình học túy khác dường đã không còn tác dụng Tiếp tục nói việc nghiên cứu các bài toán HHP mới, chúng ta thấy điều rằng: muốn tự nghĩ bài HHP hoàn toàn độc lập với các bài đã có là chuyện không đơn giản; ta có thể sử dụng tương tự các yếu tố đường và điểm tạo các bài toán độc đáo Chẳng hạn bài toán trên, các bạn có thể tự hỏi không dựng các tam giác phía ngoài tam giác mà dựng phía ngược lại thì kết trên sao, định lý có còn đúng hay không Vẫn còn nhiều ví dụ phát này như: • Tam giác có hai đường trung tuyến, đường cao là tam giác cân; phân giác thì sao? • Giao điểm các đường chia ba phía các góc tam giác là tam giác là tam giác (định lý Morley); thì chia ba phía ngoài thì sao? • Ta có bài toán quen thuộc là: “Cho tam giác ABC nội tiếp (O), có trọng tâm G Giả sử các tia GA, GB, GC cắt (O) A0 , B , C Chứng minh GA + GB + GC ≤ GA0 + GB + GC ”; thì thay G trực tâm hay tâm đường tròn nội tiếp thì sao? • Quỹ tích trực tâm H tam giác ABC có BC cố định và ∠BAC không đổi là đường tròn, trọng tâm G là đường tròn; tâm đường tròn nội tiếp là gì? • Nếu biết trung điểm các cạnh có thể dựng các đỉnh tam giác, biết chân đường cao có thể dựng các đỉnh tam giác; biết chân các đường phân giác thì có dựng không và dựng nào? Chẳng hạn từ vấn đề cuối vừa nêu trên, chúng ta có bài toán sau: “Cho tam giác ABC nội tiếp đường tròn tâm O có các phân giác AD, BE, CF đồng quy I Gọi x, y, z là các tiếp tuyến (O) song song với các đoạn thẳng EF, F D, DE Giả sử x cắt y P, y cắt z M, z cắt x N Gọi H, K, L là chân đường phân giác kẻ từ góc M, N, P tam giác M N P Chứng minh (a) Các đoạn thẳng M D, N E, P F đồng quy Gọi điểm đó là R (b) Các đoạn thẳng HD, KE, LF đồng quy Gọi điểm đó là S (c) Ba điểm R, S, O thẳng hàng Gọi đường thẳng qua các điểm này là d (d) Đường thẳng d qua tâm đường tròn nội tiếp bốn tam giác ABC, DEF, M N P và HKL.” Còn nhiều điều gần gũi, quen thuộc mà chúng ta chưa tìm hiểu nhiều chúng để có thể phát thêm thú vị rèn luyện cho mình kỹ giải toán HHP Tại chúng ta không thử bắt đầu với bài HHP đơn giản nào đó để tìm đến điều thú vị? (67) 66 Chuyên đề Toán học số Tóm lại, muốn học tốt môn HHP, chúng ta cần phải có quá trình rèn luyện đầy đủ cùng với cách học tập phù hợp Chúng ta nên rèn luyện tư hình học mình từ nhiều dạng toán và nên tập trung vào các công cụ chính; đừng quá sâu vào phương pháp, công cụ hỗ trợ đặc biệt nào đó Ta học thật nhiều định lý, bổ đề cần chúng ta không thể nào nhớ hết chúng và không biết lựa chọn công cụ nào cho phù hợp để giải Chúng ta nên biết các bài toán HHP các kỳ thi thường không giải dựa trên bổ đề, định lý khó nào đó để đánh giá kỹ nhớ, thuộc bài mà dùng các công cụ thông thường, quen thuộc để có thể đánh giá khả tư duy, lập luận học sinh Hãy trang bị cho mình thứ cần thiết để có thể đối đầu với các bài toán HHP khó khăn phía trước; tất nhiên, hành trang tốt là hành trang đầy đủ và gọn gàng, có thể sử dụng nhiều tình không phải hành trang quá cồng kềnh, quá phức tạp để cần dùng thứ nào đó không biết tìm kiếm đâu, Chúc các bạn có thể rèn luyện tốt và thành công môn HHP thật thú vị và hấp dẫn này! Các bài toán rèn luyện Sau đây xin mời các bạn hãy tham khảo thử 16 bài toán kỳ thi chọn đội tuyển quốc gia Việt Nam dự thi IMO đây và hãy thử vận dụng các hướng vừa để tìm cách giải chúng Đây là các bài toán hay và khó! (Các bài toán xếp cách tương đối từ dễ đến khó) Bài tập (Đề TST 2000) Hai đường tròn (C1 ) và (C2 ) cắt hai điểm P và Q Tiếp tuyến chung hai đường tròn gần P Q tiếp xúc với (C1 ) A và tiếp xúc với (C2 ) B Các tiếp tuyến (C1 ), (C2 ) kẻ từ P cắt đường tròn E và F (E, F khác P ) Gọi H, K là các điểm nằm trên các đường thẳng AF, BE cho AH = AP và BK = BP Chứng minh năm điểm A, H, Q, K, B cùng thuộc đường tròn Bài tập (Đề TST 2003) Trên các cạnh tam giác ABC lấy các điểm M1 , N1 , P1 cho các đoạn M M1 , N N1 , P P1 chia đôi chu vi tam giác, đó M, N, P là trung điểm các đoạn BC, CA, AB Chứng minh (a) Các đường thẳng M M1 , N N1 , P P1 đồng quy điểm Gọi điểm đó là K (b) Trong các tỉ số KA KB KC , , có ít tỉ số không nhỏ √ BC CA AB Bài tập (Đề TST 2006) Cho tam giác ABC có H là trực tâm Đường phân giác ngoài góc BHC cắt các cạnh AB, AC D và E Đường phân giác góc BAC cắt đường tròn ngoại tiếp tam giác ADE điểm K Chứng minh đường thẳng HK qua trung điểm đoạn BC Bài tập (Đề TST 2006) Trong mặt phẳng cho góc xOy Gọi M, N là hai điểm nằm trên các tia Ox, Oy Gọi d là đường phân giác góc ngoài góc xOy và I là giao điểm trung trực M N với đường thẳng d Gọi P, Q là hai điểm phân biệt nằm trên đường thẳng d cho IM = IN = IP = IQ, giả sử K là giao điểm M Q và N P (a) Chứng minh K nằm trên đường thẳng cố định (b) Gọi d1 là đường thẳng vuông góc với IM M và d2 là đường thẳng vuông góc với IN N Giả sử các đường thẳng d1 , d2 cắt đường thẳng d E, F Chứng minh các đường thẳng EN, F M và OK đồng quy (68) Rèn luyện kỹ giải các bài toán Hình học phẳng 67 Bài tập (Đề TST 2009) Cho tam giác nhọn ABC nội tiếp đường tròn (O) Gọi A1 , B1 , C1 và A2 , B2 , C2 là các chân đường cao tam giác ABC hạ từ các đỉnh A, B, C và các điểm đối xứng với A1 , B1 , C1 qua trung điểm các cạnh BC, CA, AB Gọi A3 , B3 , C3 là các giao điểm đường tròn ngoại tiếp các tam giác AB2 C2 , BC2 A2 , CA2 B2 với đường tròn (O) Chứng minh A1 A3 , B1 B3 , C1 C3 đồng quy Bài tập (Đề TST 2001) Trong mặt phẳng cho hai đường tròn cắt hai điểm A, B Gọi P T là hai tiếp tuyến chung hai đường tròn đó P, T là các tiếp điểm Tiếp tuyến P và T đường tròn ngoại tiếp tam giác AP T cắt S Gọi H là điểm đối xứng với B qua đường thẳng P T Chứng minh các điểm A, S, H thẳng hàng Bài tập (Đề TST 1999) Cho tam giác ABC nội tiếp đường tròn Γ Một đường tròn θ tiếp xúc với các cạnh AB, AC và tiếp xúc với đường tròn Γ các điểm M1 , N1 , P1 Các điểm M2 , N2 , P2 và M3 , N3 , P3 xác định cách tương tự Chứng minh các đoạn thẳng M1 N1 , M2 N2 , M3 N3 cắt trung điểm đường Bài tập (Đề TST 1995) Cho tam giác ABC và điểm M nằm tam giác Gọi A0 , B , C là ảnh các điểm A, B, C qua phép đối xứng tâm M (a) Chứng minh tồn điểm điểm P mặt phẳng cách hai đầu mút các đoạn thẳng AB , BC , CA0 (b) Gọi D là trung điểm đoạn AB Chứng minh M thay đổi tam giác ABC và không trùng với D thì đường tròn ngoại tiếp tam giác M N P, đó N là giao điểm DM và AP, luôn qua điểm cố định Bài tập (Đề TST 2004) Trong mặt phẳng cho hai đường tròn (O1 ), (O2 ) cắt A và B Các tiếp tuyến A, B đường tròn (O1 ) cắt K Xét điểm M không trùng với A, B nằm trên đường tròn (O1 ) Gọi P là giao điểm thứ hai đường thẳng M A với đường tròn (O2 ) Gọi C là giao điểm thứ hai đường thẳng M K với đường tròn (O1 ) Gọi Q là giao điểm thứ hai đường thẳng CA với đường tròn (O2 ) Chứng minh (a) Trung điểm đoạn thẳng P Q nằm trên đường thẳng M C (b) Đường thẳng P Q luôn qua điểm cố định M di động trên (O1 ) Bài tập 10 (Đề TST 2003) Cho tam giác ABC có O là tâm đường tròn ngoại tiếp Gọi H, K, L là chân các đường vuông góc kẻ từ các đỉnh A, B, C tam giác ABC Gọi A0 , B0 , C0 là trung điểm các đường cao AH, BK, CL Đường tròn nội tiếp tâm I tam giác ABC tiếp xúc với các đoạn BC, CA, AB D, E, F Chứng minh A0 D, B0 E, C0 F cùng qua điểm và điểm đó nằm trên đường thẳng OI Bài tập 11 (Đề TST 2006) Cho tam giác ABC là tam giác nhọn, không cân, nội tiếp đường tròn tâm O bán kính R Một đường thẳng d thay đổi cho d luôn vuông góc với OA và luôn cắt các tia AB, AC Gọi M, N là giao điểm đường thẳng d và các đoạn AB, AC Giả sử các đường thẳng BN và CN cắt K; giả sử đường thẳng AK cắt đường thẳng BC (a) Gọi P là giao đường thẳng AK và đường thẳng BC Chứng minh đường tròn ngoại tiếp tam giác M N P luôn qua điểm cố định d thay đổi (69) 68 Chuyên đề Toán học số (b) Gọi H là trực tâm tam giác AM N Đặt BC = a và l là khoảng cách từ điểm A đến đường thẳng HK Chứng minh √ đường thẳng HK luôn qua trực tâm tam giác ABC Từ đó suy l ≤ 4R2 − a2 Đẳng thức xảy và nào? Bài tập 12 (Đề TST 2009) Cho đường tròn (O) đường kính AB và M là điểm nằm (O), M không nằm trên đoạn thẳng AB Gọi N là giao điểm phân giác góc M tam giác AM B với đường tròn (O) Đường phân giác ngoài góc ∠AM B cắt các đường thẳng N A, N B P, Q Đường thẳng M A cắt đường tròn đường kính N Q R, đường thẳng M B cắt đường tròn đường kính N P S và R, S khác M Chứng minh đường trung tuyến ứng với đỉnh N tam giác N RS luôn qua điểm cố định M di động phía đường tròn Bài tập 13 (Đề TST 2005) Cho tam giác ABC có (I) và (O) là các đường tròn nội tiếp, ngoại tiếp tam giác Gọi D, E, F là tiếp điểm đường tròn (I) trên các cạnh BC, CA, AB Gọi ωA , ωB , ωC là các đường tròn tiếp xúc với hai đường tròn (I) và (O) các điểm D, K (với đường tròn ωA ); E, M (với đường tròn ωB ) và F, N (với đường tròn ωC ) Chứng minh các đường thẳng DK, EM, F N đồng quy P và trực tâm tam giác DEF nằm trên đoạn OP Bài tập 14 (Đề TST 2007) Cho tam giác nhọn ABC với đường tròn tâm I nội tiếp Gọi (Ka ) là đường tròn qua A, AKa vuông góc với BC và (Ka ) tiếp xúc với (I) A1 Các điểm B1 , C1 xác định tương tự (a) Chứng minh AA1 , BB1 , CC1 đồng quy P (b) Gọi (Ja ), (Jb ), (Jc ) tương ứng là các đường tròn đối xứng với các đường tròn bàng tiếp các góc A, B, C tam giác ABC qua trung điểm BC, AC, AB Chứng minh P là tâm đẳng phương ba đường tròn (Ja ), (Jb ), (Jc ) Bài tập 15 (Đề TST 1995) Cho tam giác ABC có AB = c, BC = a, CA = b Lấy sáu điểm A1 , A2 , B1 , B2 , C1 , C2 phân biệt không trùng với A, B, C và các điểm A1 , A2 thuộc đường thẳng BC, B1 , B2 thuộc đường thẳng CA, các điểm C1 , C2 thuộc đường thẳng AB Gọi α, β, −−−→ −−→ −−−→ −→ −−−→ −−→ γ là các số thực xác định a · A1 A2 = α · BC, b · B1 B2 = β · CA, c · C1 C2 = γ · AB Xét các đường tròn ngoại tiếp các tam giác AB1 C1 , AB2 C2 , BC1 A1 , BC2 A2 , CA1 B1 , CA2 B2 và gọi dA , dB , dC là các trục đẳng phương cặp đường tròn qua A, B, C Chứng minh dA , dB , dC đồng quy và aα + bβ + cγ 6= Bài tập 16 (Đề TST 2008) Cho k là số thực Cho tam giác ABC nhọn, không cân có O là tâm đường tròn ngoại tiếp và AD, BE, CF là các đường phân giác tam giác Trên các đường thẳng AD, BE, CF lấy các điểm L, M, N cho AL BM CN = = = k AD BE CF Gọi (O1 ), (O2 ), (O3 ) là các đường tròn qua L, tiếp xúc với OA A; qua M tiếp xúc với OB B và qua N tiếp xúc với OC C (a) Chứng minh với k = , ba đường tròn (O1 ), (O2 ), (O3 ) có đúng điểm chung (b) Tìm tất các giá trị k cho đường tròn (O1 ), (O2 ), (O3 ) có đúng điểm chung (70) NHÌN HÌNH HỌC BẰNG CON MẮT ĐẠI SỐ Từ Nguyễn Thái Sơn HS chuyên Toán khóa 2008 - 2011 Hình học là phận hấp dẫn Toán học sơ cấp Chắc hẳn các bạn cảm thấy cái hay và thú vị các bài toán hình học chính là lời giải vô cùng đẹp túy hình học Thế để tìm cách giải không phải làm Và hướng tự nhiên hình học ta không thể tìm lời giải chính là đại số (Trong bài viết này chúng ta quan tâm các lời giải đại số, không trình bày các cách giải hình học túy) Mục đích bài viết này chính là góp phần tìm hướng đã bế tắc đường hình học túy Để có thể ứng dụng tốt đại số hình học, nói chung có khá nhiều công thức cần phải nắm vững (đa số là bản), ta có thể tóm gọn vài công thức sau (các công thức quen thuộc khác các bạn có thể đọc nhiều tài tiệu) Các công thức lượng giác Cho tam giác ABC Ta ký hiệu • a, b, c là độ dài các cạnh BC, CA, AB tam giác • 2p = a + b + c là chu vi tam giác và S = SABC là diện tích tam giác • R, r là bán kính đường tròn ngoại tiếp và nội tiếp tam giác • , rb , rc là bán kính các đường tròn bàng tiếp góc A, B, C • la , lb , lc là độ dài các đường phân giác góc A, B, C Khi đó A • sin = r (p − b)(p − c) A , cos = bc r p(p − a) A , tan = bc s (p − b)(p − c) p(p − a) • pr = (p − a)ra = S • − r = 4R sin2 A • Giả sử AH là đường cao tam giác Thế thì BH = c + a2 − b , 2a 69 CH = a2 + b − c 2a (71) 70 Chuyên đề Toán học số • Nếu các góc α, β, α0 , β ∈ [0◦ , 90◦ ] thỏa mãn α + β = α0 + β và sin α sin α0 = , sin β sin β thì ta có α = α0 và β = β Định lý Stewart Nếu đường AD = d thuộc tam giác ABC chia cạnh BC thành đoạn BD = m và CD = n thì d2 a = b2 m + c2 n − amn Định lý Van Aubel Cho tam giác ABC, M là điểm nằm tam giác AM, BM, CM cắt BC, CA, AB A0 , B , C Khi đó ta có hệ thức MA C 0A B0A + = C 0B B0C M A0 Các công thức khoảng cách Nếu gọi G, H, I, O là trọng tâm, trực tâm, tâm đường tròn nội tiếp, ngoại tiếp tam giác ABC, thì ta có • 9IG2 = 9r2 − 3p2 + 2(a2 + b2 + c2 ) p bcp(p − a) • la = b+c • 9OG2 = 9R2 − (a2 + b2 + c2 ) • OH = 3OG • OI = R2 − abc a+b+c • HI = 4R2 − a3 + b3 + c3 + abc a+b+c • a + b + c = 2p, ab + bc + ca = p2 + r2 + 4Rr, abc = 4pRr, Các định lý: điểm, Carno, Menelaus, Céva và Céva-sin • Định lý điểm: Cho điểm A, B, C, D mặt phẳng Khi đó AB vuông góc CD và AC − AD2 = BC − BD2 • Định lý Carno: Cho tam giác ABC và ba điểm M, N, P thuộc các đường thẳng BC, CA, AB Các đường thẳng qua M vuông góc BC, qua N vuông góc CA, qua P vuông góc AB đồng quy và M B − M C + N C − N A2 + P A2 − P B = (72) 71 Nhìn Hình học mắt Đại số • Định lý Menelaus: Cho tam giác ABC và ba điểm M, N, P thuộc các đường thẳng BC, CA, AB Ta có M, N, P thẳng hàng và MB NC P A · · = MC NA P B • Định lý Céva: Cho tam giác ABC và ba điểm M, N, P thuộc các đường thẳng BC, CA, AB Ta có AM, BN, CP đồng quy và MB NC P A · · = −1 MC NA P B Hệ thức trên còn có thể viết dạng lượng giác là sin ∠M AB sin ∠N BC sin ∠P CA · · = sin ∠M AC sin ∠N BA sin ∠P CB Hàng điểm điều hòa và các hệ thức liên quan Cho (ABCD) = −1, I là trung điểm AB, K là trung điểm CD Khi đó ta có các hệ thức sau • CA DA : = −1 CB DB • Hệ thức Decartes: 1 = + AB AC AD • Hệ thức Newton: IA = IC · ID • Hệ thức Macloranh: AB · AK = AC · AD Ngoài ra, ta có còn có hệ thức đơn giản sau: Cho (ABCD) = −1 và K là trung điểm CD Khi đó  2 KC KC CA DA KA =− = , = KB KB CB DB KB Chứng minh Do (ABCD) = −1 và K là trung điểm CD nên theo hệ thức Newton, 2 ta có KC = KD2 = KC = KD = KA · KB Từ đó suy KC AK AC DA KA = = = và = KB CK CB DB KB  KC KB 2 Các hệ thức chứng minh Sau đây là ứng dụng nhỏ hai hệ thức vừa chứng minh (73) 72 Chuyên đề Toán học số Ví dụ (Đường thẳng Gauss) Cho tứ giác toàn phần ABCDEF có ba đường chéo EF, AC, BD Gọi H, I, K là các trung điểm EF, BD, AC Chứng minh các điểm H, I, K cùng nằm trên đường thẳng Lời giải E B I M A O K H F C D G Gọi G là giao điểm BD và EF, O là giao điểm AC và BD Hướng tự nhiên để tiếp cận bài toán trên chính là sử dụng định lý Menelaus cho tam giác M GO để chứng minh I, H, K thẳng hàng Chú ý (GM EF ) = −1, (M OAC) = −1, Áp dụng các hệ thức trên, ta thu  2  2 IM FM HO CO = , = , FG CM IG HM (GODB) = −1 KG = KO  DG DO 2 Nhân lại theo vế, ta thu IM HO KG · · = IG HM KO  F M CO DG · · F G CM DO 2 Đến đây, áp dụng định lý Menelaus cho tam giác M GO với cát tuyến CDF, ta có F M CO DG · · = F G CM DO Từ đó suy điều phải chứng minh Bài toán trên có nhiều cách giải khác độc đáo cách dựng yếu tố phụ ta có thể thấy hướng trên là tự nhiên (74) 73 Nhìn Hình học mắt Đại số Một số hệ thức khác Bổ đề Cho tứ giác ABCD M, N là các điểm nằm trên hai đoạn AD, BC cho MA NB m = = Khi đó MD NC n −→ −−→ −−→ nAB + mDC MN = m+n nAB + mDC Đặc biệt ABCD là hình thang thì M N = m+n Chứng minh bổ đề trên là đơn giản, ta thử xét ví dụ sau là bài toán dễ để bước đầu thấy ứng dụng bổ đề Ví dụ Cho tam giác ABC, đường cao AA0 Hạ A0 E, A0 F vuông góc với BC E, F Từ B và C, kẻ các đường vuông góc với BC cắt A0 E, A0 F M, N Chứng minh M N qua trực tâm H tam giác ABC Lời giải A N F H H1 M E G C B A0 Do M B, N C song song với AA0 nên MB EB A0 B A0 B = = , suy M B = Tương AA0 EA AA02 AA0 A0 C AA0 Giả sử M N cắt AA0 H Theo bổ đề ta có tự ta có M C = A0 C · BM + A0 B · CM A0 B · A0 C AH = = BC BC Suy A0 H · AA0 = A0 B · A0 C, hay tam giác BA0 H đồng dạng tam giác AA0 C Từ đây ta có BH vuông góc với AC, hay H ≡ H (đpcm) Kế đến, ta có bổ đề liên quan đến đường thẳng Euler tam giác (75) 74 Chuyên đề Toán học số Bổ đề Cho tam giác ABC O, H, G là tâm đường tròn ngoại tiếp, trực tâm và trọng tâm tam giác Giả sử AC > AB, AC > BC Khi đó ta có BC(AC − BC ) sin(B − A) sin ∠OHC = = 2 AB(AC − AB ) sin(B − C) sin ∠OHA Chứng minh A J O F B H D I C Ta có sin(B − A) = sin B cos A − sin A cos B AC AB + AC − BC BC AB + BC − AC = · − · 2R 2AB · AC 2R 2AB · BC AC − BC = · 2R AB Tương tự, ta có sin(B − C) = AC − AB · Do đó 2R BC sin(B − A) BC(AC − BC ) = sin(B − C) AB(AC − AB ) Ta còn phải chứng minh BC(AC − BC ) sin ∠OHC = 2 AB(AC − AB ) sin ∠OHA Gọi I, J là trung điểm BC, BA Dễ thấy sin ∠OHC = sin ∠HOJ và sin ∠OHA = sin ∠HOI, đó sin ∠OHC sin ∠HOJ F J OH FJ = = · = sin ∠OHA sin ∠HOI OH DI DI (76) Nhìn Hình học mắt Đại số 75 Từ đây suy ta cần chứng minh FJ BC(AC − BC ) = DI AB(AC − AB ) Ta có JF = JB − BF = (1) AB AB + BC − AC AC − BC − = , và tương tự 2AB 2AB ID = AC − AB 2BC Do đó (1) hiển nhiên đúng Bổ đề chứng minh Hệ thức trên cho ta cái nhìn đại số đường thẳng Euler (HO chính là đường thẳng Euler) Việc dựng thêm các đường vuông góc trên là cách làm quen thuộc phương pháp dùng đại số để giải hình học Sau đây, ta xem xét vài ứng dụng nhỏ hệ thức trên Ví dụ Cho tam giác ABC có B > A, B > C Về cùng nửa mặt phẳng bờ AC, dựng các tia Ax, Cy cho ∠BAx = ∠ABC = ∠BCy, Cy cắt Ax M Chứng minh M nằm trên đường thẳng Euler tam giác ABC Lời giải Nhận xét bài toán trên có cách phát biểu khá túy hình học, có lẽ đại số là cách tiếp cận tự nhiên cho bài toán này A M J O F B H D I C Một cách tiếp cận khá tự nhiên là tìm cách chứng minh ∠AHM = ∠AHO Với góc ∠AHM thì ta hoàn toàn xác định khá đẹp mắt và gọn gàng với bổ đề trên, từ đó ta hoàn toàn có thể tin tưởng bài toán có lời giải ngắn gọn đại số (77) 76 Chuyên đề Toán học số Để xử lý ∠AHO, ta dùng định lý Céva dạng sin cho tam giác AHM (ý tưởng hoàn toàn đơn giản và tự nhiên) Nối HM, đó các đường thẳng Ax, HM, Cy đồng quy M Từ đó suy sin ∠M AH sin ∠M CA sin ∠M HC · · = sin ∠M AC sin ∠M CH sin ∠M HA sin ∠M CA sin ∠M HC · = 1, suy Do ∠M CH = ∠M AH nên ta có sin ∠M AC sin ∠M HA sin ∠M CA sin ∠M HA sin(B − C) = = sin ∠M AC sin ∠M HC sin(B − A) Theo bổ đề trên, ta thấy M nằm trên đường thẳng Euler tam giác ABC Nhận xét Một hướng giải khác tự nhiên cho bài toán trên là sử dụng phương pháp vector: • Gọi E là giao điểm CM và AB, F là giao điểm AM và BC, G là trọng tam tam giác ABC Sau đó, sử dụng định lý Menelaus cho các tam giác cân, −−→ −−→ −−→ −→ −−→ −→ ta biểu diễn các vector AM , BM , CM theo AB, BC, CA −−→ −−→ −−→ −−→ −−→ −→ −−→ • Sử dụng hệ thức AM + BM + CM = 3GM để biểu diễn GM theo AB và CB −→ −−→ −→ −→ −−→ −→ • Sau đó chú ý AO + BO + CO = 3GO, từ đó chứng minh GM ↑↑ GO Đôi phương pháp này ta không cần tính tường minh các đại lượng mà nên khéo léo sử dụng cách gián tiếp, tức là để số đại lượng ẩn số và làm cho biểu thức cuối cùng, các đại lượng đó bị triệt tiêu Sau đây là ví dụ tương tự khác với kỹ thuật và hướng giải tự nhiên, dễ dàng Ví dụ Cho tam giác ABC nhọn, có H, G là trực tâm và trọng tâm tam giác Qua A, B, C kẻ các đường thẳng da , db , dc vuông góc với GA, GB, GC da cắt db C , tương tự ta có A0 và B Gọi G0 là trọng tâm tam giác A0 B C Chứng minh G0 nằm trên đường thẳng Euler tam giác ABC Lời giải Để giải bài này, trước hết ta cần gọi thêm số yếu tố phụ: • Gọi O là tâm đường tròn ngoại tiếp tam giác ABC • D, E, F là chân các đường cao hạ từ A, B, C tam giác ABC • A0 G0 , B G0 , C G0 cắt B C , C A0 , A0 B K, N, M • I, J, S là trung điểm BC, AB, CA Dễ dàng dự đoán A0 G0 ⊥ BC (tương tự các đoạn còn lại) và đây chính là thuận lợi cho ta việc dùng đại số Để chứng minh điều này, ta có khá nhiều cách, lượng giác vector, đây tôi trình bày lời giải theo hướng vector (78) 77 Nhìn Hình học mắt Đại số B0 K A C0 E L H B O G I Q G D C A0 Ta có −−0→ −−→ −− → −− → −−→ −→ −− → −−→ −−→ −→ 0 0 2A K · BC = A B + A C BG + GC = A0 B · BG + A0 C · GC Nhận xét −−→ −−→ −−→ −→ 0 cos A B , BG = − cos A0 C , GC Thật vậy, −−→ −−→ −−→ −→ −→ −−→ π −→ −−→ A0 B , BG = A0 B , GC + GC, BG = − + GC, BG (1) (mod 2π) và −−→ A0 C , −−→ nên A0 B , → −→ −−→ −→ −→ −− π −→ −−→ 0 − GC = A C , BG + BG, GC = − − GC, BG    − − → −−→ −→ BG + A0 C , GC = −π (mod 2π) Từ đây ta có (1) (mod 2π), −−→ −−→ Từ (1) suy để chứng minh 2A0 K · BC = 0, ta cần chứng minh A0 B CG A B · GC = A C · GB, hay 0 = AC BG 0 0 (79) 78 Chuyên đề Toán học số Dễ thấy A0 B sin ∠IGB sin ∠A0 C B sin ∠AGB = (2) = = 0 0 AC sin ∠A B C sin ∠AGC sin ∠IGC Mặt khác, tam giác GBC, SGBI = SGCI , suy BG sin ∠BGI = CG sin ∠CGI Do đó sin ∠IGC BG = (3) CG sin ∠IGB Từ (2) và (3), suy A0 G0 ⊥ BC Chứng minh tương tự, ta A0 G0 ⊥ BC, B G0 ⊥ AC, C G0 ⊥ AB (4) Áp dụng kỹ thuật tương tự trên, ta chứng minh G0 nằm trên đường thẳng Euler tam giác ABC cách sử dụng định lý Céva-sin tam giác AHC Ta có sin ∠G0 AH sin ∠G0 CA sin ∠G0 HC · · = sin ∠G0 AC sin ∠G0 CH sin ∠G0 HA Mà dễ thấy (4) nên ta áp dụng kỹ thuật gián tiếp sin ∠G0 AH sin ∠G0 CA sin ∠G0 AH G0 A · = · = sin ∠G0 CH sin ∠G0 AC sin ∠G0 CH G0 C  DQ G0 C · G0 A JF  · DQ G0 A = GC JF Ta đã triệt tiêu các đại lượng G0 A và G0 C Và thế, công việc còn lại bây là chứng minh AB(AC − AB ) DQ = , (5) JF BC(AC − BC ) sin ∠G0 HA DQ AB(AC − AB ) để từ đó suy = = Áp dụng bổ đề, ta có sin ∠G0 HC JF BC(AC − BC ) điều phải chứng minh DQ , ta có thể sử dụng phương pháp gián tiếp JF (dành cho các bạn), đây việc tính toán khá đơn giản và ta tính trực tiếp để thấy vài tính chất điểm G0 Ý tưởng ta là tính cụ thể các đoạn BQ và CQ Ta có ∠GBC = ∠BA0 G0 và ∠GCB = ∠G0 A0 C nên Bây ta chứng minh (5) Để tính sin ∠BC A cos ∠GBC A0 B = = 0 AC sin ∠CBA cos ∠GCB Đến đây cách sử công thức trung tuyến cho tam giác ABC và định lý hàm số cosin cho tam giác BGC, ta có A0 B BG AC + 3BC − AB BG sin ∠GBC = · (chú ý = ) 2 AC CG AB + 3BC − AC CG sin ∠GCB Lại có A0 B sin ∠BA0 G0 = BQ và A0 C sin ∠CA0 G0 = CQ, nên BQ CG A0 B A0 B sin ∠BA0 G0 · = = · A0 C sin ∠CA0 G0 CQ BG A0 C (6) (80) 79 Nhìn Hình học mắt Đại số Kết hợp với (6), ta BQ AC + 3BC − AB = CQ AB + 3BC − AC Mặt khác, BQ+CQ = BC nên từ đây ta dễ dàng tính BQ = suy 2(AC − AB ) DQ = 3BC 2(AC − BC ) Tương tự, ta có JF = Từ đó rõ ràng 3AB DQ AB(AC − AB ) = , JF BC(AC − BC ) AC + 3BC − AB , 6BC và ta có điều phải chứng minh Nhận xét Từ việc tính DQ, CQ ta còn có thể vị trí hình học điểm G0 Thật 2(AC − AB ) AC − AB mà ID = nên dễ thấy vậy, ta có DQ = 3BC BC DQ = ID HG0 DQ Từ đây sử dụng định lý Thalès, ta có = = , suy HO ID 4 HG0 = HO = · HG = 2HG 3 Vậy G là trung điểm HG0 hay G0 đối xứng H qua G Công cụ đại số còn thể sức mạnh bài toán có yếu tố đường tròn Ta xét ví dụ sau dể hiểu rõ điều này Ví dụ Cho tam giác ABC nội tiếp đường tròn (O, R), D là điểm nằm đoạn BC Gọi (O0 , r1 ), (O00 , r2 ) là đường tròn tiếp xúc với (O), BC và AD (hai đường tròn (O0 , r1 ), (O00 , r2 ) gọi là đường tròn Mixtilinear các tam giác ABD và ACD) Chứng minh r = r1 cos2 ∠ADB ∠ADC + r2 cos2 2 Lời giải Để có thể giải thành công bài toán này, ta cần có định lý Lyness cho tứ giác và bổ đề Sawayama Định lý Lyness Cho tam giác ABC nội tiếp (O) Gọi (wa , pa ) là đường tròn vừa tiếp xúc (O) vừa tiếp xúc với hai cạnh AB và AC E, F Khi đó ta có r (1) pa = ; ∠BAC cos (2) Trung điểm EF là tâm đường tròn nội tiếp tam giác ABC (81) 80 Chuyên đề Toán học số A O E I D W C B Với (1) ta có thể chứng minh (2) phương pháp đại số Bổ đề Sawayama Cho (O) và đường tròn (I) tiếp xúc với (O) M Hai dây cung AC, BD (O) tiếp xúc (I) E, F Khi đó tâm đường tròn nội tiếp các tam giác ABC và DBC nằm trên đường thẳng EF ∠ADC ∠ADB ,β= , 2 ◦ đó α + β = 90 Không tính tổng quát, giả sử DB > DC và (O ) tiếp xúc BC, AD E, M ; (O00 ) tiếp xúc BC, AD F, N Bây giờ, ta chứng minh bài toán đã cho Để cho gọn, ta đặt α = A M O0 I O O00 K N G B E H D F C (82) Nhìn Hình học mắt Đại số 81 Dễ thấy EM ⊥ N F Giả sử M E cắt O0 O00 I , M E cắt O0 D K, N F cắt O00 D G, ta có  2 I O0 OE KO0 = = sin2 α = O0 O00 O0 D O0 D Bây giờ, gọi I 00 là giao điểm O0 O00 và N F thì ta có 2  DG DF I 00 O0 = cos2 β = = O0 O00 DO00 DO00 I O0 I 00 O0 = , suy I ≡ I 00 Theo bổ đề Sawayama, ta có O0 O00 O0 O00 I ≡ I 00 ≡ I Từ đó suy IO0 = sin2 α = cos2 β O0 O00 Tương tự, ta có IO00 = sin2 β = cos2 α O0 O00 Mà sin2 α = cos2 β nên Áp dụng bổ đề 1, ta có r= O00 I O0 I O0 E · O00 I + OF · O0 I = r · + r · = r1 cos2 α + r2 cos2 β O0 O00 O0 O00 O0 O00 Phép chứng minh hoàn tất Nhận xét Bài toán chưa dừng lại đây, ta hoàn toàn có thể khai thác thêm nữa: Hạ IH ⊥ BC, ta có EH = r tan α và F H = r tan β Bây đặt DH = x, ta tính DE = r tan α + x và DF = r tan β − x Suy r1 = DE tan α = r tan2 α + x tan α, r2 = DF tan β = r tan2 β − x tan β Từ đây với các vị trí đặc biệt D ta hoàn toàn có thể xác định r1 , r2 Hệ thức trên cho ta liên hệ đẹp r1 , r2 , r và có lợi các bài toán đường tròn Mixtilinear Chẳng hạn ta có thể dùng nó để giải bài toán: Khi D là tiếp điểm đường tròn bàng tiếp góc BAC với BC thì r1 = r2 = r Để kết thúc bài viết, tôi xin nêu ví dụ cuối cùng với hai lời giải “đại số” Ví dụ Cho tam giác ABC nhọn nội tiếp đường tròn (O, R) có ∠C < ∠B < 90◦ Tiếp tuyến (O) A cắt BC S SO cắt AB, AC M, N Gọi E, F là trung điểm AB, AC Chứng minh N E, M F, AO đồng quy Lời giải Bài toán trên giải phương pháp hình học túy khá ngắn gọn và dễ dàng Ở đây ta xem xét nó góc độ đại số (83) 82 Chuyên đề Toán học số A F H E K N M O I S D B C Cách Đặt BC = a, CA = b, AB = c Dễ thấy B nằm S và C Giả sử SO cắt EF K, AO cắt EF H Hướng giải tự nhiên ta là sử dụng định lý Céva để chứng minh N E, M F, AO đồng quy, tức là cần chứng minh OM F N EA · · = ON F A EM OM KM = và từ đó ON KN áp dụng định lý Menelaus (với chú ý F, E, K thẳng hàng), ta có Muốn ta phải chứng minh (M N OK) = −1 để suy KM F N EA OM F N EA · · = · · = 1, ON F A EM KN F A EM tức là bài toán chứng minh Dễ thấy (M N OK) = −1 và (EF HK) = −1, hay HE KE = HF KF (1) Vậy ta cần chứng minh (1) là đủ Hạ OI ⊥ BC, đặt ∠IAO = α và ∠AIO = β (dễ thấy α, β < 90◦ ) Ta có SA = abc , b − c2 SB = ac2 , b2 − c Đến đây đặt tiếp OI = d = R cos A, ta SC = ab2 b2 − c MA SA sin α b sin α bR = = = , suy MB SB sin β a sin β cd ME bR − cd = MB 2cd Tương tự ta có NE cR − bd = NB 2bd (84) 83 Nhìn Hình học mắt Đại số Do KE song song với BC nên áp dụng định lý Thalès ta có KE = ac(bR − cd) , 2d(b2 − c2 ) KF = ba(cR − bd) , 2d(b2 − c2 ) suy KE c(bR − cd) = KF b(cR − bd) (2) Bây giờ, gọi D là giao điểm AO và BC, ta tính HE DE c cos C = = HF DF b cos B (3) Kết hợp (2) và (3), ta cần chứng minh c(bR − cd) c cos C = b(cR − bd) b cos B Hệ thức này hiển nhiên đúng (có thể kiểm tra dễ dàng biến đổi tương đương) Do đó bài toán chứng minh xong Cách Cũng giống cách 1, ý tưởng ta là chứng minh OM F N EA · · =1 ON F A EM để từ đó áp dụng định lý Céva suy kết bài toán Hệ thức trên tương đương với FA AC OM F N · = = ON EM EA AB Do F N = ON cos ∠AN M và EM = OM cos ∠AM N nên ta có OM ON cos ∠AN M AC cos ∠AM N AB · = , hay = ON OM cos ∠AM N AB cos ∠AN M AC (4) Chứng minh (4) đơn giản và có nhiều cách, đây chúng ta giải trên tinh thần đại số Ta có cos ∠AM N = cos(α + C) = cos C cos α − sin C sin α = cos C sin ∠AIB − sin C cos ∠AIB = sin(∠AIB − C) = sin ∠IAC Tương tự ta có cos ∠AN M = sin ∠IAB Mà AB sin ∠IAB = AC sin ∠IAC nên sin ∠IAC AB cos ∠AM N = = cos ∠AN M sin ∠IAB AC Bài toán chứng minh xong Nhận xét Bài toán trên không cần tới các giả thiết tam giác nhọn và ∠C < ∠B < 90◦ (85) 84 Chuyên đề Toán học số Bài tập Ý tưởng đại số hình học luôn tự nhiên và đẹp đẽ, hy vọng qua các ví dụ trên các bạn đã phần nào hình dung cách ứng dụng linh hoạt đại số hình học (ở đây đại số khác với tọa độ) Để kết thúc xin mời các bạn thử giải các bài toán sau cách đại số Bài tập Cho tam giác ABC có đường tròn nội tiếp tâm I; I1 , I2 , I3 là tâm các đường −→ −→ − − − tròn bàng tiếp các góc A, B, C Ký hiệu → v1 = BI + CI, tương tự với → v2 , → v3 Gọi (K1 ), − (K2 ), (K3 ) là ảnh (I1 ), (I2 ), (I3 ) qua các phép tịnh tiến theo các vector → v1 , → − → − v2 , v3 Chứng minh tâm đẳng phương (K1 ), (K2 ), (K3 ) nằm trên đường thẳng IG với G là trọng tâm tam giác ABC Bài tập Cho tam giác ABC nội tiếp đường tròn (O, R) Gọi M, N, P là trung điểm BC, CA, AB Các đường trung trực AB, AC cắt trung tuyến AM D, E và cắt F Chứng minh bốn điểm O, D, E, F cùng nằm trên đường tròn π BD cắt AC E Gọi O1 và O2 là tâm các đường tròn ngoại tiếp các tam giác ABE và CDE Chứng minh trung điểm O1 O2 nằm trên BD Bài tập Cho tứ giác ABCD có ∠DAB = ∠DCB = Tài liệu tham khảo [1] Đoàn Quỳnh, Số phức với hình học phẳng, Nhà xuất Giáo Dục, 1999 [2] Christopher J Bradley, Challenges in Geometry: for Mathematical Olympians Past and Present, Oxford University Press, 2005 [3] Paul Yiu, Introduction to the Geometry of the triangle, 2001 [LINK: http://math.fau.edu/Yiu/GeometryNotes020402.ps] [4] Kiran S Kedlaya, Notes on Euclidean Geometry, 1999 [LINK: http://math.mit.edu/∼kedlaya/geometryunbound/geom-080399.pdf] [5] Kiran S Kedlaya, Geometry Unbound, 2006 [LINK: http://math.mit.edu/∼kedlaya/geometryunbound/gu-060118.pdf] [6] Roger A Johnson, Advanced Euclidean Geometry, Dover Publications, 1960 (86) NGUỒN GỐC BÀI TOÁN HÌNH HỌC SỐ TRONG ĐỀ THI VIỆT NAM TST 2009 Lê Bá Khánh Trình Trường Đại học Khoa học Tự nhiên thành phố Hồ Chí Minh Lời tựa Seminar các phương pháp Toán sơ cấp thành lập số giáo viên chuyên Toán thành phố Hồ Chí Minh và các tỉnh lân cận, sinh hoạt đặn cách tuần vào các sáng chủ nhật (trừ số ngắt quãng bất đắc dĩ) Kể từ tháng 10/2007 đến nay, qua năm hoạt động đã tổ chức trên 50 seminar với các chủ đề đa dạng Toán học phổ thông: Giải tích, Đại số, Hình học, Tổ hợp, Số học, ứng dụng Toán học, giới thiệu Toán cao cấp Bài viết đây TS Lê Bá Khánh Trình trình bày seminar ngày 26/9, ghi lại và bổ sung bạn Lê Phúc Lữ Qua bài viết nhỏ này, chúng ta có thể nhìn thấy quá trình hình thành và kiểm định bài toán nào Học đường tư người khác chính là bước mở đầu sáng tạo 1.1 Một số định lý mở đầu Đường thẳng Gauss Cho tứ giác ABCD có E, F là giao điểm các đường thẳng chứa các cặp cạnh đối là AB và CD; AD và BC Gọi M, N, P là trung điểm các cạnh AC, BD, EF Khi đó ta có M, N, P thẳng hàng và đường thẳng qua chúng gọi là đường thẳng Gauss tứ giác ABCD E B A P N M C F D Cách chứng minh quen thuộc định lý này là dùng tỉ số diện tích Đây chính là các tính chất quan trọng tứ giác toàn phần 1.2 Đường thẳng Newton tứ giác ngoại tiếp Cho tứ giác ABCD lồi ngoại tiếp đường tròn (O) Gọi M, N là trung điểm AC, BD Khi đó ta có ba điểm M, O, N thẳng hàng và đường thẳng qua chúng gọi là đường thẳng Newton 85 (87) 86 Chuyên đề Toán học số B A N M O C D Tính chất quan trọng tứ giác ngoại tiếp này có liên quan đến bài toán tìm quỹ tích các điểm K nằm mặt phẳng cho tổng diện tích hai tam giác KAB và KCD tổng diện tích hai tam giác KBC và KDA, tập hợp đó chính là đường thẳng qua trung điểm hai đường chéo; ta dễ dàng thấy tâm O đường tròn nội tiếp tứ giác ABCD có tính chất đó nên O phải nằm trên đường thẳng M N hay M, O, N thẳng hàng Ngoài ra, ta có thể dựng thêm đường phụ để dùng phương pháp diện tích sử dụng định lý nhím để giải vấn đề này Đường thẳng Newton đúng tứ giác lõm ngoại tiếp đường tròn sau (trong hình bên dưới, tứ giác lõm ABCD “ngoại tiếp” đường tròn tâm O và M, N là hai trung điểm các đường chéo AC và BD) O C A M D N B Từ đó, ta thấy tứ giác ngoại tiếp thì đường thẳng Gauss, đường thẳng Newton nó là trùng và trên đó có chứa điểm đặc biệt Vận dụng nhận xét này, ta có nhiều bài toán hay và khó liên quan đến thẳng hàng các điểm, đồng quy các đường thẳng tứ giác ngoại tiếp 1.3 Tứ giác Pedal Cho tứ giác ABCD có I là điểm nằm tứ giác Gọi M, N, P, Q là hình chiếu I lên các cạnh AB, BC, CD, DA Khi đó, tứ giác M N P Q là tứ giác Pedal điểm I tứ giác đã cho (88) Nguồn gốc bài toán Hình học số đề thi Việt Nam TST 2009 87 B M A N I Q C D P Cũng giống tam giác Pedal, tứ giác loại này có nhiều tính chất thú vị Trong phần này, chúng ta quan tâm đến tính chất nội tiếp hay ngoại tiếp nó với số vị trí đặc biệt điểm I và tứ giác ABCD đã cho Các bài toán liên quan Bài toán Cho tứ giác ABCD nội tiếp đường tròn (O) có hai đường chéo AC và BD cắt I Gọi M N P Q là tứ giác Pedal điểm I tứ giác ABCD (M, N, P, Q là hình chiếu I lên các cạnh AB, BC, CD, DA) Chứng minh M N P Q là tứ giác ngoại tiếp Lời giải A M Q O B D I P N C Ta có ∠IM B + ∠IN B = 180◦ nên tứ giác IM BN nội tiếp, suy ∠IM N = ∠IBN Tương tự, ∠IM Q = ∠IAQ Mặt khác, ABCD nội tiếp nên ∠IAQ = ∠IBN, đó ∠IM N = ∠IM Q, hay M I là phân giác ∠N M Q Tương tự, IN, IP, IQ là phân giác các góc tương ứng tứ giác M N P Q Từ đó suy I cách các cạnh tứ giác M N P Q hay M N P Q là tứ giác ngoại tiếp đường tròn tâm I Ta có điều phải chứng minh Nhận xét Nếu thêm vào đề bài điều kiện AC và BD vuông góc với P thì ta có thêm kết sau: “Tứ giác M N P Q vừa ngoại tiếp vừa nội tiếp đường tròn.” (89) 88 Chuyên đề Toán học số Ta cần chứng minh thêm tứ giác M N P Q nội tiếp Ta có ∠AM Q = ∠AIQ = ∠ADI = ∠QP I Tương tự, ∠BM N = ∠N P I Do đó ∠N M Q + ∠N P Q = ∠N M Q + ∠IP N + ∠IP Q = ∠N M Q + ∠AM Q + ∠BM N = 180◦ , hay tứ giác M N P Q nội tiếp Vậy tứ giác M N P Q vừa nội tiếp vừa ngoại tiếp Nhận xét Ta thay đổi giả thiết chút và quan tâm đến các tính chất đã nêu: “Trong đường tròn (O) cho điểm I cố định Hai dây cung AC và BD đường tròn (O) thay đổi và cắt I Gọi M, N, P, Q là hình chiếu I lên các cạnh AB, BC, CD, DA tứ giác ABCD Gọi E, F là trung điểm M P, N Q Chứng minh đường thẳng qua EF luôn qua điểm cố định.” Vì là kết hợp trực tiếp định lý đã nêu trên nên nói chung kết thu còn thô, chưa có điểm lạ hay đặc biệt gì Bài toán Cho tứ giác ABCD nội tiếp có hai đường chéo vuông góc với I Gọi M, N, P, Q là hình chiếu I lên các cạnh AB, BC, CD, DA Giả sử E là giao điểm hai đường thẳng M N, P Q; F là giao điểm hai đường thẳng P N, QM Chứng minh trung điểm EF, M P, N Q thẳng hàng và đường thẳng đó qua I A M O Q B E I D P N C F Kết bài toán này không khó để tìm đã nắm các nội dung trên Khi đó tứ giác lõm N F QE ngoại tiếp đường tròn (I) theo cách hiểu 1.2 và dẫn đến ba trung điểm EF, M P, N Q và điểm I cùng nằm trên đường thẳng Nếu chưa nắm hướng đã xác định ban đầu thì đây thật không phải là bài toán đơn giản (90) Nguồn gốc bài toán Hình học số đề thi Việt Nam TST 2009 89 Dễ dàng thấy cách dùng định lý Menelaus, ta có thể chứng minh M N, AC, P Q đồng quy và M Q, BD, P N đồng quy Từ điều này, ta có thể phát triển thêm nhiều kết liên quan đến tính chất nội tiếp này tứ giác M N P Q Dựa vào cấu trúc trên, ta giữ lại số điểm, đoạn thẳng cần thiết và bỏ các điểm còn lại nhằm che giấu chất “tứ giác ngoại tiếp” đề Rõ ràng bài toán vừa phát biểu, tứ giác quan trọng cần xét là N F QE, để có thể đặt các vấn đề liên quan đến nó thì phải thông qua các tứ giác ABCD, M N P Q đã xuất trước, điều này khiến cho bài toán không còn nhiều giá trị Ta dựng lại tứ giác N F QE thay vì thông qua các điểm A, B, C, D có sẵn, ta dựng thông qua đoạn thẳng EF và điểm P Thực điều này không quá khó, ta có thể làm sau • Giả sử ta đã có đoạn thẳng EF và điểm P trên mặt phẳng (P không thuộc EF ) • Dựng đường tròn (O) đường kính EF và gọi I là giao điểm phân giác góc EP F với (O) (I và P nằm cùng phía với P so với EF ) • Gọi C, D là các giao điểm đường phân giác ngoài góc EP F với các đường thẳng IE, IF • Tia F P cắt đường tròn đường kính ID Q (khác P ), tia EP cắt đường tròn đường kính IC N (khác P ) Đến đây rõ ràng là yêu cầu nêu trên đã thực Không có các tứ giác ABCD, M N P Q giúp định hướng, bài toán trở nên xa lạ và ta có thể đặt các vấn đề cũ trước đó: chứng minh trung điểm EF, QN và điểm P thẳng hàng I N C Q P D E F P E F O Từ cách dựng đó, ta phát biểu bài toán sau Bài toán Cho đoạn thẳng EF và điểm P không nằm trên EF Gọi I là giao điểm phân giác góc EP F với đường tròn đường kính EF (I nằm cùng phía với P so với EF ) Gọi D, C là giao điểm các đường phân giác ngoài góc EP F với các đoạn IE, IF Gọi N là giao điểm tia EP với đường tròn đường kính IC và Q là giao điểm tia F P với đường tròn đường kính ID Chứng minh trung điểm EF, QN và I là ba điểm thẳng hàng (91) 90 Chuyên đề Toán học số Lời giải I N Q C K D P E F O Để giải bài toán này, ta cần chứng minh tứ giác lõm N EQF ngoại tiếp đường tròn tâm I để áp dụng đường thẳng Newton vào suy trung điểm hai đường chéo N Q, EF và I là thẳng hàng Điều này có thể thực trực tiếp mà không cần phải khôi phục lại tất các điểm đề bài nêu Thật vậy, gọi K là giao điểm P Q và DI Do P I, P D là các phân giác ngoài và phân giác tam giác KP E nên DE IE = = k, DK IK suy đường tròn đường kính DI chính là đường tròn Appollonius hai điểm D, I ứng với tỉ số k xác định trên Do Q thuộc đường tròn này nên QK DK =k= , QE DE suy QD là phân giác ∠EQK và QI là phân giác ngoài góc ∠EQK Hoàn toàn tương tự, ta thấy N I là phân giác ngoài góc ∠P N F Do đó, I cách các cạnh EN, F Q, QE, N F tứ giác lõm QEQF hay I là tâm đường tròn nội tiếp tứ giác này Từ đó ta có kết cần chứng minh Tiếp theo, hình vẽ thoáng hơn, ta thay đổi đề bài chút, ta định nghĩa lại điểm I nằm khác phía với P so với EF và đặt tên các điểm bài cho phù hợp Ta xét bài toán sau Bài toán Cho đường tròn (O) đường kính AB và M là điểm nằm (O), M không nằm trên đoạn AB Gọi N là giao điểm tia phân giác góc M tam giác AM B với đường tròn (O) Đường phân giác ngoài góc AM B cắt các đường thẳng N A, N B P, Q Đường thẳng M A cắt đường tròn đường kính N Q R, đường thẳng M B cắt đường tròn đường kính N P S và R, S khác M Chứng minh đường trung tuyến ứng với đỉnh N tam giác N RS luôn qua điểm cố định M di động phía đường tròn Đây chính là bài số quen thuộc đề thi chọn đội tuyển học sinh giỏi quốc gia dự thi IMO năm 2009 Việc xây dựng các điểm theo thứ tự ngược lại dùng tính chất ngoại (92) Nguồn gốc bài toán Hình học số đề thi Việt Nam TST 2009 91 tiếp tứ giác lõm đã biến bài toán kết hợp thô từ các kết quen thuộc thành bài toán không đơn giản Trên thực tế, có thí sinh kỳ thi đó giải trọn vẹn bài này, có thí sinh đã tiếp cận chất bài toán Đặc biệt, có thí sinh khác đã giải bài này phương pháp tọa độ và chứng minh bài toán đúng (tức là trung tuyến đỉnh N tam giác N RS luôn qua O) thay N điểm thuộc phân giác góc ∠AM B không thiết là N phải thuộc đường tròn (O) Các lời giải khác cho bài số đề Việt Nam TST 2009 Ngoài hướng giải cách dùng tứ giác lõm ngoại tiếp và đường thẳng Newton, đây có nêu thêm ba lời khác và điều này cho ta thấy hướng tiếp cận, đánh giá khác cho nhiều lời giải khác có nội dung khá thú vị Lời giải R I D C Q S P M A O B N Qua R kẻ đường thẳng song song với P Q cắt N A C, qua S kẻ đường thẳng song song với P Q cắt N B D Gọi I là trung điểm CD Ta chứng minh CD k AB Thật vậy, N nằm trên đường tròn đường kính AB nên ∠AN B = 90◦ , suy AN ⊥ BN Từ đây ta có BN là tiếp tuyến đường tròn đường kính P N Do đó 4BM N ∼ 4BN S (g.g) Vì P Q là đường phân giác góc ngoài AM N nên ∠SM P = ∠AM P = ∠QM R = ∠BM Q Mặt khác, lại có ∠SM P = ∠SN P (góc nội tiếp cùng chắn cung P S đường tròn đường kính P N ) và ∠QM R = ∠QN R (góc nội tiếp cùng chắn cung QR đường tròn đường kính QN ) Do đó ∠SN P = ∠QN R, suy ∠SN P + ∠SN R = ∠QN R + ∠SN R và ∠CN R = ∠SN B Bây giờ, xét hai tam giác 4BN S và 4RN C, ta có ∠RCN = ∠M P N = ∠N SM = ∠N SB và ∠CN R = ∠SN B nên 4BN S ∼ 4RN C (g.g) Suy 4BM N ∼ 4BN S ∼ 4RN C (93) 92 Chuyên đề Toán học số Tương tự, ta có 4DSN ∼ 4RAN ∼ 4N AM Ta thấy NB NS = , suy N B · N C = N R · N S NR NC ND NS = , suy N A · N D = N R · N S • Do 4DSN ∼ 4RAN nên NA NR NC NA = và AB k CD Do vậy, trung Từ kết này ta có N B · N C = N A · N D, suy NB ND điểm AB, trung điểm CD và N là điểm thẳng hàng, tức là N, O, I thẳng hàng (1) • Do 4BN S ∼ RN C nên Hơn nữa, MN BN NB · NC = , suy RC = NC RC MN DS NA · ND DN = , suy DS = • Do 4DSN ∼ 4N AM nên MN NA MN • Do 4BM N ∼ 4RN C nên Kết hợp các điều trên, ta RC = DS, mà RC k DS (cùng song song với P Q) nên tứ giác RCSD là hình bình hành Do đó, hai đường chéo CD và RS tứ giác cắt trung điểm đường Suy I là trung điểm CD là trung điểm RS Khi đó N I chính là đường trung tuyến tam giác N RS (2) Từ (1) và (2), suy trung tuyến N I tam giác N RS luôn qua O Vậy trung tuyến ứng với đỉnh N tam giác N RS luôn qua I là điểm cố định M di động khắp phía đường tròn (O) Đây chính là điều phải chứng minh Lời giải F N E B O A P Q M S I R Dễ thấy điểm cố định cần tìm chính là trung điểm O đoạn AB Gọi điểm đối xứng M qua N Q là E, điểm đối xứng M qua N P là F Vì N thuộc đường tròn đường kính nên ∠P N Q = ∠AN B = 90◦ , theo tính đối xứng qua đường thẳng nên ta ∠EN M + ∠F N M = 2(∠P N M + ∠QN M ) = 2∠P N Q = · 90◦ = 180◦ (94) Nguồn gốc bài toán Hình học số đề thi Việt Nam TST 2009 93 Suy E, N, F thẳng hàng Hơn N E = N M = N F nên N là trung điểm EF Do N là trung điểm cung AB nên SN là phân giác ∠ASB, đó ∠M SA = 2∠M SN = 2∠M P N = ∠M P E Tứ giác M SP E nội tiếp nên ∠M P E = ∠M SE, suy ∠M SA = ∠M SE; đó S, A, E thẳng hàng Tương tự, R, B, F thẳng hàng Ta có AS SN P A SN SA + SN P A SN SA + SN P A SN SP SN SA = = = = = AE SESA SEP A SESA + SEP A SESA + SEP A SN EP Tương tự, SN RQ BR Nhân tương ứng hai đẳng thức này lại, ta có = BF SN F Q SN SP SN F Q AS BF · = · SN EP SN RQ AE BR (1) Hai tam giác 4N SP, 4N RQ là tam giác vuông, đồng thời 1 ∠P N S = ∠P M S = ∠AM S = ∠BM R = ∠QM R = ∠QN R, 2 nên chúng đồng dạng, suy SN SP = SN RQ  NP NQ 2 Mặt khác, hai tam giác 4N QF, 4N P E vuông nên SN QF FN · FQ FQ MQ = = = SN P E EN · EP EP MP Nhân tương ứng hai đẳng thức này lại, ta có     SN SP SN F Q NP FQ SN SP SN F Q NP FQ · = · hay · = · SN RQ SN P E NQ EP SN P E SN RQ NQ EP (2) So sánh (1) và (2), ta có AS BF · = AE BR  NP NQ 2 · MQ MP Ta thấy tam giác N P Q vuông N có đường cao N M nên 4N M P ∼ 4QM N, suy   N P SN M P MN · MP MP = = = NQ SQM N MN · MQ MQ Từ đây kết hợp với trên, ta AS BF · = AE BR suy  NP NQ 2 · MQ MP MQ = · = 1, MP MQ MP −→ −→ −−→ −−→ BR AS = Suy tồn số thực k cho AS = k AE, BR = k BF Do đó AE BF −→ −→ −−→ k −→ −−→ −−→ OI = AS + BR = AE + BF = k ON , 2 hay O, N, I thẳng hàng Vậy trung tuyến N I tam giác N RS qua O là điểm cố định Ta có điều phải chứng minh (95) 94 Chuyên đề Toán học số Lời giải N K E F A O P B Q M S I R Gọi I là trung điểm RS, F là giao điểm đường thẳng M A với (P M N ), E là giao điểm đường thẳng M B với (QM N ) Ta chứng minh AB k EF Điều này tương đương với việc chứng minh SQM N MA SP M N SP M N SP M N SP SN MB SP M N = = = hay = = hay AF BE SP F N SP SN SQEN SN RQ SQM N SQRN (1) Xét hai tam giác vuông P N S và QN R có ∠P N S = ∠P M S = ∠QM R = ∠QN R nên chúng   SP N S PN đồng dạng, suy = ; đồng thời, hai tam giác P M N và QM N có chung SQN R QN SP M N MP đường cao N M nên = Từ đó suy (1) tương đương với SQM N MQ MP = MQ  PN QN 2 hay PM · PQ PN2 = QM · QP QN Điều này đúng tính chất phương tích các điểm, từ đó suy AB k EF Gọi K là giao điểm EF với AN thì ∠KF N = ∠KSN Ta có ∠AKF = ∠N KE = ∠N AO = ∠AN O, mà ∠AKF = ∠AKS nên ∠AKS = ∠AN O, suy KS k ON Do đó ∠ON S = ∠KSN = ∠N F K = ∠N F E Tương tự ∠ON R = ∠OEF Từ đó, ta sin ∠IN R NS NF sin ∠N EF sin ∠ON R = = = = sin ∠IN S NR NE sin ∠N F E sin ∠ON S Dễ thấy các tia N R, N S nằm các tia N P, N Q nên ∠RN S < ∠P N Q = 90◦ , suy góc sin ∠T N R ∠RN S các góc ∠SN I, ∠RN I nhọn và tỉ số xác định cách sin ∠T N S (96) Nguồn gốc bài toán Hình học số đề thi Việt Nam TST 2009 95 với tia N T nào đó nằm N R, N S Suy hai tia N O, N I trùng hay N, O, I thẳng hàng Vậy trung tuyến N I tam giác N RS luôn qua điểm O cố định Ta có điều phải chứng minh Như đã nêu trên, phương pháp tọa độ, chúng ta có thể chứng minh bài toán đúng với N là điểm trên phân giác góc AM B Các bạn thử giải bài toán sau phương pháp tọa độ lẫn phương pháp hình học phẳng túy: “Trên mặt phẳng cho đoạn thẳng AB cố định và M là điểm không nằm trên đường thẳng AB Gọi N là điểm trên phân giác góc AM B Đường phân giác ngoài góc AM B cắt các đường thẳng NA, N B P, Q Đường thẳng M A cắt đường tròn đường kính N Q R, đường thẳng M B cắt đường tròn đường kính N P S và R, S khác M Chứng minh đường trung tuyến ứng với đỉnh N tam giác N RS luôn qua điểm cố định với vị trí M và N.” Ta thấy xuất phát từ bài toán quen thuộc, đơn giản, việc chọn hướng khác quá trình xây dựng các điểm, ta có thể phát thêm nhiều bài toán thú vị và hấp dẫn Học hỏi các cách khác nhau, ý tưởng mẻ việc giải các bài toán đó là điều vô cùng bổ ích Các bạn hãy thử thực điều này để thu cho mình nhiều kinh nghiệm nhằm củng cố, rèn luyện thêm môn hình học phẳng này (97) 96 d Chuyên đề Toán học số (98) NHỎ MÀ KHÔNG NHỎ Võ Quốc Bá Cẩn SV Đại học Y Dược Cần Thơ Tóm tắt Trong bài này chúng ta cùng bàn bổ đề “nhỏ”, ứng dụng nó thì lại không “nhỏ” chút nào Đó là công cụ hỗ trợ đắc lực giúp làm tăng thêm tính hiệu bất đẳng thức Cauchy-Schwarz giải toán Giới thiệu Toán học luôn ẩn chứa nhiều điều thú vị và bất ngờ đằng sau nó Một bài toán cũ, mặc dù ta đã giải giải lại nhiều lần, lần xem lại, nó lại gợi cho ta nhiều ý tưởng lạ Những kết thú vị đó đã khiến cho sinh viên ngành Y tôi không thể nào xao lãng với đam mê mình, quay lưng lại với Toán Và tôi tin có nhiều bạn nghĩ Trở lại với bài viết này, bài toán cũ mà tôi muốn đề cập là bài toán số 12 mà M Lascu đã đề nghị [3]: Bài toán Cho n là số nguyên không nhỏ 2, a là số thực dương Chứng minh rằng, các số thực x1 , x2 , , xn thỏa mãn đồng thời các điều kiện x1 + x2 + · · · + xn = a, thì ta có  2a xi ∈ 0, , n x21 + x22 + ··· + x2n a2 , ≤ n−1  ∀i = 1, 2, , n Một bài toán đơn giản và khá dễ giải Như nó có điểm gì đặc biệt gì đây? Thật ra, thời gian đầu, tiếp cận bài toán này tôi chẳng có ý tưởng thú vị nào từ nó Sau này, quá trình viết kỹ thuật “thêm – bớt” bất đẳng thức Cauchy-Schwarz sách [8], tôi nhớ nó Ý tưởng kỹ thuật thêm – bớt là thêm vào các phân thức (có mẫu số dương) bớt lượng thích hợp cho tử số phân thức nhận là số không âm càng nhỏ càng tốt, áp dụng bất đẳng thức Cauchy-Schwarz Như thì khả thành công cao Để nắm rõ tư tưởng kỹ thuật này, ta lấy ví dụ bài toán thi thử Đại học Bài toán Cho a, b, c là các số dương thỏa mãn a2 + b2 + c2 = Chứng minh 1 + + ≥ 2−a 2−b 2−c 97 (99) 98 Chuyên đề Toán học số Nếu sử dụng Cauchy-Schwarz trực tiếp thông thường, ta không thu kết mong muốn Thật vậy, theo bất đẳng thức Cauchy-Schwarz thì 1 (1 + + 1)2 + + ≥ = 2−a 2−b 2−c (2 − a) + (2 − b) + (2 − c) − (a + b + c) Thế 9 p ≤ = − (a + b + c) − 3(a + b2 + c2 ) Do đó không thể sử dụng đánh giá trên để giải bài này > 2−a vế trái bất đẳng thức đã cho trừ cho Bây giờ, ta quan sát và để ý , ∀a > Như vậy, lấy phân thức thì ta các phân thức dương, và lúc đó bất đẳng thức có thể viết lại thành a b c + + ≥ 2−a 2−b 2−c Một điều đặc biệt là đây sử dụng bất đẳng thức Cauchy-Schwarz theo “bản năng” thông thường, ta lại đến kết b c a2 b2 c2 a + + = + + 2−a 2−b 2−c 2a − a2 2b − b2 2c − c2 (a + b + c)2 ≥ (2a − a2 ) + (2b − b2 ) + (2c − c2 ) (a + b + c)2 (a + b + c − 3)2 = =3+ ≥ 2(a + b + c) − 2(a + b + c) − Đó chính là tư tưởng kỹ thuật thêm – bớt Một tư tưởng đơn giản mà hiệu Giống đứa trẻ vui mừng hớn hở người lớn cho quà bánh, tôi vô cùng hứng khởi bắt gặp kỹ thuật thú vị này Tôi bắt đầu thử giải lại các bài toán mà tôi đã giải trước đây ý tưởng này để kiểm tra xem nó “mạnh” cỡ nào Và tôi đã thất bại trước bài toán, đó là bài toán (xem [4], bài số 30, trang 154) Bài toán Cho a, b, c là các số thực dương thỏa mãn a + b + c = Chứng minh   1 + + + ≥ 10(a2 + b2 + c2 ) a b c   1 Oái oăm chỗ, bất đẳng thức này có dấu (a, b, c) = 2, , , và kỹ thuật 2 trên không hiệu với loại đẳng thức “kỳ lạ” này (100) Nhỏ mà không nhỏ 99 Trong lúc “bí”, chẳng biết tình cờ hay vô ý, tôi lại lật lại bài toán ngày nào Tôi nhìn nó ngẫm nghĩ hồi và phát ý tưởng Trước hết là tôi nhận nguyên nhân vì mình làm không ra, việc trừ phân thức cho số cụ thể làm ta khó điều chỉnh gặp phải bài toán có nhiều dấu đẳng thức có thuộc loại lạ (giống bài này) “Tại không thử thêm và bớt cho biểu thức chứa biến nhỉ? Như dễ điều chỉnh hơn!” – Tôi nghĩ “Nếu thì ta cần tiến hành đánh giá cho biến.” Bài toán đánh giá cho biến, đó tôi nghĩ: Tại lại không thử đánh giá tương tự? Sau thời gian nghiền ngẫm, tôi tìm bổ đề sau với cách đánh giá tương tự Bổ đề Cho x1 , x2 , , xn là các số thực thỏa mãn x1 + x2 + · · · + xn = n Đặt x21 + x22 + · · · + x2n = n + n(n − 1)t2 , với t ≥ 0.1 Khi đó, với i = 1, 2, , n, ta có − (n − 1)t ≤ xi ≤ + (n − 1)t Chứng minh Ta chứng minh khẳng định đúng cho i = Các trường hợp khác chứng minh tương tự Sử dụng bất đẳng thức Cauchy-Schwarz, ta có x22 + x23 + ··· + x2n (n − x1 )2 (x2 + x3 + · · · + xn )2 = ≥ n−1 n−1 Do x21 (n − x1 )2 + ≤ n + n(n − 1)t2 n−1 Giải bất phương trình bậc hai với biến x1 , ta có − (n − 1)t ≤ x1 ≤ + (n − 1)t Bổ đề chứng minh Với kết vừa tìm này, tôi đã giải thành công bài toán công cụ đơn giản là bất đẳng thức Cauchy-Schwarz Lời giải bài toán Do a, b, c > và a+b+c = nên có thể đặt a2 +b2 +c2 = 3+6t2 với ≤ t < Khi đó theo bổ đề 1, ta có − 2t ≤ a, b, c ≤ + 2t 1 ≥ , và ta nghĩ đến việc biến đổi bất đẳng thức lại sau a + 2t        1 1 1 − + − + − + + ≥ 30(1 + 2t2 ), a + 2t b + 2t c + 2t + 2t Từ đây suy Ta có thể đặt vầy là vì x21 + x22 + · · · + x2n ≥ n Ngoài ra, có thể thấy ≤ t ≤ xi là các số không âm vì trường hợp này ta có x21 + x22 + · · · + x2n ≤ (x1 + x2 + · · · + xn )2 = n2 (101) 100 Chuyên đề Toán học số + 2t   + 2t − a + 2t − b + 2t − c + + + + ≥ 30(1 + 2t2 ) a b c Nếu t = thì a2 + b2 + c2 = 3, suy a = b = c = và bất đẳng thức hiển nhiên đúng Vì ta cần xét < t < là đủ (bạn đọc hiểu rõ vì lại phải xét t = đây) Sử dụng bất đẳng thức Cauchy-Schwarz, ta + 2t − a + 2t − b + 2t − c + + ≥ a b c [(1 + 2t − a) + (1 + 2t − b) + (1 + 2t − c)]2 ≥ a(1 + 2t − a) + b(1 + 2t − b) + c(1 + 2t − c) 36t2 36t2 6t = = = 2 3(1 + 2t) − (3 + 6t ) 6t − 6t 1−t (∗) Và thế, ta đưa bài toán chứng minh bất đẳng thức biến   6t + + ≥ 30(1 + 2t2 ) + 2t − t Thực biến đổi tương đương, ta 8(1 + t) + ≥ 10(1 + 2t2 ), (1 + 2t)(1 − t) 8(1 + t) ≥ 20t2 + 7, (1 + 2t)(1 − t) + 8t ≥ (7 + 20t2 )(1 + t − 2t2 ), + 8t ≥ + 7t + 6t2 + 20t3 − 40t4 , 40t4 − 20t3 − 6t2 + t + ≥ 0, (10t2 + 5t + 1)(2t − 1)2 ≥ Bất đẳng thức cuối hiển nhiên đúng nên ta có điều phải chứng minh Nhận xét Có thể thấy đánh giá bổ đề là đánh giá chặt vì ta cần có n − biến thì đẳng thức xảy Điều này phù hợp với đặc điểm phần lớn các bất đẳng thức đối xứng, đó là đẳng thức xảy có n − biến Như vậy, biết sử dụng bổ đề này cách thích hợp, chúng ta có thể giải nhiều bài toán khó Và kỹ thuật thêm – bớt bất đẳng thức Cauchy-Schwarz là công cụ phối hợp hiệu với bổ đề Ta lấy đánh giá (∗) để giải thích nguyên nhân vì Chúng ta đã biết bất đẳng thức Cauchy-Schwarz (a21 + a22 + · · · + a2n )(b21 + b22 + · · · + b2n ) ≥ (a1 b1 + a2 b2 + · · · + an bn )2 (102) Nhỏ mà không nhỏ 101 có dấu đẳng thức xảy và a1 a2 an = = ··· = , b1 b2 bn đây hiểu theo nghĩa mẫu thì tử Như vậy, (∗), chúng ta có đẳng thức (1 + 2t − b)2 (1 + 2t − c)2 (1 + 2t − a)2 = = a(1 + 2t − a) b(1 + 2t − b) c(1 + 2t − c) Ta biết rằng, mấu chốt quan trọng để biết việc sử dụng đánh giá để giải bài toán bất đẳng thức có đạt hiệu suất cao hay không là chỗ: Liệu đánh giá đó có đảm bảo dấu đẳng thức bất đẳng thức ban đầu hay không? Do ta thử kiểm tra xem điểm đẳng thức bất đẳng thức ban đầu có thỏa mãn điều kiện trên hay không Bất đẳng thức ban đầu có đẳng thức a = 2, b = c = Với giá trị này ta có t = và + 2t − a = 0, suy mẫu và tử phân thức thứ nên ta loại nó đi, và điều kiện để (∗) xảy đẳng thức là (1 + 2t − b)2 (1 + 2t − c)2 = b(1 + 2t − b) c(1 + 2t − c) Nhưng điều này là hiển nhiên, vì b = c Tóm lại, có thể thấy việc sử dụng bổ đề kết hợp với kỹ thuật thêm – bớt bất đẳng thức Cauchy-Schwarz đã giữ nguyên điểm đẳng thức bất đẳng thức ban đầu Chính điều này đã làm tăng thêm tính hiệu kỹ thuật Các bài toán ứng dụng Phần trên tôi đã giới thiệu cùng các bạn kỹ thuật thêm – bớt bất đẳng thức Cauchy-Schwarz và bổ đề 1, cùng với phối hợp hiệu chúng Dưới đây chúng ta cùng sử dụng kỹ thuật phối hợp này để giải lại số bài toán khó Bài toán Cho a, b, c là các số thực không âm thỏa mãn ab + bc + ca > Chứng minh bất đẳng thức sau b c 3(a2 + b2 + c2 ) a + + ≥ + b+c c+a a+b (a + b + c)2 Lời giải Bất đẳng thức có tính ba biến a, b, c, vì không giảm tính tổng quát, ta có thể giả sử a + b + c = Đặt a2 + b2 + c2 = + 6t2 với ≤ t < Ta cần chứng minh a b c 3(3 + 6t2 ) + + ≥ + 3−a 3−b 3−c 32 (103) 102 Chuyên đề Toán học số Bất đẳng thức này tương đương với       b c a +1 + +1 + + ≥ + 2t2 , 3−a 3−b 3−c 3 + + ≥ + 2t2 , 3−a 3−b 3−c 2 + + ≥ + t2 3−a 3−b 3−c Nếu t = thì a = b = c = 1, bất đẳng thức trở thành đẳng thức Vì ta cần xét < t ≤ Theo bổ đề 1, ta có − 2t ≤ a, b, c ≤ + 2t, suy 1 ≥ = , 3−a − (1 − 2t) 2(1 + t) và ta nghĩ đến việc biến đổi bất đẳng thức sau       1 2 − − − ≥ + t2 , + + + 3−a 1+t 3−b 1+t 3−c 1+t 1+t   a + 2t − b + 2t − c + 2t − 3t + + + t2 ≥ 1+t 3−a 3−b 3−c 1+t Sử dụng bất đẳng thức Cauchy-Schwarz, ta có a + 2t − b + 2t − c + 2t − + + ≥ 3−a 3−b 3−c [(a + 2t − 1) + (b + 2t − 1) + (c + 2t − 1)]2 ≥ (3 − a)(a + 2t − 1) + (3 − b)(b + 2t − 1) + (3 − c)(c + 2t − 1) 36t2 = 9(2t − 1) + (4 − 2t)(a + b + c) − (a2 + b2 + c2 ) 36t2 6t = = 9(2t − 1) + 3(4 − 2t) − (3 + 6t2 ) 2−t Và bài toán đưa chứng minh 6t 3t ≥ + t2 (1 + t)(2 − t) 1+t Thực biến đổi và rút gọn, ta   3t − ≥ t2 , 1+t 2−t 3t2 ≥ t2 , (1 + t)(2 − t) ≥ 4(1 + t)(2 − t) (104) Nhỏ mà không nhỏ 103 Bất đẳng thức cuối cùng đúng vì theo AM-GM, ta có 4(1 + t)(2 − t) ≤ [(1 + t) + (2 − t)]2 = Bài toán chứng minh xong Đẳng thức xảy và a = b = c, a = b, c = (và các hoán vị tương ứng) Bài toán Chứng minh với a, b, c dương, ta có 8abc a2 + b + c + ≥ ab + bc + ca (a + b)(b + c)(c + a) Lời giải Trong trường hợp (a − b)2 + (b − c)2 + (c − a)2 = 0, ta có đẳng thức xảy Vì cần xét (a − b)2 + (b − c)2 + (c − a)2 > là đủ Không tính tổng quát, giả sử a + b + c = Đặt a2 + b2 + c2 = + 6t2 , với < t < Khi đó (a + b + c)2 − (a2 + b2 + c2 ) − (3 + 6t2 ) = = 3(1 − t2 ) 2 Bất đẳng thức cần chứng minh trở thành ab + bc + ca = + 6t2 8abc + ≥ 2, 3(1 − t ) (a + b)(b + c)(c + a) + 2t2 8abc + ≥ 2, − t2 (a + b)(b + c)(c + a) 8abc − 4t2 ≥ (a + b)(b + c)(c + a) − t2 1 − 4t2 thì ≤ nên bất đẳng thức hiển nhiên đúng Xét trường hợp < t < 2 1−t 1 Ta tìm cách đánh giá biểu thức A = + + Theo bổ đề 1, ta có a b c < − 2t ≤ a, b, c ≤ + 2t, 1 suy ≤ , và ta nghĩ đến việc biến đổi biểu thức A sau a − 2t       1 1 1 A= − + − + − + a − 2t b − 2t c − 2t − 2t   a + 2t − b + 2t − c + 2t − =− + + + − 2t a b c − 2t Nếu t ≥ Sử dụng bất đẳng thức Cauchy-Schwarz, ta có a + 2t − b + 2t − c + 2t − + + ≥ a b c [(a + 2t − 1) + (b + 2t − 1) + (c + 2t − 1)]2 ≥ a(a + 2t − 1) + b(b + 2t − 1) + c(c + 2t − 1) 36t2 = (a + b2 + c2 ) + (2t − 1)(a + b + c) 36t2 6t = = (3 + 6t ) + 3(2t − 1) t+1 (105) 104 Chuyên đề Toán học số Từ đó suy A≤− 6t 3(1 − t) + = (1 − 2t)(1 + t) − 2t (1 − 2t)(1 + t) Sử dụng đánh giá này, ta (a + b + c)(ab + bc + ca) − abc (a + b)(b + c)(c + a) = abc   abc 1 + + −1 = (a + b + c) a b c 3(1 − t) 2(2 − t)2 ≤3· −1= , (1 − 2t)(1 + t) (1 − 2t)(1 + t) suy 8abc 4(1 − 2t)(1 + t) ≥ (a + b)(b + c)(c + a) (2 − t)2 Vậy ta cần chứng minh − 4t2 4(1 − 2t)(1 + t) ≥ (2 − t)2 − t2 Do − 2t > nên bất đẳng thức này tương đương với 4(1 + t) + 2t ≥ , (2 − t) − t2 4(1 + t)2 + 2t − ≥ − 1, (2 − t)2 1−t 3t 3t(4 + t) ≥ , (2 − t) 1−t (4 + t)(1 − t) ≥ (2 − t)2 , − 3t − t2 ≥ − 4t + t2 , t(1 − 2t) ≥ nên bất đẳng thức cuối hiển nhiên đúng Bài toán chứng minh xong Đẳng thức xảy và a = b = c Vì < t < Bài toán Cho a, b, c, d, e là các số thực dương thỏa mãn a + b + c + d + e = Chứng minh 1 1 20 + + + + + ≥ a b c d e a + b + c2 + d2 + e2 (106) Nhỏ mà không nhỏ 105 Lời giải Đặt a2 + b2 + c2 + d2 + e2 = + 20t2 , với ≤ t < Ta cần chứng minh 1 1 20 + + + + + ≥ 9, a b c d e + 20t2 hay 1 1 + + + + + ≥ a b c d e + 4t2 Bất đẳng thức này hiển nhiên đúng với t = (vì đó a = b = c = d = e = 1, bất đẳng thức trở thành đẳng thức) nên ta cần xét < t < Theo bổ đề thì − 4t ≤ a, b, c, d, e ≤ + 4t, suy 1 ≥ , và ta nghĩ đến việc biến đổi bất đẳng thức sau a + 4t  X 1 − + + ≥ 9, a + 4t + 4t + 4t2 X + 4t − a + + ≥ + 4t a + 4t + 4t2 Sử dụng bất đẳng thức Cauchy-Schwarz, ta hX i2 (1 + 4t − a) X + 4t − a 202 t2 X X ≥ X = a (1 + 4t) a− a2 a(1 + 4t − a) = 202 t2 20t = 5(1 + 4t) − (5 + 20t ) 1−t Do đó cần chứng minh 20t + + ≥ (1 − t)(1 + 4t) + 4t + 4t2 Bất đẳng thức này tương đương với 20t + (1 − t)(1 + 4t)   −5 ≥4− , + 4t + 4t2 20t 20t 16t2 − ≥ , (1 − t)(1 + 4t) + 4t + 4t2 20t2 16t2 ≥ , (1 − t)(1 + 4t) + 4t2 5(1 + 4t2 ) ≥ 4(1 − t)(1 + 4t) Ta có 5(1 + 4t2 ) − 4(1 − t)(1 + 4t) = + 20t2 − 4(1 + 3t − 4t2 ) = 36t2 − 12t + = (6t − 1)2 ≥ 0, (107) 106 Chuyên đề Toán học số nên bất đẳng thức cuối hiển nhiên đúng Phép chứng minh hoàn tất Đẳng thức xảy và a = b = c = d = e = 1, 5 a = b = c = d = , e = (và các hoán vị tương ứng) Bài toán Cho x1 , x2 , , xn là các số thực không âm thỏa x1 + x2 + · · · + xn = n Chứng minh x31 + x32 + · · · + x3n + n2 ≤ (n + 1)(x21 + x22 + · · · + x2n ) Lời giải Đặt x21 + x22 + · · · + x2n = n + n(n − 1)t2 với ≤ t ≤ Ta phải chứng minh   x31 + x32 + · · · + x3n + n2 ≤ (n + 1) n + n(n − 1)t2 , hay x31 + x32 + · · · + x3n ≤ n + n(n2 − 1)t2 Nếu t = thì x21 + x22 + · · · + x2n = n, suy x1 = x2 = · · · = xn = 1, bất đẳng thức trở thành đẳng thức Vì cần xét < t ≤ Theo bổ đề 1, ta có − (n − 1)t ≤ xi ≤ + (n − 1)t, ∀i = 1, 2, , n, suy x3i ≤ [1 + (n − 1)t] x2i , và ta nghĩ đến việc biến đổi bất đẳng thức sau n X i=1 − n X x3i − [1 + (n − 1)t] n X x2i + [1 + (n − 1)t] i=1 n X x2i ≤ n + n(n2 − 1)t2 , i=1   x2i [1 + (n − 1)t − xi ] + [1 + (n − 1)t] n + n(n − 1)t2 ≤ n + n(n2 − 1)t2 i=1 Sử dụng bất đẳng thức Cauchy-Schwarz, ta ( n )2 X xi [1 + (n − 1)t − xi ] n X i=1 x2i [1 + (n − 1)t − xi ] ≥ n X i=1 [1 + (n − 1)t − xi ] i=1 ( [1 + (n − 1)t] = n X i=1 xi − n X )2 x2i i=1 n(n − 1)t {n [1 + (n − 1)t] − [n + n(n − 1)t2 ]}2 = n(n − 1)t = n(n − 1)t(1 − t)2 Do đó bài toán đưa chứng minh   −n(n − 1)t(1 − t)2 + [1 + (n − 1)t] n + n(n − 1)t2 ≤ n + n(n2 − 1)t2 (108) 107 Nhỏ mà không nhỏ Thực biến đổi và rút gọn, ta −n(n − 1)t(1 − t)2 + n + n(n − 1)t + n(n − 1)t2 [1 + (n − 1)t] ≤ n + n(n2 − 1)t2 , −n(n − 1)t(1 − t)2 + n(n − 1)t + n(n − 1)t2 [1 + (n − 1)t] ≤ n(n2 − 1)t2 , −(1 − t)2 + + t [1 + (n − 1)t] ≤ (n + 1)t, 2t − t2 + t + (n − 1)t2 ≤ (n + 1)t, (n − 2)t(1 − t) ≥ Do < t ≤ và n ≥ nên bất đẳng thức cuối hiển nhiên đúng Bài toán chứng minh xong Ta có đẳng thức xảy n = 2, x1 = x2 = · · · = xn = x1 = x2 = · · · = xn−1 = 0, xn = n (và các hoán vị tương ứng) n > Bài toán Cho x1 , x2 , , xn là các số thực không âm thỏa x1 + x2 + · · · + xn = n Chứng minh   1 n + + ··· + ≥ n(n − 2)2 + 4(n − 1)(x21 + x22 + · · · + x2n ) x1 x2 xn Lời giải Đặt x21 + x22 + · · · + x2n = n + n(n − 1)t2 với ≤ t ≤ Ta phải chứng minh     1 n + + ··· + ≥ n(n − 2)2 + 4(n − 1) n + n(n − 1)t2 , x1 x2 xn hay 4(n − 1)2 1 (n − 2)2 + 4(n − 1) 4(n − 1)2 + t =n+ t + + ··· + ≥ x1 x2 xn n n n Nếu t = thì x21 + x22 + · · · + x2n = n, suy x1 = x2 = · · · = xn = 1, bất đẳng thức trở thành đẳng thức Vì cần xét < t ≤ Theo bổ đề 1, ta có − (n − 1)t ≤ xi ≤ + (n − 1)t, suy ∀i = 1, 2, , n, 1 ≥ , và ta nghĩ đến việc biến đổi bài toán sau xi + (n − 1)t  n  X 1 n 4(n − 1)2 − + ≥n+ t, xi + (n − 1)t + (n − 1)t n i=1 n X + (n − 1)t − xi n 4(n − 1)2 + ≥n+ t + (n − 1)t i=1 xi + (n − 1)t n Áp dụng bất đẳng thức Cauchy-Schwarz, ta có ( n )2 X [1 + (n − 1)t − xi ] n X + (n − 1)t − xi n2 (n − 1)2 t2 ≥ i=1 = n n n X X X xi i=1 xi [1 + (n − 1)t − xi ] [1 + (n − 1)t] xi − x2i i=1 i=1 = 2 n (n − 1) t n(n − 1)t = n [1 + (n − 1)t] − [n + n(n − 1)t ] 1−t i=1 (109) 108 Chuyên đề Toán học số Từ đó bài toán đưa chứng minh n(n − 1)t n 4(n − 1)2 + ≥n+ t (1 − t) [1 + (n − 1)t] + (n − 1)t n Bất đẳng thức này tương đương với   n(n − 1)t n 4(n − 1)2 ≥ n− + t, (1 − t) [1 + (n − 1)t] + (n − 1)t n n(n − 1)t n(n − 1)t 4(n − 1)2 ≥ + t, (1 − t) [1 + (n − 1)t] + (n − 1)t n n(n − 1)t2 4(n − 1)2 ≥ t, (1 − t) [1 + (n − 1)t] n n2 ≥ 4(n − 1)(1 − t) [1 + (n − 1)t] Theo bất đẳng thức AM-GM, ta có 4(n − 1)(1 − t) [1 + (n − 1)t] ≤ {(n − 1)(1 − t) + [1 + (n − 1)t]}2 = n2 , nên bất đẳng thức cuối hiển nhiên đúng Phép chứng minh hoàn tất Đẳng thức xảy và x1 = x2 = · · · = xn = 1, n n , xn = (và các hoán vị tương ứng) x1 = x2 = · · · = xn−1 = 2(n − 1) Bài toán Cho x1 , x2 , , xn là các số thực không âm thỏa x1 + x2 + · · · + xn = n Chứng minh (n − 1)(x31 + x32 + · · · + x3n ) + n2 ≥ (2n − 1)(x21 + x22 + · · · + x2n ) Lời giải Đặt x21 + x22 + · · · + x2n = n + n(n − 1)t2 với ≤ t ≤ Ta phải chứng minh   (n − 1)(x31 + x32 + · · · + x3n ) + n2 ≥ (2n − 1) n + n(n − 1)t2 , hay x31 + x32 + · · · + x3n ≥ n(2n − 1) − n2 (2n − 1)n(n − 1) + t = n + n(2n − 1)t2 n−1 n−1 Nếu t = thì x1 = x2 = · · · = xn = nên bất đẳng thức trở thành đẳng thức Vì cần xét < t ≤ Lúc này, ta có hai trường hợp xảy • Trường hợp t ≥ Sử dụng bất đẳng thức Cauchy-Schwarz, ta có n−1 x31 + x32 + · · · + x3n ≥  2 (x21 + x22 + · · · + x2n )2 = n + (n − 1)t2 x1 + x + · · · + x n (110) 109 Nhỏ mà không nhỏ Suy ta cần chứng minh  2 n + (n − 1)t2 ≥ n + n(2n − 1)t2 Bất đẳng thức này tương đương với 2  + (n − 1)t2 ≥ + (2n − 1)t2 , 2(n − 1)t2 + (n − 1)2 t4 ≥ (2n − 1)t2 ,   t2 (n − 1)2 t2 − ≥ 0, hiển nhiên đúng t ≥ • Trường hợp < t < n−1 Sử dụng bổ đề 1, ta có n−1 < − (n − 1)t ≤ xi ≤ + (n − 1)t, ∀i = 1, 2, , n, suy x3i ≥ [1 − (n − 1)t] x2i , và ta nghĩ đến việc biến đổi bất đẳng thức sau n X i=1 n X x3i − [1 − (n − 1)t] n X x2i + [1 − (n − 1)t] i=1 n X x2i ≥ n + n(2n − 1)t2 , i=1   x2i [xi + (n − 1)t − 1] + [1 − (n − 1)t] n + n(n − 1)t2 ≥ n + n(2n − 1)t2 i=1 Sử dụng bất đẳng thức Cauchy-Schwarz, ta ( n )2 X xi [xi + (n − 1)t − 1] n X i=1 x2i [xi + (n − 1)t − 1] ≥ n X i=1 [xi + (n − 1)t − 1] i=1 ( n X x2i + [(n − 1)t − 1] i=1 n X )2 i=1 n(n − 1)t {[n + n(n − 1)t2 ] + n [(n − 1)t − 1]}2 = n(n − 1)t = n(n − 1)t(1 + t)2 Do đó ta cần chứng minh   n(n − 1)t(1 + t)2 + [1 − (n − 1)t] n + n(n − 1)t2 ≥ n + n(2n − 1)t2 Thực khai triển và rút gọn, ta n(n − 1)t(1 + t)2 + n − n(n − 1)t + n(n − 1)t2 [1 − (n − 1)t] ≥ n + n(2n − 1)t2 , (111) 110 Chuyên đề Toán học số n(n − 1)t(1 + t)2 − n(n − 1)t + n(n − 1)t2 [1 − (n − 1)t] ≥ n(2n − 1)t2 , (n − 1)(1 + 2t + t2 ) − (n − 1) + (n − 1)t − (n − 1)2 t2 ≥ (2n − 1)t,   3(n − 1)t + (n − 1) − (n − 1)2 t2 ≥ (2n − 1)t, n − ≥ (n − 1)(n − 2)t Do t ≤ và n ≥ nên bất đẳng thức cuối hiển nhiên đúng n−1 Phép chứng minh hoàn tất Đẳng thức xảy và x1 = x2 = · · · = xn = 1, n , xn = (và các hoán vị tương ứng) x1 = x2 = · · · = xn = n−1 Lời bàn Qua các ví dụ ứng dụng trên, ta có thể thấy kỹ thuật phối hợp bổ đề và kỹ thuật thêm – bớt bất đẳng thức Cauchy-Schwarz là kỹ thuật mạnh và hiệu Tuy vậy, kỹ thuật này còn nhiều điểm hạn chế cần khắc phục Nó khó áp dụng cho các bất đẳng thức với điều kiện dạng tích, các bất đẳng thức chứa hay dạng phân thức phức tạp, Ví dụ bài toán sau Bài toán 10 Cho a, b, c là các số thực thỏa mãn a + b + c = Chứng minh 1 1 + + ≤ + 2b + 2c + 2a2 Tôi cố gắng cải tiến kỹ thuật này để nó trở nên hiệu Rất mong nhận trao đổi từ bạn bè yêu Toán gần xa Mọi góp ý xin gửi về: Võ Quốc Bá Cẩn, C65 khu dân cư Phú An, phường Phú Thứ, quận Cái Răng, thành phố Cần Thơ, qua hòm thư điện tử babylearnmath@yahoo.com Bài tập tự luyện Bài tập Cho a, b, c là các số thực dương thỏa a + b + c = Chứng minh b2 c2 a2 + + ≤ a+2 b+2 c+2 ab + bc + ca Bài tập Cho a, b, c là các số thực dương thỏa a + b + c = Chứng minh   1 12 + + ≥ 4(a3 + b3 + c3 ) + 21 a b c Bài tập Cho a, b, c là ba số thực dương tùy ý Chứng minh a b c 2(ab + bc + ca) + + + ≥ b c a a2 + b + c (112) Nhỏ mà không nhỏ 111 Bài tập Cho các số thực a, b, c thỏa mãn a + b + c = và a2 + b2 + c2 = + 6t2 với t ≥ Chứng minh (1 − 2t)(1 + t)2 ≤ abc ≤ (1 + 2t)(1 − t)2 Bài tập Cho a, b, c là các số thực dương Tìm tập hợp tất các số thực k cho bất đẳng thức sau đúng a3 + b + c k(ab + bc + ca) k + + ≥ (a + b)(b + c)(c + a) (a + b + c)2 Bài tập Cho a, b, c là các số thực dương Chứng minh 1 + + ≥ b2 + bc + c2 c2 + ca + a2 a2 + ab + b2 (a + b + c)2 Bài tập Cho a, b, c là các số không âm thỏa mãn ab + bc + ca > Chứng minh b2 b c a+b+c a + + + ≥ 2 2 + bc + c c + ca + a a + ab + b a +b +c a+b+c Bài tập Cho a, b, c là các số không âm thỏa mãn ab + bc + ca > Chứng minh (a2 + bc)(b + c) (b2 + ca)(c + a) (c2 + ab)(a + b) + + ≥ (a + b + c) 2 2 2 b + bc + c c + ca + a a + ab + b Bài tập Cho a, b, c là các số thực thỏa mãn điều kiện (a + b + c)(ab + bc + ca) 6= Chứng minh bc ca ab 4(a2 + b2 + c2 ) + + + ≥ 2 2 2 b + bc + c c + ca + a a + ab + b (a + b + c) Bài tập 10 Cho các số thực không âm a, b, c, d thỏa mãn điều kiện a2 +b2 +c2 +d2 = Chứng minh a3 + b3 + c3 + d3 + ab + ac + ad + bc + bd + cd ≤ a + b + c + d Bài tập 11 Cho a, b, c, d, e là các số thực dương thỏa mãn a2 + b2 + c2 + d2 + e2 = Chứng minh 1 1 + + + + ≤ − 2a − 2b − 2c − 2d − 2e Bài tập 12 Cho x1 , x2 , , x10 là các số thực dương thỏa x1 + x2 + · · · + x10 = 10 Chứng minh 1 + + · · · + ≥ x21 + x22 + · · · + x210 x1 x2 x10 (113) 112 Chuyên đề Toán học số Bài tập 13 Cho x1 , x2 , , xn là các số thực không âm thỏa x1 + x2 + · · · + xn = n Chứng minh 2(x31 + x32 + · · · + x3n ) + n2 ≤ (2n + 1)(x21 + x22 + · · · + x2n ) Bài tập 14 Chứng minh rằng, a1 , a2 , , an là các số thực dương thỏa mãn a1 + a2 + · · · + an = n, thì √ √ 1 2n n − ≥ n + n − + + ··· + + 2 a1 a2 an a1 + a2 + · · · + an Tài liệu tham khảo [1] G H Hardy, J E Littlewood, G Pólya, Inequalities, Cambridge University Press, 1967 [2] J Michael Steele, The Cauchy-Schwarz Master Class, Cambridge University Press, 2004 [3] T Andreescu, V Cˆırtoaje, G Dospinescu, M Lascu, Old and New Inequalities, Volume 1, GIL Publishing House, 2004 [4] V Cˆırtoaje, Algebraic Inequalities: Old and New Methods, GIL Publishing House, 2006 [5] Phạm Kim Hùng, Sáng tạo Bất đẳng thức, Nhà xuất Tri Thức, 2006 [6] V Q B Can, C Pohoat¸ă, Old and New Inequalities, Volume 2, GIL Publishing House, 2008 [7] Võ Quốc Bá Cẩn, Trần Quốc Anh, Bất đẳng thức và lời giải hay, Nhà xuất Hà Nội, 2009 [8] Võ Quốc Bá Cẩn, Trần Quốc Anh, Sử dụng phương pháp Cauchy-Schwarz để giải toán Bất đẳng thức, Nhà xuất Đại học Sư phạm Hà Nội, 2010 [9] Một số tạp chí Toán học: • Crux Mathematicorum • American Mathematical Monthly • Mathematical Reflections [10] Các diễn đàn thảo luận Toán: • http://math.vn • http://mathlinks.ro • http://mathoverflow.net • http://mathscope.org (114) BẤT ĐẲNG THỨC BERNOULLI Trương Tấn Sang HS chuyên Toán khóa 2009 - 2012 Tóm tắt lý thuyết Bất đẳng thức Bernoulli phát biểu sau: Định lý (Bất đẳng thức Bernoulli) Với số thực x > −1 và số thực r > 1, ta có bất đẳng thức (1 + x)r ≥ + rx Đẳng thức xảy và x = Trong trường hợp < r < 1, ta có bất đẳng thức với chiều ngược lại, tức là (1 + x)r ≤ + rx Nếu lấy x + → x thì ta có Định lý Với x > và r > 1, ta có xr ≥ rx + − r Đẳng thức xảy và x = Bất đẳng thức này thường sử dụng bất đẳng thức gốc Nếu đặt r = a với a > b > và cho x → xb , ta có bất đẳng thức b Định lý Cho a > b là hai số thực dương Khi đó với x > 0, ta có a a xa ≥ xb + − b b Bất đẳng thức này sử dụng phổ biến vì đánh giá chênh lệch số mũ Điểm mạnh bất đẳng thức Bernoulli là đánh giá các bất đẳng thức với số mũ vô tỉ và bất đẳng thức không đồng 2.1 Các dạng toán đặc trưng Kỹ thuật đánh giá qua chênh lệch lũy thừa Các dạng toán đặc trựng bất đẳng thức Bernoulli dễ nhận ra, vì đó là bất đẳng thức với số mũ vô tỉ dương hay chuyển đổi số mũ vô tỉ Để mở đầu chuyên đề, ta chứng minh bất đẳng thức sau 113 (115) 114 Chuyên đề Toán học số Ví dụ Cho x1 , x2 , , xk là các số thực dương Chứng minh với m > n > 0, ta có bất đẳng thức  m m xm + x2 + · · · + xk n 1  m ≥ xn1 + xn2 + · · · + xnk n 1 n Lời giải Ý tưởng chính việc sử dụng bất đẳng thức Bernoulli là thể chênh lệch số mũ bé với số mũ lớn Để thực điều này, ta chuẩn hóa xn1 + xn2 + · · · + xnk = n Khi đó, ta cần chứng minh m m xm + x2 + · · · + xk ≥ n Áp dụng bất đẳng thức Bernoulli, ta có m n n ≥ + xm i = [1 + (xi − 1)] m n (x − 1), n i Do đó m m xm + x2 + · · · + xk ≥ n + ∀i = 1, 2, , k m n (x + xn2 + · · · + xnk − n) n Với điều kiện chuẩn hóa xn1 + xn2 + · · · + xnk = n, ta có điều phải chứng minh Đẳng thức xảy và x1 = x2 = · · · = xk Để nắm rõ kỹ thuật này, ta lấy ví dụ với bất đẳng thức Nesbitt Ví dụ Chứng minh với số dương a, b, c ta có bất đẳng thức  a b+c  √3  + b c+a  √3  + c a+b √3 ≥ √ √ Lời giải Ta đánh giá số mũ thông qua số mũ bất đẳng thức Bernoulli, chú ý sử dụng các lượng đánh giá thích hợp để đẳng thức xảy    2a b+c 2b c+a 2c a+b √3 ≥ √3 ≥ √3 ≥ √ √ √    2a b+c  2b c+a  2c a+b  +1− +1− +1− √ 3, √ √ 3, Cộng ba bất đẳng thức trên lại và áp dụng bất đẳng thức Nesbitt, ta có điều phải chứng minh Đẳng thức xảy và a = b = c Ví dụ Cho tam giác ABC có độ dài các đường trung tuyến là ma , mb , mc và các phân giác la , lb , lc Chứng minh  ma la  √3  + mb lb √3  + mc lc √3 ≥ (116) 115 Bất đẳng thức Bernoulli Lời giải Ta biết rằng: Trong tam giác, độ dài đường trung tuyến luôn không nhỏ độ dài phân giác Do đó ma ≥ la , mb ≥ lb , mc ≥ lc , suy ma mb mc + ≥ + la lb lc √ Từ đây, thực đánh giá số mũ thông qua số mũ bất đẳng thức Bernoulli, ta dễ dàng suy điều phải chứng minh Ví dụ Cho A, B, C là ba góc tam giác Chứng minh với < k < 1, ta có √ !k B C A cosk + cosk + cosk ≤ 2 2 Lời giải Để ý đẳng thức xảy A = B = C = π và với k = là bất đẳng thức sở Do đó, ta đánh giá thông qua số mũ Áp dụng bất đẳng thức Bernoulli, ta có   A k A 2k √ cos ≤ √ cos + − k, 2 3   2k B k B √ cos ≤ √ cos + − k, 2 3  k C C 2k √ cos ≤ √ cos + − k 2 3 Cộng các bất đẳng thức trên lại theo vế và áp dụng bất đẳng thức sở √ A B C 3 cos + cos + cos ≤ , 2 2 ta có điều phải chứng minh Đẳng thức xảy và tam giác ABC Ta có bài toán tương tự Ví dụ Nếu A, B, C là ba góc tam giác, thì √ √ ! sin A + sin B + sin C ≤ √ √ √ Lời giải Dạng phát biểu bài toán gợi cho ta nhớ đến kết quen thuộc: sin2 A + sin2 B + sin2 C ≤ , ∀4ABC (1) Như vậy, ý tưởng ta là đánh giá thông qua số mũ Áp dụng bất đẳng thức Bernoulli, ta có  √ sin A 2 ≥√  √ sin A √2 √ +1− √ = 2  √ sin A √2 +1− √ 2, (117) 116 Chuyên đề Toán học số suy  √ sin A √2 ≤√  √ sin A 2 √ 2 1 sin2 A + − √ +1− √ = 2 Cộng bất đẳng thức này với hai bất đẳng thức tương tự và sử dụng (1), ta thu kết cần chứng minh Đẳng thức xảy và tam giác ABC 2.2 Kỹ thuật chọn điểm rơi Cũng các bất đẳng thức AM-GM, Cauchy-Schwarz, đôi đẳng thức xảy bất đẳng thức Bernoulli khá chênh lệch các biến Nhưng cần cần kỹ thuật nhỏ, ta có thể giải vấn đề này Ví dụ Cho a, b, c, k, r là các số dương, r > và x, y, z là các số dương thay đổi cho ax + by + cz = k Tìm giá trị nhỏ biểu thức P = xr + y r + z r Lời giải Giả sử đẳng thức xảy x =x 0, y = y, z = z0 Chú ý đẳng thức xảy bất đẳng thức Bernoulli biến số 1, nên ta đánh giá sau  r   x x ≥r + − r x0 x0 Nhân hai vế cho xr0 , ta thu xr ≥ rxxr−1 + (1 − r)xr−1 0 Xây dựng bất đẳng thức tương tự với hai biến còn lại cộng ba bất đẳng thức lại, suy + yy0r−1 + zz0r−1 ) + (1 − r)(xr0 + y0r + z0r ) P ≥ r(xxr−1 Do đó, ta chọn x0 , y0 , z0 > cho  r−1 y r−1 z r−1  x0 = = =λ a b c  ax0 + by0 + cz0 = k Giải hệ trên ta tìm   ka r−1   x =  r r r   a r−1 + b r−1 + c r−1     kb r−1 y0 = r r r  r−1 + b r−1 + c r−1  a      kc r−1    z0 = r r r a r−1 + b r−1 + c r−1 Thay x0 , y0 , z0 vào bất đẳng thức trên, ta tìm giá trị nhỏ P Ví dụ Cho a, b, c là các số thực dương thỏa mãn điều kiện a + 2b + c = Tìm giá trị nhỏ biểu thức √ √ √ P = 2a + b + 2c (118) 117 Bất đẳng thức Bernoulli √ √ √ √ √ √ Lời giải Bằng phép đổi biến x = 2a , y = b , z = 2c dạng: Cho x, y, z là các số thực dương thỏa mãn điều kiện x √1 √ + 2y z + √ 3, ta chuyển bài toán = Tìm giá trị nhỏ biểu thức √ P =x √ +y √ +z Đây chính là trường hợp riêng ví dụ trên Bất đẳng thức Bernoulli khảo sát tính tăng giảm hàm số Bất đẳng thức Bernoulli còn sử dụng để khảo sát tính tăng giảm hàm số có chất bất đẳng thức Ví dụ Chứng minh với a, b, c dương, hàm số sau tăng trên (0, +∞)  x  x  x b c a + + f (x) = bc ca ab Lời giải Để khảo sát hàm số bất đẳng thức Bernoulli đơn giản, cần xét bất đẳng thức dạng chênh lệch số mũ Lấy x1 > x2 > 0, ta chứng minh f (x1 ) ≥ f (x2 ) Áp dụng bất đẳng thức Bernoulli, dễ thấy  x1  x x1 a2 x1 a ≥ +1− , bc x2 bc x2  x1  x b x1 b2 x1 ≥ +1− , ca x2 ca x2  x1  x2 c x1 c x1 ≥ +1− ab x2 ab x2 Cộng ba bất đẳng thức trên lại theo vế, ta có   x1 x1 f (x1 ) ≥ f (x2 ) + − x2 x2 Như ta cần chứng minh   x1 x1 f (x2 ) + − ≥ f (x2 ) hay f (x2 ) ≥ x2 x2 s      a2 x2 b2 x2 c2 x2 = Bất đẳng thức này đúng vì theo AM-GM ta có f (x2 ) ≥ 3 bc ca ab Nhận xét Tổng quát hơn, ta có thể chứng minh kết sau: Cho a, b, c là các số thực dương Xét f (x) = ax + bx + cx Nếu f (x) ≥ 3, ∀x ≥ x0 > thì ta có f (x) tăng trên (0, +∞) (119) 118 Chuyên đề Toán học số Bài toán tổng quát cho n biến phát biểu tương tự Bạn đọc có thể tự kiểm tra lại kết này Sử dụng kết này, ta chứng minh các hàm số sau tăng trên (0, +∞) :  f (x) = bc a2 x +  và hàm số k(x) =  ca x 2a b+c b2 x ab b2 x 2b c+a x  +  +  , g(x) =  + 2c a+b b+c 2a x  + c+a 2b x  + a+b 2c x  tăng tăng trên x ,  , +∞ Bất đẳng thức Weierstrass Bất đẳng thức Weierstrass là dạng tương tự bất đẳng thức Bernoulli, nó phát biểu sau Định lý (Bất đẳng thức Weierstrass) Với số thực a1 , a2 , , an cùng dấu và lớn −1, ta có bất đẳng thức (1 + a1 )(1 + a2 ) · · · (1 + an ) ≥ + a1 + a2 + · · · + an Đây là bất đẳng thức có nhiều ứng dụng Nó có thể giúp ta đưa bất đẳng thức gồm tích nhiều thành phần phức tạp trở thành bất đẳng thức đơn giản và thuận tiện cho việc chứng minh Để rõ ứng dụng nó, ta xét số ví dụ Ví dụ Cho a, b, c là các số thực dương Chứng minh (a2 + 2)(b2 + 2)(c2 + 2) ≥ 3(a + b + c)2 Lời giải Để ý ba số a2 − 1, b2 − 1, c2 − luôn có ít hai số cùng dấu Giả sử hai số đó là a2 − và b2 − Khi đó, theo bất đẳng thức Weierstrass, ta có       a2 − b2 − 2 (a + 2)(b + 2) = + (a − 1) + (b − 1) = + 1+ 3   2 a −1 b −1 + = 3(a2 + b2 + 1) ≥9 1+ 3 2 Vậy ta còn phải chứng minh (a2 + b2 + 1)(c2 + 2) ≥ (a + b + c)2 Bất đẳng thức này hiển nhiên đúng theo bất đẳng thức Cauchy-Schwarz (a2 + b2 + 1)(1 + + c2 ) ≥ (a + b + c)2 Đẳng thức xảy và a = b = c = Ví dụ 10 Cho a, b, c là các số thực dương Chứng minh bất đẳng thức sau (a2 + 3)(b2 + 3)(c2 + 3) ≥ 4(a + b + c + 1)2 (120) Bất đẳng thức Bernoulli 119 Lời giải Tương tự ví dụ trên, ta giả sử a và b là hai số cho a2 − 1, b2 − có cùng dấu Khi đó, áp dụng bất đẳng thức Weierstrass, ta       a2 − b2 − 2 2 (a + 3)(b + 3) = + (a − 1) + (b − 1) = 16 + 1+ 4   2 a −1 b −1 = 4(a2 + b2 + 2) ≥ 16 + + 4 Bài toán quy chứng minh (a2 + b2 + 2)(c2 + 3) ≥ (a + b + c + 1)2 Và giống bất đẳng thức trên, ta thấy bất đẳng thức này hiển nhiên đúng theo bất đẳng thức Cauchy-Schwarz (a2 + b2 + + 1)(1 + + c2 + 1) ≥ (a + b + c + 1)2 Phép chứng minh hoàn tất Đẳng thức xảy và a = b = c = Ví dụ 11 Cho a, b, c, d là các số thực dương Chứng minh (a2 + 1)(b2 + 1)(c2 + 1)(d2 + 1) ≥ 16 (a + b + c + d)2 27 x y z t Lời giải Đặt a = √ , b = √ , c = √ , d = √ Khi đó bất đẳng thức có thể viết lại thành 3 3 (x2 + 3)(y + 3)(z + 3)(t2 + 3) ≥ 16(x + y + z + t)2 Bây giờ, ta có để ý bốn số x2 − 1, y − 1, z − 1, t2 − luôn có hai số cùng dấu Giả sử hai số đó là x2 − và y − 1, thì theo ví dụ trên, ta dễ thấy (x2 + 3)(y + 3) ≥ 4(x2 + y + 2) Sử dụng đánh giá này, ta đưa bài toán chứng minh (x2 + y + 2)(z + 3)(t2 + 3) ≥ 4(x + y + z + t)2 Đến đây, áp dụng bất đẳng thức Cauchy-Schwarz   (z + t)2 (x + y + z + t)2 ≤ (x2 + y + 2) + + , ta đến việc chứng minh bất đẳng thức sau   (z + t)2 2 (z + 3)(t + 3) ≥ + = + 2(z + t)2 Bất đẳng thức này tương đương với z t2 + z + t2 + ≥ 4zt, hiển nhiên đúng theo AM-GM √ z t2 + z + t2 + ≥ z t2 · z · t2 · = 4zt Bài toán chứng minh xong Đẳng thức xảy và a = b = c = d = √ (121) 120 Chuyên đề Toán học số 5.1 Các phương pháp sử dụng bất đẳng thức Bernoulli Các bài toán minh họa Để chuẩn bị bước sang hai kỹ thuật đánh giá chênh lệch số mũ và Bernoulli ngược, ta làm quen dần với các dạng toán sử dụng bất đẳng thức Bernoulli, là mở đầu cho hai kỹ thuật nói trên Ví dụ 12 (Olympic 30-4) Cho hai số dương a, b thỏa mãn a + b = Tìm nghiệm khoảng (0, 1) bất phương trình x(1 + b) xa > + bx Lời giải Thực chất đây là bài bất đẳng thức thông thường vì với x thuộc (0, 1), bất phương trình trên luôn đúng Thật vậy, áp dụng bất đẳng thức Bernoulli, ta có xb = [1 − (1 − x)]b ≤ − b(1 − x) = − b + bx Từ đây suy (1 + b)xb ≤ (1 + b)(1 − b + bx) < + bx Mà b = − a nên ta có (1 + b)x1−a < + bx, hay xa > (1 + b)x + bx Vậy bất phương trình thỏa mãn với x ∈ (0, 1) Ví dụ 13 (Tạp chí Toán học và Tuổi trẻ) Giải phương trình q 3x − 4x = − (1 + x2 )3 Lời giải Ta chứng minh 3x4 − 4x3 + p (1 + x2 )3 ≥ 1, ∀x ∈ R Áp dụng bất đẳng thức Bernoulli, ta có p 3 (1 + x2 )3 = (1 + x2 ) ≥ + x2 Từ đó suy 3x4 − 4x3 + p (1 + x2 )3 ≥ 3x4 − 4x3 + x2 + = x2 (6x2 − 8x + 3) + ≥ 2 Dấu đẳng thức xảy và x = Vậy phương trình có nghiệm x = Ví dụ 14 Chứng minh với số tự nhiên n ≥ 2, ta có bất đẳng thức r r r n(n + 2) 3 n+1 n + + + ··· + ≤ n n+1 (122) 121 Bất đẳng thức Bernoulli Lời giải Để khử các bậc k + 1, ta áp dụng bất đẳng thức Bernoulli sau k+1  k+1 1 , ∀k ≥ = + (k + 1) · ≤ 1+ k k(k + 1) k(k + 1) Từ đó lấy bậc k + vế, ta r 1 k+1 k + ≤1+ =1+ − , k k(k + 1) k k+1 ∀k ≥ Cho k = 1, 2, , n, cộng các bất đẳng thức lại theo vế, ta r r r       1 1 1 3 n+1 n + + 1+ − + ··· + + − + + ··· + ≤ 1+ − n 2 n n+1 n(n + 2) =n+1− = n+1 n+1 Bài toán chứng minh Nhận xét Nếu đặt f (n) là vế trái bất đẳng thức đã cho thì dễ thấy f (n) > n (do n(n + 2) n hạng tử nó > 1) Mà theo kết bài toán trên thì f (n) ≤ = n+ Do n+1 n+1 n < f (n) ≤ n + n n+1 Từ đây, ta có thể thấy [f (n)] = với n ≥ Tác giả đã đọc hệ bất đẳng thức H¨older chứng minh bất đẳng thức Bernoulli độc đáo Ví dụ 15 Cho , bi (i = 1, 2, , n) là các số thực dương Chứng minh với k > 1, ta có bất đẳng thức bk1 ak−1 + bk2 ak−1 + ··· + bkn (b1 + b2 + · · · + bn )k ≥ (a1 + a2 + · · · + an )k−1 ank−1 Lời giải Đây là bất đẳng thức với a1 , a2 , , an và với b1 , b2 , , bn nên không tính tổng quát, ta có thể giả sử a1 + a2 + · · · + an = b1 + b2 + · · · + bn = n Khi đó bất đẳng thức trở thành bk1 bk2 bkn + + · · · + ≥ n ak−1 ak−1 ank−1 Áp dụng bất đẳng thức Bernoulli, ta có  k   bi bi ≥k + − k, ai ∀i = 1, 2, , n Từ đó suy bki aik−1  = bi k   bi ≥ k + − k = kbi + (1 − k)ai , ∀i = 1, 2, , n (123) 122 Chuyên đề Toán học số Cho i = 1, 2, , n cộng các bất đẳng thức lại với nhau, ta thu bk1 ak−1 + bk2 ak−1 bkn ≥ k(b1 + b2 + · · · + bn ) + (1 − k)(a1 + a2 + · · · + an ) ak−1 n + ··· + = kn + (1 − k)n = n Bất đẳng thức chứng minh  1+ x Ví dụ 16 Chứng minh hàm số f (x) =   x+1 thì lại giảm trên khoảng này số g(x) = + x x tăng trên (0, +∞), đó hàm Lời giải (a) Chứng minh f (x) tăng Lấy x1 > x2 > 0, ta cần chứng minh f (x1 ) ≥ f (x2 ), hay     x1 x2 1+ ≥ 1+ x1 x2 Bất đẳng thức này tương đương với  x1  1 x2 ≥1+ , 1+ x1 x2 hiển nhiên đúng theo bất đẳng thức Bernoulli   x1 x2 x1 1+ · =1+ ≥1+ x1 x2 x1 x2 Vậy f (x) tăng trên (0, +∞) (b) Chứng minh g(x) giảm Lấy x1 > x2 > 0, ta cần chứng minh g(x1 ) ≤ g(x2 ), hay     x1 +1 x2 +1 1+ ≤ 1+ x1 x2 Bất đẳng thức này tương đương với x1 + ≤ x1  Do <  x2 x2 +  x2 + x2  1+x2 1+x1 ≤  1+x2 1+x1 , x1 x1 + 1 + x2 < nên theo bất đẳng thức Bernoulli, ta có + x1 x2 x2 +  1+x2 1+x1  = 1− x2 +  1+x2 Vậy g(x) là hàm giảm trên (0, +∞) 1+x1 ≤1− + x2 1 x1 · =1− = + x1 x2 + x1 + x1 + (124) Bất đẳng thức Bernoulli 123 Nhận xét Có thể dễ dàng chứng minh f (x) < < g(x) với x > Tuy nhiên, chưa phải là chặn trên nhỏ f (x), không phải là chặn lớn g(x) Với trợ giúp máy tính điện tử, người ta đã chứng minh lim f (x) = lim g(x) = e ≈ 2.718281828459 x→+∞ x→+∞ Từ đây ta có thể thấy chặn trên nhỏ f (x) và chặn lớn g(x) phải là e, tức ta có     x x+1 1+ ≤e≤ 1+ x x Cho x → 1 , ta (1 + x) x ≤ e ≤ (1 + x) x +1 , hay x (1 + x)r ≤ erx ≤ (1 + x)r(1+x) , ∀x > 0, r > Ví dụ 17 Cho a, b, c là các số thực dương thỏa mãn abc = Chứng minh 1 + b + c ≤ + c) b (c + a) c (a + b) aa (b Lời giải Đây là bất đẳng thức đẹp Các số mũ thực a, b, c gợi ý cho chúng ta sử dụng bất đẳng thức Bernoulli Tuy nhiên a, b, c lại không có đánh giá thích hợp với nên ta giả sử c = min{a, b, c} Khi đó c ≤ Áp dụng bất đẳng thức Bernoulli, ta có 1 ≤ · c + − c = − c cc c Mặt khác, ta lại có aa ≥ a, bb ≥ b.1 Do ta cần chứng minh 1 2−c + + ≤ a(b + c) b(c + a) a + b   1 1 Vì abc = nên ta có + = 2c − c + Mà a(b + c) b(c + a) a+c b+c √  √ 2 √ a − b ab − c 1 √  ≥ + −√ = a+c b+c ab + c (a + c)(b + c) ab + c 2c2 + ≤ 2c − √ Hơn nữa, − c > nên áp dụng bất đẳng thức a(b + c) b(c + a) ab + c AM-GM, ta có 2−c 2−c ≤ √ a+b ab Do đó Có thể chứng minh bất đẳng thức xx ≥ x, ∀x > sau: Bất đẳng thức này tương đương với xx−1 ≥ Nếu x ≥ thì bất đẳng thức hiển nhiên đúng Còn < x < 1, ta viết lại nó thành x1−x ≤ 1, 1−x và áp dụng bất đẳng thức Bernoulli, x1−x = [1 + (x − 1)] ≤ + (1 − x)(x − 1) = − (x − 1)2 ≤ (125) 124 Chuyên đề Toán học số Kết hợp các đánh giá này lại, ta đưa bài toán chứng minh 2c − √ Đặt t = √ c ≤ và thay √ 2c2 2−c + √ ≤ ab + c ab 1 ab = √ = , bất đẳng thức trên trở thành t c 2t2 − 2t4 + t2 t + t(2 − t2 ) ≤ 2 Sau khai triển và rút gọn, ta bất đẳng thức hiển nhiên đúng (t4 + 2t3 + t2 + 4t + 3)(t − 1)2 ≥ Đẳng thức xảy và a = b = c = 5.2 Kỹ thuật đánh giá chênh lệch số mũ Kỹ thuật đánh giá chênh lệch số mũ là kỹ thuật hay sử dụng Tuy không quá mạnh bất đẳng thức Bernoulli cho phép ta thay đổi các số mũ mà không cần nhất, chí có thể chuyển từ số mũ nhỏ sang số mũ lớn với dấu ≥ (xem phần 5.3 bên dưới) Ví dụ 18 Cho a, b, c là các số thực dương Chứng minh r r r r a b c + + ≤ 4a + b + c 4b + c + a 4c + a + b Lời giải Đây là bất đẳng thức dạng chênh lệch số mũ Áp dụng bất đẳng thức Bernoulli (với chú ý điều kiện xảy đẳng thức), ta có r 6a 6a 3a ≤ · +1− = + 4a + b + c 4a + b + c 4a + b + c Mặt khác, theo bất đẳng thức Cauchy-Schwarz, ta lại có   1 1 1 = ≤ + = + 4a + b + c 3a + (a + b + c) (1 + 1) 3a a + b + c 12a 4(a + b + c) Kết hợp với trên, ta r 6a 3 a ≤ + · 4a + b + c 4 a+b+c Cộng bất dẳng thức này với hai bất đẳng thức tương tự, ta ! r r r √ a b c a+b+c + + ≤ + · = 4a + b + c 4b + c + a 4c + a + b 4 a+b+c Từ đó suy điều phải chứng minh Đẳng thức xảy và a = b = c Trong bài toán trên, ta đã chuyển từ số mũ sang số mũ (126) 125 Bất đẳng thức Bernoulli Ví dụ 19 Cho x, y là hai số dương thỏa mãn x2 + y ≥ x3 + y Tìm giá trị lớn P = x3 + y Lời giải Bài toán trên đã giải theo nhiều cách, chủ yếu là chia trường hợp, xét vị trí x, y so với số Nhưng với kỹ thuật chênh lệch số mũ, lời giải sau đây thật ấn tượng: Để tìm giá trị lớn x3 + y , ta chuyển các số mũ khác thành mũ Áp dụng bất đẳng thức Bernoulli x2 ≤ x3 + , y ≥ y − 3 3 Do đó x + + y ≥ x3 + y − 3 3 Từ đây ta suy P = x3 + y ≤ Đẳng thức xảy và x = y = Ví dụ 20 Chứng minh với x, y > 0, ta có x4 y4 x2 y2 x y + − − + + ≥ y4 x4 y2 x2 y x Lời giải Bài toán trên thể rõ ý tưởng kỹ thuật đánh giá chênh lệch số mũ Ta chuyển hết số mũ Áp dụng bất đẳng thức Bernoulli, ta có x4 x2 x2 ≥ · + − = · − 1, y4 y2 y2 x4 x x ≥ · + − = · − y y y Từ hai bất đẳng thức này suy x4 x4 x4 = · + · ≥ y y y     x2 x x2 x 2· −1 + · − = + · − y y y y Chứng minh tương tự, ta có y4 y2 y ≥ + · − x x x Do đó x4 y4 x2 y2 x y + − − + + ≥3 y x y x y x  x y + y x  r −4≥3·2 x y · − = y x Đẳng thức xảy và x = y Ví dụ 21 Cho a, b, c là các số thực dương Chứng minh (a + b)c (b + c)a (c + a)b ≤ 2a+b+c aa bb cc Lời giải Ở đây, ta không biết a, b, c lớn hay bé so với nên không thể áp dụng trực tiếp bất đẳng thức Bernoulli Tuy nhiên, ta có thể “mượn” số mũ khác chắn đánh giá với Áp dụng bất đẳng thức Bernoulli, ta có   a b + c a+b+c b+c a a 3(b + c) ≤ · +1− = 2a 2a a + b + c a+b+c 2(a + b + c) (127) 126 Chuyên đề Toán học số Xây dựng các bất đẳng thức tương tự nhân vế theo vế, ta  b+c 2a  a a+b+c  c+a 2b  b a+b+c  a+b 2c  c a+b+c ≤ 27(a + b)(b + c)(c + a) ≤ 8(a + b + c)3 Từ đó suy (b + c)a (c + a)b (a + b)c ≤ 2a+b+c aa bb cc Bất đẳng thức chứng minh Đẳng thức xảy và a = b = c Nhận xét Cũng cách sử dụng bất đẳng thức Bernoulli, ta còn có thể chứng minh bất đẳng thức mạnh là a+b+c (a + b) (b + c) (c + a) ≤ (a + b + c) ≤ 2a+b+c aa bb cc c a b  Sau đây là bài toán thú vị Ví dụ 22 Chứng minh với a ≥ b > 0, ta có     a b b a ≤ + b + a 3 Lời giải Tại đây là bất đẳng thức thú vị? Ta thử làm bài toán cách tự nhiên Bất đẳng thức cần chứng minh tương đương với (32a + 1)b ≤ (32b + 1)a Bất đẳng thức trên cùng chiều, thật may mắn Áp dụng bất đẳng thức Bernoulli b (32a + 1) a ≤ + b 2a ·3 a Do đó, ta cần chứng minh 1+ b 2a · ≤ 32b a Bất đẳng thức này tương đương với b ≤ 9b−a a Thử vài giá trị, ta thấy bất đẳng thức trên không đúng Vậy kế hoạch chúng ta đã phá sản? Không thật thế, nhìn kỹ bất đẳng thức cuối cùng Vế trái nhỏ 1, vế phải nhỏ Nếu vế phải lớn thì thật tốt Nhưng thật bất ngờ, điều đó lại có thể Chỉ cần ta thấy bình đẳng và bất đẳng b a−b thức cần chứng minh thì ta có thể chuyển đổi bất đẳng thức cuối ≤ , bất đẳng thức a này hiển nhiên đúng Để làm rõ ý tưởng lời giải trên, ta hãy cùng xét bài toán tổng quát (128) Bất đẳng thức Bernoulli 127 Ví dụ 23 Cho a ≥ b là hai số dương và k là số thực dương tùy ý Chứng minh     b a a b k + a ≤ k + b k k Lời giải Sau thực các đánh giá tương tự trên, ta cần chứng minh b ≤ k 2(b−a) a Đây là chỗ đau đầu ta Nhưng với số k tổng quát, ta có thể thấy cách xử lý dễ dàng, cho < k ≤ thì vế phải hiển nhiên lớn Lí có thể đặt là bình đẳng k và k Ví dụ 24 (IMO Shortlist 2004) Cho a, b, c là các số dương thỏa mãn ab + bc + ca = Chứng minh bất đẳng thức sau r r r 3 + 6b + + 6c + + 6a ≤ a b c abc Lời giải Bài toán này không quá khó, cần đánh giá chênh lệch lũy thừa Ta có  13    + 6b 1  a + 6b + ,  √  ≤ √ 3 a  31   + 6c 1 b  + 6c + ,  √  ≤ √ 3 b   31   + 6a c  √ √ + 6a +   ≤ c 3  Cộng ba bất đẳng thức này lại theo vế, ta ! r r r   1 1 1 1 3 √ + 6b + + 6c + + 6a ≤ √ + + + √ (a + b + c) + a b c c a b 3 = √ + √ (a + b + c) + 3abc 3 Mặt khác, sử dụng các đánh giá √ ab + bc + ca ≥ a2 b2 c2 , ta có 1≤ √ , 3abc (ab + bc + ca)2 ≥ 3abc(a + b + c), a+b+c≤ 3abc Kết hợp với trên, ta r r r   √ 1 3 √ + 6b + + 6c + + 6a ≤ + √ · +2· √ = a b c abc 3abc 3 3abc 3abc Bất đẳng thức chứng minh Đẳng thức xảy và a = b = c = √ (129) 128 Chuyên đề Toán học số Ví dụ 25 (Japan 2005) Cho các số dương a, b, c thỏa mãn a + b + c = Chứng minh √ √ √ 3 a + b − c + b + c − a + c + a − b ≤ Lời giải Dễ thấy các biểu thức dấu dương Áp dụng bất đẳng thức Bernoulli 1 (1 + b − c) ≤ (1 + b − c) + = (b − c) + 1, 3 từ đó suy √ a + b − c ≤ (ab − ca) + a Cộng bất đẳng thức này với hai bất đẳng thức tương tự, ta có điều phải chứng minh Đẳng thức xảy và a = b = c = 5.3 Kỹ thuật “Bernoulli ngược” Như ta đã biết, bất đẳng thức Bernoulli cho phép đánh giá số mũ lớn dương trội số mũ bé dương, đa số bất đẳng thức, là bất đẳng thức mạnh, ta cần đánh giá ngược lại Khi đó, kỹ thuật nhỏ không chú ý không thể đánh giá các bất đẳng thức ngược chiều Tác giả đặt tên là kỹ thuật Bernoulli ngược, đây là kỹ thuật mà tác giả tâm đắc Ví dụ 26 Chứng minh với a, b ∈ (0, 1), ta có ab + ba > Lời giải Đây là bài toán khá tiếng, đặc trưng cho bất đẳng thức Bernoulli Tuy nhiên lần nhìn qua, thật tiếc là số mũ a, b nhỏ nên bất đẳng thức ngược chiều Nhưng cần kỹ thuật nhỏ, ta có thể giải vấn đề này Áp dụng bất đẳng thức Bernoulli, ta có 1 a a  = ab =  b ≥ > a + b − ab a + b 1+b −1 a a Tương tự, ta có ba > b a b Do đó ab + ba > + = a+b a+b a+b Như trường hợp số mũ cho bất đẳng thức ngược chiều thì ta xét nghịch đảo biểu thức cần đánh giá Để rõ kỹ thuật này, ta xét tiếp ví dụ sau đây, mà theo tác giả việc sử dụng Bernoulli ngược sáng Ví dụ 27 (MOP 2002) Chứng minh bất đẳng thức sau đúng với a, b, c dương  2a b+c 2  + 2b c+a 2  + 2c a+b 2 ≥ (130) Bất đẳng thức Bernoulli 129 Lời giải Với số mũ 1, bài toán là bất đẳng thức Nesbitt quen thuộc Tuy nhiên, đây là số mũ và công việc chúng ta là chuyển số mũ 1, đó bất đẳng thức trái dấu Vì ta áp dụng kỹ thuật Bernoulli ngược Ta có  2   b+c b+c a+b+c ≤ + = 2a 2a 3a Từ đây suy  2a b+c 2 ≥ 3a a+b+c Cộng bất đẳng thức này với hai bất đẳng thức tương tự, ta  2  2  2 3 2a 2b 2c 3a + 3b + 3c + + ≥ = b+c c+a a+b a+b+c Đẳng thức xảy và a = b = c Trong trường hợp tổng quát, ta có bài toán sau Ví dụ 28 Cho a, b, c là các số thực không âm thỏa mãn ab + bc + ca > Chứng minh với n ≥ 2, ta có r r r a b c n n + + n ≥ b+c c+a a+b Lời giải Vế trái có dạng mũ nhỏ so với bậc vế phải n càng lớn Do đó, theo n tác giả, bài toán này là áp dụng tinh tế kỹ thuật Bernoulli ngược kết hợp với kỹ thuật đánh giá chênh lệch lũy thừa Nếu ba số a, b, c có số 0, giả sử a = 0, đó bất đẳng thức trở thành r r c n b + n ≥ 2, c b hiển nhiên đúng theo bất đẳng thức AM-GM Xét trường hợp a, b, c > Áp dụng bất đẳng thức Bernoulli, ta có r a 1 n = (đánh giá số mũ sang số mũ ) ≥    b+c n n b+c n b+c n + a a 2 2 Ngoài ra, ta chứng minh (b + c) n ≤ b n + c n (cũng bất đẳng thức Bernoulli) Do đó, kết hợp với trên, ta r a 2a n n ≥ = 2 2 b+c bn + cn an + bn + cn + a n2 Cộng bất đẳng thức này với hai bất đẳng thức tương tự, ta thu kết cần chứng minh Đẳng thức xảy và ba số a, b, c có số 0, và hai số còn lại Sau đây là bài toán có dạng tương tự (131) 130 Chuyên đề Toán học số Ví dụ 29 Cho a, b, c là độ dài ba cạnh tam giác Chứng minh s 2 s 2 s 2 3a 3b 3c 3√ 3 3 45 + + ≥ 3b + 3c − a 3c + 3a − b 3a + 3b − c Lời giải Bất đẳng thức đã cho có thể viết lại thành  5a 3b + 3c − a 2  + 5b 3c + 3a − b 2  + 5c 3a + 3b − c 2 ≥ Áp dụng bất đẳng thức Bernoulli, ta có  3b + 3c − a 5a 2 ≤ 3b + 3c − a a + 2b + 2c · +1− = 5a 5a Do đó  5a 3b + 3c − a 2 ≥ 5a a + 2b + 2c Từ bất đẳng thức này và hai bất đẳng thức tương tự, ta đưa bài toán chứng minh a b c + + ≥ a + 2b + 2c b + 2c + 2a c + 2a + 2b Bất đẳng thức này đúng vì theo bất đẳng thức Cauchy-Schwarz, ta có a b c (a + b + c)2 + + ≥ a + 2b + 2c b + 2c + 2a c + 2a + 2b a(a + 2b + 2c) + b(b + 2c + 2a) + c(c + 2a + 2b) (a + b + c)2 = (a + b + c)2 + 2(ab + bc + ca) (a + b + c)2 ≥ = (a + b + c)2 + (a + b + c)2 Đẳng thức xảy và a = b = c Kỹ thuật Bernoulli ngược thường áp dụng với bất đẳng thức Cauchy-Schwarz hiệu Và đây là đường để sáng tạo bất đẳng thức Ví dụ 30 Cho n là số tự nhiên lớn và a1 , a2 , , an là các số thực dương Chứng minh v u a1 a2 an u + + ··· + n t (a1 + a2 + · · · + an )2 a2 a3 a1 ≥ n a1 a2 + a2 a3 + · · · + an a1 + (n − 1)(a21 + a22 + · · · + a2n ) Lời giải Rõ ràng đây là bất đẳng thức có dạng khá lạ việc giải các bất đẳng thức thông thường đây là dạng cho kỹ thuật Bernoulli ngược Ta chứng minh bất đẳng thức mạnh r r r a1 a2 an n(a1 + a2 + · · · + an )2 n n + + ··· + n ≥ a2 a3 a1 a1 a2 + a2 a3 + · · · + an a1 + (n − 1)(a21 + a22 + · · · + a2n ) (132) Bất đẳng thức Bernoulli 131 Áp dụng bất đẳng thức Bernoulli, dễ thấy r a1 1 na21 na1 n =  1 ≥ = = a2 a2 a2 + (n − 1)a1 a1 a2 + (n − 1)a21 a2 n +1− na1 n a1 Xây dựng các bất đẳng thức tương tự, cộng lại và áp dụng bất đẳng thức Cauchy-Schwarz, ta có điều phải chứng minh Ví dụ 31 Cho < a ≤ b ≤ và 1 + ≥ Chứng minh a b ab ≤ ba Lời giải Áp dụng bất đẳng thức Bernoulli, ta có  b a b b a2 + b − ab ≥ · +1− = a a a a a2     a a b Mà a + b − ab = b +1−a ≥b + − a = (a − 2)2 ≥ 0, nên từ trên ta có b 4 b aa ≤ a2 a2 + b − ab Vậy ta cần chứng minh a2 ≤ b b + a2 − ab Bất đẳng thức này tương đương với (b + a − ab)(a − b) ≤ 0, hay   1 + − (a − b) ≤ (đúng theo giả thiết) a b Đẳng thức xảy và a = b Nhận xét Nếu thay điều kiện chặn là < a ≤ ≤ b thì bài toán đúng Ví dụ 32 Cho các số dương x, y thỏa mãn x + y = Chứng minh √ √ x y +√ ≥ 1−y 1−x Lời giải Áp dụng bất đẳng thức Bernoulli, ta có 1 p ≥ = 1 2(1 − x) (1 − x) + y+ 2 Từ đó suy √ √ x 2x 2x √ ≥ = 1 1−x y+ xy + x 2 (133) 132 Chuyên đề Toán học số Tương tự, ta có y √ ≥ 1−y √ 2y xy + y Sử dụng hai đánh giá này kết hợp với các bất đẳng thức Cauchy-Schwarz và AM-GM, ta   √  x2 y2  √ x y (x + y)2 √ + +√ ≥ 2 ≥ 2·  1 1−y 1−x xy + x xy + y 2xy + (x + y) 2 √ √ √ 2 2 = ≥ = 4xy + (x + y)2 + 1 Đẳng thức xảy và x = y = Ví dụ 33 Cho a, b, c là các số dương thỏa mãn ab + bc + ca = Chứng minh với n ≥ 1, ta có r r p r 6n 9(a2 + b2 + c2 ) n b + c n a + b n c + a + + ≥ 2a 2b 2c 3(n + 1) + (n − 1)(a2 + b2 + c2 ) Lời giải Đây là bất đẳng thức sử dụng phương pháp Bernoulli-Cauchy-Schwarz rõ vì bậc n chuyển thành số hạng khiến chúng ta dự đoán sử dụng Bernoulli ngược và dạng biểu thức vế phải quen thuộc với bất đẳng thức Cauchy-Schwarz Áp dụng bất đẳng thức Bernoulli, ta có r n b + c = 1 ≥ 2a 2a n b+c = n(b + c) = 2a 2a + (n − 1)(b + c) +1− n(b + c) n n(b + c)2 2a(b + c) + (n − 1)(b + c)2 Tương tự, ta có các bất đẳng thức r n(c + a)2 n c + a ≥ , 2b 2b(c + a) + (n − 1)(c + a)2 r n a+b n(a + b)2 ≥ 2c 2c(a + b) + (n − 1)(a + b)2 Cộng ba bất đẳng thức lại và áp dụng bất đẳng thức Cauchy-Schwarz, suy r r r 4n(a + b + c)2 n b + c n a + b n c + a + + ≥ 2a 2b 2c 2(n − 1)(a2 + b2 + c2 ) + 2(n + 1)(ab + bc + ca) = 2n(a + b + c)2 (n − 1)(a2 + b2 + c2 ) + 3(n + 1) Bài toán quy chứng minh p (a + b + c)2 ≥ 3 9(a2 + b2 + c2 ) (134) 133 Bất đẳng thức Bernoulli Bất đẳng thức này đúng theo AM-GM (a + b + c)2 = (a2 + b2 + c2 ) + (ab + bc + ca) + (ab + bc + ca) p p ≥ 3 (a2 + b2 + c2 )(ab + bc + ca)2 = 3 9(a2 + b2 + c2 ) Vậy bài toán chứng minh xong Đẳng thức xảy và a = b = c = Ví dụ 34 Cho hai số dương x, y thỏa mãn x + y = Tìm giá trị nhỏ biểu thức P = xx + y y Lời giải Sử dụng bất đẳng thức Bernoulli, ta có (2x)x =  1 x ≥ = , ·x+1−x −x 2x 2x (2y)y =  1 y ≥ = ·y+1−y −y 2y 2y Nhận xét đến đây ta cộng vế theo vế hai bất đẳng thức trên lại thì vế trái còn 2x và 2y trước các số hạng Nhưng đặc biệt, tích hai “phần thừa” này lại Do đó, ta nhân vế theo vế hai bất đẳng thức trên, suy 2xx y y ≥  Từ đây ta có −x  −y  ≥   −x +  −y 2 = √ p √ √ P = xx + y y ≥ xx y y = · 2xx y y ≥ Đẳng thức xảy và x = y = Ví dụ 35 Cho n ≥ và a, b, c là các số dương thỏa mãn s s s 6 6 + + ≥ a + 3(n − 1) b + 3(n − 1) c + 3(n − 1) Chứng minh r n + a r n + b r n 3n ≥ c Lời giải Ta có nhận xét: Với n càng lớn thì số mũ vế trái bất đẳng thức cần chứng minh càng nhỏ, ta không thể đánh giá chênh lệch với , ta đánh giá chênh lệch với số mũ Áp dụng bất đẳng thức Bernoulli, ta có r 1 3n n =  1 ≥ = a a a + 3(n − 1) a n +1− 3n n (135) 134 Chuyên đề Toán học số Cộng bất đẳng thức này với hai bất đẳng thức tương tự, ta suy r r r   1 n n n + + ≥ 3n + + a b c a + 3(n − 1) b + 3(n − 1) c + 3(n − 1) Đến đây, ta sử dụng bất đẳng thức Bernoulli đánh giá với số mũ s s s 6 ≤ + , a + 3(n − 1) a + 3(n − 1) 6 ≤ + , b + 3(n − 1) b + 3(n − 1) 6 ≤ + c + 3(n − 1) c + 3(n − 1) Cộng ba bất đẳng thức này lại và sử dụng giả thiết, ta 1 1 + + ≥ a + 3(n − 1) b + 3(n − 1) c + 3(n − 1) Từ đó, kết hợp với đánh giá trên, ta có điều phải chứng minh Đẳng thức xảy và a = b = c = và n = Ví dụ 36 Chứng minh với x, y, z ∈ (0, 1), ta có x2y + y 2z + z 2x > Lời giải Áp dụng bất đẳng thức Bernoulli, ta có x 1 x xy =  y ≥ y = > x + y − xy x+y +1−y x x Do đó x 2y  ≥ x x+y 2 Đánh giá tương tự với y, z, ta suy x2y + y 2z + z 2x >  x x+y 2  + y y+z 2  + z z+x Vậy ta cần chứng minh  x x+y 2  + y y+z 2  + z z+x 2 ≥ 2 (136) 135 Bất đẳng thức Bernoulli Đây là bất đẳng thức quen thuộc và có nhiều cách chứng minh cho nó Xin nêu đây y z x cách chứng minh phổ biến: Đặt a = , b = , c = , ta có abc = và bất đẳng thức trên x y z viết lại thành 1 + + ≥ 2 (1 + x) (1 + y) (1 + z) Sử dụng bất đẳng thức Cauchy-Schwarz, ta 1 1   + + ≥ y 2 x (1 + x) (1 + y) 1+ (1 + xy) 1+ (1 + xy) x y x z y + = = = (x + y)(1 + xy) (x + y)(1 + xy) + xy z+1 Từ đó suy 1 z z2 + z + 3 (z − 1)2 + + ≥ = = ≥ + + 2 2 2 (1 + x) (1 + y) (1 + z) z + (1 + z) (z + 1) 4(z + 1) Bài toán chứng minh xong Nhận xét Từ cách chứng minh trên, ta có thể mở rộng bài toán cho n biến: Với x1 , x2 , , xn ∈ (0, 1), ta có n no 2x3 2x1 x2x + x + · · · + x > 1, n Thực cách làm tương tự trên, dễ thấy ta cần chứng minh n no 1 + + · · · + ≥ 1, (1 + a1 )2 (1 + a2 )2 (1 + an )2 Phần chứng minh dành cho bạn đọc Ví dụ 37 Cho a, b, c là ba cạnh tam giác Chứng minh √ √ a b c +√ +√ ≥ 2b2 + 2c2 − a2 2c2 + 2a2 − b2 2a2 + 2c2 − b2 Lời giải Bất đẳng thức đã cho có thể viết lại thành  3a2 2b2 + 2c2 − a2  12  + 3b2 2c2 + 2a2 − b2  21  + 3c2 2a2 + 2b2 − c2  12 ≥ Áp dụng bất đẳng thức Bernoulli, ta có  2b2 + 2c2 − a2 3a2  12 ≤ a2 + b2 + c2 2b2 + 2c2 − a2 · + − = 3a2 3a2 Từ đó suy  3a2 2b2 + 2c2 − a2  21 ≥ 3a2 a2 + b2 + c2 Cộng bất đẳng thức này với hai bất đẳng thức tương tự, ta có kết cần chứng minh Đẳng thức xảy và a = b = c (137) 136 Chuyên đề Toán học số Ví dụ 38 Chứng minh với a, b, c dương, ta có r 2a2 + bc 2b2 + ca 2c2 + ab (a2 + b2 + c2 )(a + b + c) + + ≤ a + 2bc b + 2ca c + 2ab abc Lời giải Bài toán trên không giải đơn giản các bài toán trước vì trước dấu bé là tổng hàm Lời giải sau: Áp dụng bất đẳng thức Bernoulli, ta có  2a2 + bc 3bc  21 = 2a2 + bc = ≥ 1 bc a2 + 2bc 3bc + 2a2 + bc 2a2 + bc Cộng bất đẳng thức này với hai bất đẳng thức tương tự, ta 2a2 + bc 2b2 + ca 2c2 + ab + + ≤ a2 + 2bc b2 + 2ca c2 + 2ab  2a2 + bc 3bc  12  + 2b2 + ca 3ca  21  + 2c2 + ab 3ab  12 (1) Mặt khác, theo bất đẳng thức Cauchy-Schwarz, ta lại có X  2a2 + bc  3bc hX i X  12  X X  21 X  2 2 ≤ (2a + bc) = a + bc 3bc 3bc 1      X 1 X (a2 + b2 + c2 )(a + b + c) 2 ≤ a2 = (2) 3bc abc Từ (1) và (2), ta có điều phải chứng minh Đẳng thức xảy và a = b = c Nhận xét Có thể chứng minh 2a2 + bc 2b2 + ca 2c2 + ab + + ≥ a2 + 2bc b2 + 2ca c2 + 2ab Thật vậy, sử dụng bất đẳng thức AM-GM, ta có X 2a2 + bc X  2a2 + bc  3X a2 = − + = + 2 a + 2bc a + 2bc 2 a + 2bc 2 X a + = ≥ 2 2 a + (b + c ) Như vậy, có thể thấy bài toán trên cho ta đánh giá chặt cho bất đẳng thức quen thuộc (a2 + b2 + c2 )(a + b + c) ≥ abc Ví dụ 39 Cho a, b, c là các số thực dương Chứng minh " 4a2 + (b − c)2 a2 + b2 + c2 + bc #2 " 4b2 + (c − a)2 + a + b2 + c2 + ca #2 " 4c2 + (a − b)2 + a + b2 + c2 + ab #2 ≥ (138) 137 Bất đẳng thức Bernoulli Lời giải Áp dụng bất đẳng thức Bernoulli, ta có " #2 4a2 + (b − c)2 4a2 + (b − c)2 = ≥ 2  a2 + b2 + c2 + bc b2 + c2 + 2a2 a + b2 + c2 + bc 4a2 + (b − c)2 Đánh giá tương tự với hai biểu thức còn lại, ta suy #2 " X 4a2 + (b − c)2 X 4a2 + (b − c)2 ≥ a2 + b2 + c2 + bc b2 + c2 + 2a2 Vậy ta cần chứng minh 4a2 + (b − c)2 4b2 + (c − a)2 4c2 + (a − b)2 + + ≥ 2a2 + b2 + c2 2b + c2 + a2 2c + a2 + b2 Do (b + c)2 4a2 + (b − c)2 = − nên bất đẳng thức trên có thể viết lại thành 2a2 + b2 + c2 2a2 + b2 + c2 (c + a)2 (a + b)2 (b + c)2 + + ≤ 2a2 + b2 + c2 2b2 + c2 + a2 2c2 + a2 + b2 Áp dụng bất đẳng thức Cauchy-Schwarz, ta có (b + c)2 (b + c)2 b2 c2 = ≤ + 2a2 + b2 + c2 (a2 + b2 ) + (c2 + a2 ) a2 + b2 c2 + a2 Từ đó suy X X (b + c)2 ≤ 2a2 + b2 + c2  c2 b2 + a2 + b2 c2 + a2  = Bài toán chứng minh xong Đẳng thức xảy và a = b = c Nhận xét Có thể chứng minh kết chặt  4a2 a2 + b2 + c2 + bc  23  + 4b2 a2 + b2 + c2 + ca  23  + 4c2 a2 + b2 + c2 + ab  32 ≥ Ví dụ 40 Cho a, b, c là các số dương Chứng minh s 3 s 3 s 3 20a 20b 20c 4 + + ≥ 2a + 9b + 9c 2b + 9c + 9a 2c + 9a + 9b Lời giải Ta chứng minh bất đẳng thức mạnh là   20   20   20 27 27 27 20a 20b 20c + + ≥ 2a + 9b + 9c 2b + 9c + 9a 2c + 9a + 9b Áp dụng bất đẳng thức Bernoulli, ta có   20 2a + 9b + 9c 27 20 2a + 9b + 9c 20 a+b+c ≤ · +1− = , 20a 27 20a 27 3a (139) 138 Chuyên đề Toán học số từ đó suy  20a 2a + 9b + 9c  20 27 ≥ 3a a+b+c Cộng bất đẳng thức này với hai bất đẳng thức tương tự, ta thu kết trên Đẳng thức xảy và a = b = c Nhận xét Ngoài cách giải trên, ta có thể chứng minh trực tiếp bất đẳng thức gốc kỹ thuật Bernoulli ngược và bất đẳng thức Cauchy-Schwarz Bài tập đề nghị Bài tập Chứng minh với n ≥ 1, ta có √ n+1 nn ≥ n+1 Bài tập Chứng minh với n nguyên dương, ta có bất đẳng thức 1≤ √ n n≤2− n Bài tập Cho a, b, c là độ dài ba cạnh tam giác Chứng minh r r r   a b c a b c + + ≥2 + + b+c−a c+a−b a+b−c b+c c+a a+b Bài tập Cho a, b, c là các số thực dương Chứng minh r r r 2a 2b 2c (a + b + c)2 + + ≥ b+c c+a a+b a + b2 + c2 Bài tập Cho a và b là các số thực không âm thỏa mãn a + b = Chứng minh   a−b 2b 2a a +b + ≤ 2 Bài tập Cho hai số dương a, b thỏa mãn a + b = Chứng minh aa + bb ≥ + (a − b)2 Bài tập Cho x1 , x2 , , xn là các số thực dương Chứng minh (x21 + 1)(x22 + 1) · · · (x2n + 1) nn−2 ≥ (x1 + x2 + · · · + xn )2 (n − 1)n−1 Bài tập Cho a, b, c là các số dương thỏa mãn điều kiện 2a + 3b + 4c = Tìm giá trị nhỏ √ √ √ 2 biểu thức P = 2a + 3b + 4c Bài tập Chứng minh với a, b, c dương, ta có ab+c + bc+a + ca+b ≥ (140) Bất đẳng thức Bernoulli 139 Tài liệu tham khảo [1] Trần Phương, Những viên kim cương bất đẳng thức toán học, Nhà xuất Tri Thức, 2009 [2] Phạm Kim Hùng, Sáng tạo bất đẳng thức, Nhà xuất Tri Thức, 2006 [3] Trần Nam Dũng, Chuyên đề bồi dưỡng học sinh giỏi [4] Nguyễn Văn Nho, Lê Hoành Phò, Tìm giá trị lớn nhất, nhỏ nhất, Nhà xuất [5] Tuyển tập các đề thi truyền thống 30-4 [6] Tạp chí Toán học và Tuổi trẻ [7] Wikipedia, Bernoulli’s inequality [LINK: http://en.wikipedia.org/wiki/Bernoulli’s_inequality] [8] Các diễn đàn Toán học: • http://diendantoanhoc.net • http://mathlinks.ro • http://mathscope.org • http://math.vn (141) 140 Chuyên đề Toán học số d MỘT SỐ CÂU CHUYỆN VUI VỀ TOÁN HỌC Con đường các nhà Toán học Một nhà Vật lý qua hành lang thì thấy nhà Toán học lúi húi bò bò lại trên sàn Nhà Vật lý tò mò lên tiếng hỏi: – Ông làm gì đây đấy? – À, tôi tìm cái kim, tôi vừa đánh rơi Nhà Vật lý hỏi tiếp: – Thế ông đánh rơi chỗ nào? – Ở phòng tôi thôi Nhà Vật lý ngạc nhiên quá hỏi: – Đánh rơi phòng ông lại đây tìm Nhà Toán học đáp: – Ừ, phòng tối quá, tôi ngoài này tìm cho sáng! Toán học nhiều là Khi gặp vấn đề hóc búa ta hay nghĩ đến đường đến lời giải, đề định nghĩa Số ảo i, hay hàm Dirac-delta là hai số vô vàn ví dụ Nhà Toán học thông minh Một nhà Toán học và nhà văn bị tộc da đỏ bắt Tù trưởng lạc này là người thông minh và đã học hành Sau bỏ đói ba ngày, tù trưởng cho lính dắt nhà văn vào phòng và bảo ông ta ăn Nhà văn đặt ngồi trên ghế góc phòng, bụng khấp khởi mừng nhìn thấy mâm sơn hào hải vị đặt góc phòng bên Tên tù trưởng giải thích: – Mày phải ngồi yên trên ghế, phút mày lại quyền kéo cái ghế nửa quãng đường tới mâm cơm Ông nhà văn giãy nảy: – Tao không tham gia trò giễu cợt này, không thằng nào không thấy là tao chẳng đến chỗ mâm cơm Tù trưởng không làm khó dễ gì nhà văn, ông này cắp bụng đói phòng nhốt mình Tới lượt nhà Toán học đưa với điều kiện tương tự Khi nghe tên tù trưởng giải thích luật chơi, mắt ông này sáng rực và ngồi vào ghế Tù trưởng vờ ngạc nhiên hỏi: – Chẳng nhẽ mày không thấy là mày chẳng đến tới chỗ mâm cơm hay sao? Nhà Toán học mỉm cười: – Tao không tới tận chỗ mâm cơm, tao có thể đến gần đủ để ăn cơm Ngồi tù, nhà văn nhìn thấy nhà Toán học ăn cơm và xỉu (142) ENLIGHTENING TRIGONOMETRICAL SUBSTITUTIONS Vardan Verdiyan - Daniel Campos Salas Trigonometry has always been an indivisible part of algebra There are certain algebraic inequalities deemed to be highly complex; yet the use of substitutions as a key unquestionably makes them look more straightforward Moreover, trigonometrical substitutions create such amazing results along the process of reducing the problem that immediately leads to a direct solution Besides, trigonometrical functions have some well-known properties that are highly useful while solving inequalities As a result, many algebraic problems can be solved easily using a trigonometrical substitution The article aims to present some of such sustitutions Initially, we start this paper introducing the readers to these substitutions After that we will present some well-known trigonometrical identities and inequalities that are highly constructive while solving algebraic inequalities with the help of trigonometrical substitutions Last but not least, we will discuss and introduce some Olympiad problems to the readers Lemmas on identities Lemma Let α, β, γ be angles in (0, π) Then, we have α, β, γ are the angles of a triangle if and only if tan β β γ γ α α tan + tan tan + tan tan = 2 2 2 Proof Since < α+β < 2π, it follows that there exists an angle in (−π, π), say   πγ , such −x , that α + β + γ = π Using the addition formulas and the fact that tan x = cot we have α β − tan tan γ α+β 2 tan = cot = α β 2 tan + tan 2 From this it results that tan α β β γ0 γ0 α tan + tan tan + tan tan = 2 2 2 (1) tan α β β γ γ α tan + tan tan + tan tan = 1, 2 2 2 (2) Now suppose that for some α, β, γ in (0, π) 141 (143) 142 Chuyên đề Toán học số We will prove that γ = γ , and this would imply that α, β, γ are the angles of(144) a triangle (145) (146) γ − γ0 (147) γ γ0 (148) = kπ for From (1) and (2) it follows that tan = tan This implies that (149) (150) 2 (151) some nonnegative integer k But (152) (153) (154) (155) (156) γ − γ (157) (158) (159) γ (160) (161) (162) γ (163) (164) (165) (166) (167) (168) (169)(170) (171) + (172) (173) < π, so it follows that k = 0, this is γ = γ , as desired Lemma Let α, β, γ be angles in (0, π) Then, we have α, β, γ are the angles of a triangle if and only if sin2 β γ α α β γ + sin2 + sin2 + sin sin sin = 2 2 2 Proof Since < α + β < 2π, it follows that there exists an angle in (−π, π), say γ , such that α + β + γ = π Using the product-to-sum and the double angle formulas, it results that   α β γ0 α+β α+β α β γ + sin sin sin = cos cos + sin sin sin 2 2 2 2    α+β α−β α+β α+β = cos + cos − cos cos 2 2 α+β α−β cos α + cos β = cos cos = 2 2   α β − sin2 + − sin2 2 = α β = − sin2 − sin2 , 2 and this proves that α α β γ0 β γ sin + sin + sin + sin sin sin = 2 2 2 (1) Now suppose that sin2 α β γ α β γ + sin2 + sin2 + sin sin sin = 1, 2 2 2 (2) for some α, β, γ in (0, π) We will prove that γ = γ , and this would imply that α, β, γ are the angles of a triangle From (1) and (2), it follows that   α β γ γ0 γ γ − sin + sin sin sin − sin = 0, sin 2 2 2 (174) Enlightening Trigonometrical Substitutions this is, 143    γ γ0 γ γ0 α β sin − sin sin + sin + sin sin = 2 2 2 The second factor equals sin γ γ0 α−β α+β γ α−β + sin + cos − cos = sin + cos , 2 2 2 which is evidently greater than It results that sin γ = γ and the proof is completed γ0 γ = sin , and this implies that 2 Lemma (Half-angle or Briggs formulas) Let ABC be a triangle, then, r r A (s − b)(s − c) A s(s − a) sin = and cos = bc bc Proof From the cosine law and the double-angle formulas, we have − cos A 2bc − (b2 + c2 − a2 ) a2 − (b − c)2 A = = = 2 4bc 4bc (s − b)(s − c) (a + b − c)(a − b + c) = , = 4bc bc r A (s − b)(s − c) from where we conclude sin = Analogously it can be proven that bc r A s(s − a) , cos = bc sin2 and this completes the proof Substitutions and Transformations π−α π−α π−α ,B= ,C= , this 2 π implies that A + B + C = π, and < A, B, C < This shows that A, B, C are angles of an acute angled triangle Note that     α α cyc sin = cos A , cyc cos = sin A , 2     α α cyc tan = cot A , cyc cot = tan A , 2 T1 Let α, β, γ be angles of a triangle Let A = where by cyc we denote a cyclic permutation of angles T2 Let x, y, z be positive real numbers Then, x + y, y + z, z + x are the sidelengths of a triangle This is x + y + z = s and (x, y, z) = (s − a, s − b, s − c) for some triangle ABC with sidelengths a, b, c and semiperimeter s (175) 144 Chuyên đề Toán học số S1 Let a, b, c be arbitrary positive real numbers, such that ab + bc + ca = The fact π that for x ∈ 0, , tan x takes all values from R+ , and Lemma allows us to substitute A B C a = tan , b = tan , c = tan , 2 where A, B, C are the angles of an arbitrary triangle ABC S2 Applying T1 to S1 we have that if a, b, c are arbitrary positive real numbers, such that ab + bc + ca = 1, then we can perform the substitution a = cot A, b = cot B, c = cot C, where A, B, C are the angles of an acuted angled triangle ABC S3 Let a, b, c be arbitrary positive real numbers such that a + b + c = abc Dividing 1 by abc it follows that + + = Due to S1 we can make the substitution bc ca ab A = tan , a B = tan , b C = tan , c this is B C A , b = cot , c = cot , 2 where A, B, C are the angles of a triangle ABC a = cot S4 Applying T1 to S3 we have that if a, b, c are arbitrary positive real numbers such that a + b + c = abc, then we can perform the substitution a = tan A, b = tan B, c = tan C, where A, B, C are the angles of an acuted angled triangle ABC S5 Let a, b, c be arbitrary positive real numbers such that a2 + b2 + c2 + 2abc = Note that since all the numbers are positive, it follows that a, b, c < The fact x that for x ∈ (0, π), sin takes all values from (0, 1), and Lemma allows us to substitute A B C a = sin , b = sin , c = sin , 2 where A, B, C are the angles of an arbitrary triangle ABC S6 Applying T1 to S5 we have that if a, b, c are arbitrary positive real numbers such that a2 + b2 + c2 + 2abc = 1, then we can perform the substitution a = cos A, b = cos B, c = cos C, where A, B, C are the angles of an acuted angled triangle ABC (176) Enlightening Trigonometrical Substitutions 145 S7 Let x, y, z be positive real numbers Applying T2 to the expressions r r r yz zx xy , , , (x + y)(x + z) (y + z)(y + x) (z + x)(z + y) this can be substituted by r (s − b)(s − c) , bc r (s − c)(s − a) , ca r (s − a)(s − b) , ab where a, b, c are the sidelengths of a triangle According to Lemma 3, these expressions equal A B C sin , sin , sin , 2 where A, B, C are the angles of a triangle ABC S8 Analogously to S7, the expressions s s x(x + y + z) y(x + y + z) , , (x + y)(x + z) (y + z)(y + x) s z(x + y + z) , (z + x)(z + y) can be substituted by A B C , cos , cos , 2 where A, B, C are the angles of a triangle ABC cos Some well-known inequalities For any triangle ABC, we have that A B C + sin + sin ≤ ; 2 2 √ A B C 3 • sin A + sin B + sin C ≤ cos + cos + cos ≤ ; 2 2 • cos A + cos B + cos C ≤ sin A B C sin sin ≤ ; 2 √ A B C 3 • sin A sin B sin C ≤ cos cos cos ≤ ; 2 √ A B C • cot + cot + cot ≥ 3; 2 • cos A cos B cos C ≤ sin • cos2 A + cos2 B + cos2 C ≥ sin2 A B C + sin2 + sin2 ≥ ; 2 • sin2 A + sin2 B + sin2 C ≤ cos2 A B C + cos2 + cos2 ≤ ; 2 (177) 146 Chuyên đề Toán học số • cot A + cot B + cot C ≥ tan √ A B C + tan + tan ≥ 2 For acute-angled triangles, we have that • sec A + sec B + sec C ≥ csc A B C + csc + csc ≥ 6; 2 • sin A + sin B + sin C > 2; • cos2 A + cos2 B + cos2 C < 1; • tan A + tan B + tan C ≥ cot √ A B C + cot + cot ≥ 3 2 Some well-known identities For arbitrary angles α, β, γ, we have that • sin α + sin β + sin γ − sin(α + β + γ) = sin β+γ γ+α α+β sin sin ; 2 • cos α + cos β + cos γ + cos(α + β + γ) = cos α+β β+γ γ+α cos cos ; 2 • cos(α+β+γ) = cos α cos β cos γ−cos α sin β sin γ−sin α cos β sin γ−sin α sin β cos γ For any triangle ABC, we have that • cos A + cos B + cos C = + sin • sin A + sin B + sin C = cos B C A sin sin ; 2 B C A cos cos ; 2 • sin 2A + sin 2B + sin 2C = sin A sin B sin C; • −(cos 2A + cos 2B + cos 2C) = + cos A cos B cos C; • − 2(cos2 A + cos2 B + cos2 C) = −(cos 2A + cos 2B + cos 2C); • sin2 A + sin2 B + sin2 C = + cos A cos B cos C Applications Problem (Poland 1999) Let a, b, c be positive real numbers such that a + b + c = Prove the following inequality √ a2 + b2 + c2 + 3abc ≤ (178) Enlightening Trigonometrical Substitutions r Solution Let us denote cyc x = bc a 147 ! This implies that cyc(a = yz) The problem turns to prove the inequality √ (xy)2 + (yz)2 + (zx)2 + 3xyz ≤ 1, given that xy + yz + zx = 1, where x, y, z are arbitrary positive real numbers Note that the inequality is equivalent to √ (xy + yz + zx)2 + 3xyz ≤ + 2xyz(x + y + z), √ ≤ x + y + z   A Perform the substitution S1: cyc x = tan , where A, B, C are the angles of a tri2 angle So the last inequality is equivalent to √ A B C tan + tan + tan ≥ 2 x Since the function tan is convex on (0, π), it follows from Jensen’s inequality that √ B C A+B+C √ A =3· = 3, tan + tan + tan ≥ tan 2 and this completes the proof Problem (Crux Mathematicorum) Let x, y, z be positive real numbers Prove that x y z p p p + + ≤ x + (x + y)(x + z) y + (y + z)(y + x) x + (z + x)(z + y) Solution The inequality is equivalent to r 1+ (x + y)(x + z) x2 + r 1+ 1 r + ≤ (y + z)(y + x) (z + x)(z + y) 1+ y2 z2 Since the inequality  is homogenous  we can assume that xy + yz + zx = Perform the A substitution S1: cyc x = tan , where A, B, C are angles of a triangle Note that    A B A C tan + tan tan + tan (x + y)(x + z) 2 2 = = , x2 A A tan sin 2 and similar forms for the other terms Then, the inequality is equivalent to sin A A + sin sin + B B + sin sin + C C + sin ≤ 1, (179) 148 Chuyên đề Toán học số that is 2≤ A + sin + B + sin + 1 + sin C A B C On the other hand, using the well-known inequality sin + sin + sin ≤ , it follows 2 2 from Cauchy-Schwarz that 1     ≤ + + , 2≤  A B C A B C + sin + sin + sin + sin + + sin + + sin 2 2 2 and the proof is completed Problem (Nesbitt’s Inequality) For any positive real numbers x, y, z prove the following inequality x y z + + ≥ y+z z+x x+y r  A yz Solution Perform the substitution S7: cyc = sin , where A, B, (x + y)(x + z) C are angles of a triangle Note that r r yz zx B A sin sin (x + y)(x + z) (y + z)(y + x) z 2, r = = C xy x+y sin (z + y)(z + x) and similar forms for the other terms So, we have to show that sin A B C A B C sin sin sin sin sin 2 + 2 + 2 ≥ C A B sin sin sin 2 Using the known inequalities (u + v + w)2 ≥ 3(uv + vw + wu), sin2 A B C + sin2 + sin2 ≥ , 2 it follows that   A B B C C A  2 sin sin sin sin sin sin A B C   2 2 + 2 + 2 ≥ sin + sin + sin ≥  C A B  2 sin sin sin 2 Taking square root of each side the conclusion follows, and the proof is completed (180) Enlightening Trigonometrical Substitutions 149 1 + + = Prove that x y z p √ √ √ x − + y − + z − ≤ x + y + z Problem (Iran 1997) Let x, y, z > such that Solution Let (x, y, z) = (a + 1, b + 1, c + 1), with a, b, c positive real numbers Note that the hypothesis is equivalent to ab + bc + ca + 2abc = Then, we have to prove that √ √ √ √ a + b + c ≤ a + b + c + Squaring the inequality and cancelling some terms yields √ √ √ ab + bc + ca ≤   A B C , sin , where The hypothesis and S5 allow us to take (ab, bc, ca) = sin , sin 2 ABC is an arbitrary triangle Then, we have to show that sin A B C + sin + sin ≤ , 2 2 which is well-known, and the proof is completed Problem (Open Olympiad of FML No-239, Russia) Let a, b, c be positive real numbers such that a + b + c = Prove the following inequality r r r ab bc ca + + ≤ c + ab a + bc b + ca Solution The inequality is equivalent to s s r ab bc ca + + ≤ , c(a + b + c) + ab a(a + b + c) + bc b(a + b + c) + ca or s ab + (c + a)(c + b) s bc + (a + b)(a + c) r ca ≤ (b + c)(b + a) A B C Using S7 we can substitute the three terms for the expressions sin , sin , sin So, 2 we have to prove that A B C sin + sin + sin ≤ , 2 2 which is well-known, and the conclusion follows Problem (Romania 2005) Let a, b, c be positive real numbers such that (a + b)(b + c)(c + a) = Prove that ab + bc + ca ≤ (181) 150 Chuyên đề Toán học số Solution Let us notice that our inequality is equivalent to  3 3 [(a + b)(b + c)(c + a)]2 (ab + bc + ca) ≤ Since the last inequality is homogenous, we can assume without   loss of generality that A ab + bc + ca = Perform the substitution S1: cyc a = tan , where A, B, C are the angles of a triangle Then the inequality is equivalent to show that  3 ≤ [(a + b)(b + c)(c + a)]2 On the other hand,    C A B cos cos cos     2 (a + b)(b + c)(c + a) =  A B  B C  C A cos cos cos cos cos cos 2 2 2 = A B C cos cos cos 2  It suffices to prove that   3  ≤ 2  , A B C cos cos cos 2 or √ A B C 3 cos cos cos ≤ , 2 which is well-known, and the conclusion follows Problem Let x, y, z be positive real numbers such that xy + yz + zx = Prove that y z 2x(1 − x2 ) 2y(1 − y ) 2z(1 − z ) x + + ≥ + + + x2 + y + z (1 + x2 )2 (1 + y )2 (1 + z )2   A Solution Perform the substitution S1: cyc x = tan , where A, B, C are angles of a triangle Note that x A A = sin cos , 1+x 2 2x(1 − x2 ) = sin A cos A, (1 + x2 )2 and analogous forms for the terms y, z Then the inequality is equivalent to show that sin A A B B C C cos + sin cos + sin cos ≥ sin A cos A + sin B cos C + sin C cos C, 2 2 2 (182) Enlightening Trigonometrical Substitutions 151 or sin A + sin B + sin C ≥ sin 2A + sin 2B + sin 2C B C A From the well-known identities sin A + sin B + sin C = cos cos cos and 2 sin 2A + sin 2B + sin 2C = sin A sin B sin C, it follows that the inequality is equivalent to prove that cos A B C cos cos ≥ sin A sin B sin C, 2 or A B C ≥ sin sin sin 2 The last inequality is well-known and the conclusion follows Problem Let x, y, z be positive real numbers Prove the following inequality hp i√ p p p x(y + z) + y(z + x) + z(x + y) x + y + z ≥ (x + y)(y + z)(z + x) Solution Rewrite the inequality as s s s x(x + y + z) y(x + y + z) z(x + y + z) + + ≥ (x + y)(x + z) (y + z)(y + x) (z + x)(z + y) Applying S8 to the inequality, it follows that we have to prove that cos α β γ + cos + cos ≥ 2, 2   π−α where α, β, γ are angles of a triangle Perform the transformation T1: cyc A = , where A, B, C are angles of an acute-angled triangle So, the inequality is equivalent to prove that sin A + sin B + sin C ≥ On the other hand, using the fact that A, B, C are angles of an acute-angled triangle and the well-known identity sin2 A + sin2 B + sin2 C = + cos A cos B cos C, we have that sin A + sin B + sin C ≥ sin2 A + sin2 B + sin2 C = + cos A cos B cos C > 2, which completes our proof Remark The inequality sin A+sin B+sin C ≥ has other different proofs For example, suppose without loss of generality that A ≥ B ≥ C From the fact that π π  , ,  (A, B, C) 2 (183) 152 Chuyên đề Toán học số and that the function sin x is concave on (0, π), we can apply Karamata’s inequality to obtain that π π sin A + sin B + sin C ≥ sin + sin + sin = 2 Another approach is to use Jordan’s inequality for acute angles, this is  π 2α , ∀α ∈ 0, sin α ≥ π Problem (Turkey TST 2006) Let x, y, z > and xy + yz + zx = Prove that √ 2 √ √ √ 27 (x + y)(y + z)(z + x) ≥ x + y + y + z + z + x ≥   A B C Solution Perform the substitution S1: (x, y, z) = tan , tan , tan , where A, 2 B, C are angles of a triangle Note that A B x + y = tan + tan = 2 cos C A B cos cos 2 , and analogous forms for the terms y + z, z + x Then, the inequality is equivalent to v v 2 v u u u u cos C u cos A u cos B u u √  u 27 2 u u u  ≥ ≥ + + t t t  B C B C A A A B C cos cos cos cos cos cos cos cos cos 2 2 2 2 We will prove first the left hand side inequality From Cauchy-Schwarz we have that 3(a2 + b2 + c2 ) ≥ (a + b + c)2 , then it suffices to prove that  cos C cos A cos B  27   ≥ 3 + + A B C A B B C C A cos cos cos cos cos cos cos cos cos 2 2 2 2 This inequality is equivalent to A B C ≥ cos2 + cos2 + cos2 , which is well-known 2 Now we will prove the right hand side inequality From the inequality (a + b + c)2 ≥ 3(ab + bc + ca), it suffices to prove that   3  cos A + cos B + cos √  ≥  C (184) Enlightening Trigonometrical Substitutions 153 From Cauchy-Schwarz and Jensen’s inequalities, we have that cos A + cos B + cos C ≥ √ ≥ √ = 3, B C A cos + cos + cos 2 2 as desired Problem 10 (Crux Mathematicorum) Let a, b, c be positive real numbers such that a + b + c = abc Prove that √ a5 (bc − 1) + b5 (ca − 1) + c5 (ab − 1) ≥ 54 Solution The hypothesis and S3 allows us to perform the substitution   B C A (a, b, c) = cot , cot , cot 2 We will prove more, namely, that for any nonnegative real number k the following inequality holds k ak (bc − 1) + bk (ca − 1) + ck (ab − 1) ≥ · +1 Note that A ak (bc − 1) = cotk = sink−1 B+C   cos B C A cot cot − = cotk · B C 2 sin sin 2 A cosk A B C sin sin 2 Then, from AM-GM it follows that v v k k u u u u A A B C B C u cos cos cos u cot cot cot X u u 2 2 2 k a (bc − 1) ≥ 3u k+1 = 3u u t A B C t A B C sin sin sin sin sin sin 2 2 2 From the inequalities cot √ A B C A B C cot cot ≥ 3, sin sin sin ≤ it follows that 2 2 2 v k u u A B C u cot cot cot p u 3 k 2 3k 3u ≥ 3 · = · +1 , t A B C sin sin sin 2 as we wanted to prove (185) 154 Chuyên đề Toán học số Problem 11 (Mathematical Reflections) Let a, b, c be positive real numbers, such that a + b + c + = 4abc Prove that 1 1 1 + + ≥3≥ √ +√ +√ a b c ca ab bc Solution Rewrite the condition as 1 1 + + + = bc ca ab abc This and S5 implies that we can take     1 A B C , , = sin , sin , sin , bc ca ab 2 for an arbitrary triangle ABC Note that this implies that   B C C A B A   sin sin sin sin sin sin 1  2, 2, 2 , , =  A B C a b c sin sin sin 2 Then, we have to prove that sin B C B C A A sin sin sin sin sin 2 + 2 + 2 ≥ ≥ sin A + sin B + sin C A B C 2 2 sin sin sin 2 The right hand side inequality is well-known The left hand side inequality has already been proved in Problem 3, and we are done Problem 12 (Iran 1996) Let x, y, z be positive real numbers Prove that   1 (xy + yz + zx) + + ≥ 2 (x + y) (y + z) (z + x) Solution Since this inequality is homogenous we can assume  without loss of generality  α β γ that xy + yz + zx = Perform the substitution S1: (x, y, z) = tan , tan , tan , 2 where α, β, γ are angles of an acute angled triangle So the inequality is equivalent to 1  2 +  2 +  2 ≥ , γ α α β β γ tan + tan tan + tan tan + tan 2 2 2 or α β γ β γ α cos2 cos2 cos2 cos2 cos2 2 + 2 + 2 ≥ γ α β cos2 cos2 cos2 2 cos2 (1) (186) Enlightening Trigonometrical Substitutions 155   π−α Perform the tranformation T1: cyc A = , where A, B, C are angles of an acute angled triangle Thus the inequality is equivalent to  2  2  2 sin A sin B sin B sin C sin C sin A + + ≥ sin C sin A sin B Suppose without loss of generality that f (A, B, C) ≥ π π > A ≥ Rewrite the inequality in the form + 2(sin2 A + sin2 B + sin2 C), where sin A sin B sin B sin C sin C sin A + + sin C sin A sin B From Jensen’s inequality we have that f (A, B, C) = sin2 B + sin2 C ≤ sin2 A B+C = cos2 2 (2) From AM-GM and (2) we have that sin B sin C ≤ cos2 A (3) Consider the difference     sin2 B − C sin2 A sin2 A B+C B+C  2 − 1 d = f (A, B, C) − f A, , =   2 sin A sin B sin C Since π π > A ≥ , note that (3) is equivalent to A ≥ 16 sin4 A ≥ sin B sin C sin2 A sin2 Hence d ≥ 0, so we only need to prove that   B+C B+C f A, , ≥ + 2(sin2 A + sin2 B + sin2 C) 2 By (2) we have that sin2 A + sin2 B + sin2 C ≤ sin2 A + cos2 A , so it is enough to prove that     B+C B+C 2 A f A, , ≥ + sin A + cos 2 (187) 156 Chuyên đề Toán học số This inequality is equivalent to  A   cos   2 A ,  ≥ + sin A + cos 2 sin A + sin A  or cos A(cos A + 1)(2 cos A − 1)2 ≥ 0, which is true, because A is an acute angle, and this completes our proof Problem 13 (MOSP) Let x, y, z be nonnegative real numbers such that xy+yz+zx = 1, and no two of them are equal to zero Prove the following inequality 1 + + ≥ x+y y+z z+x Solution Since this inequality is symmetric, we can assume without loss of generality that x ≥ y ≥ z If z = it results xy = and our inequality is equivalent to 1 + + ≥ , x y x+y or (x + y)2 + ≥ (x + y), or   (x + y − 2) ≥ 0, x+y− √ which is true because we have x + y ≥ xy = from AM-GM Let assume that x, y, z are positive real numbers Perform the substitution S1:   A cyc x = tan , where A, B, C are the angles of triangle ABC Note that A A B B cos cos cos cos 2 = 2, = A+B C x+y sin cos 2 and similar expressions for the other terms Rewrite the inequality as A B B C C A cos cos cos cos cos 2 + 2 + 2 ≥ C A B cos cos cos 2 cos (188) Enlightening Trigonometrical Substitutions 157 Squaring both sides we get A B cos2 X A 25 2 +2 cos2 ≥ C cos2 Using the inequality (1) from Problem 12 it is enough to prove that X cos2 cos2 A B C + cos2 + cos2 ≥ 2, 2 which is true because A B C A B C cos2 + cos2 + cos2 = + sin sin sin ≥ 2 2 2 The proof is completed Problem 14 Let a, b, c be positive real numbers, with a, b, c ∈ (0, 1) such that ab + bc + ca = Prove that   b c − a2 − b − c a + + ≥ + + − a2 − b2 − c2 a b c   A Solution Perform the substitution S1: cyc a = tan , where A, B, C are angles of A B C a triangle Since a, b, c ∈ (0, 1) it follows that tan , tan , tan ∈ (0, 1), this is A, 2 B, C are angles of an acute angled triangle Note that   A A sin cos  a 2 = tan A  cyc  =  1−a cos A So the inequality is equivalent to show that tan A + tan B + tan C ≥ 3 + + , tan A tan B tan C or tan A tan B tan C(tan A + tan B tan C) ≥ 3(tan A tan B + tan B tan C + tan C tan A) Applying T1 to Lemma 1, since A, B, C are angles of an acute-angled triangle we have tan A + tan B + tan C = tan A tan B tan C So, it is sufficient to show that (tan A + tan B tan C)2 ≥ 3(tan A tan B + tan B tan C + tan C tan A) This inequality reduces to (tan A − tan B)2 + (tan B − tan C)2 + (tan C − tan A)2 ≥ The proof is completed (189) 158 Chuyên đề Toán học số Problem 15 (Mathematical Reflections) Let a, b, c be positive real numbers Prove that the following inequality holds s r r r b+c c+a a+b 16(a + b + c)3 + + ≥ a b c (a + b)(b + c)(c + a) Solution Note that the inequality is equivalent to s s s (a + b)(c + a) (b + c)(b + a) (c + a)(c + b) + (c + a) + (a + b) ≥ (b + c) a(a + b + c) b(a + b + c) c(a + b + c) √ 4(a + b + c) ≥ From T2 and S8 it follows that we can perform the substitutions s (a + b)(c + a) x (b + c) = X a(a + b + c) cos and √ √ 4(a + b + c) 4s = , 3 where XY Z is a triangle with sidelengths x, y, z and semiperimeter s Analogously, equivalent expressions are obtained for the other terms Using the sine law and the formulas for the double-angle it yields that we have to prove that √   sin Y sin Z X Y Z sin X + + = sin + sin + sin (sin X + sin Y + sin Z), ≥ X Y Z 2 cos cos cos 2 or equivalently, √   Y Z X Y Y Z Z X X (1) sin + sin + sin ≥ sin cos + sin cos + sin cos 2 2 2 2 2 X X Given that the function sin is increasing on [0, π] and cos is decreasing on [0, π], 2 from Chebyschev’s inequality we have that    X Y Z X Y Z sin + sin + sin cos + cos + cos X X X 2 2 2 ≥ sin cos (2) 2 X Since the function cos is concave on [0, π], from Jensen’s inequality it follows that X Y Z √ cos + cos + cos 2 ≥ (3) From (1), (2) and (3), the conclusion follows (190) 159 Enlightening Trigonometrical Substitutions Exercises Exercise Let a, b, c be positive real numbers Prove the following inequality p p p p ab(a + b) + bc(b + c) + ca(c + a) ≥ 4abc + (a + b)(b + c)(c + a) Exercise Let a, b, c be positive real numbers such that ab + bc + ca = Prove that + a2 b + b c + c a2 + + ≥ 2 (a + b) (b + c) (c + a) Exercise Prove that if x, y, z > satisfy the condition x + y + z = xyz, then p √ √ xy + yz + zx ≥ + + x2 + + y + + z Exercise Let x, y, z be positive real numbers such that x + y + z = xyz Prove that √ (x − 1)(y − 1)(z − 1) ≤ − 10 Exercise Let a, b, c be positive real numbers Prove that   b+c c+a a+b a b c + + ≥4 + + a b c b+c c+a a+b Exercise Let x, y, z be positive real numbers Prove that √ √ √ 4(x + y + z) y+z z+x x+y + + ≥p x y z (x + y)(y + z)(z + x) Exercise Let a, b, c be nonnegative real numbers such that ab + bc + ca = Prove that the following inequality holds √ 1 1 +√ +√ ≥2+ √ c+a a+b b+c Exercise (Crux Mathematicorum) Let x, y, z be positive real numbers such that xy + yz + zx = Prove that √ x(1 − y )(1 − z ) + y(1 − z )(1 − x2 ) + z(1 − x2 )(1 − y ) ≤ Exercise (APMO 2002) Let x, y, z be positive real numbers such that 1 + + = x y z Prove that √ √ √ √ √ √ √ x + yz + x + yz + x + yz ≥ xyz + x + y + z Exercise 10 (APMO 2004) Prove that for all positive real numbers a, b, c, we have (a2 + 2)(b2 + 2)(c2 + 2) ≥ 9(ab + bc + ca) (191) 160 Chuyên đề Toán học số Exercise 11 (Belarus 1996) Let x, y, z be positive real numbers such that √ x + y + z = xyz Prove that xy + yz + zx ≥ 9(x + y + z) Exercise 12 (Iran 2005) If a, b, c are nonnegative real numbers such that 1 + + = 2, 2 1+a 1+b + c2 then prove that ab + bc + ca ≤ Exercise 13 (Romania 2005, Unused) Let a, b, c be positive real numbers such that a + b + c = Show that r a b c √ +√ +√ ≥ c+a b+c a+b Exercise 14 (Ukraine 2005) Let a, b, c be positive real numbers such that a+b+c = Prove that r r r r r r 1 1 1 −1 −1+ −1 −1+ −1 − ≥ a b b c c a Exercise 15 (USAMO 2001) Let a, b, c be nonnegative real numbers numbers such that a2 + b2 + c2 + abc = Prove that ≤ ab + bc + ca − abc ≤ References [1] CMS, Crux Mathematicorum and Mathematical Mayhem [2] T Andreescu, V Cirtoaje, G Dospinescu, M Lascu, Old and New Inequalities, GIL Publishing House, 2004 [3] Tran Phuong, Diamonds in Mathematical Inequalities, Ha Noi Publishing House, 2007 [4] N M Sedrakyan, Geometricheskie Neravenstva, Yerevan, 2004 [5] E Specht, Collected Inequalities [LINK: http://www.imo.org.yu/othercomp/Journ/ineq.pdf] [6] H Lee, Topics in Inequalities – Theorems and Techniques, 2006 [LINK: http://ultrametric.googlepages.com/ineq_hl.pdf] [7] H Lee, Inequalities through problems, 2006 [LINK: http://www.scribd.com/doc/13889461] (192) VỀ MỘT BÀI TOÁN BẤT ĐẲNG THỨC Nguyễn Văn Huyện SV Đại học Giao thông Vận tải thành phố Hồ Chí Minh Trước hết ta xét bài toán sau đây Bài toán Cho a, b, c là các số thực dương Chứng minh bất đẳng thức sau luôn thỏa mãn a2 + b2 + c2 + 2abc + ≥ 2(ab + bc + ca) (∗) Lời giải Đặt a = x + 1, b = y + 1, c = z + 1, ta có x, y, z > −1 Thay vào, bất đẳng thức đã cho có thể viết lại thành x2 + y + z + 2xyz ≥ Vì xy · yz · zx = x2 y z ≥ nên ít ba số xy, yz, zx phải không âm Do tính đối xứng nên ta có thể giả sử xy ≥ 0, đó sử dụng bất đẳng thức AM-GM, ta thu x2 + y + z + 2xyz ≥ 2xy + z + 2xyz = 2xy(z + 1) + z = 2xyc + z ≥ Bài toán chứng minh xong Đẳng thức xảy và a = b = c = Lời giải Theo nguyên lý Dirichlet, ta thấy ba số a, b, c có hai số cùng ≥ cùng ≤ Giả sử hai số đó là a, b, đó (a − 1)(b − 1) ≥ Từ đây, cách sử dụng đẳng thức a2 + b2 + c2 + 2abc + − 2(ab + bc + ca) = (a − b)2 + (c − 1)2 + 2c(a − 1)(b − 1) ≥ 0, ta thu bất đẳng thức (∗) Lời giải Ta sử dụng phương pháp dồn biến để chứng minh bài toán Giả sử c = min{a, b, c} và đặt f (a, b, c) = a2 + b2 + c2 + 2abc + − 2(ab + bc + ca), ta có f (a, b, c) − f Do đó f (a, b, c) ≥ f √  √  √ √ 2  √ ab, ab, c = a− b a + b + ab − 2c ≥ √ ab, √  ab, c Vậy ta cần chứng minh √  √ f ab, ab, c ≥ 161 (193) 162 Chuyên đề Toán học số Thật vậy, đặt t = √ ab thì ta có f (t, t, c) = 2t2 + c2 + 2t2 c − 2(t2 + 2tc) + = (c − 1)2 + 2c(t − 1)2 ≥ Bài toán chứng minh xong Lời giải Sử dụng bất đẳng thức AM-GM, ta có √ 2abc + = abc + abc + ≥ a2 b2 c2 ≥ 9abc a+b+c Do đó, ta cần chứng minh a2 + b + c + 9abc ≥ 2(ab + bc + ca) a+b+c Thực phép khai triển trực tiếp, ta có bất đẳng thức tương đương với a3 + b3 + c3 + 3abc ≥ a2 (b + c) + b2 (c + a) + c2 (a + b), đúng vì đây chính là bất đẳng thức Schur dạng bậc ba Bất đẳng thức (∗) Darij Grinberg đề xuất vào năm 2004 Mặc dù là kết đơn giản bất đẳng thức này lại có nhiều ứng dụng việc chứng minh các bất đẳng thức ba biến Sau đây, chúng ta vào xét các bài toán cụ thể để hiểu rõ vì chúng tôi lại nói Bài toán (Moscow 2000) Cho a, b, c là các số dương thỏa mãn abc = Chứng minh bất đẳng thức sau a2 + b2 + c2 + a + b + c ≥ 2(ab + bc + ca) Lời giải Sử dụng bất đẳng thức AM-GM, ta có √ a + b + c ≥ abc = = 2abc + Vì để chứng minh bài toán, ta cần chứng minh a2 + b2 + c2 + 2abc + ≥ 2(ab + bc + ca) Đây chính là bất đẳng thức (∗) nên ta có điều phải chứng minh Đẳng thức xảy và a = b = c = Bài toán (VMO 2006) Tìm số k lớn để bất đẳng thức 1 + + + 3k ≥ (k + 1)(a + b + c) a b c đúng với a, b, c dương thỏa mãn abc = (194) Về bài toán Bất đẳng thức 163 , đó a, b, c là các số dương và t2 abc = Do đó, theo yêu cầu bài toán ta phải có   + t + 3k ≥ (k + 1) 2t + t2 t Lời giải Cho a = b = t (t > 0, t 6= 1) và c = Bất đẳng thức này tương đương với   + t − ≥ (k + 1) 2t + − , t2 t t6 − 3t2 + (k + 1)(2t3 − 3t2 + 1) ≥ , t2 t2 (t2 − 1)2 (t2 + 2) (k + 1)(t − 1)2 (2t + 1) ≥ , t2 t2 (t + 1)2 (t2 + 2) ≥ k + 1, ∀t > 2t + Cho t → 0+ , ta ≥ k + 1, suy k ≤ Ta chứng minh chính là số cần tìm, tức là 1 + + + ≥ 2(a + b + c) a b c 1 Đặt x = , y = , z = thì ta có xyz = 1, = 2xyz + và a b c a+b+c= 1 xyz xyz xyz + + = + + = xy + yz + zx x y z x y z Do đó bất đẳng thức trên có thể viết lại thành x2 + y + z + 2xyz + ≥ 2(xy + yz + zx), hiển nhiên đúng theo (∗) Vậy ta có kết luận kmax = Bài toán (Romania 2005) Cho các số dương a, b, c thỏa mãn điều kiện (a + b)(b + c)(c + a) = Chứng minh ab + bc + ca ≤ Lời giải Đặt x = a + b, y = b + c, z = c + a thì ta có xyz = và a= z+x−y , b= x+y−z , c= y+z−x Bất đẳng thức cần chứng minh viết lại sau z+x−y x+y−z x+y−z y+z−x y+z−x z+x−y · + · + · ≤ 2 2 2 (195) 164 Chuyên đề Toán học số Sau thu gọn, ta x2 + y + z + ≥ 2(xy + yz + zx), hay là x2 + y + z + 2xyz + ≥ 2(xy + yz + zx) Đây chính là bất đẳng thức (∗) nên ta có điều phải chứng minh Đẳng thức xảy và a = b = c = Bài toán Chứng minh với số thực dương a, b, c, ta có a2 + b2 + c2 + 2abc + ≥ (a + 1)(b + 1)(c + 1) Lời giải Sau khai triển và rút gọn, ta có bất đẳng thức tương đương a2 + b2 + c2 + abc + ≥ ab + bc + ca + a + b + c, hay là 2(a2 + b2 + c2 ) + 2abc + ≥ 2(ab + bc + ca + a + b + c) Theo (∗), ta có a2 + b2 + c2 + 2abc + ≥ 2(ab + bc + ca) Sử dụng đánh giá này, ta đưa bài toán chứng minh a2 + b2 + c2 + ≥ 2(a + b + c) Bất đẳng thức này tương đương với bất đẳng thức hiển nhiên đúng (a − 1)2 + (b − 1)2 + (c − 1)2 ≥ Bài toán chứng minh xong Đẳng thức xảy và a = b = c = Bài toán (APMO 2004) Chứng minh bất đẳng thức sau luôn thỏa mãn với số thực dương a, b, c (a2 + 2)(b2 + 2)(c2 + 2) ≥ 9(ab + bc + ca) Lời giải Bất đẳng thức cần chứng minh tương đương với a2 b2 c2 + 3(a2 + b2 + c2 ) + (a2 + b2 + c2 ) + 2(a2 b2 + b2 c2 + c2 a2 + 3) + ≥ 9(ab + bc + ca) Theo bất đẳng thức AM-GM, ta có   a + b b + c c + a2 2 + + ≥ 3(ab + bc + ca) 3(a + b + c ) = 2 và   2(a2 b2 + b2 c2 + c2 a2 + 3) = (a2 b2 + 1) + (b2 c2 + 1) + (c2 a2 + 1) ≥ 4(ab + bc + ca) (196) 165 Về bài toán Bất đẳng thức Từ đó bài toán quy chứng minh a2 + b2 + c2 + a2 b2 c2 + ≥ 2(ab + bc + ca) (∗∗) Bất đẳng thức này có thể viết lại thành   a + b2 + c2 + 2abc + − 2(ab + bc + ca) + (abc − 1)2 ≥ 0, hiển nhiên đúng theo (∗) Đẳng thức xảy và a = b = c = Nhận xét Bài toán còn đúng trường hợp a, b, c là các số thực Thật vậy, từ chứng minh trên ta thấy (a2 + 2)(b2 + 2)(c2 + 2) = (|a|2 + 2)(|b|2 + 2)(|c|2 + 2) ≥ 9(|a||b| + |b||c| + |c||a|) ≥ 9(ab + bc + ca) Ngoài ra, bất đẳng thức trên còn có thể làm chặt lên thành Bài toán (Mở rộng APMO 2004) Chứng minh bất đẳng thức sau luôn đúng với số thực dương a, b, c (a2 + 2)(b2 + 2)(c2 + 2) ≥ 3(a + b + c)2 Lời giải Tương tự trên, ta sử dụng phép khai triển trực tiếp và viết lại bất đẳng thức dạng a2 b2 c2 + a2 + b2 + c2 + 2(a2 b2 + b2 c2 + c2 a2 + 3) + ≥ 6(ab + bc + ca) Đến đây, ta sử dụng bất đẳng thức AM-GM 2(a2 b2 + b2 c2 + c2 a2 + 3) ≥ 4(ab + bc + ca), và đưa bất đẳng thức chứng minh a2 + b2 + c2 + a2 b2 c2 + ≥ 2(ab + bc + ca) Đây chính là bất đẳng thức (∗∗) Bài toán chứng minh xong Đẳng thức xảy a = b = c = Bài toán Cho a, b, c là các số thực dương Chứng minh (a2 + 2)(b2 + 2)(c2 + 2) ≥ 3(a + b + c)2 + (abc − 1)2 Lời giải Sau khai triển và rút gọn, ta bất đẳng thức tương đương 2(a2 b2 + b2 c2 + c2 a2 + 3) + a2 + b2 + c2 + 2abc + ≥ 6(ab + bc + ca) Theo bất đẳng thức AM-GM thì 2(a2 b2 + b2 c2 + c2 a2 + 3) ≥ 4(ab + bc + ca) Do đó ta cần chứng minh a2 + b2 + c2 + 2abc + ≥ 2(ab + bc + ca) Đây chính là bất đẳng thức (∗) Bài toán chứng minh xong Đẳng thức xảy và a = b = c = (197) 166 Chuyên đề Toán học số Bài toán (Iran 2002) Cho các số dương a, b, c thỏa mãn a2 + b2 + c2 + abc = Chứng minh a + b + c ≤ Lời giải Từ giả thiết sử dụng bất đẳng thức (∗), ta có = 2(a2 + b2 + c2 + abc) + = a2 + b2 + c2 + (a2 + b2 + c2 + 2abc + 1) ≥ a2 + b2 + c2 + 2(ab + bc + ca) = (a + b + c)2 , từ đó suy a + b + c ≤ Đẳng thức xảy và a = b = c = Bài toán 10 (Hello IMO 2007) Chứng minh với a, b, c dương, ta có 2(a2 + b2 + c2 ) + abc + ≥ 5(a + b + c) Lời giải Sử dụng bất đẳng thức AM-GM, ta có a+b+c=  1 · · (a + b + c) ≤ + (a + b + c)2 Do đó ta cần chứng minh   12(a2 + b2 + c2 ) + 6abc + 48 ≥ (a + b + c)2 + Bất đẳng thức này tương đương với 7(a2 + b2 + c2 ) + 6abc + ≥ 10(ab + bc + ca),   4(a2 + b2 + c2 − ab − bc − ca) + a2 + b2 + c2 + 2abc + − 2(ab + bc + ca) ≥ 0, đúng vì ta có a2 + b2 + c2 ≥ ab + bc + ca (theo AM-GM) và a2 + b2 + c2 + 2abc + ≥ 2(ab + bc + ca) (theo (∗)) Bài toán chứng minh xong Đẳng thức xảy và a = b = c = Tài liệu tham khảo [1] Phạm Kim Hùng, Sáng tạo Bất đẳng thức, Nhà xuất Tri Thức, 2006 [2] Võ Quốc Bá Cẩn, Trần Quốc Anh, Bất đẳng thức và lời giải hay, Nhà xuất Hà Nội, 2009 [3] Võ Quốc Bá Cẩn, Trần Quốc Anh, Sử dụng phương pháp Cauchy-Schwarz để giải toán Bất đẳng thức, Nhà xuất Đại học Sư phạm Hà Nội, 2010 [4] http://mathlinks.ro (198) TỔ HỢP VÀ CÔNG THỨC Ck2 Đặng Hoàng Linh HS chuyên Toán khóa 2008 - 2011 Ví dụ Có 80 ủy ban tạo 1600 ủy viên, ủy ban có đúng 80 ủy viên Chứng minh tồn hai ủy ban có ít ủy viên chung Lời giải Thay vì đếm số cặp ủy ban bất kỳ, ta đếm số “cặp” ủy ban mà ủy viên tham dự Đánh số thứ tự các ủy viên 1, 2, , 1600, các ủy viên này có số ủy ban tham gia là k1 , k2 , , k1600 Có n ủy ban (n = 80), ủy ban có đúng 80 ủy viên nên k1 + k2 + · · · + k1600 = 80n Với ủy viên i, số cặp ủy ban mà ủy viên đó tham gia là Ck2i Vậy tổng cộng có 1600 X Ck2i i=1 Bây ta chứng minh bài toán phản chứng, giả sử cặp ủy ban có nhiều thành viên chung, nên có nhiều 3Cn2 cặp ủy ban Do đó, ta có 1600 X Ck2i ≤ 3Cn2 i=1 Mặt khác, lại có 1600 X 1600 X i=1 Ck2i = 1600 X ki (ki − 1) i=1 = i=1 ki2 − 1600 X ki i=1 1600 X ≥ !2 1600 X ki i=1 1600 − i=1 ki = 2n2 − 40n Từ đây ta có 2n2 − 40n ≤ n(n − 1), suy n ≤ 77 (vô lý) Vậy bài toán giải xong Ta nhận thấy nhiều bài toán thay vì đếm số phần tử thì ta đếm số cặp phần tử, đôi lại giúp ta giải nhiều bài toán khó, ta thử bắt đầu bài toán khó Ví dụ Một hội nghị sử dụng ngôn ngữ chính thức Biết với hai đại biểu luôn có ngôn ngữ mà hai biết, chứng minh có ngôn ngữ biết ít 60% đại biểu Khi nhìn vào đề bài trên, có lẽ khó định hướng để giải, là số 60% khá mơ hồ Nhưng ta thử quên cách đếm thông thường mà thử tìm số cặp phần tử nó Lời giải Nếu có người nào đó biết ngôn ngữ thì theo giả thiết ta suy tất người còn lại biết ngôn ngữ đó, và kết luận bài toán là hiển nhiên 167 (199) 168 Chuyên đề Toán học số Giả sử tất đại biểu biết ít ngôn ngữ Gọi số đại biểu là n và tập hợp A, B, C, D là tập người biết các ngôn ngữ I, II, III, IV Số cặp người biết ngôn ngữ A, B, C, D là CA2 , CB2 , CC2 , CD Do người biết ít ngôn ngữ, cho nên ta có bất đẳng thức sau CA2 + CB2 + CC2 + CD ≥ 2Cn2 Bất đẳng thức này tương đương với  A(A − 1) + B(B − 1) + C(C − 1) + D(D − 1) ≥ 2n(n − 1), 2  2  2  2 1 1 + B− + C− + D− ≥ 2n(n − 1) + A− 2 2 (1) Không tính tổng quát, ta giả sử A là tập lớn các tập A, B, C, D Khi đó, từ (1) ta có  2 1 A− ≥ (2n2 − 2n + 1) Từ bất đẳng thức này dễ dàng chứng minh A > n với n ≥ 10 Qua hai ví dụ trên, hẳn các bạn đã thấy hữu dụng việc sử dụng công thức Ck2 việc giải số bài toán tổ hợp Nhờ nó mà chúng ta có thể giải số bài toán chứng minh đẳng thức khá hóc búa bài IMO 1998 Ví dụ (IMO 1998) Trong kỳ thi có m thí sinh và n giám khảo, đó n là số lẻ Cho biết thí sinh phải chấm tất giám khảo hai giá trị “chấm đỗ” và “chấm hỏng” Giả sử cặp giám khảo chấm nhiều k thí sinh Chứng minh n−1 k ≥ m 2n Lời giải Chúng ta đếm số cặp giám khảo chấm thí sinh Xét thí sinh thứ i (1 ≤ i ≤ m) chấm đỗ xi giám khảo và bị chấm hỏng yi giám khảo Rõ ràng xi + yi = n Lúc đó số cặp giám khảo chấm thí sinh thứ i là xi (xi − 1) yi (yi − 1) x2 + yi2 xi + yi + = i − 2 2  (xi + yi )2 xi + yi 1 ≥ − = n − n= (n − 1)2 − 2 4 Cx2i + Cy2i = Vì n lẻ và Cx2i + Cy2i là số nguyên nên Cx2i + Cy2i ≥ (n − 1)2 Ta có tất n giám khảo và cặp giám khảo chấm nhiều nhất k thí sinh nên có nhiều kCn2 cặp giám khảo Vậy ta có bất đẳng thức kCn2 ≥ m X i=1 (Cx2i + Cy2i ) ≥ m(n − 1)2 (200) Tổ hợp và công thức Ck2 169 k (n − 1)2 n−1 k Từ đây suy ≥ ≥ Bài toán chứng minh , hay m 4Cn m 2n Ví dụ (Đề kiểm tra đội dự tuyển Đại học Sư phạm) Cho n là số nguyên lớn và P1 , P2 , , Pn là các tập có hai phần tử và đôi phân biệt tập hợp S = {1, 2, , n} thỏa mãn tính chất: i 6= j mà Pi ∩ Pj khác rỗng thì tồn k để Pk = {i, j} Chứng minh với số i ∈ S xuất đúng hai lần các tập Pj với j = 1, 2, , n Lời giải Gọi x1 , x2 , , xn là số lần xuất 1, 2, , n các tập Pj với j = 1, 2, , n Khi đó x1 + x2 + · · · + xn = 2n Tổng số các cặp Pi , Pj phân biệt mà giao chúng khác rỗng là ! n n X X x2i − 2n ≥ n Cx2i = i=1 i=1 Dấu xảy và x1 = x2 = · · · = xn = (đpcm) Sau đây là số bài toán để luyện tập Bài tập (Liên Xô 1965) Một hội thảo có 40 họp, họp có 10 thành viên tham dự Cho biết thành viên cùng dự họp với tối đa lần, chứng minh hội thảo có nhiều 60 thành viên Bài tập (IMO 1989) Cho n, k là hai số nguyên dương thỏa mãn n ≥ k và S là tập hợp gồm n điểm mặt phẳng có hai tính chất: (a) Không có ba điểm nào S thẳng hàng; (b) Với điểm P ∈ S có không ít k điểm S cách P √ Chứng minh k < + 2n Bài tập Cho dãy số hữu hạn có n phần tử, số n gọi là độ dài dãy Giả sử dãy số gồm toàn các chữ số và 1, ta gọi khoảng cách hai dãy là số các vị trí khác hai dãy Ví dụ 1101011 và 1011000 là hai dãy có độ dài và có khoảng cách vì có các vị trí số 2, số 3, số và số khác Bây cho m dãy trên Giả sử hai dãy có khoảng cách nhỏ d, chứng minh m≤ 2d 2d − n Bài tập Cho n điểm thẳng hàng A1 , A2 , , An (n ≥ 4) Đặt M = {Ai Aj | ≤ i < j ≤ n} Biết Ai Aj có độ dài nguyên với i 6= j, chứng minh có ít có độ dài chia hết cho |M | đoạn Ai Aj (201) 170 Chuyên đề Toán học số Bài tập Trong mặt phẳng cố định hệ tọa độ Oxy, chúng ta xét tập hợp R gồm điểm với tọa độ (x, y), đây x, y là số nguyên và ≤ x ≤ 12, ≤ y ≤ 10 Mỗi điểm tô màu trắng, xanh đỏ Chứng minh tồn hình chữ nhật có các cạnh song song với các trục tọa độ, mà đỉnh nó là điểm R sơn cùng màu (202) MỞ RỘNG TỪ MỘT BÀI TOÁN Từ Nguyễn Thái Sơn HS chuyên Toán khóa 2008 - 2011 Trong kỳ thi IMO lần thứ 29 năm 1987 có bài toán sau Bài toán Gọi pn (k) là số hoán vị tập {1, 2, , n} có đúng k điểm cố định Chứng minh n X kpn (k) = n! k=0 Bài toán này có lời giải khá đơn giản với phương pháp đếm hai cách Ta cùng xem xét các lời giải sau Lời giải Ta tính số các cặp (f, i) với f là hoán vị và i là điểm cố định cách • Cách Xét f là hoán vị bảo tồn đúng k điểm, i là các điểm cố định f Rõ ràng i có k cách chọn, còn số các hoán vị có k điểm cố định theo định n X nghĩa là pn (k) nên ta có số các cặp (f, i) là kpn (k) k=0 • Cách Cố định phần tử x Với giá trị x ta có (n − 1)! hoán vị nhận x làm điểm cố định, mà x có n cách chọn nên ta có tất n! cặp (f, i) Từ hai cách tính trên, ta suy n! = n X kpn (k) Bài toán chứng minh k=0 Lời giải Ta có đẳng thức sau n X pn (k) = n! (1) k=0 Thật vậy, ta biết tập {1, 2, , n} có tất n! hoán vị, gồm các loại không bảo tồn phần tử nào loại bảo tồn n phần tử, đó ta có (1) Mặt khác, ta có đẳng thức sau với k > kpn (k) = npn−1 (k − 1) (2) Thật vậy, ta đếm số các cặp (f, i) với f là hoán vị có đúng k điểm cố định và i là các điểm đó, rõ ràng số các cặp là kpn (k) Mặt khác ta có thể cố định giá trị tùy ý và tính số hoán vị bảo tồn đúng k − phần tử n − phần tử còn lại, và số các cặp là npn−1 (k − 1) Từ đây ta có (2) 171 (203) 172 Chuyên đề Toán học số Từ (1) và (2) ta có n X kpn (k) = n k=0 n X pn−1 (k − 1) = n · (n − 1)! = n! k=1 Bài toán chứng minh Thực chất hai cách giải trên không quá khác Cách có lẽ ngắn gọn cách 2, ta có mối liên hệ (2) kpn (k) = npn−1 (k − 1) Liên hệ này khiến ta nghĩ không thử tìm cách tính trực tiếp các số pn (k)? Từ (2), ta có n n(n − 1) · · · (n − k + 1) pn−1 (k − 1) = · · · = pn−k (0) k k! n! = pn−k (0) = Cnk pn−k (0) k!(n − k)! pn (k) = (3) Từ đây ta thu các công thức (có thể xem bài toán mới) n X pn (k) = k=0 n X k=0 Cnk pn−k (0) = n!, n X k=0 kpn (k) = n X kCnk pn−k (0) = n! k=0 Điều thú vị là từ pn (k) = Cnk pn−k (0), ta có thể quy việc tính pn (k) việc tính pn (0) Dường khả tính pn (k) là có thể thực và đã đơn giản nhiều Giờ ta tìm cách tính pn (0) Một cách tự nhiên, ta tính vài giá trị đầu pn (0) : • p1 (0) = 0; • p2 (0) = 1; • p3 (0) = 2; • p4 (0) = 9; • Ta có các nhận xét: = · (2 + 1), = · (1 + 0), Vậy phải pn+1 (0) = n [pn (0) + pn−1 (0)]? Thực tế điều này đúng và chứng minh dễ dàng Xét S = {1, 2, , n, n + 1} Ta tìm cách tính số hoán vị không có điểm cố định S Hoán vị này có thể thành lập sau: Đầu tiên lấy số n + 1, có n cách đặt nó vào n vị trí (1, 2, , n) để f (n + 1) 6= n + Giả sử n + đặt vào vị trí i (i ∈ {1, 2, , n}), ta cần xếp n số {1, 2, , n} vào n vị trí {1, 2, , n + 1} \ {i} (204) 173 Mở rộng từ bài toán Đặt G = {1, 2, , n + 1} \ {i} Nhận thấy G và {1, 2, , n} có đúng n − số giống nhau, hai số khác là G có n + và tập {1, 2, , n} có i Ta có hai khả xảy xếp n số {1, 2, , n} vào n vị trí {1, 2, , n + 1} \ {i} • Nếu i xếp vào n + thì số các cách xếp n − số còn lại chính là số các hoán vị không có điểm cố định n − phần tử Trường hợp này có pn−1 (0) hoán vị • Nếu i không xếp vào n + 1, ta coi i giống n + 1, số các hoán vị trường hợp này là pn (0) Từ các lập luận trên, ta có pn+1 (0) = n [pn (0) + pn−1 (0)] (Ví dụ, xét tập {1, 2, 3, 4, 5}, ta cần tính p5 (0) Giờ ta cần tìm vị trí để đặt số 5, chẳng hạn |{z} Giờ ta xếp bốn số 1, 2, 3, vào các vị trí 2, 3, 4, Có hai khả • Nếu số rơi vào vị trí : |{z} 5 |{z} Giờ ta phải xếp {2, 3, 4} vào {2, 3, 4} cho không có điểm cố định Rõ ràng có p3 (0) hoán vị sinh • Nếu số không rơi vào vị trí 5, ta coi “là” và số hoán vị sinh là p4 (0) Do ban đầu có cách chọn vị trí nên ta có p5 (0) = [p4 (0) + p3 (0)]) Vấn đề đây rõ ràng, ta cần tìm công thức tổng quát pn (0) với ( p1 (0) = 0, p2 (0) = pn+1 (0) = n [pn (0) + pn−1 (0)] Để cho gọn, ta đặt yn = pn (0) Khi đó, ta có ( y1 = 0, y2 = yn+1 = n(yn + yn−1 ), ∀n ≥ (4) Hệ thức liên hệ yn+1 , yn và yn−1 có thể viết lại thành yn+1 − (n + 1)yn = −(yn − nyn−1 ) Đặt zn = yn − nyn−1 , ta z2 = y2 − 2y1 = và zn+1 = −zn Dễ dàng tìm công thức tổng quát zn là zn = (−1)n , ∀n ≥ 2, suy yn − nyn−1 = (−1)n , hay yn = nyn−1 + (−1)n , ∀n ≥ (205) 174 Chuyên đề Toán học số Từ đây ta có (chú ý y1 = 0) yn = nyn−1 + (−1)n = n(n − 1)yn−2 + n1 (−1)n−1 + (−1)n = n(n − 1)(n − 2)yn−3 + 2!Cn2 (−1)n−2 + 1!Cn1 (−1)n−1 + 0!Cn0 (−1)n = · · · = n(n − 1) · · · (n − k)yn−k−1 + n−k X i!Cni (−1)n−i i=0 = n(n − 1) · · · (2)y1 + n−2 X i!Cni (−1)n−i i=0 = n−2 X i!Cni (−1)n−i i=0 Như vậy, ta đã tính pn (0), cụ thể là pn (0) = yn = n−2 X i!Cni (−1)n−i (5) i=0 Liệu công thức này có rút gọn hay không? Bản thân tôi chưa trả lời câu hỏi này, hy vọng trao đổi thêm với các bạn Bây giờ, quay trở lại bài toán ta Từ (3) ta có pn (k) = Cnk pn−k (0) = Cnk n−k−2 X i i!Cn−k (−1)n−k−i (6) i=0 Từ công thức (6) này, ta có thể tạo nhiều bài toán khá khó, ví dụ n X k=0 ( Cnk n−k−2 X )  i  i!Cn−k (−1)n−k−i = n! i=0 Tuy nhiên, bài toán này thường không tự nhiên và khá khó, điều mà ta quan tâm là ta thu  kp (k) = npn−1 (k − 1)   n   p (k) = C k p (0) n n n−k  p (0) = n [p (0) + p (0)] n+1 n n−1    pn (0) − npn−1 (0) = (−1)n (7) Để kết thúc bài viết, ta xem xét ứng dụng (7) toán sơ cấp Bài toán Cho S = {1, 2, , n} Gọi xn là số hoán vị có đúng điểm cố định và yn là số hoán vị không có điểm cố định nào S Chứng minh |xn − yn | = (206) Mở rộng từ bài toán 175 Lời giải Với các kết trên, bài toán này khá đơn giản Ta có xn = pn (1) = npn−1 (0) = nyn−1 và yn = pn (0) = npn−1 (0) + (−1)n = nyn−1 + (−1)n Từ đây suy xn − yn = (−1)n , đó hiển nhiên ta có |xn − yn | = (đpcm) Nếu không biết pn (0) = npn−1 (0) + (−1)n , ta có thể giải bài toán sau: Do xn − yn = nyn−1 − (n − 1)(yn−1 + yn−2 ) = yn−1 − (n − 1)yn−2 = −(xn−1 − yn−1 ) nên |xn − yn | = |xn−1 − yn−1 | = · · · = |x1 − y1 | = Công thức (7) sâu tiếp tục, các bạn còn tìm thấy nhiều đẳng thức thú vị Việc đào sâu vào kết luôn tạo nhiều điều thú vị, xin chúc các bạn tìm nhiều điều thú vị cho riêng mình Tài liệu tham khảo [1] Nguyễn Văn Mậu, Trần Nam Dũng, Vũ Đình Hòa, Đặng Huy Ruận, Đặng Hùng Thắng, Chuyên đề chọn lọc Tổ hợp và Toán rời rạc, Nhà xuất Giáo Dục, 2008 [2] Titu Andreescu, Zuming Feng, A Path to Combinatorics for Undergraduates: Counting Strategies, Birkhauser, 2004 [3] Titu Andreescu, Zuming Feng, 102 Combinatorial Problems: From the Training of the USA IMO Team, Birkhauser, 2002 [4] Gerald Berman, K D Fryer, Introduction to Combinatorics, Waterloo – Academic Press, 1972 (207) 176 Chuyên đề Toán học số d THƠ HAY TOÁN HỌC Pi The admirable number pi: three point one four one All the following digits are also initial, five nine two because it never ends It can’t be comprehended six five three five at a glance eight nine by calculation, seven nine or imagination, not even three two three eight by wit, that is, by comparison four six to anything else two six four three in the world The longest snake on earth calls it quits at about forty feet More than Counting One added forever joined by zero, paired to opposites– these build the integers, base for construction of more new numbers from old: ratios, radical roots and transcendentals, transfinite cardinals–constructions bold! (208) ỨNG DỤNG CỦA TOÁN HỌC TRONG VIỆC HIỂN THỊ CÁC ĐỐI TƯỢNG HÌNH ẢNH BẰNG MÁY VI TÍNH Phạm Mộng Bảo SV Đại học Sư phạm thành phố Hồ Chí Minh Bằng máy vi tính chúng ta có thể thực nhiều công việc với hình ảnh, và cuối cùng là thể hình ảnh qua các in Chắc hẳn bạn đã chiêm ngưỡng in chất lượng cao có hình ảnh sắc nét, độc đáo Và có bạn tự hỏi: “Máy vi tính đã thể các in đó nào? Có bao nhiêu cách thể vậy? Cách nào là tối ưu?”, bài viết này chúng ta cùng tìm hiểu các vấn đề này Để tiện cho việc tìm hiểu, ta hạn chế các hình ảnh mà ta đề cập đây là ảnh “trắng đen”, nghĩa là không đề cập đến biến “màu sắc” Phương pháp dùng hệ tọa độ Có thể nói đây là phương pháp đơn giản nhất, nhằm gây niềm tin cho phương pháp này chúng ta cùng xem qua tác phẩm “Chiều chủ nhật trên đảo La grande Jatte” Georges Seurat Để thể ảnh này Georges không dùng bút lông thông thường mà dùng vô số chấm chấm nhỏ có màu sắc theo phong cách hội họa gọi là trường phái “chấm chấm” (pointillism) Bạn có thể nhìn thấy chấm bạn đứng khá gần tranh, bạn xa khỏi tranh thì các chấm cuối cùng pha trộn lẫn không còn phân biệt Từ đó tạo nên hình mà bạn chiêm ngưỡng đây Hình Chiều chủ nhật trên đảo La grande Jatte 177 (209) 178 Chuyên đề Toán học số Hiệu tranh “Chiều chủ nhật trên đảo La grande Jatte” có sở Vật lý là “nhiễu xạ” Nhưng nói tóm lại là cần thể dấu chấm thích hợp, đủ nhiều, nơi cần thiết là ta có thể thu hình ảnh không quá tầm thường đã miêu tả Vậy các kiến thức hệ tọa độ Descartes hữu dụng phương pháp này Và ta có thể xem phương pháp này khiến máy tính có cách hiểu hình ảnh là tập hợp các điểm có tọa độ (x, y) kiểu tập A A = {(1, 1); (3, 1); (1, 2); (3, 2); (2, 2); (1.64, 1.64); (1.5, 2); (2.5, 2); (3, 1.5); (2, 1); (1.5, 1); (2.5, 1); (2.75, 1); (2.25, 1); (1.75, 1); (1.25, 1); (1, 1.5); (1.74, 1.66); (1, 1.75); (1, 1.25); (3, 1.75); (3, 1.25); (2.75, 2); (2.25, 2); (1.75, 2); (1.25, 2); (2, 1.2); (2.54, 1.34); (1.52, 1.34); (2.3, 1.24); (1.75, 1.24); (1.87, 1.21); (1.63, 1.28); (1.58, 1.31); (1.69, 1.26); (1.93, 1.2); (2.31, 1.26); (2.19, 1.22); (1, 1.88); (1, 1.63); (1, 1.38); (1, 1.13); (1.13, 1); (1.38, 1); (1.63, 1); (1.88, 1); (1.13, 2); (1.38, 2); (1.63, 2); (1.88, 2); (1.06, 2); (1, 1.94); (1, 1.81); (1, 1.69); (1, 1.56); (1, 1.44); (1, 1.31); (1, 1.19); (1, 1.06); (1.06, 1); (1.19, 1); (1.31, 1); (1.44, 1); (1.56, 1); (1.69, 1); (1.81, 1); (2.25, 1.24); (1.81, 1.22); (2.22, 1.63); (1.94, 1); (2.07, 1.2); (2.81, 2); (1.94, 2); (1.81, 2); (1.69, 2); (1.56, 2); (1.44, 2); (1.31, 2); (1.19, 2); (2.36, 1.64); (1.69, 1.68); (1.78, 1.63); (1.66, 1.67); (2.06, 2); (2.13, 2); (2.19, 2); (2.26, 1.66); (2.31, 1.68); (2.34, 1.67); (2.13, 1.21); (2.37, 1.28); (2.42, 1.31); (2.48, 1.34); (2.38, 2); (2.44, 2); (2.31, 2); (2.63, 2); (2.56, 2); (2.69, 2); (2.88, 2); (2.94, 2); (3, 1.88); (3, 1.81); (3, 1.94); (3, 1.63); (3, 1.56); (3, 1.69); (3, 1.38); (3, 1.13); (2.88, 1); (2.63, 1); (2.38, 1); (2.13, 1); (2.06, 1); (2.19, 1); (2.31, 1); (2.44, 1); (2.56, 1); (2.69, 1); (2.81, 1); (2.94, 1); (3, 1.06); (3, 1.19); (3, 1.31); (3, 1.44)} Bảng Liệt kê các phần tử tập hợp A Và hình là cái mà tập A thể hệ trục tọa độ Descartes, với x là hoành độ, y là tung độ y x Hình Hình vẽ minh họa tập hợp A Phương pháp này khá dễ hiểu nên chúng ta chẳng cần phải giải thích dong dài, cái ta quan tâm đây là làm theo phương pháp này cái lợi gì? Nhưng trước chúng ta tìm cho nó nhận xét phù hợp, bạn đọc hãy tham khảo thêm cách biểu diễn điểm hệ tọa độ khác Hệ tọa độ độc cực Định nghĩa Trong mặt phẳng chọn điểm O cố định gọi là cực và tia Ox gọi là tia cực Vị trí điểm M trên mặt phẳng hoàn toàn xác định hai đại lượng  −−→ r = OM, ϕ = Ox, OM , đó r là bán kính vector và ϕ là góc cực điểm M (Chú ý rằng, ϕ là góc định hướng có chiều dương ngược với chiều quay kim đồng hồ.) (210) Ứng dụng Toán học việc hiển thị hình ảnh máy vi tính 179 Cặp (r, ϕ) gọi là các tọa độ cực điểm M Để biểu diễn tất các điểm mặt phẳng, rõ ràng cần hạn chế r ≥ 0, ≤ ϕ ≤ 2π Khi ấy, điểm M (≡ O) có cặp (r, ϕ) tương ứng và cặp (r, ϕ) tương ứng với điểm Với góc O thì r = và ϕ y M ϕ x O Hình Hệ tọa độ độc cực Bây lấy trục tọa độ Descartes cho gốc tọa độ trùng với cực và nửa dương trục hoành trùng với trục cực (xem hình) Vậy ta có  p (  x2 + y r = x = r cos ϕ (∗) hay ngược lại  tan ϕ = y y = r sin ϕ x Để từ (∗) xác định góc ϕ, ta cần chọn ϕ cho sin ϕ cùng dấu với y √ Ví dụ, tọa độ điểm M hệ tọa độ Descartes là x = , y = Khi đó r = 1, 2 √ tan ϕ = 3, ta có hai góc π π ϕ1 = , ϕ2 = + π 3 √  π π π > Vậy M : 1, Ta chọn ϕ = vì sin > cùng dấu với y = 3 Thoạt nhìn đây là hệ thống tọa độ khác mà thôi, nhiên đây là hệ thống tọa độ mà các nhà lập trình phần mềm hay dùng, ta nên tìm hiểu qua, từ đó nhìn tổng quan ta có đánh giá mang tư tưởng “thuật toán” cho phương pháp này sau ◦ Ưu điểm: dễ hiểu, trực quan, số lượng biến phải dùng ít, biến vị trí (cho x, y), biến màu sắc (nếu có) ◦ Nhược điểm: số lượng điểm cần phải dùng cho đối tượng quá nhiều dẫn đến tốn dung lượng cách đáng kể, và vì mà muốn sửa chữa hình ảnh đó bạn phương hướng Không thể phóng đại (zoom) lên nhiều lần bạn không muốn hình ảnh mình trở nên vô nghĩa Vì lẽ đó mà phương pháp này thường dùng để mô tả các hình ảnh mà ta không cần quan tâm nhiều đến độ nét và chi tiết, nhiều phần mềm vẽ hình, xem ảnh dùng phương pháp này Để cải tiến đáng kể việc phóng đại (zoom) phương pháp dùng hệ tọa độ, người ta bắt đầu suy nghĩa đến dùng đồ thị hàm số, nhiên lời khuyên là ta phải nhìn nhận cách nghiêm túc liên tục hàm số đó ta muốn đạt hiệu tốt (211) 180 Chuyên đề Toán học số Phương pháp đồ thị Ở đây ta không dùng đồ thị hàm số biểu diễn dạng bảng giá trị kiểu x f (x) −1 3 −2 đơn giản là vì đồ thị chúng rời rạc, làm chẳng khác nào √ phương pháp hệ tọa độ Cái mà ta cần tìm hiểu là các đồ thị các hàm số kiểu y = − x2 hay đồ thị biểu diễn hợp thành hai hàm số  p 3√ y= x2 − − x2   p 3√ y= x2 + − x2 mà đồ thị chúng hình là y 1.5 y 2.0 1.0 1.5 0.5 1.0 -1.0 0.5 -2 -1 -0.5 x -0.5 0.5 1.0 x -0.5 -1.0 Hình Đồ thị miêu tả vài hàm số Từ đó ý tưởng nảy là ta làm cho máy vi tính hiểu cách vẽ đồ thị hàm số, để chúng vẽ hình này cho ta Ban đầu việc làm này có quá vô nghĩa, vì học các lớp THCS, ta biết muốn vẽ đồ thị hàm số nào đó, ta phải xác định số lượng điểm đủ nhiều để có thể nối chúng lại tạo thành đồ thị, làm chính là tạo điểm hệ tọa độ để vẽ hình (212) Ứng dụng Toán học việc hiển thị hình ảnh máy vi tính 181 (quay lại phương pháp hệ tọa độ) còn gì! Tuy nhiên cách làm này có khác là ta cụ thể a số lượng điểm (của hình vẽ) cần vẽ chính xác đây với tỉ lệ phóng đại cho trước Ví dụ để vẽ đường tròn, thay vì tạo hình vẽ cho tập hợp có số lượng điểm là B1 (có các tọa độ rời rạc, xem chừng vô ý thức), ta vẽ hình cho tập hợp là B2 có số lượng điểm (phân biệt) tùy ý thích (ta cho trước) rải trên biến hoành độ (làm vầy để tránh việc điểm ảnh hình √ vẽ phần nhỏ đồ thị, mà không trải dài khắp hình) và thỏa mãn điều kiện y =a − x2 (xem hình 5), điều này thật là ứng dụng triệt để khái niệm hàm số liên tục (xem lại sách giáo khoa Đại số và Giải tích 11) qua việc liên hệ số ε với tỉ lệ phóng đại cho trước, từ đó ta nhận thấy cho dù có phóng đại cỡ nào nữa, ta có cảm giác đồ thị chúng ta liền nét y y x −1 x −1 Hình Đồ thị miêu tả tập B1 và B2 (56 điểm ảnh) tỉ lệ 1.2 : Một vấn đề phát sinh là không phải đối tượng hình ảnh nào chúng ta mô tả cách trơn tru hàm số cả, việc tham khảo qua các dạng hàm số có dạng khác là điều nên làm chẳng hạn Hàm khúc Thường cho biểu diễn hình thức  f1 (x)      f (x) y=  ···     fn (x) sau x ∈ D1 x ∈ D2 ··· x ∈ Dn đó fi là hàm số xác định trên Di và ∀(i, j),p i 6= j ⇒ Di ∩ Dj = ∅y (hay là các miền Di rời nhau) Đây là hàm số không sơ cấp, thường sử dụng phương pháp này Một đại diện cho loại hàm này chính là hàm ( −x x ≥ y = |x| = −x x < Ta phân tích hiệu phương pháp này: ◦ Ưu điểm: trực quan, số lượng biến phải dùng tương đối ít (nhưng nhiều phương pháp hệ tọa độ, thêm các biến phân hoạch theo hoành độ), có thể chỉnh sửa đối tượng hình ảnh cách thay đổi hàm số chúng, cho phép khả phóng đại hình ảnh (213) 182 Chuyên đề Toán học số ◦ Nhược điểm: số lượng điểm cần phải dùng quá nhiều (biến thiên theo tỉ lệ phóng đại), khó mã hóa liệu đầu vào cho phù hợp (khó tìm hàm số phù hợp để vẽ đồ thị chúng), thuật toán để thực còn phức tạp Phương pháp này cải tiến số điểm so với phương pháp tọa độ, mặt khác nó có hạn chế riêng mình Các phần mềm Toán học thường dùng phương pháp này để vẽ đồ thị (như Geogebra chẳng hạn1 ) y y 10 30 20 10 -10 -5 10 x -20 -10 10 20 x -5 -10 -10 -20 Hình ( Đồ thị miêu tả hàm số r = 2(1 + cos 7ϕ) (hàm số theo tọa độ độc cực) x = 10(2 + sin 5t) cos t và , t ∈ [−10, 10] (hàm số f (x, y) = theo tham số t) y = 10(2 + sin 5t) sin t Phương pháp vừa cải tiến khiến ta có cái nhìn khá thiện cảm, nhiên vấn đề lộ rõ ta bắt đầu vẽ hình không trơn tru, gồ ghề, gãy vỡ, đó chính là lúc ta tìm đến phương pháp mới, khá trừu tượng, đẹp đẽ, Phương pháp tổ chức hình ảnh theo kiểu tự đồng dạng Rời xa giới Hình học Euclide chút để đến với Hình học Fractal (xem định nghĩa và các vấn đề liên quan sách giáo khoa Toán 11) Ngồi ngẫm nghĩ lại mớ kiến thức Hình học Fractal đã biết, bạn hẳn thích thú với cách dựng hình đẹp mắt, mà hiệu hình gồ ghề, dị dạng công thức ban đầu lại vô cùng gọn gàng, “bông tuyết Von Koch” (xem hình 7) chẳng hạn Chính vì mà thể chế dạy học phổ thông nước ta, phần mềm này không cho kết phù hợp hay nói nặng là vẽ sai (214) Ứng dụng Toán học việc hiển thị hình ảnh máy vi tính 183 Hình Bông tuyết Von Koch qua lần tự đồng dạng Ta ngẫm lại thuật toán tí xíu: Lấy tam giác có cạnh 1, gọi là hình ban đầu K0 • Bước Chia cạnh có ba đoạn và thay đoạn hai đoạn nó cho chúng tạo với đoạn bỏ tam giác phía ngoài ta bông tuyến K1 • Bước Cứ tiếp tục lặp lại theo nguyên tắc: Từ bông tuyết Kn (n ≥ 0) để có bông tuyết Kn+1 , ta chia cạnh Kn thành ba đoạn và thay đoạn hai đoạn nó, cho chúng tạo với đoạn bỏ tam giác phía ngoài • Quá trình trên lặp đi, lặp lại cho ta dãy các bông tuyết K0 , K1 , K2 , , Kn , Và là ta đã có hình ảnh không tồi chút nào Nhưng làm nào để làm máy vi tính hiểu thuật toán dựng hình đẹp đẽ trên đây trên đây! Chúng ta hoàn toàn có thể làm điều đó nhờ ý tưởng sau đây Đám bụi Cantor Định nghĩa d • Hình ban đầu E0 là đoạn thẳng đơn vị [0, 1] • Quy tắc sinh: Chia đoạn thẳng [0, 1] ba đoạn nhau, bỏ đoạn nhỏ chính giữa, hình E1 • Lập lại quy tắc sinh đoạn thẳng E1 , E2 , , tiếp tục đến vô tận (xem hình 8) Ta tập hợp gồm toàn hạt bụi, có tổng chiều dài 0, không chứa đoạn thẳng nào thuộc [0, 1] dù là nhỏ bao nhiêu nữa, gồm vô hạn không đếm các điểm, có thể cho tương ứng − với toàn đoạn thẳng [0, 1] 3 E E1 E2 E3 E4 E5 F FL FR Hình Đám bụi Cantor (215) 184 Chuyên đề Toán học số Ngoài định nghĩa hình ảnh trên ta còn có thể định nghĩa Số học (người ta chứng minh hai định nghĩa này là tương đương) là tập hợp tất các số viết dạng phân số tam phân 0, a1 , a2 , , an , (phân số hệ đếm tam phân) mà chữ số 2(không có chữ số 1) Tuy nhiên điều đáng quan tâm đây là tương ứng − 1, chính vì điều đẹp đẽ đó mà ta có định lý tổng quát sau Định lý (Định lý đường Fractal) Mỗi Fractal là ảnh liên tục tập Cantor, nghĩa là Fractal tìm ánh xạ ϕ liên tục biến tập Cantor thành Fractal Theo nghĩa đó ta có thể nói tập Cantor là Fractal “nguyên thủy” Định lý này cho phép ta có thể “số hóa” Fractal Và chính vì lẽ đó mà hình ảnh có vẻ trừu tượng, hỗn độn, vô trật tự (như hình đám mây, hình dãy núi, sóng, nhành cây, ) lại mang mã hóa vô cùng gọn gàng Hình Cây San hô Fractal, có khởi đầu là đoạn thẳng gốc Hình 10 Vài nhánh cây thông qua phép biến đổi affine tự đồng dạng Đó là ý tưởng phương pháp tổ chức hình ảnh theo kiểu tự đồng dạng, việc nhận xét phương pháp này hoàn toàn tùy thuộc vào bạn đọc (hãy liên tưởng đến ADN môn Sinh học di truyền), đại diện ưu tú cho phương pháp này là kỹ thuật IFS (Iterated Function Scheme) (216) Ứng dụng Toán học việc hiển thị hình ảnh máy vi tính 185 Vài kết luận và hướng gợi mở Trong bài viết này là sơ nét phương pháp tiếp cận chính Toán học, việc hiển thị các đối tượng hình ảnh máy vi tính Mỗi cách làm có ưu nhược điểm riêng! Có thể sau đọc bài viết này bạn đọc tự đề thuật toán, phương pháp khác hay cho vấn đề trên Nhưng điều đáng lưu ý đây là “Tại phải cần hình ảnh quá chi tiết và có độ chính xác cao?”, có vẻ ta không quá cần thiết đến điều này Thật khó câu trả lời chính xác, bạn hãy tưởng tượng gì diễn bạn có thể phóng đại ảnh lên tỉ lệ tùy ý mà không làm giảm chất lượng ảnh • Có khả năng, bạn là người đầu tiên nhìn thấy vật thể nhỏ tự nhiên (nhỏ hạt bản) • Có khả năng, bạn là người đầu tiên nhìn thấy hành tinh khác, có sống (ngoài địa cầu ra) cái máy quay phim có sử dụng thuật toán ghi ảnh, ghi hình cho chính bạn tạo Điều đó là có ý nghĩa quan trọng và đáng để lưu tâm đúng không bạn nhỉ?! Tài liệu tham khảo [1] Đỗ Công Khanh (chủ biên), Toán cao cấp – Giải tích hàm biến (Toán 1), Nhà xuất Đại học Quốc gia thành phố Hồ Chí Minh, 2006 [2] Hoàng Chúng (chủ biên), Tìm hiểu Fractal – Một hình học lạ, Nhà xuất Giáo Dục, 2003 [3] Kenneth Falconer, Fractal Geometry: Mathematical Foundations and Applications, Wiley Publishing House, 2004 [4] http://www.geogebra.org (217) 186 d Chuyên đề Toán học số (218) CẤP SỐ CỘNG VÀ PHƯƠNG TRÌNH HÀM TRÊN N Nguyễn Trọng Tuấn Trường Phổ thông Năng khiếu - Đại học Quốc gia thành phố Hồ Chí Minh Chúng ta biết dãy số fn = an + b, đây a, b ∈ N, là cấp số cộng với công sai là a Một hàm số cảm sinh cấp số cộng nó có dạng tuyến tính Theo cách tiếp cận này, có khá nhiều hàm trên N liên quan đến cấp số cộng Một dấu hiệu để dự đoán hàm số f tuyến tính là bậc các biến không vượt quá Bài viết sau đây trình bày kỹ thuật để giải phương trình hàm trên N liên quan đến cấp số cộng Hy vọng nó có ích cho các bạn học sinh chuyên Toán Bài toán Tìm tất các hàm f : N∗ → N∗ thỏa mãn điều kiện f (f (m) + f (n)) = m + n, ∀m, n ∈ N∗ Lời giải Dễ dàng chứng minh f là hàm có tính chất đơn ánh Giả sử m, n, p, q là số nguyên dương thỏa mãn m + n = p + q Khi đó f (f (m) + f (n)) = f (f (p) + f (q)) Do f là đơn ánh nên ta có f (m) + f (n) = f (p) + f (q) Từ đẳng thức trên suy f (m + 1) − f (m) = f (m) − f (m − 1) = · · · = f (3) − f (2) = f (2) − f (1) Như thế, ta thấy dãy {f (m)} là cấp số cộng với số hạng đầu là f (1) và công sai là d = f (2) − f (1) Và vậy, để xác định f (m) ta cần xác định f (2) và f (1) Ta có f (m) = f (1) + [f (2) − f (1)] (m − 1), ∀m ∈ N∗ Thay biểu thức f (m) vào hệ thức đầu bài, ta f (2f (1) + (m − 1)d + (n − 1)d) = m + n, hay f (1) + [2f (1) + (m − 1)d + (n − 1)d − 1] d = m + n Suy f (1) = d = Từ đó hàm số cần tìm là f (n) = n với n ∈ N∗ Sử dụng kỹ thuật bài toán giúp chúng ta giải nhiều phương trình hàm cách ngắn gọn 187 (219) 188 Chuyên đề Toán học số Bài toán Tìm tất hàm f : N → N thỏa mãn điều kiện f (f (m) + f (n) + f (p)) = m + n + p, ∀m, n, p ∈ N Lời giải Dễ thấy f đơn ánh Cho n = p, ta có f (f (m) + 2f (n)) = m + 2n Với số tự nhiên m và n > 0, ta có m + 2n = (m + 2) + 2(n − 1) Từ đó f (f (m) + 2f (n)) = f (f (m + 2) + 2f (n − 1)) Vì f đơn ánh nên f (m) + 2f (n) = f (m + 2) + 2f (n − 1) hay f (m + 2) − f (m) = 2f (n) − 2f (n − 1), ∀m, n ∈ N, n ≥ Thay n = m + 2, ta có f (m + 2) − f (m) = 2f (m + 2) − 2f (m + 1) Do đó f (m + 2) − f (m + 1) = f (m + 1) − f (m), ∀m ∈ N Đẳng thức trên chứng tỏ {f (m)} là cấp số cộng Do đó f (m) = am + b với a, b ∈ N Thay vào hệ thức đầu bài, ta f (am + b + an + b + ap + b) = m + n + p hay a2 (m + n + p) + 3ab + b = m + n + p, ∀m, n, p ∈ N Suy a = 1, b = Vậy hàm số cần tìm là f (n) = n với n ∈ N (thỏa mãn) Có nhiều hàm số thỏa mãn hệ thức f (f (n)) = an + b với n ∈ N∗ lại không cảm sinh cấp số cộng, nghĩa là dãy {f (n)} không phải là cấp số cộng Bài toán đề cập hàm số Bài toán Chứng minh tồn hàm số f : N∗ → N∗ thỏa mãn đồng thời hai điều kiện: (i) f tăng thực sự; (ii) f (f (n)) = 2n + với n ∈ N∗ Hơn nữa, dãy f (n) không phải là cấp số cộng Lời giải Chú ý f tăng thực nên với m, n ∈ N∗ , m > n, ta luôn có f (m) − f (n) ≥ m − n Từ (1) suy f (f (n + 1)) − f (f (n)) ≥ f (n + 1) − f (n) Như f (n + 1) − f (n) ≤ 2, ∀n ∈ N∗ Lại áp dụng (1) ta f (n) − n ≤ f (f (n)) − f (n) = 2n + − f (n) Do đó f (n) ≤ 3n + , ∀n ∈ N∗ (1) (220) 189 Cấp số cộng và Phương trình hàm trên N Mặt khác, từ (1) ta có f (f (n)) − f (n) ≤ f (f (f (n))) − f (f (n)) , suy 4n + − f (n) ≤ 2f (n) + hay f (n) ≥ 4n + , ∀n ∈ N∗ Từ đó với số nguyên dương n, ta có 4n + 3n + ≤ f (n) ≤ (2) Từ (2), ta có f (1) = 3, f (2) = và f (3) = f (f (1)) = 5, f (4) = f (f (2)) = Bây giờ, giả sử f (n) < m < f (n + 1) và không tồn k cho f (k) = m Khi đó, ta có f (f (n)) < f (m) < f (f (n + 1)) hay 2n + < f (m) < 2n + Vậy f (m) = 2n + = f (f (n)) + Từ đó, ta có thuật toán xây dựng hàm f (n) sau • f (1) = Ta có f (3) = và từ đó f (2) = • Giả sử đã xác định f (1), f (2), , f (k) Ta xác định f (k + 1) sau – Nếu có m mà f (m) = k + (số m là nhất) thì đặt f (k + 1) = 2m + – Nếu không có số m thì đặt f (k + 1) = f (k) + Từ đó ta có dãy f (n) sau f (1) = 3, f (2) = 4, f (3) = 5, f (4) = 7, f (5) = 9, f (6) = 10, Rõ ràng dãy f (n) xác định thỏa mãn yêu cầu bài toán và không phải là cấp số cộng Bài toán giải xong Như vậy, f tăng thực và thỏa mãn f (f (n)) = an + b với n ∈ N∗ thì {f (n)} chưa là cấp số cộng hay nói cách khác, f có thể không có dạng tuyến tính Do đó, muốn có dạng tuyến tính, ta cần phải bổ sung điều kiện cho hàm f Một điều kiện đó nêu bài toán sau đây Bài toán Tìm tất các hàm tăng thực f : N∗ → N∗ thỏa mãn điều kiện: (i) f (f (n)) = 4n + với n ∈ N∗ ; (ii) f (f (n) + 1) = 4n + 11 với n ∈ N∗ (221) 190 Chuyên đề Toán học số Lời giải Trước hết, dễ dàng chứng minh các kết f (4k + 9) = 4f (k) + 9, f (4k + 10) = 4f (k) + 11 Tương tự bài toán 3, ta có f (n + 1) − f (n) ≤ 4, ∀n ∈ N∗ Gọi k là số tự nhiên cho f (k + 1) − f (k) = M là số lớn Ta có f (f (f (k + 1))) − f (f (f (k))) = 4f (k + 1) + − 4f (k) − = 4M Mặt khác, f (f (f (k + 1))) − f (f (f (k))) = [f (f (f (k + 1))) − f (f (f (k + 1)) − 1)] + + [f (f (f (k + 1)) − 1) − f (f (f (k + 1)) − 2)] + + · · · + [f (f (f (k)) + 1) − f (f (f (k)))] ≤ [f (f (k + 1)) − f (f (k))] M = 4M Từ đó các tổng ngoặc vuông phải và M Đặc biệt ta có M = f (f (f (k)) + 1) − f (f (f (k))) = f (4k + 10) − f (4k + 9) = Chú ý f (n + 1) = f (n) + thì f (f (n + 1)) = f (f (n) + 1) hay 4(n + 1) + = 4n + 11 (mâu thuẫn) Vậy ta phải có f (n + 1) = f (n) + với n ∈ N∗ Như f (n) = 2n + b Thay biểu thức f (n) vào f (f (n)) = 4n + 9, ta có 2(2n + b) + b = 4n + 9, đó b = Ta thấy f (n) = 2n + thỏa mãn hệ thức f (f (n) + 1) = 4n + 11 Vậy hàm số cần tìm là f (n) = 2n + với n ∈ N∗ Nhận xét Nếu gọi k là số tự nhiên cho f (k + 1) − f (k) = m là số nhỏ thì ta dẫn đến kết m = Bài toán sau đây sử dụng kỹ thuật bài toán để chứng minh dãy {f (n)} là cấp số cộng kết M = m Bài toán (IMO 2009) Cho s1 , s2 , , sn , là dãy tăng nghiêm ngặt các số nguyên dương thỏa mãn: Các dãy ss1 , ss2 , , ssn , và ss1 +1 , ss2 +1 , , ssn +1 , là các cấp số cộng Chứng minh thân dãy s1 , s2 , , sn , là cấp số cộng Lời giải Để dễ xử lý, ta phát biểu bài toán dạng tương đương: Cho f : N∗ → N∗ là hàm tăng thực và thỏa mãn: Các dãy {f (f (n))} và {f (f (n) + 1)} là các cấp số cộng Chứng minh dãy {f (n)} là cấp số cộng (222) Cấp số cộng và Phương trình hàm trên N 191 Từ giả thiết suy f (f (n)) = an + b và f (f (n) + 1) = cn + d (ở đây a, b, c, d là số nguyên dương) Ta có f (f (n)) < f (f (n) + 1) ≤ f (f (n + 1)) , suy an + b < cn + d ≤ a(n + 1) + b hay a + d a+b b <c+ ≤a+ n n n Bằng cách cho n → +∞, ta a = c Tóm lại ta có f (f (n)) = an + b, f (f (n) + 1) = an + d, ∀n ∈ N∗ Dễ dàng chứng minh f (f (an + b)) = af (n) + b, f (f (an + b + 1)) = af (n) + d Ta có < f (n + 1) − f (n) ≤ a với n ∈ N∗ Gọi r, t là các số tự nhiên cho M = f (r + 1) − f (r) lớn và m = f (t + 1) − f (t) nhỏ Với kỹ thuật tương tự bài toán 4, ta có M = f (f (f (r)) + 1) − f (f (f (r))) = f (ar + b + 1) − f (ar + b) = d − b và m = f (f (f (t)) + 1) − f (f (f (t))) = f (at + b + 1) − f (at + b) = d − b Từ đó suy M = m Suy f (n + 1) − f (n) = d − b với n ∈ N∗ Ta có điều phải chứng minh (223) 192 d Chuyên đề Toán học số (224) SÁNG TẠO PHƯƠNG TRÌNH HÀM TỪ CÁC HẰNG ĐẲNG THỨC Lê Việt Hải - Đào Thái Hiệp HS chuyên Toán khóa 2009 - 2012 Như ta đã biết, hàm số là phần quan trọng Toán học Việc giải phương trình hàm (PTH) vì mà quan tâm Trong bài viết này, nhóm tác giả muốn trình bày đến cho các bạn cách sáng tạo phương trình hàm đơn giản, lại có nhiều điều đặc biệt và đôi lúc thực “hóc búa” Đó là việc đề xuất các bài phương trình hàm dựa trên các đẳng thức, đẳng thức Qua việc giải chúng, các bạn có thêm kỹ giải toán, và thực sự, đây là điều có ích Những bài phương trình hàm trình bày bài viết này đã nhóm tác giả tìm kiếm và chọn lọc, có bài dễ và có bài khó, bài có cái hay riêng mình, việc phát chúng và giải chúng là công việc thú vị mà các tác giả muốn chia sẻ với bạn đọc Bài viết chắn không tránh khỏi sai sót Mong các bạn quan tâm xem xét, và liên hệ với các tác giả để trao đổi nhiều Bài toán “chìa khóa” Những PTH sáng tác từ đẳng thức có ít hàm thoả là f (x) = x Chính vì vậy, việc tìm điểm chung các bài PTH này là điều quan trọng Sau thời gian xem xét kỹ, xin trình bày đến các bạn bài toán quen thuộc sau, có thể áp dụng để giải nhiều bài PTH dạng này Bài toán Tìm tất các hàm f : R → R thoả mãn f (x + y) = f (x) + f (y), ∀x, y ∈ R các trường hợp sau (a) f (x) là hàm liên tục; (b) f (x) là hàm đơn điệu; (c) |f (x)| < M, ∀x ∈ [a, b]; (d) f (x2 ) = f (x), ∀x ∈ R; (e) f (x3 ) = f (x), ∀x ∈ R Lời giải Cho x = y = 0, ta f (0) = 2f (0), suy f (0) = Hơn thay y −x, ta f (0) = f (x) + f (−x), đó f (−x) = −f (x), ∀x ∈ R, hay f là hàm lẻ 193 (225) 194 Chuyên đề Toán học số Đặt f (1) = c = const Khi đó ta có     1 1 c = f (1) = f + + ··· + = nf n n n n   m Như nf = c, ∀n ∈ N Từ đây xét x = với m, n ∈ N và (m, n) = 1, ta có n n     m 1 1 m =f + + ··· + = mf =c· f (x) = f n n n n n n Như f (x) = cx với số hữu tỉ dương x Mặt khác f là hàm lẻ và f (0) = nên từ đây ta suy f (x) = cx, ∀x ∈ Q (a) Nếu f là hàm liên tục: Với số vô tỉ x luôn tồn dãy số hữu tỉ {xn } hội tụ x Như vậy, dựa vào tính liên tục f, ta có f (x) = lim f (xn ) = lim (cxn ) = cx xn →x xn →x Vậy f (x) = x là hàm thoả (b) Nếu f là hàm đơn điệu (ở đây các tác giả trình bày phần chứng minh cho f là hàm đơn điệu tăng Trường hợp f là hàm đơn điệu giảm, bạn đọc có thể chứng minh tương tự.): Trước hết, vì f (x) đồng biến nên c = f (1) ≥ f (0) = Giả sử tồn x0 là số vô tỉ cho f (x0 ) > cx0 Do x0 là số vô tỉ nên dễ dàng suy f (x0 ) và cx0 là số vô tỉ Khi đó tồn số hữu tỉ y cho f (x0 ) > cy > cx0 Tuy nhiên đó f (x0 ) > f (y) nên x0 > y > x0 (vô lý) Chứng minh tương tự trường hợp f (x0 ) < cx0 , ta có điều vô lý Mâu thuẫn này chứng tỏ f (x0 ) = cx0 Như f (x) = cx, ∀x ∈ R là hàm thoả (c) Nếu |f (x)| < M, ∀x ∈ [a, b]: Ta chứng minh f (x) bị chặn trên [0, b − a] Thật vậy, với x ∈ [0, b − a] thì x + a ∈ [a, b] Khi đó ta có f (x + a) = f (x) + f (a), suy f (x) = f (x + a) − f (a), −2M < f (x) < 2M, suy |f (x)| < 2M Đặt b − a = d > 0, f (x) bị chặn trên [0, d] Đặt c = f (d) , g(x) = f (x) − cx Suy d g(x + y) = f (x + y) − c(x + y) = f (x) − cx + f (y) − cy = g(x) + g(y), ∀x, y ∈ R f (d) · d = Vậy g(x + d) = g(x), ∀x ∈ R, đó g là hàm tuần d hoàn Hơn nữa, g(x) = f (x) − cx nên g bị chặn trên [0, d], cộng thêm tính tuần Hơn g(d) = f (d) − (226) Sáng tạo Phương trình hàm từ các đẳng thức 195 hoàn chu kỳ d g, ta suy g bị chặn trên R Giả sử tồn x0 cho g(x0 ) 6= Khi đó, ta có với số tự nhiên n thì g(nx0 ) = ng(x0 ) suy |g(nx0 )| = n|g(x0 )| Do g(x0 ) 6= nên chọn n đủ lớn ta có thể cho n|g(x0 )| lớn tuỳ ý, suy |g(nx0 )| lớn tuỳ ý, trái với điều kiện bị chặn g Vậy g(x) = 0, ∀x ∈ R Do đó, f (x) = cx, ∀x ∈ R (d) Nếu f (x2 ) = f (x), ∀x ∈ R: Từ giả thiết ta suy f (x) ≥ với số thực không âm x Khi đó, với x > y ≥ thì f (x) − f (y) = f (x − y) ≥ Như f (x) ≥ f (y), f (x) là hàm không giảm Áp dụng kết câu (b), ta có f (x) = cx, ∀x ∈ R Thay vào giả thiết ta tìm c = ∨ c = Như vậy, f (x) = 0, ∀x ∈ R và f (x) = x, ∀x ∈ R là các hàm thoả (e) Nếu f (x3 ) = f (x), ∀x ∈ R: Ta có   [f (x) + f (y)]3 = f (x + y) = f (x + y)3 = f x3 + y + 3xy(x + y) Từ đây suy f (x) + f (y) + 3f (x)f (y)f (x + y) = f (x3 ) + f (y ) + 3f (xy(x + y)) , hay f (x)f (y)f (x + y) = f (xy(x + y)) , ∀x, y ∈ R Do f (x) = f (x3 ), ∀x ∈ R nên ta có f (1) = f (1), suy f (1) = 0∨f (1) = 1∨f (1) = −1 Ta xét hai trường hợp sau • Trường hợp f (1) = Khi đó ta có f (x2 + x) = f (x)f (1)f (x + 1) = 0, ∀x ∈ R 1 nên ta suy f (x) = 0, ∀x ≥ − 4 Mặt khác ta lại có f (x) là hàm lẻ nên f (x) = 0, ∀x ∈ R Dễ thấy hàm này thoả Do x2 + x nhận giá trị không bé − • Trường hợp f (1) = ∨ f (1) = −1 Khi đó, đặt f (1) = c, ta có f (x2 + x) = f (x)f (1)f (x + 1), hay tương đương f (x2 ) + f (x) = cf (x) [f (x) + c] = cf (x) + c2 f (x) = cf (x) + f (x) Như f (x2 ) = cf (x) Tùy vào c = hay c = −1, và làm tương tự câu (d) ta suy hai hàm tương ứng thỏa là f (x) = x, ∀x ∈ R và f (x) = −x, ∀x ∈ R Vậy với giả thiết (e), ta có hàm số thỏa mãn là f (x) = 0, f (x) = x và f (x) = −x Nhận xét Điều kiện (d) và (e) bổ đề có thể thay đổi thành f n (x) = f (xn ), ∀x ∈ R với n là số tự nhiên và n > Ngoài ra, còn có hướng tổng quát khác cho các điều kiện này sau: Cho đa thức P (x) có bậc lớn thỏa P (f (x)) = f (P (x)) , ∀x ∈ R Hãy tìm tất hàm f thỏa (227) 196 Chuyên đề Toán học số Các bài toán áp dụng Trước vào bàn luận các bài toán cụ thể, xin trình bày với các bạn cách đơn giản để sáng tạo PTH từ đẳng thức Kể từ phần này ta quy ước P (x, y) là phép (x, y) vào phương trình hàm giả thiết bài toán Chắc hẳn chúng ta biết vài đẳng thức bản, ví dụ (x + y)2 = x2 + 2xy + y , (x + y)3 = x3 + 3xy(x + y) + y , Việc thay x f (x) hay f (f (x)) , đặc biệt là f (x + y) − y, đã cho đời nhiều bài toán hay và khó Tất các bài toán mà tác giả giới thiệu bài viết này đề xuất từ chính việc “đơn giản” Và chúng ta hãy cùng thưởng thức thú vị bài PTH dạng này Việc sáng tạo PTH không dựa trên các đẳng thức đã biết, đôi xuất phát từ điều “khá hiển nhiên”, không vì mà tính thú vị, ta đến với hai bài toán sau Bài toán (IMC 2010) Tìm tất các hàm liên tục f : R → R thoả mãn f (xy + x + y) = f (x) + f (y) + f (xy), ∀x, y ∈ R Lời giải Bài toán có cách phát biểu đơn giản, không vì mà nó “dễ xơi” Ta cùng đến với cách giải sau đây Ta có P (−x, x) : f (−x2 ) = f (−x2 ) + f (x) + f (−x), f (x) + f (−x) = 0, ∀x ∈ R Ta suy f là hàm lẻ Để ý xy + x + y = (x + 1)(y + 1) − Một cách tự nhiên, ta lựa chọn các số có nhiều hai cách phân tích để từ đẳng thức này ta tìm các (x, y) phù hợp để thay vào phương trình, qua đó hy vọng có các đánh giá dẫn tới kết bài toán Cũng từ ý tưởng này, ta nghĩ đến việc chứng minh f (3x) = 3f (x), ∀x ∈ R sau: Ta có • P (x, 1) : f (2x + 1) = 2f (x) + f (1), ∀x ∈ R Như vậy, ta có f (3) = 3f (1) và f (2x − 1) = 2f (x − 1) + f (1), ∀x ∈ R • P (3, x2 − 1) : f (4x2 − 1) = f (3(x2 − 1)) + f (3) + f (x2 − 1) (1) • Xét P (1, 2x2 − 1) :   f (4x2 − 1) = 2f (2x2 − 1) + f (1) = 2f (x2 − 1) + f (1) + f (1) = 4f (x2 − 1) + 3f (1) (2) (228) Sáng tạo Phương trình hàm từ các đẳng thức 197 Từ (1) và (2) ta suy  f 3(x2 − 1) = 3f (x2 − 1) Mặt khác, với x ≥ 0, luôn tồn a ∈ R cho x = a2 − Như f (3x) = 3f (x), ∀x ≥ Do f là hàm lẻ, nên suy f (3x) = 3f (x), ∀x ∈ R Bên cạnh đó ta chứng minh f (x + 1) = f (x) + f (1), ∀x ≥ Thật vậy, • Xét P (3, −x2 − 1), ta có  f (−4x2 − 1) = f −3(x2 + 1) + f (3) + f (−x2 − 1) = 4f (x2 + 1) + 3f (1), ∀x ∈ R (3) • Xét P (1, −2x2 − 1) : f (−4x2 − 1) = 2f (−2x2 − 1) + f (1) = −2(2f (x2 ) + f (1)) + f (1) = −4f (x2 ) − f (1), ∀x ∈ R (4) Từ (3) và (4) ta suy f (x2 + 1) = f (x2 ) + f (1) Vì x2 ≥ nên f (x + 1) = f (x) + f (1), ∀x ≥ (5) Đặt f (1) = c Dựa vào (5), dễ dàng có f (n) = nc, ∀n ∈ N∗ Quy ước n là số tự nhiên > Ta có P (x, n) : f ((n + 1)x + n) = f (x) + f (n) + f (nx), suy f ((n + 1)x) = f (x) + f (nx), ∀x ∈ R Từ điều trên, theo quy nạp, ta dễ dàng có f (nx) = nf (x), ∀x ∈ R Suy f (n) = nc và f (x) = cx, ∀x ∈ Q+ Vì f là hàm liên tục và lẻ nên ta có f (x) = cx, ∀x ∈ R Vậy đây là hàm thoả Nhận xét Cách giải trên dựa vào việc để ý giả thiết tính liên tục hàm f, ta cố gắng chứng minh f (x) = cx với x hữu tỉ, để từ đó suy hàm thoả Tất phép trên xuất phát từ ý tưởng đó Nhận xét PTH trên tương đương với PTH Cauchy, và đây là bài tập dành cho các bạn: Cho hàm f : R → R, chứng minh f (xy + x + y) = f (xy) + f (x) + f (y) và f (x + y) = f (x) + f (y) với số thực x, y Trên thực tế, các tác giả đã tiếp cận bài toán đầu trước biết kết trên (229) 198 Chuyên đề Toán học số Bài toán Tìm tất các hàm liên tục f : R → R thoả mãn f2009 (x) = x, ∀x ∈ R, đó fn (x) = f (f ( (f (x)) )) , n lần f Lời giải Trước tiên ta chứng minh f là đơn ánh Thật vậy, f (x1 ) = f (x2 ), ta có f2009 (x1 ) = f2009 (x2 ) suy x1 = x2 Vậy f đơn ánh Bây ta chứng minh bổ đề sau Bổ đề Nếu f : R → R vừa là hàm đơn ánh, vừa là hàm liên tục, thì f đơn điệu Chứng minh Vì f là đơn ánh, ta chứng minh tồn x < y cho f (x) < f (y) thì f đồng biến (nếu với x < y mà f (x) > f (y) thì hiển nhiên f nghịch biến) Giả sử f không đồng biến, tức là có ba trường hợp sau có thể xảy ra, tồn z cho (1) z < x < y và f (z) > f (x), f (x) < f (y); (2) x < y < z và f (z) < f (y), f (x) < f (y); (3) x < z < y và [f (z) − f (x)] [f (z) − f (y)] > Dễ thấy, ta cần chứng minh (1) sai Chọn M cho f (x) < M < {f (y), f (z)} Theo tính chất hàm liên tục, tồn a cho z < a < x và f (a) = M, đồng thời tồn b cho x < b < y và f (b) = M Suy f (a) = f (b), suy a = b f đơn ánh, điều này không xảy vì a < x < b Vậy ta có điều giả sử là sai, tóm lại f là hàm đơn điệu Quay trở lại bài toán, ta chứng minh f là hàm đồng biến Thật vậy, giả sử f nghịch biến, ta có với x < y thì f (x) > f (y) Suy f2 (x) = f (f (x)) < f (f (y)) = f2 (y) Cứ tiếp tục thế, ta f2009 (x) > f2009 (y), suy x > y (mâu thuẫn) Vậy f là hàm đồng biến Bây ta giả sử tồn x cho f (x) > x, đó f2 (x) = f (f (x)) > f (x), ta suy x = f2009 (x) > f2008 (x) > · · · > f2 (x) > f (x) > x (mâu thuẫn) Tương tự với f (x) < x suy mâu thuẫn Vậy f (x) = x, ∀x ∈ R Nhận xét Với cách giải trên, ta dễ dàng giải bài toán thay số 2009 số tự nhiên lẻ Tuy nhiên, các bạn hãy thử suy nghĩ xem thay 2009 số tự nhiên chẵn khác thì kết nào? Qua các ví dụ trên, có thể thấy chúng có cách phát biểu đơn giản lời giải lại không đơn giản, chính điều đó đã làm nên thú vị PTH dạng này Quay trở lại với dạng chính PTH mà chúng ta xem xét, bài toán sau đây phần trích từ các kỳ thi Olympic, phần là các tác giả sáng tác Ta bắt đầu đẳng thức quen thuộc xn − y n = (x − y)(xn−1 + xn−2 y + · · · + xy n−2 + y n−1 ) (230) 199 Sáng tạo Phương trình hàm từ các đẳng thức Bài toán Tìm tất các hàm f : R → R thoả mãn f (xn ) − f (y n ) = [f (x) − f (y)] (xn−1 + xn−2 y + · · · + xy n−2 + y n−1 ), ∀x, y ∈ R, đó n là số tự nhiên và lớn Lời giải Trước hết ta thử giải bài toán trường hợp cụ thể n Qua đó, ta thấy rõ hướng cần phải làm giải bài toán với các số n • Trường hợp n = Phương trình hàm ban đầu lúc này trở thành f (x2 ) − f (y ) = [f (x) − f (y)] (x + y), ∀x, y ∈ R Ta không thể tính f (0), vì trên thực tế f (x) = ax + b là lớp hàm thỏa mãn yêu cầu bài toán, giá trị f (0) là không cố định Trong trường hợp này, ta đặt hàm số g : R → R cho g(x) = f (x) − f (0) Khi đó g(0) = 0, và ta thu phương trình hàm tương tự g(x2 ) − g(y ) = [g(x) − g(y)] (x + y), ∀x, y ∈ R (1) Thay y = 0, ta có g(x2 ) = xg(x), ∀x ∈ R Thay trở lại vào phương trình (1), ta có g(x)y = g(y)x, ∀x, y ∈ R Đến đây cho y = 1, ta có g(x) = xg(1) = ax, và đó f (x) = ax + f (0) = ax + b, ∀x ∈ R • Trường hợp n = Đây là bài toán đề thi Moldova năm 2004 f (x3 ) − f (y ) = [f (x) − f (y)] (x2 + xy + y ), ∀x, y ∈ R Cũng lập luận tương tự trường hợp n = 2, ta xét hàm số g : R → R thoả mãn g(x) = f (x) − f (0) Khi đó g(0) = và g(x3 ) − g(y ) = [g(x) − g(y)] (x2 + xy + y ), ∀x, y ∈ R (2) Thay y = 0, ta có g(x3 ) = x2 g(x) Thay trở lại vào phương trình (2), ta có (x + y)g(x)y = (x + y)g(y)x, ∀x, y ∈ R Ở đây, chúng ta không quyền triệt tiêu (x + y) hai vế phương trình hàm trên, vì sau đó ta không thể so sánh g(x)y và g(y)x với x + y = Tuy vậy, ta có thể chứng minh g(x)y = g(y)x, ∀x, y ∈ R Thật vậy, g(x) = 0, ∀x ∈ R là hàm thỏa nên ta có thể giả sử g không đồng với Khi đó, dễ dàng chứng minh g(x) = ⇔ x = (231) 200 Chuyên đề Toán học số Với số thực x, y bất kỳ, luôn tồn số thực x0 cho x + x0 và y + x0 khác không Từ đó ta suy g(x)x0 = g(x0 )x và g(y)x0 = g(x0 )y, g(x)y = g(y)x, ∀x, y ∈ R Tương tự trường hợp n = 2, ta chứng minh f (x) = ax + b, ∀x ∈ R là các hàm thoả đề • Trường hợp tổng quát xem là bài tập cho các bạn Bài toán trên giới thiệu cho chúng ta phương pháp giải phương trình hàm đó là xét hàm liên quan, từ đó suy các phương trình từ các phương trình hàm cũ, mà việc giải chúng dễ dàng Đồng thời cho ta thấy rằng, không phải lúc nào ta tính f (0) mong muốn, vì ta cần phải xử lý cách linh hoạt Tiếp theo là bài thi đề IMO, bắt nguồn từ đẳng thức quen thuộc, dùng khá nhiều việc chứng minh bất đẳng thức (x2 + y )(a2 + b2 ) = (xa + yb)2 + (xb − ya)2 Bài toán (IMO 2002) Tìm tất các hàm f : R → R thoả mãn (f (x) + f (z))(f (y) + f (t)) = f (xy − zt) + f (xt + yz), ∀x, y, z, t ∈ R Lời giải Cho x = y = z = t = 0, ta 4f (0) = 2f (0) suy f (0) = ∨ f (0) = Đến đây ta xét hai trường hợp • Trường hợp f (0) = Thay z = y = t = 0, ta có    1 1 + f (x) + = + , 2 2 ∀x ∈ R Suy f (x) = , ∀x ∈ R Thử lại thấy hàm này thỏa • Trường hợp f (0) = Thay z = t = 0, ta thu f (xy) = f (x)f (y), ∀x, y ∈ R Do f nhân tính, suy f (x2 ) = f (x), ∀x ∈ R Điều này chứng tỏ f nhận giá trị không âm với x không âm Cho x = 0, t = 1, ta có f (z)(f (y) + f (1)) = f (−z) + f (yz), suy f (z) = f (−z), ∀z ∈ R Như f là hàm chẵn Dễ thấy f (x) = 0, ∀x ∈ R là hàm thỏa Xét trường hợp f không đồng với Khi đó, ta chứng minh f (x) = ⇔ x = (232) Sáng tạo Phương trình hàm từ các đẳng thức 201 Giả sử ∃x1 6= 0, x2 6= 0, x1 6= x2 cho f (x1 ) = 0, f (x2 ) 6= Vì f nhân tính nên     x2 x2 f (x2 ) = f · x1 = f f (x1 ) = (mâu thuẫn) x1 x1 Như f (x) = ⇔ x = Do đó, ta có f (x) > ⇔ ∀x 6= Xét x, y > 0, cho z = t = √ xy, ta thu √ √ √ √ [f (x) + f ( xy)] [f (y) + f ( xy)] = f (x xy + y xy) Mà f nhân tính nên ta có  √  √ √ √ 2 √ x + f ( y) [f (x) + f ( xy)] [f (y) + f ( xy)] = f ( xy) f và √ √ √ √ f (x xy + y xy) = f ( xy(x + y)) = f ( xy) f (x + y)  √ √ 2 Từ đó ta suy f (x + y) = f ( x) + f y hay p √  √ f (x + y) = f x + f ( y) , ∀x, y > p √ √ Hơn nữa, f (x) = f ( x) và f (x) > 0, ∀x > nên ta có f (x) = f ( x) , ∀x > Kết hợp với trên, ta p p p f (x + y) = f (x) + f (y), ∀x, y > p Đến đây ta xét hàm g : R+ → R+ cho f (x) = g(x), ∀x > Dễ dàng suy g vừa là hàm nhân tính, vừa là hàm cộng tính Khi đó, ta dễ dàng có g(x) = x, ∀x > 0, dẫn đến f (x) = x2 , ∀x > 0, mà f lại là hàm chẵn và f (0) = 0, ta có hàm cần tìm là f (x) = x2 , ∀x ∈ R Tóm lại, có hàm thỏa phương trình đã cho là f (x) = x2 , f (x) = và f (x) = Nhận xét Điểm nhấn lời giải trên chính là việc suy tính chẵn hàm số f và tính không âm với các giá trị biến không âm Đây coi là hai điều quan trọng, không bài toán này, mà còn hữu dụng nhiều bài phương trình hàm khác Ngoài ra, còn điều đáng để để ý lời giải trên chính là phép √ z = t = xy, điều này bắt nguồn từ việc ta muốn đơn giản hoá vế phải phương trình hàm Việc ta dự đoán p“khá chính xác” f (x) = x là hàm thỏa đã đưa ta đến việc xét hàm số g(x) = f (x), và thực điều này giúp ta giải bài toán cách khá dễ dàng Đây là kinh nghiệm nhỏ, ta dự đoán hàm nào đó thỏa phương (233) 202 Chuyên đề Toán học số trình hàm đã cho, ta có thể xét hàm liên quan, mà việc giải hàm này “dễ dàng” hơn, từ đó suy hàm cần tìm Lời giải chính thức bài toán trên dựa trên phương trình hàm sau f (x − y) + f (x + y) = [f (x) + f (y)] , ∀x, y ∈ R Đồng thời ta có thể chứng minh f (x) là hàm đồng biến trên [0, +∞) Các bạn hãy thử giải phương trình hàm trên dựa trên ý tưởng này Chúng ta cùng qua bài toán khác kỳ thi China MO, xuất phát từ đẳng thức quen thuộc x3 + y = (x + y)(x2 − xy + y ) Điều đặc biệt là đề bài không yêu cầu ta tìm các hàm thỏa Bài toán (China MO 1996) Cho hàm số f : R → R thoả mãn   f (x3 + y ) = (x + y) f (x) − f (x)f (y) + f (y) , ∀x, y ∈ R Chứng minh f (1996x) = 1996f (x), ∀x ∈ R Lời giải Ta chứng minh f (nx) = nf (x), ∀n ∈ N, x ∈ R Việc làm này mang lại cho bài toán cái nhìn tổng quát hơn, đồng thời các bạn không bị bối rối gặp số 1996 hay số nào khác Ta có • P (0, 0) : f (0) = • P (x, 0) : f (x3 ) = xf (x), ∀x ∈ R (1) Từ đây, ta suy f (x) ≥ ∀x ≥ và f (x) ≤ ∀x ≤ Gọi X = {a | f (ax) = af (x), ∀x ∈ R} Ta chứng minh a ∈ X và a > thì a + ∈ X Thật vậy, giả sử f (ax) = af (x), ∀x ∈ R √ Vì f [(a + 1) · 0] = (a + 1)f (0) = nên ta xét x 6= Xét P (x, ax) , ta có √  √  √  f (x3 + ax3 ) = x + ax f (x) − f (x)f ax + f ax Theo hệ thức (1) và (2), ta lại có axf (x) = af (x3 ) = f (ax3 ) = f  √  √ √  3 ax = axf ax , hay tương đương √ ax √ √  √ √  3 af (x) − f ax af (x) + f ax = (2) (3) (234) Sáng tạo Phương trình hàm từ các đẳng thức Do a > nên x, √ 203 √ √ ax cùng dấu, suy af (x) + f ( ax) 6= Từ đó, theo trên ta √ √  af (x) = f ax , ∀x ∈ R∗ Sử dụng kết này, ta viết (3) lại thành i √  √ h √ f (a + 1)x3 = x + a f (x) − f (x) af (x) + a2 f (x)  √ √  √ = xf (x) + a − a + a2 = f (x3 )(a + 1) Do số thực có bậc nên ta suy f ((a + 1)x) = (a + 1)f (x), ∀x ∈ R∗ Vì ∈ X nên n ∈ X, ∀n ∈ N∗ Vậy f (nx) = nf (x), ∀n ∈ N Cho n = 1996, ta có kết bài toán Nhận xét Việc suy dấu f (x) từ hệ thức (1) là quan trọng, nó giúp ta triệt tiêu bình phương mà không cần xét dấu, đây là điều đáng lưu ý nhiều bài toán khác Nhận xét Với đề bài tương tự, liệu có thể tìm các hàm f thoả mãn không? Nếu không, cần phải thêm vào bài toán điều kiện gì? Những bài phương trình hàm trên là bước khởi động cho bài phương trình hàm sau đây, tất dựa trên đẳng thức quen thuộc (x + y)2 = x2 + 2xy + y Tất lời giải trình bày đây có ý tưởng “sử dụng lại”, mà công việc các bạn là tìm ra, hiểu và áp dụng chúng Bài toán sau lấy đề China TST, có chút “biến thể”, là xuất phát từ 2  x+y = 2+ + xy x xy y Bài toán (China TST 2007) Tìm tất các hàm f : Q+ → Q+ thoả mãn f (x) + f (y) + 2xyf (xy) = f (xy) , f (x + y) ∀x, y ∈ Q+ Lời giải Việc hàm f không xác định x = đã gây chút khó khăn “dự đoán” hàm f là hàm gì Trong trường hợp này, ta thường hay thử tính các giá trị f các giá trị đặc biệt nào đó, và thường là f (1), f (2), (do ta làm việc trên tập các số hữu tỉ dương) Ta có (235) 204 Chuyên đề Toán học số • P (1, 1) : 4f (1) = • P (2, 2) : f (1) , suy f (2) = f (2) 1 f (4) + + 8f (4) = , suy f (4) = 4 f (4) 16 • P (1, 2) : f (1) + 1 +1= 4f (3) • P (1, 3) : f (1) + f (3) + 6f (3) = (1) f (3) = 16f (3) f (4) (2) Từ (1) và (2), ta dễ dàng giải f (1) = 1, f (3) = Như ta có “dự đoán”: hàm f (x) = là hàm thoả mãn yêu cầu bài toán Trên thực tế, đây chính là kết x cần tìm, và vì vậy ta chứng minh f (x) = , x2 ∀x ∈ Q+ Nên nhớ, ta làm trên tập hữu tỉ dương, cách tốt là chứng minh trên tập số nguyên dương trước suy trên tập hữu tỉ Xét P (x, 1), ta có f (x) + + 2xf (x) = f (x) , f (x + 1) hay tương đương 1 + 2x + = f (x) f (x + 1) Từ đẳng thức này, ta có thể chứng minh quy nạp f (n) = , n2 ∀n ∈ N∗ Thật vậy, dễ thấy khẳng định đúng với n = Giả sử khẳng định đúng với n = k > 1, tức là f (k) = Khi đó với n = k + 1, ta có k 1 = + 2k + = k + 2k + = (k + 1)2 , f (k + 1) f (k) Như khẳng định đúng với n = k + Theo (k + 1)2 nguyên lý quy nạp ta có khẳng định đúng với số nguyên dương n từ đó suy f (k + 1) = Tiếp đến, ta chứng minh khẳng định sau quy nạp f (nx) = f (x) , n2 ∀n ∈ N∗ , x ∈ Q+ (236) Sáng tạo Phương trình hàm từ các đẳng thức 205 f (x) nên ta có khẳng định đúng với n = Giả sử khẳng 12 định đúng với n = k > 1, tức là f (x) f (kx) = k Khi đó với n = k + 1, ta có 1 f (xk) • P (x, k) : f (x) + + 2xkf (xk) = , suy k + 2xk + = k f (x + k) f (x) f (x + k) Hiển nhiên f (1 · x) = f (x) = • P (x, 1) : 1 1 + 2x + = , suy + 2(x + k) + = f (x) f (x + 1) f (x + k) f (x + k + 1) Từ hai điều trên ta suy 1 = k + 2xk + + 2(x + k) + f (x + k + 1) f (x) Ta lại có P (x, k + 1) : f (x) + f (x(k + 1)) , hay + 2x(k + 1)f (x(k + 1)) = f (x + k + 1) (k + 1) f (x) 1 + + 2x(k + 1) = f (x(k + 1)) (k + 1) f (x(k + 1)) f (x + k + 1) Kết hợp với trên, ta f (x) + + 2(x + k) + + 2x(k + 1) = k + 2xk + f (x(k + 1)) (k + 1) f (x(k + 1)) f (x) Từ đó f (x) 1 + = (k + 1)2 + f (x(k + 1)) (k + 1) f (x(k + 1)) f (x) Với g(x) = f (x(k + 1)) , coi đây là phương trình theo ẩn f (x), ta dễ dàng giải f (x) = (k + 1)2 g(x) Như f (x(k + 1)) = f (x) (k + 1)2 Khẳng định đó đúng với n = k + Theo nguyên lý quy nạp ta có khẳng định đúng với số nguyên dương n m   f m m n , từ đó Bây giờ, cho x = bất kỳ, đó m, n ∈ N∗ , ta có f n · = n n n2 m n2 f (x) = f = n2 f (m) = = n m x Như vậy, hàm thỏa phương trình hàm ban đầu là f (x) = , ∀x ∈ Q+ x Bài toán sau đây là chút “sáng tạo” các tác giả, dựa trên hoàn toàn đẳng thức (x + y)2 = x2 + 2xy + y , tìm lời giải, chúng tôi lại không ngờ nó khá “hóc búa”, và thực bài toán này chứa đựng nhiều điều thú vị mà chúng ta cần học hỏi (237) 206 Chuyên đề Toán học số Bài toán Tìm tất các hàm số f : R → R thỏa mãn f (x + y) = f (x2 ) + 2f (x)f (y) + f (y ), ∀x, y ∈ R Lời giải Cho x = y = 0, ta có f (0) = 2f (0) + 2f (0), suy f (0) = ∨ f (0) = −2 Như vậy, ta cần xét hai trường hợp sau • Trường hợp f (0) = Cho y = 0, ta thu f (x) = f (x2 ), ∀x ∈ R Ta suy f (x) ≥ 0, ∀x ≥ Thay f (x2 ) = f (x) và f (y ) = f (y) vào phương trình ban đầu, ta f (x + y) = [f (x) + f (y)]2 , ∀x, y ∈ R (1) Thay y = −x, ta dễ dàng suy f là hàm lẻ Như f (x) ≥ 0, ∀x ≥ và f (x) ≤ 0, ∀x ≤ Do đó, (1) tương đương với f (x + y) = f (x) + f (y), ∀x, y ∈ R, xy ≥ Ta cần chứng minh f cộng tính x, y trái dấu (hiển nhiên đúng hai số 0) Thật vậy, x, y là hai số trái dấu, thì x + y phải cùng dấu với −x −y, suy f (x + y) + f (−y) = f (x) f (x + y) + f (−x) = f (y), Cả hai điều này kết hợp với tính lẻ hàm f, ta thu f (x + y) = f (x) + f (y) Như f là hàm cộng tính trên R, kết hợp với f (x2 ) = f (x) ta suy f (x) = và f (x) = x, ∀x ∈ R là hai hàm thỏa • Trường hợp f (0) = Khi đó ta có P (x, 0) : f (x) = f (x2 ) − 4f (x) − Từ đó f (x2 ) = f (x) + 4f (x) + Thay trở lại vào phương trình ban đầu, ta f (x + y) = f (x) + 4x + + 2f (x)f (y) + f (y) + 4f (y) + = [f (x) + f (y) + 2]2 , ∀x, y ∈ R.(2) (2) Bây thay y = −x phương trình (2), ta có [f (x) + f (−x) + 2]2 = f (0) = (3) Đến đây, ta xét ba trường hợp sau ◦ f (x) + f (−x) + = 2, ∀x ∈ R ◦ f (x) + f (−x) + = −2, ∀x ∈ R ◦ f (x) + f (−x) + = 2, ∀x ∈ X và f (x) + f (−x) + = −2, ∀x ∈ Y, đó X, Y là hai tập hợp cho X ∪ Y = R (238) Sáng tạo Phương trình hàm từ các đẳng thức 207 Giả sử tồn số a cho f (a) + f (−a) + = 2, hay f (a) + f (−a) = Dễ thấy a 6= 0, và ta có thể giả sử a > Gọi F (x) là phép x vào (2) Ta có ◦ F (−a) : f (a) = f ((−a)2 ) − 4f (−a) − ◦ F (−a) : f (−a) = f ((−a)2 ) − 4f (−a) − = f (a2 ) − 4f (−a) − Hơn f (a) = f (−a) f (a) + f (−a) = 0,√nên từ đó ta suy f (a) = f (−a), f (a) = f (−a) = Mặt khác, xét F ( a) , ta có √  √ √  f a = f (a) − 4f ( a) − = −4f a − 2, hay √  √  a + 4f a + = √ √ √ √ √ Coi f ( a) là ẩn, ta giải f ( a) = − ∨ f ( a) = − − f2 (4) Ta tiếp tục xét các phép sau ◦ F (a) : f (a) = f (a2 ) − 4f (a) − 2, suy f (a2 ) = (vì f (a) = 0) ◦ P (a, a) : f (2a) = 2f (a2 ) + · · = ◦ P (2a, −a) : = f (2a − a) = f (4a2 ) + 2f (2a)f (−a) + f (a2 ) = f (4a2 ) + 2, suy f (4a2 ) = −2 ◦ P (2a, 2a) : f (4a) = 2f (4a2 ) + 2f (2a) = 2(−2) + · = 4, suy f (4a) = ∨ f (4a) = −2 (5) √ √ √ ◦ F (2 a) : f (2 a) = f (4a) − 4f (2 a) − 2, suy √  √  √  √  f 2 a = −4f a ∨ f 2 a = −4f a − (do (5)) √ Coi f (2 a) là ẩn, ta giải √  √  √  f a = ∨ f a = −4 ∨ f a = −2 (6) √ √ √ √ Tuy nhiên, ta lại có P ( a, a) : f (2 a) = 2f (a) + 2f ( a) , suy f √ 2 √ 2 √  √  2 a =2 2−2 ∨f a =2 + (do (4)) Kết hợp với (6), ta suy điều vô lý Mâu thuẫn này chứng tỏ f (x)+f (−x)+2 6= 2, ∀x ∈ R, đó f (x) + f (−x) + = −2, hay f (x) + f (−x) = −4, ∀x ∈ R Bây ta xét hàm g : R → R cho g(x) = f (x) + Khi đó ta có g(x) + g(−x) = 0, ∀x ∈ R (7) (239) 208 Chuyên đề Toán học số Từ phương trình hàm ban đầu, ta thu [g(x + y) − 2]2 = g(x2 ) + [g(x) − 2] [g(y) − 2] + g(y ) − 4, hay tương đương g (x + y) − 4g(x + y) = g(x2 ) + g(y ) + 2g(x)g(y) − [g(x) + g(y)] (8) Lại có f (x) = f (x2 ) − 4f (x) − nên [g(x) − 2]2 = g(x2 ) − − [g(x) − 2] − 2, hay g (x) = g(x2 ), ∀x ∈ R (9) Thế −x vào x và −y vào y hệ thức (8), kết hợp với (7) và (9), ta có g (−x − y) − 4g(−x − y) = g(x2 ) + g(y ) + 2g(−x)g(−y) − [g(−x) + g(−y)] Phương trình này tương đương với g (x + y) + 4g(x + y) = g(x2 ) + g(y ) + 2g(x)g(y) + [g(x) + g(y)] , ∀x, y ∈ R Kết hợp với (8), ta suy g(x + y) = g(x) + g(y), ∀x, y ∈ R Kết hợp với (9) và sử dụng kết “bài toán chìa khóa”, ta suy g(x) = x ∨ g(x) = 0, ∀x ∈ R, đó f (x) = x − ∨ f (x) = −2, ∀x ∈ R Dễ dàng kiểm tra đây là hai hàm thoả Tóm lại, phương trình hàm ban đầu có hàm thỏa là f (x) = 0, f (x) = −2, f (x) = x và f (x) = x − 2, ∀x ∈ R Nhận xét Có lẽ xem lời giải, tự hỏi là “Tại lại giải ?” Nhưng để ý kỹ, tác giả đã theo hướng hoàn toàn tự nhiên: Tính f (0), đoán hàm → Thử trường hợp → Cố gắng áp dụng “bài toán chìa khóa” Có thể thấy, lời giải trên hoàn toàn sơ cấp, và hai trường hợp sử dụng kết “bài toán chìa khóa” để tìm hàm thoả Tuy lời giải là vậy, mong bạn đọc gần xa quan tâm và cố gắng tìm lời giải khác Bây chúng ta cùng đến với bài toán khác, dựa trên (x+y)2 = x2 +2xy +y , thú vị sau Bài toán Tìm tất các hàm số f : R → R thoả mãn f (x + y) = f (x2 ) + 2f (xy) + f (y ), ∀x, y ∈ R Lời giải Ta có P (0, 0) : f (0) = 4f (0), suy f (0) = ∨ f (0) = Như vậy, ta cần xét hai trường hợp (240) Sáng tạo Phương trình hàm từ các đẳng thức 209 • Trường hợp f (0) = Ta xét hai phép ◦ P (x, 0) : f (x) = f (x2 ) + 3f (0) = f (x2 ) + 12, ∀x ∈ R ◦ P (−x, 0) : f (−x) = f (x2 ) + 3f (0) = f (x2 ) + 12, ∀x ∈ R Vậy f (x) = f (−x), ∀x ∈ R Ta lại có P (x, −x) : f (0) = 2f (x2 ) + 2f (−x2 ), hay = f (x2 ) + f (−x2 ) Do ta có = f (x) + f (−x), ∀x ∈ R Coi f (x) và f (−x) là hai ẩn, từ hai điều trên ta suy f (x) = f (−x) = 4, ∀x ∈ R Vậy f (x) = 4, ∀x ∈ R là hàm thoả • Trường hợp f (0) = Ta có P (x, 0) : f (x) = f (x2 ), ∀x ∈ R (1) Suy f (x) ≥ 0, ∀x ∈ R Hơn nữa, P (x, −x) : f (0) = 2f (x2 ) + 2f (−x2 ), hay = f (x2 ) + f (−x2 ) Từ đây, ta có = f (x) + f (−x), ∀x ∈ R Vậy f là hàm lẻ Dễ thấy hàm f (x) = 0, ∀x ∈ R là hàm thoả Ta xét hàm f không đồng với Khi đó ta chứng minh f (x) = ⇔ x = Giả sử tồn x1 , x2 ∈ R∗ , x1 6= x2 cho f (x1 ) 6= và f (x2 ) = Do f là hàm lẻ nên ta giả sử x1 , x2 > Từ phương trình hàm ban đầu và hệ thức (1), ta f (x + y) ≥ f (y), ∀x, y > 0, suy f (x) ≥ f (y), ∀x > y > Ta có P (x1 , x2 ) : f (2x2 ) = 2f (x2 ) + 2f (x2 ) =, suy f (2x2 ) = Như f (x2 ) = thì f (2x2 ) = Khi đó dễ dàng có f (2n x2 ) = 0, ∀n ∈ N Chọn n đủ lớn cho 2n x2 > x1 Khi đó ta có = f (2n x2 ) ≥ f (x1 ) > 0, vô lý Vậy f (x) = ⇔ x = Khi đó, từ hệ thức (1) suy f (1) = f (1), suy f (1) = Ta chứng minh f (nx) = nf (x), ∀x > 0, n ∈ N (2) Trước hết, ta chứng minh k ∈ N thoả (2) thì k + thoả (2) Thật vậy, xét phép P (kx, x) : f ((k + 1)x) = f (kx) + 2f (kx2 ) + f (x) = k f (x) + 2kf (x) + f (x) = (k + 2k + 1)f (x) = (k + 1)2 f (x), ∀x > (241) 210 Chuyên đề Toán học số Suy f ((k + 1)x) = (k + 1)f (x), ∀x > Vì thoả (2) nên n ∈ N∗ thoả (2) Do f là hàm lẻ, ta suy f (nx) = nf (x), ∀x ∈ R, n ∈ N Từ đây ta dễ dàng có f (q) = q, ∀q ∈ Q Bằng cách làm tương tự “bài toán chìa khóa”, sử dụng điều sau ◦ f là hàm đồng biến trên R ◦ f (q) = q, ∀q ∈ Q Ta suy f (x) = x, ∀x ∈ R Dễ thấy đây là hàm thoả Tóm lại, PTH ban đầu có hàm thoả là f (x) = 0, f (x) = và f (x) = x, ∀x ∈ R Nhận xét Việc dự đoán hàm thỏa bài này là khá dễ dàng Tuy nhiên trường hợp (tức f (0) = 0), có điều thú vị là ta không suy f cộng tính, lại có tính chất hàm cộng tính, đó là f (nx) = nf (x), và từ đó suy kết bài toán Qua đó, cho ta kinh nghiệm, không suy “tính chất” nào đó hàm f thì hãy thử tìm tính chất khác có mối quan hệ mật thiết với “tính chất” mà mình cần Đây là ví dụ, cho thấy hiệu từ việc chứng minh trên tập N, dẫn đến tập Q và kết luận tập R Xin giới thiệu bài toán tổng quát, đây xem là bài tập dành cho các bạn: Với n > 1, tìm tất các hàm f : R → R thỏa mãn f (x1 + x2 + · · · + xn ) = n X f (x2i ) + i=1 X f (xj xk ), ∀x1 , x2 , , xn ∈ R 1≤j<k≤n Tổng kết và bài tập đề nghị Bài viết đến đây là kết thúc Các tác giả đã cố gắng trình bày điều thú vị PTH dạng này đến bạn đọc, chắn là chưa đủ Mong qua bài viết này, các bạn có thêm niềm yêu thích môn Toán nói chung và PTH nói riêng Sau đây là số bài tập cho các bạn rèn luyện Bài tập Tìm tất các hàm f : R → R thoả mãn (i) f (x + y) = f (x) + f (y), ∀x, y ∈ R   = 1, ∀x 6= (ii) f (x)f x Bài tập (IMO 1992) Tìm tất các hàm f : R → R thoả mãn  f x2 + f (y) = y + f (x), ∀x, y ∈ R Bài tập Tìm tất các hàm f : R → R thoả mãn  f x2 + f (y) − y = f (x), ∀x, y ∈ R (242) Sáng tạo Phương trình hàm từ các đẳng thức Bài tập Tìm tất các hàm f : R → R thoả mãn f (x − y) = f (x2 ) − 2f (x)f (y) + f (y ), ∀x, y ∈ R Bài tập Tìm tất các hàm f : R → R thoả mãn f (2x + f (y)) = f (2x) + xf (2y) + f (f (y)) , ∀x, y ∈ R Bài tập Tìm tất các hàm f : R → R thoả mãn f (x − f (y)) = f (x) + xf (y) + f (f (y)) − 1, ∀x, y ∈ R 211 (243) 212 d Chuyên đề Toán học số (244) LỜI GIẢI ĐỀ THI VÀO LỚP 10 CHUYÊN TOÁN NĂM HỌC 2010 - 2011 Bài (a) Cho a, b, c là các số thực thoả mãn điều kiện a + b + c = a3 + b3 + c3 = Chứng minh ba số a, b, c có ít số (1 điểm) (b) Giải hệ phương trình  (1)  x + y + z = xy + yz + zx = −1 (2)   x3 + y + z + = 3(x2 + y + z ) (3) (1 điểm) Lời giải (a) Từ a + b + c = suy c = −(a + b) Từ đó ta có = a3 + b3 + c3 = a3 + b3 − (a + b)3 = −3a2 b − 3ab2 = −3ab(a + b) = 3abc Vậy abc = 0, suy ba số a, b, c phải (đpcm) (b) Cách Đặt x = a + 1, y = b + 1, z = c + Thay vào phương trình (1) ta a + b + c = Thay vào (2) với chú ý a + b + c = 0, ta ab + bc + ca = −4 (4) Thay vào (3) với chú ý a + b + c = 0, ta a3 + b3 + c3 = Áp dụng câu (a), ta suy ba số a, b, c Không tính tổng quát, giả sử a = Khi đó b = −c và thay vào (4) ta tìm b = ±2 Từ đây tìm x, y, z Kết luận: Phương trình có nghiệm (1, −1, 3) và các hoán vị (6 nghiệm) Cách Từ phương trình (1) và phương trình (2) ta suy x2 + y + z = (x + y + z)2 − 2(xy + yz + zx) = 11 Thay vào phương trình (3), ta x3 + y + z = 27 213 (5) (245) 214 Chuyên đề Toán học số Từ (1) và (5) ta suy = (x + y + z)3 − (x3 + y + z ) = 3(x + y)(y + z)(z + x) Từ đó suy ba số x, y, z có hai số có tổng Không tính tổng quát, giả sử x + y = Từ (1) suy z = Thay vào phương trình (2), ta tìm x = −1, y = x = 1, y = −1 Kết luận: Phương trình có nghiệm (1, −1, 3) và các hoán vị (6 nghiệm) Bài (a) Giải phương trình √ (2x − 1)2 = 12 x2 − x − + (1 điểm) (b) Cho tam giác ABC vuông A và có diện tích Chứng minh ta có bất đẳng thức √ ≤ BC ≤ 2(AB + AC − 1) (1 điểm) Lời giải (a) Điều kiện x2 − x − ≥ ⇔ x ≤ −1 ∨ x ≥ Ta biến đổi phương trình dạng √ √ (2x − 1)2 = 12 x2 − x − + ⇔ 4x2 − 4x + = 12 x2 − x − + √ ⇔ x2 − x = x2 − x − √ Đặt t = x2 − x − ≥ thì t2 = x2 − x − Thay vào phương trình, ta t2 + − 3t = ⇔ t = ∨ t = √ ± 13 • Với t = 1, ta x − x − = 0, suy x = • Với t = 2, ta x2 − x − = 0, suy x = −2, x = Các nghiệm này thỏa mãn điều kiện Vậy phương trình đã cho có nghiệm là x = −2, x = 3, x = 1± √ 13 (b) Đặt AB = c, AC = b thì theo điều kiện đề bài, ta có ab = Ngoài ra, theo định lý Pythagore, ta có √ BC = a2 + b2 Vế thứ bất đẳng thức cần chứng minh có thể viết lại thành √ √ 2ab ≤ a2 + b2 ⇔ a2 + b2 ≥ 2ab ⇔ (a − b)2 ≥ (đúng, đây có thể dùng bất đẳng thức AM-GM) (246) 215 Lời giải đề thi vào lớp 10 chuyên Toán năm học 2010 - 2011 Vế thứ hai bất đẳng thức có thể viết lại thành √ √ √ a2 + b2 + ≤ 2(a + b) ⇔ a2 + b2 + a2 + b2 + ≤ 2(a2 + b2 + 2ab) √ 2 √ ⇔ a2 + b − a2 + b + ≥ ⇔ a2 + b2 − ≥ Bất đẳng thức cuối cùng hiển nhiên đúng Bài toán giải hoàn toàn Bài (a) Hãy số nguyên dương phân biệt mà tổng ba số chúng là số nguyên tố (0.5 điểm) (b) Chứng minh không tồn số nguyên dương phân biệt cho tổng ba số chúng là số nguyên tố (1 điểm) Lời giải (a) Có thể (1, 3, 7, 9) (b) Do các số nguyên dương là phân biệt nên tổng ba số lớn Ta chứng minh các tổng đó chia hết cho 3, từ đó không thể là số nguyên tố, suy đpcm Xét số dư phép chia các số này cho • Nếu các số dư 0, 1, xuất thì ta lấy ba số tương ứng, ta số có tổng chia hết cho • Nếu có số dư nào đó không xuất thì có số và có nhiều số dư, suy tồn số có cùng số dư Ba số này có tổng chia hết cho Bài toán giải Bài √ Cho trường tròn tâm O, bán kính R và dây cung BC có độ dài BC = R A là điểm thay đổi trên cung lớn BC Gọi E là điểm đối xứng B qua AC và F là điểm đối xứng C qua AB Các đường tròn ngoại tiếp các tam giác ABE và ACF cắt K (K 6= A) (a) Chứng minh K luôn thuộc đường tròn cố định (1 điểm) (b) Xác định vị trí điểm A để tam giác KBC có diện tích lớn và tìm giá trị lớn đó theo R (1 điểm) (c) Gọi H là giao điểm BE và CF Chứng minh tam giác ABH đồng dạng với tam giácAKC và đường thẳng AK luôn qua điểm cố định (1 điểm) (247) 216 Chuyên đề Toán học số Lời giải E A F O H C B C0 K B0 (a) Ta có ∠AKC = ∠AF C (cùng chắn cung AC) Mặt khác ∠AF C = ∠F CA (do F đối xứng C qua AB) và ∠F CA = 90◦ − A, nên ta có ∠AKC = 90◦ − A Hoàn toàn tương tự, ta có ∠AKB = 90◦ − A Suy ∠BKC = 180◦ − 2A Suy K luôn thuộc cung chứa góc nhìn đoạn BC góc 180◦ − 2A √ (b) Tam giác KBC có đáy BC = R không đổi và K nằm trên cung chứa góc 180◦ − 2A nên diện tích tam giác KBC lớn K là điểm K0 cung chứa góc, tức là tam giác KBC cân K Khi đó A chính là trung điểm cung lớn BC √ Để tính giá trị lớn diện tích tam giác K0 BC, ta chú ý vì BC = R nên A = 60◦ Suy ∠BKC = 180◦ − 2A = 60◦ Suy√ra tam giác K0 BC là tam giác có √ 3R2 cạnh BC = R Vậy diện tích lớn (c) Kéo dài AC cắt đường tròn ngoại tiếp tam giác ABE C Khi đó AC là đường kính Tương tự, kéo dài AB cắt đường tròn ngoại tiếp ACF B thì AB là đường kính Suy AK, B C, C B là các đường cao tam giác AB C Suy tứ giác B BCC nội tiếp và ∠AC B = ∠ABC Ta có ∠BAH = 90◦ − ∠ABC = 90◦ − ∠AC B = ∠KAC = ∠KAC Mặt khác theo chứng minh phần (a), ta đã có ∠AKC = ∠F CA = ∠ABH Từ đây suy tam giác ABH đồng dạng với tam giác AKC (248) Lời giải đề thi vào lớp 10 chuyên Toán năm học 2010 - 2011 217 Vì ∠BAH = ∠KAC nên theo tính chất quen thuộc tam giác, ta có AK qua tâm đường tròn ngoại tiếp O tam giác ABC (đpcm) Bài Trong giải bóng đá có 12 đội tham dự, thi đấu vòng tròn lượt (hai đội thi đấu với đúng trận) (a) Chứng minh sau vòng đấu (mỗi đội thi đấu đúng trận) luôn tìm ba đội bóng đôi chưa thi đấu với (1 điểm) (b) Khẳng định trên còn đúng không các đội đã thi đấu trận? (0.5 điểm) Lời giải (a) Xét đội bóng A Sau vòng đấu, A chưa đấu với đội bóng Gọi S là tập hợp tất các đội bóng chưa đấu với A Xét đội bóng B thuộc S Do B đấu trận nên B thi đấu nhiều với đội bóng thuộc S Suy B chưa thi đấu với ít đội bóng thuộc S Giả sử B chưa thi đấu với C thuộc S Khi đó A, B, C đôi chưa thi đấu với (đpcm) (b) Kết luận là không Ta chia 12 đội thành nhóm, nhóm đội Cho các đội thi đấu vòng tròn nhóm thì sau năm vòng, đội thuộc nhóm đã thi đấu với Lấy đội bóng bất kỳ, theo nguyên lý Dirichlet có hai đội cùng nhóm, và vì các đội này đã thi đấu với Suy không tồn đội bóng đôi chưa thi đấu với (249) 218 Chuyên đề Toán học số d DANH NGÔN TOÁN HỌC • “Give me a place to stand, and I will move the earth.” Archimedes of Syracus (287-212 B C E) • “This, therefore, is Mathematics: She reminds you of the invisible forms of the soul; she gives life to her own discoveries; she awakens the mind and purifies the intellect; she brings to light our intrinsic ideas; she abolishes oblivion and ignorance which are ours by birth ” Diadochus Proclus (410-485) • “Mathematics is the door and key to the sciences.” Roger Bacon (1214-1294) • “Perfect numbers like perfect men are very rare.” Rene Descartes (1596-1650) • “Mathematics is the science of what is clear by itself.” Carl Gustav Jacobi (1804-1851) • “Do not worry about your difficulties in mathematics, I assure you that mine are greater.” Albert Einstein (1879-1955) • “What remains to be resovled is the question of knowing to what extent and up to what point these hypotheses are found to be confirmed by experience.” Bernard Riemann (1826-1866) • “Logic is the hygiene the mathematician practices to keep his ideas healthy and strong.” Hermann Klaus Hugo Weyl (1885-1955) • “If there is a problem you can’t solve, then there is an easier problem you can solve: find it.” George Polya (1887-1985) • “When introduced at the wrong time or place, good logic may be the worst enemy of good teaching.” George Polya (1887-1985) (250) ĐÁP ÁN ĐỀ THI CHỌN ĐỘI TUYỂN TOÁN NĂM HỌC 2008 - 2009 Ngày thứ nhất, 21/11/2008 Bài (a) Chứng minh tồn số n chẵn, n > 2008 cho 2009n − 49 là số chính phương (b) Chứng minh không tồn số nguyên m cho 2009m − 147 là số chính phương Lời giải Chú ý 2009 = 49 · 41 = 72 · 41 nên yêu cầu bài toán tương đương với việc chứng minh (a) Tồn số n chẵn, n > 2008 cho 41n − là số chính phương (b) Không tồn số nguyên m cho 41m − là số chính phương (a) Trước hết ta tìm số a cho a2 + chia hết cho 41 Điều này có thể thực cách thử Ta dễ dàng tìm a = thoả mãn Từ đây, ta thấy các số (82k + 9)2 + là số chẵn và chia hết cho 41 Bây cần chọn n= (82k + 9)2 + 41 với k đủ lớn là ta tìm số n thoả mãn điều kiện đề bài (b) Giả sử tồn m cho 41m − = a2 Khi đó ta có −3 ≡ a2 (mod 41) Từ đó theo định lý nhỏ Fermat (−3)20 ≡ a40 ≡ (mod 41) (1) Nhưng mặt khác ta lại có (−3)4 ≡ −1 (mod 41), suy (−3)20 ≡ (−1)5 = −1 (mod 41) (2) Do (1) và (2) mâu thuẫn với nên điều giả sử trên là sai Vậy không tồn số nguyên m cho 41m − là số chính phương Bài Cho số nguyên dương n Có bao nhiêu số chia hết cho 3, có n chữ số và các chữ số thuộc {3, 4, 5, 6}? Lời giải Bài toán này có ba cách giải sau Cách Gọi An tập hợp các số có n chữ số lập từ các chữ số {3, 4, 5, 6} và chia hết cho 3, và Bn là tập hợp các số có n chữ số lập từ các chữ số {3, 4, 5, 6} và không chia hết cho Ta cần tìm an = |An | Đặt bn = |Bn | Ta thấy số thuộc An+1 có thể thu (và có thể thu được) hai cách sau đây: 219 (251) 220 Chuyên đề Toán học số (1) Lấy số thuộc An thêm vào phía sau (cả hai được) (2) Lấy số thuộc Bn thêm 4, vào phía sau, nữa, có cách thêm Từ đây suy an+1 = 2an + bn (1) Lý luận hoàn toàn tương tự với Bn+1 , ta bn+1 = 2an + 3bn (2) Rút bn = an+1 − 2an (và bn+1 = an+2 − 2an+1 ) từ (1) và thay vào (2), ta có an+2 − 2an+1 = 2an + 3(an+1 − 2an ) Sau thu gọn, ta an+2 − 5an+1 + 4an = (3) Giải phương trình sai phân (3) với điều kiện a1 = (có hai số là 3, 6), a2 = (có các số 4n + 33, 66, 36, 63, 45, 54), ta an = Cách Lý luận tương tự trên chú ý rằng, số tất các số có n chữ số lập từ {3, 4, 5, 6}, theo quy tắc nhân, 4n nên ta có bn = 4n − an Và có thể thu công thức truy hồi an+1 = 2an + 4n − an Từ đó an = 4n−1 + an−1 = 4n−1 + 4n−2 + an−2 = · · · = 4n−1 + · · · + + a1 = 4n − 4n + +1 = 3 Cách Chú ý là số tự nhiên chia hết cho và tổng các chữ số nó chia hết cho Do số các số có n chữ số lập từ {3, 4, 5, 6} và chia hết cho (mà ta gọi là an ) tổng các hệ số x3k khai triển đa thức F (x) = (x3 + x4 + x5 + x6 )n Gọi ε là nghiệm phương trình ε2 + ε + = thì ε3 = Từ đây dễ dàng chứng minh ε2k + εk + = k không chia hết cho và = k chia hết cho Từ đây ta suy F (1) + F (ε) + F (ε2 ) = 3an 4n + n Nhưng F (1) = , F (ε) = F (ε ) = nên ta suy an = Bài Cho tam giác ABC có A cố định và B, C thay đổi trên đường thẳng d cố định cho gọi A0 là hình chiếu A lên d thì A0 B · A0 C âm và không đổi Gọi M là hình chiếu A0 lên AB (a) Chứng minh tâm I đường tròn ngoại tiếp tam giác BCM thuộc đường thẳng cố định (252) Đáp án đề chọn đội tuyển Toán năm học 2008 - 2009 221 (b) Gọi N là hình chiếu A0 lên AC, K là giao điểm các tiếp tuyến đường tròn ngoại tiếp tam giác A0 M N M và N Chứng minh K thuộc đường thẳng cố định Lời giải A F N0 M Z N C A0 E Y B I K P (a) Đặt A0 B · A0 C = −k , k > 0, (I, R) = (BM N ) Từ I hạ IE vuông góc với AA0 Gọi N , P là giao điểm (I, R) và AA0 Ta có AM · AB = AN · AP = AA02 Do AN = AA0 + A0 N và AP = AA0 + A0 P nên (AA0 + A0 N )(AA0 + A0 P ) = AA02 Thu gọn lại, ta AA0 (A0 N + A0 P ) = −A0 N · A0 P Mà A0 N · A0 P = −k nên từ trên ta có AA0 (A0 N + A0 P ) = k , từ đó suy 2A0 E = k2 AA0 Suy E cố định Vậy I chạy trên đường thẳng qua E vuông góc AA0 cố định (253) 222 Chuyên đề Toán học số (b) Gọi F là trung điểm AA0 , ta có F là tâm đường tròn ngoại tiếp tứ giác AN A0 M Gọi Z là giao điểm M N và AA0 , ta có ZA · ZA0 = ZM · ZN = −k Suy Z cố định Bây giờ, từ K hạ KY vuông góc với AA0 Ta có F, M, N, K, Y cùng nằm trên đường tròn, suy ZF · ZY = ZM · ZN = −k Từ đây dễ thấy Y cố định Vậy K di động trên đường thẳng qua Y và vuông góc AA0 cố định Bài Cho đa thức f (x) = x2 + ax + b Biết phương trình f (f (x)) = có bốn nghiệm phân biệt x1 , x2 , x3 , x4 và x1 + x2 = −1 Chứng minh b ≤ − Lời giải Từ điều kiện đề bài suy phương trình f (x) = có hai nghiệm phân biệt c, d và x1 , x2 , x3 , x4 là các nghiệm cặp phương trình f (x) = c, f (x) = d Ta xét các trường hợp • x1 , x2 là nghiệm cùng phương trình, chẳng hạn phương trình f (x) = c • x1 , x2 là nghiệm hai phương trình khác nhau, chẳng hạn f (x1 ) = c, f (x2 ) = d Trong trường hợp thứ nhất, áp dụng định lý Viette ta suy a = và c là nghiệm phương trình f (x) = ta có c2 + c + b = Phương trình f (x) = c ⇔ x2 + x + b − c = có hai nghiệm phân biệt suy ∆ = − 4(b − c) > Tương tự − 4(b − d) > 0, suy + 4(c + d) > 8b Nhưng c + d = −1 nên ta b<− Xét trường hợp thứ hai Trong trường hợp này ta có x21 + ax1 + b = c, x22 + ax2 + b = d Cộng hai đẳng thức này vế theo vế, với chú ý c + d = −a và x1 + x2 = −1, ta x21 + x22 + 2b = (x1 + x2 )2 + (x1 − x2 )2 Suy b = − ≤− 4 (254) Đáp án đề chọn đội tuyển Toán năm học 2008 - 2009 223 Ngày thứ hai, 22/11/2008 Bài Giả sử P (x) = (x + 1)p (x − 3)q = xn + a1 xn−1 + a2 xn−2 + · · · + an , đó p, q là các số nguyên dương Chứng minh a1 = a2 thì 3n là số chính phương Lời giải Ta có P (x) = (x + 1)p (x − 3)q = (xp + Cp1 xp−1 + Cp2 xp−2 + · · · )(xq − 3Cq1 xq−1 + 9Cq2 xq−2 + · · · ) Từ đó suy a1 = Cp1 − 3Cq1 , a2 = Cp2 + 9Cq2 − 3Cp1 Cq1 Như đẳng thức a1 = a2 tương đương với p − 3q = p(p − 1) 9q(q − 1) + − 3pq, 2 2p − 6q = p2 − p + 9q − 9q − 6pq, 3(p + q) = (p + 3q)2 , 3n = (p + 3q)2 Đẳng thức cuối cùng chứng tỏ 3n là số chính phương (đpcm) Bài (a) Cho a, b, c là các số thực dương Chứng minh ta có bất đẳng thức 8abc a2 + b + c + ≥ ab + bc + ca (a + b)(b + c)(c + a) (b) Chứng minh tồn các số thực dương a, b, c cho ab + bc + ca (a + b)(b + c)(c + a) + < a2 + b + c 8abc Lời giải (a) Xin nêu hai cách chứng minh cho bất đẳng thức này Cách Không tính tổng quát, ta có thể giả sử c = min{a, b, c} Khi đó, với chú ý a2 + b2 + c2 ≥ ab + bc + ca, ta có a2 + b + c a2 + b2 + c2 − ab − bc − ca a2 + b2 + c2 − ab − bc − ca =1+ ≥1+ ab + bc + ca ab + bc + ca ab + bc + ca + c2 a2 + b2 + 2c2 a2 + b2 + 2c2 = = ab + bc + ca + c2 (a + c)(b + c) (255) 224 Chuyên đề Toán học số Vậy ta cần chứng minh a2 + b2 + 2c2 8abc + ≥ (a + c)(b + c) (a + b)(b + c)(c + a) Bất đẳng thức này tương đương với (a2 + b2 + 2c2 )(a + b) + 8abc ≥ 2(a2 b + a2 c + b2 a + b2 c + c2 a + c2 b + 2abc), a3 + b3 + a2 b + b2 a + 2c2 a + 2c2 b + 8abc ≥ 2(a2 b + a2 c + b2 a + b2 c + c2 a + c2 b + 2abc), a3 + b3 + 4abc ≥ a2 b + b2 a + 2a2 c + 2b2 c, (a − b)2 (a + b − 2c) ≥ Do c = min{a, b, c} nên bất đẳng thức cuối hiển nhiên đúng Phép chứng minh hoàn tất Đẳng thức xảy và a = b = c Cách Sử dụng phương pháp SOS Ta viết lại bất đẳng thức sau 8abc a2 + b + c −1≥1− ab + bc + ca (a + b)(b + c)(c + a) Do a2 + b2 + c2 − (ab + bc + ca) = [(a − b)2 + (b − c)2 + (c − a)2 ] và (a + b)(b + c)(c + a) − 8abc = a(b − c)2 + b(c − a)2 + c(a − b)2 , nên bất đẳng thức trên có thể viết lại dạng Sa (b − c)2 + Sb (c − a)2 + Sc (a − b)2 ≥ 0, đó 2a − , ab + bc + ca (a + b)(b + c)(c + a) 2b Sb = − , ab + bc + ca (a + b)(b + c)(c + a) 2c Sc = − ab + bc + ca (a + b)(b + c)(c + a) Sa = Không tính tổng quát, giả sử a ≥ b ≥ c Khi đó ta dễ thấy Sa ≤ Sb ≤ Sc Lại có Sa + Sb = 2 2c2 − = > 0, ab + bc + ca (a + c)(b + c) (a + c)(b + c)(ab + bc + ca) nên 2Sb ≥ Sa + Sb > 0, suy Sc ≥ Sb ≥ Từ đây với chú ý (a − c)2 ≥ (b − c)2 , ta Sa (b − c)2 + Sb (c − a)2 + Sc (a − b)2 ≥ Sa (b − c)2 + Sb (b − c)2 = (Sa + Sb )(b − c)2 ≥ (b) Kiểm tra trực tiếp, dễ thấy số (a, b, c) = (2, 1, 1) thỏa mãn yêu cầu đề bài (256) Đáp án đề chọn đội tuyển Toán năm học 2008 - 2009 225 Bài Cho góc Oxy và điểm P bên nó γ là đường tròn thay đổi luôn qua O và P, γ cắt các tia Ox, Oy M, N Tìm quỹ tích trọng tâm G và trực tâm H tam giác OM N Lời giải (a) Quỹ tích trọng tâm G tam giác OM N x X M I G P O Y N y −→ −→ Gọi I là trung điểm M N, ta có OG = OI nên G là ảnh I qua phép vị tự tâm O tỉ số Từ đây suy {G} = VO3 ({I}), tức quỹ tích G luôn là ảnh quỹ tích I qua phép vị tự tâm O tỉ số Ta đưa bài toán tìm quỹ tích I Phần thuận Gọi X là giao điểm thứ hai (IP M ) và Ox, Y là giao điểm thứ hai (IP N ) và OY Ta có (IX, IP ) ≡ (M X, M P ) (I, M, X, P đồng viên) ≡ (M O, M P ) ≡ (N O, N P ) (O, P, M, N đồng viên) ≡ (N Y, N P ) ≡ (IY, IP ) (I, N, Y, P đồng viên) Suy (IX, IY ) ≡ (mod π) Vậy X, Y, I thẳng hàng Mặt khác, ta lại có (XY, XP ) ≡ (XI, XP ) ≡ (M I, M P ) (I, M, X, P đồng viên) ≡ (Oy, OP ) (O, P, M, N đồng viên), (257) 226 Chuyên đề Toán học số nên (XY, XP ) không đổi Tương tự, ta có (Y X, Y P ) không đổi Mà P cố định nên X, Y cố định Vậy I thuộc đường thẳng XY cố định Phần đảo Lấy X ∈ Ox, Y ∈ Oy cho (XY, XP ) ≡ (Oy, OP ) (mod π), (Y X, Y P ) ≡ (Ox, OP ) (mod π) và lấy I ∈ XY Ta chứng minh tồn M ∈ Ox, N ∈ Oy cho đường tròn (OM N ) qua P Thật vậy, gọi M là giao điểm thứ hai (IXP ) và Ox, N là giao điểm thứ hai (IY P ) và Oy Ta có (M X, M P ) ≡ (IX, IP ) (I, M, X, P đồng viên) ≡ (IY, IP ) (Do I ∈ XY ) ≡ (N Y, N P ) (I, N, Y, P đồng viên) Do đó O, M, P, N đồng viên Từ đây suy (OM N ) qua P Kết luận Quỹ tích I là đường thẳng XY, đó X, Y dựng (XY, XP ) ≡ (Oy, OP ) (mod π), (Y X, Y P ) ≡ (Ox, OP ) (mod π) (b) Quỹ tích trực tâm H tam giác OM N Trước hết, ta chứng minh bổ đề sau Bổ đề Cho tam giác ABC có góc ∠BAC = α không đổi, O là tâm đường tròn ngoại HA tiếp và H là trực tâm tam giác Khi đó ta có không đổi và ∠HAB = ∠OAC OA Chứng minh A H O B C Xét các trường hợp ∠BAC = α < 90◦ , ∠BAC = α = 90◦ , ∠BAC = α > 90◦ , ta luôn có ∠HAB = ∠OAC và HA = 2R| cos A|, nên HA = 2| cos A| = const OA Bổ đề chứng minh Bây ta tìm quỹ tích trực tâm H tam giác OM N (258) Đáp án đề chọn đội tuyển Toán năm học 2008 - 2009 227 x M X I K T P O H N Y y Phần thuận Gọi • T là trung điểm OP • X, Y là hình chiếu P lên Ox, Oy • K là trực tâm tam giác OXY • I là tâm đường tròn (OM N ) Theo bổ đề ta có T, K là tâm đường tròn ngoại tiếp và trực tâm tam giác OXY nên OK = 2| cos ∠xOy|, OT ∠KOy = ∠T Ox (1) Tương tự, I, H là tâm đường tròn ngoại tiếp và trực tâm tam giác OM N nên OH = 2| cos ∠xOy|, OI Từ (1) và (2), ta có ∠IOy = ∠HOx (2) OK OH = và ∠IOT = ∠HOK, suy OT OI 4IOT ∼ 4HOK Mặt khác, vì I là tâm đường tròn (OM N P ) và T là trung điểm OP nên IT ⊥ OP hay ∠IT O = 90◦ Kết hợp với trên, ta có ∠HKO = 90◦ Vậy H thuộc đường thẳng qua K vuông góc với OK (chú ý K cố định) Phần đảo Lấy H thuộc đường thẳng qua K vuông góc với OK, đó K là trực tâm tam giác OXY và X, Y là hình chiếu P lên Ox, Oy Ta chứng minh tồn M ∈ Ox, N ∈ Oy cho (OM N ) qua P và 4OM N nhận H làm trực tâm (259) 228 Chuyên đề Toán học số Thật vậy, gọi T là trung điểm OP và dựng 4OT I ∼ 4OKH Ta có ∠OT I = ∠OKH = 90◦ , (3) ∠IOT = ∠HOK (4) Từ (3) suy I nằm trên trung trực OP Do đó vẽ (I, IO) thì đường tròn này qua P và cắt Ox, Oy M, N Ta có K, T là trực tâm và tâm đường tròn ngoại tiếp tam giác OXY nên ∠KOy = ∠T Ox Kết hợp với (4), ta ∠IOy = ∠HOx OH OT = = 2| cos ∠xOy| Kết hợp với trên, ta suy OI OK H là trực tâm tam giác OM N Lại có 4OT I ∼ 4OKH nên Kết luận Quỹ tích trực tâm tam giác OM N là đường thẳng qua K vuông góc với OK, đó K là trực tâm tam giác OXY và X, Y là hình chiếu P lên Ox, Oy Bài Với số nguyên dương n, gọi S(n) là tổng các chữ số n (a) Chứng minh các số n = 999 và n = 2999 không thể biểu diễn dạng a + b với S(a) = S(b) (b) Chứng minh với số 999 < n < 2999, ta biểu diễn dạng a + b với S(a) = S(b) Lời giải (a) Chú ý a + b = 999 thì phép cộng trên không có nhớ nên S(a) + S(b) = S(999) = 27 Như không thể có S(a) = S(b), 27 là số lẻ Lý luận tương tự 2999 (b) Trước hết ta chứng minh 999 < n < 2999 thì tồn số tự nhiên k cho S(k) + S(n − k) là số chẵn Thật vậy, S(n) là số chẵn thì ta chọn k = Nếu S(n) lẻ, giả sử n = ba2 a1 a0 , đó b = Do 999 < n < 2999 và n 6= 1999 (do S(n) lẻ) nên tồn i cho < Chọn i lớn thoả mãn điều kiện này Khi đó chọn k = 10i (ai + 1) thì S(k) = + còn S(n − k) = S(n) − − + (phép trừ có nhớ tạo số vị trí và giảm đơn vị vị trí trước đó) Từ đó suy S(k) + S(n − k) = + + S(n) − − + = S(n) + chẵn S(n) lẻ Bây giả sử ta đã tìm k cho S(k) + S(n − k) là số chẵn Khi đó, đặt k = a3 a2 a1 a0 , n − k = b3 b2 b1 b0 Do S(k) + S(n − k) chẵn nên số các số i cho + bi lẻ là chẵn Với cặp số (i, j) cho + bi = 2ki + 1, aj + bj = 2kj + lẻ, ta đổi → a0i = ki + 1, b0i = ki , a0j = kj , b0j = kj + Với các số i cho + bi = 2ki ta đổi → a0i = b0i = ki Khi đó dễ dàng nhận thấy a03 a02 a01 a00 + b03 b02 b01 b00 = a3 a2 a1 a0 + b3 b2 b1 b0 = n và S(a03 a02 a01 a00 ) = S(b03 b02 b01 b00 ), và ta có điều phải chứng minh (260) ĐÁP ÁN ĐỀ THI CHỌN ĐỘI TUYỂN TOÁN NĂM HỌC 2009 - 2010 Ngày thứ nhất, 23/11/2009 Bài Cho a, b, c là các số thực thỏa mãn điều kiện đa thức P (x) = x4 + ax3 + bx2 + cx + có ít nghiệm thực Hãy xác định tất các (a, b, c) cho tổng a2 + b2 + c2 đạt giá trị nhỏ Lời giải Gọi x0 là nghiệm P (x) (dễ thấy x0 6= 0) Do P (x0 ) = nên ta có −(x40 + 1) = ax30 + bx20 + cx0 Sử dụng bất đẳng thức Cauchy-Schwarz, ta (x40 + 1)2 = (ax30 + bx20 + cx0 )2 ≤ (a2 + b2 + c2 )(x60 + x40 + x20 ) Đặt t = x20 > thì từ trên suy a2 + b + c ≥ (t2 + 1)2 (t2 + 1)2 = t3 + t2 + t t(t2 + t + 1) Mà theo bất đẳng thức AM-GM thì t≤ Do đó t2 + , t2 + t + ≤ t2 + t2 + = (t2 + 1) 2 (t2 + 1)2 ≥ , suy t(t + t + 1) a2 + b + c ≥ Đẳng thức xảy và  x + ax30 + bx20 + cx0 + =    02  x0 =  a b c    = = x0 x0 x0 Giải hệ này ta tìm a = b = c = − 2 a = −b = c = 3 229 (261) 230 Chuyên đề Toán học số Bài Cho n là số nguyên dương và tập hợp A = {1, 2, , 2n} Một tập A gọi là tốt nó gồm đúng hai phần tử x, y cho |x − y| ∈ {1, n} Tìm số các tập hợp {A1 , A2 , , An } thỏa mãn Ai là tập tốt A với i = 1, 2, , n và A1 ∪ A2 ∪ · · · ∪ An = A Lời giải Gọi un (n = 1, 2, ) là số các tập hợp {A1 , A2 , , An } thỏa mãn yêu cầu đề bài đồng thời hai phần tử n và n + không cùng tập Ai nào Ta chia các số 1, 2, , 2n vào bảng × n sau n+1 n+2 n 2n Thế thì cách chọn liệt kê un tương ứng với cách chọn từ bảng trên các cặp gồm hai số cùng cột hai số liên tiếp trên cùng hàng Xét un+1 , vì phần tử 2(n + 1) có thể cùng với n + 2n + cùng tập Ai nào đó nên ta xét hai khả sau • 2(n + 1) và n + cùng thuộc tập Ai , giả sử đó là An+1 n+2 n+3 n+1 2(n + 1) Lúc này, cách chọn {A1 , A2 , , An } ứng với cách chọn các cặp số gồm các số cùng cột cạnh cùng hàng từ bảng × n Theo định nghĩa ta số cách chọn là un Vậy trường hợp này có un cách chọn • 2(n + 1) và 2n + cùng thuộc tập Ai , giả sử đó là An+1 n+2 n+3 n 2n + n+1 2(n + 1) Ta thấy n + có thể cùng với 2(n + 1) (trường hợp n + cùng với n + không xét un+1 ) 2(n + 1) đã cùng với 2n + nên n + phải cùng với n cùng tập Ai nào đó, giả sử là An Đến đây, sử dụng lý luận trường hợp trên ta thấy số cách chọn các tập {A1 , A2 , , An−1 } là un−1 Vậy theo quy tắc cộng thì un+1 = un + un−1 Mặt khác, u1 = 1, u2 = nên ta có thể dễ dàng tìm công thức un là   √ !n+1 √ !n+1 1− 1+  − un = √  2 Bây ta xét trường hợp sinh {A1 , A2 , , An } có n và n + cùng tập Ai nào đó (262) Đáp án đề chọn đội tuyển Toán năm học 2009 - 2010 n+1 n+2 231 n 2n Rõ ràng có thể cùng với n + n + đã cùng n nên có thể cùng với Tiếp theo, n + có thể cùng với 2, n + hay n + đã với còn n + đã với n nên n + phải với n + Tiếp tục, có thể cùng 2, hay n + đã với còn n + đã với n + nên phải với Tiếp tục lý luận trên, ta suy Ai phải có dạng {1, 2}, {3, 4}, , {n − 2, n − 1}, {n, n + 1}, {n + 1, n + 2}, , {2n − 1, 2n} Từ đó suy ra, trường hợp này cho ta {A1 , A2 , , An } n lẻ và không cho nào n chẵn Tóm lại, số các {A1 , A2 , , An } thỏa mãn yêu cầu đề bài là  n =       ! ! √ n+1 √ n+1     1+ 1−    n chẵn − √ 2    !n+1 !n+1   √ √   1+ 1−    n lẻ và n >  1+ √  −   2 Bài toán giải xong Bài Tìm tất các hàm số f : N∗ → N∗ thỏa mãn đồng thời các điều kiện: (1) f tăng ngặt trên N∗ ; (2) f (f (n)) = 4n + 9, ∀n ∈ N∗ ; (3) f (f (n) − n) = 2n + 9, ∀n ∈ N∗ Lời giải Vì f làm hàm tăng và f : N∗ → N∗ nên f (a) − f (b) ≥ a − b, ∀a, b ∈ N∗ , a > b Từ (3), ta có = 2(n + 1) + − (2n + 9) = f (f (n + 1) − (n + 1)) − f (f (n) − n)) ≥ f (n + 1) − (n + 1) − [f (n) − n] = f (n + 1) − f (n) − Suy f (n + 1) − f (n) ≤ 3, ∀n ∈ N∗ , tức là f (n + 1) − f (n) ∈ {1, 2, 3}, ∀n ∈ N∗ • Giả sử tồn n ∈ N∗ cho f (n + 1) − f (n) = thì f (n + 1) − (n + 1) = f (n) − n, suy 2(n + 1) + = f (f (n + 1) − (n + 1)) = f (f (n) − n) = 2n + 9, vô lý (263) 232 Chuyên đề Toán học số • Giả sử tồn n ∈ N∗ cho f (n + 1) − f (n) = thì f (n + 1) − (n + 1) = f (n) − n + Ta lại có f (f (n + 1) − (n + 1)) − f (f (n) − n)) = 2, nên f (n) > n thì ta đặt t = f (n) − n, có t ∈ N∗ , f (t + 2) = f (t) + 2, mà f tăng nên f (t + 1) − f (t) = 1, mâu thuẫn Vậy nên f (n) ≤ n, song vì f tăng nên f (n) ≥ n, ∀n ∈ N∗ , dẫn đến f (n) = n Nhưng từ đây ta lại có n = f (n) = f (f (n)) = 4n + 9, suy n = −3, vô lý Vậy nên f (n + 1) − f (n) 6∈ {1, 3}, ∀n ∈ N∗ và vì f (n + 1) − f (n) = 2, ∀n ∈ N∗ Do đó f (n) = 2n + k với k nguyên nào đó Thay vào f (f (n)) = 4n + ta có k = Vậy f (n) = 2n + là hàm số thỏa mãn yêu cầu đề bài Bài Cho đường tròn (O) cố định và AB là dây cố định khác đường kính (O) Gọi I là trung điểm AB P là điểm di động trên cung lớn AB (O) Các điểm M, N thuộc các tia P A, P B cho ∠P M I = ∠P N I = ∠AP B (a) Chứng minh đường cao từ P tam giác P M N qua điểm cố định (b) Chứng minh đường thẳng Euler tam giác P M N qua điểm cố định Lời giải P O H A O N I M Y S X B (264) Đáp án đề chọn đội tuyển Toán năm học 2009 - 2010 233 (a) Ký hiệu X ≡ M I ∩ P B, Y ≡ N I ∩ P A Ta có ∠P M I = ∠P N I = ∠AP B nên các tam giác P M X và P N Y cân X, Y Từ đó suy ∠P XM = ∠P Y N = 180◦ − 2∠AP B, suy M, N, X, Y đồng viên Gọi S là tâm đường tròn ngoại tiếp tam giác AOB thì S cố định Ta có ∠ISB = 180◦ − ∠AOB = 180◦ − 2∠AP B = ∠P XM và tương tự ∠ISA = ∠P Y N Suy I, S, X, B đồng viên và I, S, Y, A đồng viên Suy ∠SXB = ∠SY A = ∠SIB = 90◦ , suy IS là đường kính đường tròn ngoại tiếp tam giác P XY Mặt khác M, N, X, Y đồng viên nên M N và XY đối song với ∠AP B, suy IS ⊥ M N Nói cách khác, đường cao từ P tam giác P M N qua điểm S cố định (đpcm) (b) Trước hết ta chứng minh bổ đề sau Bổ đề Cho tam giác ABC và đường tròn (ω) qua hai điểm B, C và cắt lại các cạnh AB, AC X, Y Gọi XX , Y Y là các đường cao tam giác AXY và BB , CC là các đường cao tam giác ABC, H, H là trực tâm tam giác ABC và tam giác AXY Ký hiệu I ≡ BY ∩ CX Khi đó H, I, H thẳng hàng Chứng minh A Y0 H0 X C I X0 Y B0 (ω) H C B Ta có X, Y, X , Y đồng viên nên H X · H X = H Y · H Y , tức là PH /[BY ] = PH /[CX] (ở đây ta dùng ký hiệu [U V ] để đường tròn đường kính U V ) Ta có B, C, B , C đồng viên nên HB · HB = HC · HC , tức là PH/[BY ] = PH/[CX] (265) 234 Chuyên đề Toán học số Cuối cùng B, C, X, Y đồng viên nên IB · IY = IC · IX, tức là PI/[BY ] = PI/[CX] Suy H, I, H thẳng hàng vì cùng thuộc trục đẳng phương [BY ] và [CX] Vào bài, gọi H, O0 là trực tâm và tâm đường tròn ngoại tiếp tam giác P M N Ta có O0 P = O0 M và XP = XM nên XO0 là đường trung trực P M, suy XO0 ⊥ P Y Tương tự ta có Y O0 ⊥ P X Vậy nên O0 chính là trực tâm tam giác P XY Áp dụng bổ đề cho tam giác P XY với (ω) ≡ (M N XY ) thì ta có O0 , H, I ≡ Y N ∩ M X thẳng hàng Nói cách khác, đường thẳng Euler O0 H tam giác P M N qua điểm I cố định (đpcm) Ngày thứ hai, 24/11/2009 Bài Cho a, b, c là các số thực dương Giải hệ phương trình sau    = bc2 ax − aby +   xy    =a abz − bc2 x +  zx      = ab  bc2 y − az + yz Lời giải Đặt (m, n, p) := (a, ab, bc2 ), thì m, n, p > Hệ phương trình trở thành   1     mx − ny + mx − ny − p = − =p     xy xy       1 = m ⇔ (∗) − m + nz − px = − nz − px +   zx zx       1     =n  py − mz +  − mz − n + py = − yz yz Xem (∗) là hệ (266) (267) −x (268) D = (269) (270) −1 (271) −z phương trình tuyến tính ẩn (m, n, p), ta có (272) −y −1 (273) (274) −z −x(275) (276) = xyz − xyz − − x2 − y − z = −(1 + x2 + y + z ) 6= −1 −y (277)(278) (279) (280) (281) (282)(283) (284) xy −y −1 (285) (286) (287) (288) (289) 1 1 + x2 + y + z z x y2 (290) (291) − + − =− Dm = (292) − −z −x(293) = − − − x z zx zx y y zx (294) zx (295) (296) (297) (298)(299) −1 −y (300) yz (301) (302) 235 Đáp án đề chọn đội tuyển Toán năm học 2009 - 2010 Tính toán tương tự Dm , ta (303) (304) (305) −x (306) (307) (308) Dn = (309) (310) −1 (311) (312) (313) −z (314)(315) (316) − −1 (317) (318) xy (319) (320) + x2 + y + z (321) , − −x(322) = − yz zx (323) (324) −y (325) (326) − yz (327) (328) (329) 1(330) (331) −x −y − (332) (333) xy (334) (335) (336) (337) + x2 + y + z 1(338) Dp = (339) (340) −1 −z − (341) (342) = − xy zx (343) (344) (345) (346) (347) −z −1 − (348) (349) yz (350) Từ đó suy  (m, n, p) = Dm Dn Dp , , D D D   = 1 , , zx yz xy  Thay (m, n, p) = (a, ab, bc2 ), ta xy = , bc2 yz = , ab zx = a Nhân ba phương trình trên lại theo vế lấy bậc hai, ta có xyz = ± • Với xyz = abc z x y 1 c , ta có xyz = = = Từ đó suy x = , y = , z = abc bc ab a c bc a z x y 1 c , ta có xyz = = = Do đó x = − , y = − , z = − abc bc ab a c bc a     1 c c Vậy hệ phương trình có hai nghiệm (x, y, z) là , , và − , − , − c bc a c bc a • Với xyz = − Bài Cho dãy số {an } xác định bởi: a1 = a, an+1 = (a1 + a2 + · · · + an − 2)2 , ∀n = 1, 2, Ký hiệu Sn = a1 + a2 + · · · + an , ∀n = 1, 2, Tìm tất các giá trị a để {Sn } có giới hạn hữu hạn n → +∞ Lời giải Từ giả thiết suy Sn+1 − Sn = (Sn − 2)2 Vậy nên dãy {Sn } thực là xác định sau S1 = a, Sn+1 = f (Sn ) = Sn2 − 3Sn + (351) 236 Chuyên đề Toán học số Ta có Sn+1 − Sn = (Sn − 2)2 ≥ nên {Sn } là dãy không giảm Mặt khác, f (x) = 2x − = ⇔ x = x −∞ f (x) 3/2 − + +∞ +∞ +∞ &2 f (x) & 7/4 % 2% Xét các trường hợp a, ta thấy • Nếu a > Giả sử {Sn } có giới hạn L thì ta phải có L = f (L) nên L ∈ {1, 2}, song vì {Sn } không giảm nên L ≥ a > 2, mâu thuẫn Vậy a > thì {Sn } không hội tụ • Nếu a < thì từ bảng biến thiên suy S2 = f (a) > 2, quay trường hợp ta có {Sn } không hội tụ • Nếu ≤ a ≤ thì từ bảng biến thiên ta có ≤ Sn ≤ 2, ∀n ∈ N∗ Từ đó, {Sn } không giảm và bị chặn trên nên {Sn } hội tụ Vậy các giá tị cần tìm a là a ∈ [1, 2] Bài Tìm tất các số nguyên dương k cho phương trình x2 + y + x + y = kxy có nghiệm nguyên dương x, y Lời giải Không tính tổng quát, giả sử x ≥ y Xét giá trị k cho phương trình đã cho có nghiệm nguyên dương Trong các nghiệm ấy, ta gọi (x0 , y0 ) là nghiệm cho x0 ≥ y0 > và x0 nhỏ Xét tam thức f (x) = x2 − (ky0 − 1)x + y02 + y0 , ta có f (x0 ) = và theo định lý Viette thì f (x) còn có nghiệm x00 = ky0 − − x0 Nhưng theo cách chọn (x0 , y0 ) ta thì x00 ≥ x0 ≥ y0 nên y0 nằm ngoài khoảng hai nghiệm tam thức bậc hai f (x) có hệ số cao là số dương Từ đó f (y0 ) ≥ Do f (y0 ) = 2y02 + 2y0 − ky02 nên ta có k ≤2+ ≤ (vì y0 ≥ 1) y0 Suy k ∈ {1, 2, 3, 4} • Nếu k = thì phương trình có dạng x2 + y + x + y = xy, tương đương với  y 2 3y x− + + x + y = (vô lý vì x, y > 0) (352) 237 Đáp án đề chọn đội tuyển Toán năm học 2009 - 2010 • Nếu k = thì phương trình có dạng x2 + y + x + y = 2xy, tương đương với (x − y)2 + x + y = (vô lý vì x, y > 0) • Nếu k = thì phương trình có nghiệm (x, y) = (2, 2) • Nếu k = thì phương trình có nghiệm (x, y) = (1, 1) Vậy các giá trị cần tìm k là k = và k = Bài Cho tam giác ABC nội tiếp đường tròn (O) Ký hiệu I, I1 , I2 , I3 là tâm đường tròn nội tiếp và tâm các đường tròn bàng tiếp các góc A, B, C tam giác ABC Đường tròn ngoại tiếp tam giác II2 I3 cắt đường tròn ngoại tiếp tam giác ABC hai điểm M1 , N1 AI cắt đường tròn ngoại tiếp tam giác ABC J1 Ký hiệu d1 là đường thẳng qua J1 và vuông góc với M1 N1 Các đường thẳng d2 , d3 định nghĩa tương tự Chứng minh d1 , d2 , d3 đồng quy Lời giải I1 B J1 C N1 I S M1 O O0 I3 I2 A O1 (353) 238 Chuyên đề Toán học số Gọi (O0 ) là đường tròn ngoại tiếp tam giác I1 I2 I3 và (O1 ) là đường tròn ngoại tiếp tam giác II2 I3 Ta có AI ⊥ I2 I3 , BI ⊥ I3 I1 nên I là trực tâm tam giác I1 I2 I3 và (O) là đường tròn Euler tam giác I1 I2 I3 nên O là trung điểm IO0 Mà ∠I2 O1 I3 = 2(180◦ − ∠I2 II3 ) = 2∠I2 I1 I3 = ∠I2 O0 I3 , −−−→ −→ nên O0 đối xứng với O1 qua I2 I3 , suy O0 O1 = I1 I, tức là AIO1 O0 là hình bình hành Mà O là trung điểm IO0 nên O là trung điểm I1 O1 Hơn OO1 ⊥ M1 N1 (đường nối tâm vuông góc với dây chung) nên I1 O ⊥ M1 N1 Mặt khác, J1 là trung điểm II1 (đường tròn Euler qua trung điểm đoạn thẳng nối trực tâm tam giác với đỉnh tam giác ấy) nên phép vị tự VI2 : I1 O1 → d1 Suy d1 qua trung điểm S OI Lý luận tương tự thì d2 , d3 qua trung điểm S OI Vậy ta có d1 , d2 , d3 đồng quy trung điểm S OI (đpcm) (354) MỘT SỐ ĐỀ THI OLYMPIC TOÁN Là học sinh chuyên Toán hẳn các bạn luôn muốn tìm cho mình thử thách thật khó khăn và tìm cách vượt qua chúng Đó là dịp để các bạn kiểm tra lại kiến thức mình học khả sáng tạo thân, và qua đó biết lực mình đến đâu Các thi Olympic Toán tổ chức nhằm mục đích Và đây, chúng tôi xin trích vài đề thi Olympic Toán thú vị hai năm gần đây Các bạn hãy cùng thử sức với chúng nhé! CUỘC THI OLYMPIC TOÁN MANG TÊN LEONARD EULER (24 - 27/3/2009) Ngày thi thứ Bài Bộ tộc Mumbo và Jumbo sống bên cạnh bờ sông Có lần hai người dân lạc phải chạy gấp đến tộc bên cạnh để đưa tin: Mumbo trẻ khỏe và Jumbo thông thái Mumbo chạy với vận tốc 11km/h đến trại bè gần và bè đến tộc bên cạnh Còn Jumbo không vội vàng mà với vận tốc 6km/h đến trại bè khác và bè đến tộc bên cạnh Kết là Jumbo đến sớm Mumbo Biết dòng sông thẳng, các bè di chuyển với vận tốc dòng sông và vận tốc này không đổi trên toàn tuyến sông và là số nguyên không nhỏ 6km/h Hãy tìm giá trị lớn có thể vận tốc này Bài Phải với n nguyên dương lớn 2009, từ các phân số , n , n−1 , n−2 ., n−1 , n luôn chọn hai cặp phân số có tổng nhau? Bài Cho tam giác ABC cân B Điểm D nằm tam giác ABC cho góc ADC gấp đôi góc ABC Chứng minh hai lần khoảng cách từ B đến phân giác ngoài góc D tam giác ADC AD + DC Bài Ở đất nước Leonardia đường có chiều Mỗi đường nối hai thành phố và không qua các thành phố khác Bộ thống kê tính cho thành phố tổng số dân các thành phố mà từ đó có đường đến thành phố tính và tính tổng số 239 (355) 240 Chuyên đề Toán học số dân các thành phố mà từ thành phố tính có đường tới Chứng minh có ít thành phố mà số thứ không nhỏ số thứ hai Ngày thi thứ hai Bài Tồn hay không cách thay số nguyên liên tiếp vào các dấu “∗” để đẳng thức đúng BSCN N (∗, ∗, ∗) − BSCN N (∗, ∗, ∗) = 2009? Bài Trong tứ giác ABCD ta có AB = BD, ∠ABD = ∠DBC Trên đường chéo BD tồn điểm K cho BK = BC Chứng minh ∠KAD = ∠KCD Bài Trên bàn có 10 đống hạt dẻ với số lượng là 1, 2, 3, 4, 5, 6, 7, 8, 9, 10 hạt dẻ Hai người luân phiên bốc hạt dẻ, lần bốc hạt còn lại hạt dẻ thì dừng Nếu đây là hạt dẻ thuộc đống khác thì người sau thắng Trong trường hợp ngược lại người trước thắng Hỏi là người có chiến thuật thắng, cho dù đối thủ chơi nào? Bài Trên băng giấy vô hạn ta viết các số liền Đầu tiên là số và các số sau số trước cộng với chữ số khác nhỏ số đứng trước Hãy tìm số chữ số số đứng vị trí · 10001000 ĐỀ THI OLYMPIC TOÁN TRUNG ÂU 2010 Phần A Đề thi cá nhân Bài Tìm tất các hàm số f : R → R thỏa mãn f (x + y) + f (x)f (y) = f (xy) + (y + 1)f (x) + (x + 1)f (y), với x, y ∈ R Bài Tất các ước dương số nguyên dương N viết lên bảng đen Hai người chơi A và B tiến hành trò chơi luân phiên sau: Ở lượt đầu tiên, A xóa số N Nếu số cuối cùng xóa là d thì người chơi phải xóa ước số d bội số d Người nào không xóa thua Xác định tất các giá trị N cho A luôn có cách thắng mà không phụ thuộc vào B (356) Một số đề thi Olympic Toán 241 Bài Cho tứ giác ABCD nội tiếp có E nằm trên đường chéo AC thỏa mãn AD = AE, CB = CE Gọi M là tâm đường tròn ngoại tiếp k tam giác BDE Đường tròn k cắt đường thẳng AC E và F Chứng minh F M, AD, BC đồng quy Bài Tìm tất các số nguyên dương n thỏa mãn hai điều kiện sau (a) n phải có ít ước nguyên dương khác (b) Với hai ước a, b n thỏa mãn < a < b < n, thì (b − a) | n Phần B Đề thi đồng đội Bài Cho ba dãy tăng thực gồm các số nguyên dương a1 , a , a , , b1 , b2 , b3 , , c1 , c2 , c3 , Biết số nguyên dương thuộc đúng ba dãy nêu trên Đồng thời, với số nguyên dương n, các điều kiện sau đây thỏa mãn (a) can = bn + (b) an+1 > bn (c) cn+1 cn − (n + 1)cn+1 − ncn là số chẵn Tính a2010 , b2010 , c2010 Bài Tìm số Cn lớn cho bất đẳng thức sau đúng với a1 , a2 , , an > 0,  2 a1 + a2 + · · · + an a21 + a22 + · · · + a2n ≥ + Cn (a1 − an )2 n n Bài Tại đỉnh n-giác có đặt pháo đài Tại cùng thời điểm, pháo đài bắn vào hai pháo đài gần nó và trúng đích Ta gọi kết bắn là tập hợp các pháo đài bị bắn trúng (chú ý không phân biệt pháo đài bị bắn hay hai lần) Gọi P (n) là số lượng có thể kết bắn Chứng minh với số nguyên dương k ≥ 3, ta có P (k) và P (k + 1) là hai số nguyên tố cùng Bài Cho n là số nguyên dương Một hình vuông ABCD chia thành n2 hình vuông đơn vị Mỗi hình vuông đơn vị lại chia đôi thành hai tam giác đường chéo song (357) 242 Chuyên đề Toán học số song với cạnh BD Ta tô màu đỏ số đỉnh các hình vuông đơn vị cho tam giác 2n2 tam giác nêu trên có ít đỉnh tô màu đỏ Tìm số nhỏ các đỉnh tô màu đỏ Bài Đường tròn nội tiếp tam giác ABC tiếp xúc với các cạnh BC, CA và AB D, E và F Gọi K là điểm đối xứng với D qua tâm đường tròn nội tiếp DE cắt F K S Chứng minh AS song song với BC Bài Cho các điểm A, B, C, D, E thỏa mãn ABCD là tứ giác nội tiếp và ABDE là hình bình hành Các đường chéo AC và BD cắt S, các tia AB và DC cắt F Chứng minh ∠AF S = ∠ECD Bài Với số tự nhiên n, ta gọi an là số nguyên dương có biễu diễn thập phân là 0} |0 {z 0} 1 |0 {z 0} |0 {z n n n an luôn có thể biểu diễn thành tổng hai số lập phương không thể biểu diễn thành tổng hai số chính phương Chứng minh Bài Cho n ∈ N∗ , n không là lũy thừa Chứng minh tồn m ∈ N∗ cho (a) m là tích hai số nguyên dương liên tiếp (b) Biễu diễn thập phân m gồm hai phần giống nhau, phần có đúng n chữ số ĐỀ KIỂM TRA SỐ (10/10/2010) (CÂU LẠC BỘ TOÁN HỌC) Bài Trên hòn đảo, cư dân toàn nói thật, toàn nói dối Alice và Bob là cư dân hòn đảo này Alice nói “Đúng hai chúng tôi nói dối” Bob nói “Alice đã nói thật” Hãy xác định xem là người nói thật, là người nói dối Bài Có vật có trọng lượng là 1, 2, 3, 4, 5, 6, 7, vật bị mất, còn vật còn lại xếp thành dãy tăng dần theo trọng lượng Hãy dùng lần cân bàn (có thể kiểm tra hai nhóm vật có trọng lượng hay không) để xác định xem vật bị có trọng lượng bao nhiêu (358) Một số đề thi Olympic Toán 243 Bài Chứng minh tổng bình phương 24 số nguyên tố có ba chữ số chia hết cho 24 Bài Hai số nguyên tố p và q gọi là hai số nguyên tố sánh đôi q = p + Chứng minh p và q là hai số nguyên tố sánh đôi thì pq + q p chia hết cho p + q Bài Dãy số Fibonacci là dãy số xác định F1 = 1, F2 = 2, Fn+1 = Fn + Fn−1 với n ≥ Chứng minh với n ≥ ta có |Fn2 − Fn+1 Fn−1 | = (359) 244 d Chuyên đề Toán học số (360)

Ngày đăng: 31/12/2020, 12:09

Hình ảnh liên quan

• Làm sao đưa các kiến thức hình học sẵn có (như một phương pháp hoặc một định lý nào đó) cho việc giải một bài toán HHP? - Chuyên đề bồi dưỡng học sinh giỏi toán 9 và ôn thi vào chuyên phần số học

m.

sao đưa các kiến thức hình học sẵn có (như một phương pháp hoặc một định lý nào đó) cho việc giải một bài toán HHP? Xem tại trang 43 của tài liệu.
• Kẻ thêm các đường phụ: Dựng DE k AB (như hình vẽ trên) rồi chứng minh rằngAC=DE, BE=ADđưa vềAD+BE &gt; AB+DE. - Chuyên đề bồi dưỡng học sinh giỏi toán 9 và ôn thi vào chuyên phần số học

th.

êm các đường phụ: Dựng DE k AB (như hình vẽ trên) rồi chứng minh rằngAC=DE, BE=ADđưa vềAD+BE &gt; AB+DE Xem tại trang 44 của tài liệu.
Lời giải 2. Sử dụng hình học phẳng thuần túy (dựng nhiều đường phụ, hướng suy nghĩ hơi thiếu tự nhiên và đòi hỏi có kinh nghiệm về các bài toán có giả thiết tương tự như thế này). - Chuyên đề bồi dưỡng học sinh giỏi toán 9 và ôn thi vào chuyên phần số học

i.

giải 2. Sử dụng hình học phẳng thuần túy (dựng nhiều đường phụ, hướng suy nghĩ hơi thiếu tự nhiên và đòi hỏi có kinh nghiệm về các bài toán có giả thiết tương tự như thế này) Xem tại trang 47 của tài liệu.
Suy ra FQ = A C= AE, QE = A B= AF và tứ giác AEQF là hình bình hành. Do đóI chính là trung điểm củaAQ,màMlà trung điểm củaQDnênIMchính là đường trung bình của tam giácQAD,suy raIM=1 - Chuyên đề bồi dưỡng học sinh giỏi toán 9 và ôn thi vào chuyên phần số học

uy.

ra FQ = A C= AE, QE = A B= AF và tứ giác AEQF là hình bình hành. Do đóI chính là trung điểm củaAQ,màMlà trung điểm củaQDnênIMchính là đường trung bình của tam giácQAD,suy raIM=1 Xem tại trang 48 của tài liệu.
Gọi H, K lần lượt là hình chiếu của B ,C lên đường thẳng d. D oD là trung điểm của BC nên DH=DK,suy raADlà trung trực củaHK,do đóAH=AK. - Chuyên đề bồi dưỡng học sinh giỏi toán 9 và ôn thi vào chuyên phần số học

i.

H, K lần lượt là hình chiếu của B ,C lên đường thẳng d. D oD là trung điểm của BC nên DH=DK,suy raADlà trung trực củaHK,do đóAH=AK Xem tại trang 49 của tài liệu.
Từ việc tìm cách dựng cho điểm K, ta cũng đã có thêm trên hình một số đường phụ cần thiết, bài toán đã rõ ràng hơn nhiều - Chuyên đề bồi dưỡng học sinh giỏi toán 9 và ôn thi vào chuyên phần số học

vi.

ệc tìm cách dựng cho điểm K, ta cũng đã có thêm trên hình một số đường phụ cần thiết, bài toán đã rõ ràng hơn nhiều Xem tại trang 51 của tài liệu.
Còn đối với các bài toán mà hình vẽ không thể dựng được bằng thước và compa thì sao, chẳng hạn như định lý Morley:“Cho tam giácABC.Các đường chia ba các góc của tam giác cắt nhau tại các điểm M, N, P.Khi đó ta cóM N Plà tam giác đều.” - Chuyên đề bồi dưỡng học sinh giỏi toán 9 và ôn thi vào chuyên phần số học

n.

đối với các bài toán mà hình vẽ không thể dựng được bằng thước và compa thì sao, chẳng hạn như định lý Morley:“Cho tam giácABC.Các đường chia ba các góc của tam giác cắt nhau tại các điểm M, N, P.Khi đó ta cóM N Plà tam giác đều.” Xem tại trang 52 của tài liệu.
2.3 Về việc rút ngắn con đường đi từ giả thiết đến kết luận - Chuyên đề bồi dưỡng học sinh giỏi toán 9 và ôn thi vào chuyên phần số học

2.3.

Về việc rút ngắn con đường đi từ giả thiết đến kết luận Xem tại trang 53 của tài liệu.
Ta thấy thiếu một vài yếu tố trong hình, một yếu tố nào đó cần có để kết nối các điều ta vừa phân tích được từ giả thiết đến kết luận của bài, yếu tố đó vừa phải đảm bảo rằng có liên quan đếnN Htrong các phương tích trên, vừa đảm bảo rằng có liên hệ đến đ - Chuyên đề bồi dưỡng học sinh giỏi toán 9 và ôn thi vào chuyên phần số học

a.

thấy thiếu một vài yếu tố trong hình, một yếu tố nào đó cần có để kết nối các điều ta vừa phân tích được từ giả thiết đến kết luận của bài, yếu tố đó vừa phải đảm bảo rằng có liên quan đếnN Htrong các phương tích trên, vừa đảm bảo rằng có liên hệ đến đ Xem tại trang 53 của tài liệu.
Ta thấy rằng phép quay vector sử dụng trong bài toán đã giúp hạn chế nhiều lập luận hình học phức tạp và cho ta một lời giải hết sức nhẹ nhàng, vấn đề là chúng ta phải biết căn cứ vào các đặc trưng của bài toán để vận dụng cho phù hợp và chính xác - Chuyên đề bồi dưỡng học sinh giỏi toán 9 và ôn thi vào chuyên phần số học

a.

thấy rằng phép quay vector sử dụng trong bài toán đã giúp hạn chế nhiều lập luận hình học phức tạp và cho ta một lời giải hết sức nhẹ nhàng, vấn đề là chúng ta phải biết căn cứ vào các đặc trưng của bài toán để vận dụng cho phù hợp và chính xác Xem tại trang 65 của tài liệu.
Đặc biệt khi ABCD là hình thang thì M N= nA B+ mDC m+n . - Chuyên đề bồi dưỡng học sinh giỏi toán 9 và ôn thi vào chuyên phần số học

c.

biệt khi ABCD là hình thang thì M N= nA B+ mDC m+n Xem tại trang 74 của tài liệu.
Lời giải. Nhận xét rằng bài toán trên có cách phát biểu khá thuần túy hình học, nhưng có lẽ đại số là cách tiếp cận tự nhiên hơn cho bài toán này. - Chuyên đề bồi dưỡng học sinh giỏi toán 9 và ôn thi vào chuyên phần số học

i.

giải. Nhận xét rằng bài toán trên có cách phát biểu khá thuần túy hình học, nhưng có lẽ đại số là cách tiếp cận tự nhiên hơn cho bài toán này Xem tại trang 76 của tài liệu.
Lời giải. Bài toán trên giải bằng phương pháp hình học thuần túy khá ngắn gọn và dễ dàng - Chuyên đề bồi dưỡng học sinh giỏi toán 9 và ôn thi vào chuyên phần số học

i.

giải. Bài toán trên giải bằng phương pháp hình học thuần túy khá ngắn gọn và dễ dàng Xem tại trang 82 của tài liệu.
Đường thẳng Newton cũng đúng đối với tứ giác lõm ngoại tiếp đường tròn như sau (trong hình bên dưới, tứ giác lõmABCD“ngoại tiếp” đường tròn tâmOvàM, Nlần lượt là hai trung điểm của các đường chéoACvàBD) - Chuyên đề bồi dưỡng học sinh giỏi toán 9 và ôn thi vào chuyên phần số học

ng.

thẳng Newton cũng đúng đối với tứ giác lõm ngoại tiếp đường tròn như sau (trong hình bên dưới, tứ giác lõmABCD“ngoại tiếp” đường tròn tâmOvàM, Nlần lượt là hai trung điểm của các đường chéoACvàBD) Xem tại trang 87 của tài liệu.
lần lượt là hình chiếu củ aI lên các cạnh AB, BC, CD, DA). Chứng minh rằng MN PQ là tứ giác ngoại tiếp. - Chuyên đề bồi dưỡng học sinh giỏi toán 9 và ôn thi vào chuyên phần số học

l.

ần lượt là hình chiếu củ aI lên các cạnh AB, BC, CD, DA). Chứng minh rằng MN PQ là tứ giác ngoại tiếp Xem tại trang 88 của tài liệu.
2 Các bài toán liên quan - Chuyên đề bồi dưỡng học sinh giỏi toán 9 và ôn thi vào chuyên phần số học

2.

Các bài toán liên quan Xem tại trang 88 của tài liệu.
Tiếp theo, để cho hình vẽ thoáng hơn, ta thay đổi đề bài một chút, ta định nghĩa lại điể mI - Chuyên đề bồi dưỡng học sinh giỏi toán 9 và ôn thi vào chuyên phần số học

i.

ếp theo, để cho hình vẽ thoáng hơn, ta thay đổi đề bài một chút, ta định nghĩa lại điể mI Xem tại trang 91 của tài liệu.
RCSD là hình bình hành. Do đó, hai đường chéo CD và RS của tứ giác cắt nhau tại trung điểm của mỗi đường - Chuyên đề bồi dưỡng học sinh giỏi toán 9 và ôn thi vào chuyên phần số học

l.

à hình bình hành. Do đó, hai đường chéo CD và RS của tứ giác cắt nhau tại trung điểm của mỗi đường Xem tại trang 93 của tài liệu.
CÁC ĐỐI TƯỢNG HÌNH ẢNH BẰNG MÁY VI TÍNH - Chuyên đề bồi dưỡng học sinh giỏi toán 9 và ôn thi vào chuyên phần số học
CÁC ĐỐI TƯỢNG HÌNH ẢNH BẰNG MÁY VI TÍNH Xem tại trang 178 của tài liệu.
Bảng 1. Liệt kê các phần tử của tập hợp A. - Chuyên đề bồi dưỡng học sinh giỏi toán 9 và ôn thi vào chuyên phần số học

Bảng 1..

Liệt kê các phần tử của tập hợp A Xem tại trang 179 của tài liệu.
Hình 3. Hệ tọa độ độc cực. - Chuyên đề bồi dưỡng học sinh giỏi toán 9 và ôn thi vào chuyên phần số học

Hình 3..

Hệ tọa độ độc cực Xem tại trang 180 của tài liệu.
mà đồ thị của chúng ở hình 4 lần lượt là - Chuyên đề bồi dưỡng học sinh giỏi toán 9 và ôn thi vào chuyên phần số học

m.

à đồ thị của chúng ở hình 4 lần lượt là Xem tại trang 181 của tài liệu.
Ở đây ta không dùng đồ thị hàm số biểu diễn dưới dạng bảng giá trị kiểu như - Chuyên đề bồi dưỡng học sinh giỏi toán 9 và ôn thi vào chuyên phần số học

y.

ta không dùng đồ thị hàm số biểu diễn dưới dạng bảng giá trị kiểu như Xem tại trang 181 của tài liệu.
3 Phương pháp tổ chức hình ảnh theo kiểu tự đồng dạng - Chuyên đề bồi dưỡng học sinh giỏi toán 9 và ôn thi vào chuyên phần số học

3.

Phương pháp tổ chức hình ảnh theo kiểu tự đồng dạng Xem tại trang 183 của tài liệu.
Hình 6. Đồ thị miêu tả hàm số r= 2(1 +5 cos 7ϕ) (hàm số theo tọa độ độc cực) và - Chuyên đề bồi dưỡng học sinh giỏi toán 9 và ôn thi vào chuyên phần số học

Hình 6..

Đồ thị miêu tả hàm số r= 2(1 +5 cos 7ϕ) (hàm số theo tọa độ độc cực) và Xem tại trang 183 của tài liệu.
Hình 7. Bông tuyết Von Koch qu a5 lần tự đồng dạng. - Chuyên đề bồi dưỡng học sinh giỏi toán 9 và ôn thi vào chuyên phần số học

Hình 7..

Bông tuyết Von Koch qu a5 lần tự đồng dạng Xem tại trang 184 của tài liệu.
Ta ngẫm lại thuật toán tí xíu: Lấy một tam giác đều có cạnh bằng 1, gọi là hình ban đầu K 0. - Chuyên đề bồi dưỡng học sinh giỏi toán 9 và ôn thi vào chuyên phần số học

a.

ngẫm lại thuật toán tí xíu: Lấy một tam giác đều có cạnh bằng 1, gọi là hình ban đầu K 0 Xem tại trang 184 của tài liệu.
Ngoài định nghĩa hình ảnh trên ta còn có thể định nghĩa bằng Số học (người ta chứng minh được rằng hai định nghĩa này là tương đương) như là tập hợp tất cả các số được viết dưới dạngphân số tam phân0, a 1, a2,  - Chuyên đề bồi dưỡng học sinh giỏi toán 9 và ôn thi vào chuyên phần số học

go.

ài định nghĩa hình ảnh trên ta còn có thể định nghĩa bằng Số học (người ta chứng minh được rằng hai định nghĩa này là tương đương) như là tập hợp tất cả các số được viết dưới dạngphân số tam phân0, a 1, a2, Xem tại trang 185 của tài liệu.
Định lý này cho phép ta có thể “số hóa” một Fractal bất kỳ. Và chính vì lẽ đó mà những hình ảnh có vẻ như trừu tượng, hỗn độn, vô trật tự (như hình đám mây, hình dãy núi, ngọn sóng, nhành cây,  - Chuyên đề bồi dưỡng học sinh giỏi toán 9 và ôn thi vào chuyên phần số học

nh.

lý này cho phép ta có thể “số hóa” một Fractal bất kỳ. Và chính vì lẽ đó mà những hình ảnh có vẻ như trừu tượng, hỗn độn, vô trật tự (như hình đám mây, hình dãy núi, ngọn sóng, nhành cây, Xem tại trang 185 của tài liệu.

Từ khóa liên quan

Tài liệu cùng người dùng

Tài liệu liên quan